asdf

¡Supera tus tareas y exámenes ahora con Quizwiz!

"150. Regarding acquired upper extremity amputation in adults,:(a) the most common level is transhumeral.:(b) they account for approximately 50% of all acquired major limb amputations.:(c) they are most common in the 20- to 40-year-old age group. (d)

their most common cau se is ma lig nancy ." 149 (c) A basilar skull fracture is often associated with a permanent sensorineural hearing loss. Other traumatic brain injuries may be associated with hearing loss, but this is less common. All patients with basilar skull fracture should be assumed to hav e a hea ring los s and have audio lo gical ev alu ati on a s so on as possible.

26 Which agent administered before kidney transplantation can increase exercise tolerance? a) erythropoietin b) glucocorticoids c) coenzyme Q10 d) vitamin C

26 Reference(s) (a) Young MA, Stiens S Rehabilitation aspects of organ transplantation. In: Braddom RL, editor. Physical medicine and rehabilitation. 4th e Phi ladelphia: Elsevier; 2011. p 1385. ( b) P rovatopoulou ST , Ziroyian nis P N. Clinical use of erythropoietin in chronic kidney disease: outcomes and future prospects. Hippokratia 2011.15(2):10915. (d) Handelman GJ. New insight on Vitamin C in patients with chronic kidney disease. J Ren Nutr 2011 Jan;21(1):110-2. Option a is correct. Studies show that exercise training and treatment with erythropoiesis-stimulating agents such as epoetin (EPO) can increase exercise tolerance in patients p re- and post-renal transplant. Speci fica lly, the find in gs indicat e tha t exercise training in hemodialysis patients can increase exercise tolerance by 25%. Similar increases are observed after correction of anemia with EPO, although the increase in exercise capacity is small. Over-correction of hemoglobin (Hb) (> 13.0 g/dL) with higher doses of EPO is shown to increase morbidity and mortality. Hemoglobin normalization is not shown to have a beneficial effect on left ventricular mass and volume. Thus, close monitoring of patients with anemia secondary to chronic kidney disease is recommended, especially if they have concomitant cardiac disease. Exercise training helps counteract some of the negative side-effects of antirejection therapy with glucocorticoids including skeletal muscle atrophy, excessive weight gain, and fatigue. Coenzyme Q10 supplementation has been widely used as a complementary therapy to treat aging, stroke, neuromuscular diseases, Parkinson disease, Alzheimer disease, progressive supranuclear palsy, autosomal recessive cerebellar ataxias, amyotrophic lateral sclerosis and Huntington disease, but not decreased exercise tolerance. The role of vitamin C supplementation is currently being studied in the mobilization of iron stores in patients on hemodialysis. It has not been studied with respect to increasing exercise tolerance in this patient population.

"17 A 23

old woman with C7 ASIA B tetraplegia resulting from an accident 8 months ago is complaining of nausea for several days and has vomited non-bloody, n on-bi lio us food particles the last 3 evenings when placed back to bed after dinner. She also reports some abdominal tightness and bloating. Her symptoms are relieved when lying on the left side. Her bowel training program is going well, resulting in regular, effective bowel movements. She recently lost 25 pounds and appears quite thin on exam. Which study will confirm this patient's most likely diagnosis? a) Abdominal x-ray a. b) Head computed tomography (CT) scan b. c) Serum calcium level c. d) Upper gastrointestinal (GI) series" 17 Option d is correct. Commentary: Superior mesenteric artery (SMA) syndrome is a condition in which the third segment of the duodenum is compressed between the SMA and the aor ta. A lth ough it occurs rarely, it is more common in people with tetraplegia, especially if the person lost weight and is immobilized in the supine position. An upper GI series confirms the diagnosis with an abrupt cessation of barium in the third part of the duodenum. In addition to lying on the left side, some individuals get relief with metoclopramide (Reglan). A serum calcium level could be used to diagnose immobilization hypercalcemia, which is a common cause of nausea and vomiting in patients with tetraplegia. Hypercalcemia is not, however, alleviated with positioning and it usually occurs within the first few months after injury. Abdominal x-ray could identify chronic constipation, but since her bowel program is going well, constipation is not likely to be the cause of her symptoms. Although hydrocephalus would be identified by means of a head CT scan, it is the least likely diagnosis in this case. Reference: Kirshblum S. Rehabilitation of spinal cord injury. In: DeLisa JA, Gans BM, Walsh NE, editors. Physical medicine and rehabilitation: principles and pra ctice. 4th ed. Philadelphia: Lippincott-Raven; 2005. p 1729.

"99. Juvenile rheumatoid arthritis (JRA) differs from adult onset rheumatoid arthrit is: in JR A:(a) joint destruction occurs earlier.:(b) large joint involvement is less frequent.:(c) the cervical spine is involved less frequently. (d)

systemic features are more comm on ." 99 (d) Children with juvenile rheumatoid arthritis are more likely to have systemic features, have large joints involved, and have cervical spine involvement. Adu lts with rhe umato id arthriti s have joi nt des truc tion ea rlier.

178. Which of the following is an axonal peripheral neuropathy?:(a) Hereditary motor and sensory neuropathy type I:(b) Diphtheria neuropathy:(c) Metachromatic leukodystrophy:(d) Vincristine neuropathy

"178. (d) Vincristine can cause an axonal peripheral neuropathy. The others listed are demyelinating peripheral neuropathies."

"140. The primary advantage of mag wheels over spoked whe els in th e pe rformance of a wheelchair is

(a) lighter weight. (b) reduced maintenance. (c) more maneuverability. (d) general preference by active wheelchair users." 140. (b) Although MAG wheels require minimum maintenance and wear well, spoked wheels are substantially lighter, more responsive, and are generally preferred by active wheelchair users.

105. In regards to quality of life after spinal cord injury, which f actor has the LEAS T cor rel ation with life satisfaction? (a) Level of injury (paraplegia versus tetraplegia) (b) Access to leisure activities (c) Marriage (d) Number of hospitalizations

105. (a) The highest levels of life satisfaction for individuals with spinal cord injuries have been correlated with employment, education, number of hospitalizations, marriage, time since injury, leisure activities, social support, and adequate finances. Little correlation has been found between level of injury and life satisfaction.

137. During an independent medical examination, maximum medical improvement is defined by (a) the original symptoms having resolved. (b) expected improvement on functional gains having occurred. (c) the patient having returned to employment. (d) a period of 12 months having elapsed since the injury.

137 (b) When expected improvement on functional gains has occurred, a patient has reached maximum medical improvement.

146. Noninvasive nocturnal assisted ventilation in patients with neuromuscular disease has beenassociated with (a) increased polycythemia. (b) accelerated pulmonary hypertension. (c) decreased daytime PaCO2. (d) increased daytime fatigue.

146 (c) Assisted ventilation should be considered in pulmonary involvement from neuromuscular disease. Alternative measures such as low-flow oxygen may exacerbate hypercapnia.

147. Lumbar intradiscal pressure is lowest when a person (a) stands erect. (b) sits. (c) stands with hips flexed. (d) lies prone.

147 (d) Intradiscal pressure is lowest when a person lies prone and is much higher when standing, running, sitting, or bending.

15 Activating and strengthening which muscles may help solve iliotibial band syndrome problems? a. Hip adductors b. Quadriceps c. Tensor fascia lata d. Gluteus maximus

15 Reference(s) Hansen PA, Wilick SE. Musculoskeletal disorders of the lower lim In: Braddom RL. Physical medicine and rehabilitation, 4th e Philadelphia: Else vier; 2011; p 851-2. Option d is correct.

161. Which route of epidural steroid administr ati on is most likely to deliver steroid to the junction of the posterior disc and anterior dura? (a) Transforaminal (b) Caudal with catheter (c) Interlaminar (d) Caudal

161. (a) The subpedicular transforaminal route of epidural steroid delivery places the needle at the anterior portion of the intervertebral foramen. The retroneural route of delivery purposefully terminates needle placement at the posterior edge of the intervertebral foramen to avoid injuring radicular vasculature. The caudal and interlaminar approaches are of limited utility in delivering steroid anteriorly due to raphe within the epidural space. Ref: Irwin RW, Zuhosky JP, Sullivan WJ, et al. Interventional procedures for work-related lumbar spine conditions. Arch Phys Med Rehabil 2007;88(Suppl):S22-3.

38 In an ambulatory individual with Duchenne muscular dystr ophy, the most c ommon ly fou nd c ont rac tur es are in str uct ur es t hat aff ec t a. ankle plantar flexion. b. knee flexion. c. hip flexion. d. elbow flexion.

38 Option a is correct. In Duchenne muscular dystrophy, ankle contractures (usua lly less than 15 degr ees ) d ev el op eve n w hil e th e subj ect s are amb ulat or y. The k nee, hip and elb ow co ntr act ure s d evel op once the individual has transitioned to a wheelchair.

63. The arthropathy in persons with systemic l upu s erythematosus (SLE) generally is in the wrists, knees and small joints of the hands. It is also (a) symmetric and non-erosive. (b) symmetric and erosive. (c) asymmetric and non-erosive. (d) asymmetric and erosive.

63. (a) The arthritis in SLE is symmetric and non-erosive. It is also generally non-deforming and reducible due to its involvement of the para-articular tissues. Ref: Buyon JP. Systemic lupus erythematosus. Epidemiology, pathology, and pathogenesis. In: Klippel JH, editor. Primer on rheumatic disease. 12th ed. Atlanta (GA): Arthritis Foundation; 2001. p 337-8

"19.

Which antispasticity drug used to treat a 3-year-old child with cerebral palsy binds to GABA receptors in the spinal cord to inhibit reflexes that lead to increased tone? (a) Clonidine (b) Tizanidine (c) Dantrolene (d) Baclofen " 19 Answer: (d) Commentary: Baclofen binds to GABA receptors in the spinal cord to inhibit the reflexes that lead to increased tone. Clonidine is an alpha 2 agonist, as is tizanidine. Dantrolene works in the striated muscle at the level of the sarcoplasmic reticulum. All these drugs have pediatric application. Baclofen can be used beginning at age 2 years. Reference: McMahon M, Pruitt D, Vargus-Adams J. Cerebral palsy. In: Alexander MA, Matthews DJ, editors. Pediatric rehabilitation: principles and practice. 4th ed. New York: Demos; 2010. p 180 (Table 8.2).

178. Professionalism is the basis of medicine's contract with soc iety . Which item is a fundamental principle of medical professionalism? (a) Social justice (b) Physician paternalism (c) Patient disclosure (d) Free enterprise

178. (a) According to the Charter on Medical Professionalism, there are 3 fundamental principles of medical professionalism. They are (1) the primacy of patient welfare, (2) patient autonomy, and (3) social justice.

196. Which treatment has NOT been shown to improve epico ndylitis? (a) Low intensity laser irradiation (b) Wrist extension strengthening exercises (c) Acupuncture (d) Extracorporeal shock-wave therapy

196. (a) Wrist strengthening, acupuncture, and shock wave therapy all help in the treatment of epicondylitis. However, low intensity laser treatment is not proven beneficial.

33. A 35-year-old man with history of psoriatic arthritis complains of localiz ed low b ack pain insi dio us in o nset . The p ain i s worse in t he mo rni ng and i mpr ove s a s the day progresses. What is the most likely cause of his back pain? (a) Piriformis strain (b) Sacroiliitis (c) Quadratus lumborum strain (d) Discitis

33 (b) Sacroiliitis occurs in patients with spondyloarthropathies s uch as psoriatic arthritis, reactive arthritis, enteropathic arthritis, and ankylosing spon dylitis.

38. Where do you place the cathode when performing a median motor nerve conduction stud y, st imu lat ing a t th e e lbow?:(a) Lateral to the biceps tendon:(b) Medial to the brachial artery:(c) Lateral to brachioradialis:(d) Medial to brachioradialis

38 (b) The median nerve is just medial to the brachial artery at the elbow.

4 Of the following modalities, which is the most effective in treating pha ntom lim b p ai n? a. Iontophoresis b. Transcutaneous electrical nerve stimulation c. Short wave diathermy d. Paraffin baths

4 Option b is correct. Of the options listed, transcutaneous electrical nerve s timulation (TENS ) is the mo da li ty tha t m ay be u seful in tr eati ng phan to m limb p ain. Ion topho res is is ge ner all y u sed for dispersion of medications. Short wave diathermy is a method of deep heat. Paraffin bath is a superficial heat modality.

40 What is the overall leading cause of death for individuals with paraplegia? a) Pulmonary embolism a. b) Suicide b. c) Septicemia c. d) Heart disease

40 Option d is correct. Commentary: In paraplegia, the overall leading cause of death is heart disease, followed by septicemia and then suicide. In tetraplegia, pneumonia is the lead ing cause of death. Reference: DeVivo MJ. Epidemiology of traumatic spinal cord injury. In: Kirshblum S, Campagnola DI, DeLisa JA, editors. Spinal cord medicine. Philadelphi a: Li ppi ncott Williams & Wilkins; 2002. p 78-9.

42. The criterion scale used to describe severity of brain injury is the:(a) Disability Rating Scale.:(b) Agitated Behavior Scale.:(c) FIM™ instrument.:(d) Glasgow Coma Scale.

42 (d) The criterion to describe the severity of a traumatic brain injury is the Glascow Coma Scale (GCS). GCS score of 13-15 = mild GCS score of 9-12 = moderate GCS score of 3-8 = severe

43. A 28-year-old woman, who is 35 weeks pregnant, complains of right thigh an d groin pain wit h wei ght b earin g. Y ou di ag nose he r wit h idi op ath ic tr ansien t o ste opo ro sis of the femoral neck. What is the course of treatment? (a) Recommend labor induction (b) Prescribe protected weight bearing (c) Recommend bedrest until delivery (d) Prescribe alendronate (Fosamax)

43 (b) Patients with idiopathic transient osteoporosis of the femor al neck may ambulate as tolerated but may need protective weight bearing for pain relief. S ymptoms an d patho logy resolve within 6 months.

72. Which is the best predictor of discharge from a rehabilitation center to home f or a m an who ha s had a s tro ke?:(a) Lesion location:(b) Shoulder pain:(c) Ambulatory status:(d) Bladder incontinence

72 (d) Of the choices presented above, the most consistent predictor of good outcome and discharge home is bladder continence. Probably the strongest overall p red icto r of abi lity is admissio n function al abi lity (wh ich reflects se verity of str oke) .

8. Which radial innervated muscle is innervated by the C5 root?:(a) Anconeus:(b) Extensor carpi radialis longus:(c) Brachioradialis:(d) Triceps

8 (c) None of the other muscles listed receives C5 innervation. The supinator is the only other radial innervated muscle that has C5 innervation.

92. You are asked to evaluat e a 25-ye ar- old ma n who sustained a traumatic brain injury (TBI) in a car crash. In the emergency department, he was unable to follow commands but withdrew to pain, opened his eyes when spoken to, and was disoriented. Head computed tomography revealed a frontal contusion. According to the Glasgow Coma scale, his injury can be classified as(a) uncomplicated mild TBI.(b) complicated mild TBI.(c) moderate TBI.(d) severe TBI.

92. (c) Based on the information presented in the question, the Glasgow Coma Scale score for this patient is 11 (eyes open when spoken to 3, withdraws to pain 4, converses but is disoriented 4 = total score 11). Moderate injury is defined by GCS scores of 9 to 12.

"113.

Which factor is associated with the highest risk for postoperative disloc ation af ter a to tal h ip ar throp last y? (a) Femoral anteversion (b) Anterior approach for hip replacement (c) Long femoral component (d) Previous hip replacement" 113 (d) Patients with history of previous hip replacement and poster ior approach are at greater risk for postoperative dislocation. Posterior surgical approach has a hig her ris k of dislocation than anterior approach.

"94.

A 16-year-old with Duchenne muscular dystrophy presents to your office w ith a 3-m onth his tory of worsening shortness of breath and pressure-like chest pain. His pulmonary function testing has not changed significantly. The most likely cause for his complaints is (a) decreased cardiac output. (b) bacterial pneumonia. (c) pulmonary embolus. (d) aspiration pneumonia." 94 (a) Given that his pulmonary function has not changed, the most likely cause for his shortness of breath is cardiac decompensation due to cardiomyopathy. Respiratory failure due to neuromuscular weakness would typically correspond to worsening of pulmonary function testing. Bacterial and aspiration pneumonia would both present with much more acute symptoms. Boys and men with Duchenne muscular dystrophy rarely get deep venous thromboses or pulmonary emboli. If left untreated, the patient will likely develop congestive heart failure from cardiomyopathy. The decreased cardiac output usually responds to treatment with digoxin and ACE-inhibitors.

"21.

An 18-year-old, right-handed hockey player presents to you a fter experiencing 3 right shoulder anterior dislocations in the prior season after falls on ice. Magnetic resonance imaging shows supraspinatus tendonitis but no other lesions or tears. After 6 sessions of physical therapy, he is pain free. He has been invited to play professionally in 6 months. What is your next recommendation? (a) Tell him that he will likely dislocate again and tha t he shou ld r elocate the shoulder by forcefully pushing the anterior shoulder against a wall. (b) Refer him to a surgeon to consider shoulder stabiliz ation sur gery . (c) Tell him he should not return to any sports because of his in crea sed chance of dislocating again. (d) Stress the importance of compliance with his home ex ercise pr ogra m." 21 (b) Recurrent dislocations should be treated with surgery at some point if the athlete would like to return to contact sports. Various anterior shoulder dislocation techniques that can be applied to reduce the shoulder, most by external rotation of the shoulder or by using gravity.

"53.

In patients with osteoporosis, which treatment reduces incidence of verteb ral comp ression fract ure ? (a) Weight reduction (b) Cash brace (c) Strengthening of spinal extensors (d) Strengthening quadriceps" 53 (c) Weak extensor muscles increase risk of compression fracture. Risk increases with immobilization longer than 2 days. There is no association between ost eoporotic compres sion fractures and weight or family history.

106. In males, the energy cost of sexual activity with a married partner typically does not exceed howmany METs? (a) 2 (b) 5 (c) 10 (d) 15

106 (b) In males, the energy cost of sexual activity with a married partner does not typically exceed 5 METs and is lowest in familiar positions.

"144. Regarding Bell's palsy, which statement is TRUE?:(a) It usually develops over days.:(b) It can recur in a small percentage of patients.:(c) Women are more commonly affected than men. (d)

Bilateral cases frequently occur ." 144 (b) Most patients with Bell's palsy usually present with sudden unilateral facial weakness. Persons of any age can be affected. Men and women are affected e qua lly and i n a bout 2%-9% of pa ti ents the co ndi tion can be recurrent. Very rarely i t af fects bot h the fa cia l nerves.

"36.

The circulatory system's response to exercise is characterized by (a) parasympathetically mediated vasoconstriction of the skin. (b) vasodilatation in active muscle groups mediated by local factors. (c) sympathetically mediated vasodilatation of viscera. al perip heral va scula r r es istan ce. (d) increase in total peripheral vascular resistance" 36 (b) During vigorous exercise, sympathetically mediated vasoconst riction occurs in the skin and viscera. Vasodilatation in active muscle groups is mediated by local f actors including potassium ion concentrations, increases in osmolarity, ch an ges in adenosin e nucle otid e conc ent rations , and decreasing pH . M us cle bl ood fl ow may in crea se u p to 15 -2 0 ti mes ba sel ine. Th e va so di lata tio n i n mus cle g rou ps ca uses a reduc tion in total peripheral resistance by up to 50%.

"83.

Which steroid compound has the longest half-life? (a) Dexamethasone (b) Prednisone (c) Triamcinolone (d) Hydrocortisone" 83 (a) The least water-soluble compound will have the longest half- life.

"146.

Which technique best minimizes resistance to stret ch at tri buted to spinal reflex pathways? (a) Static stretching (b) Proprioceptive neuromuscular facilitation (c) Active isolated stretching (d) Ballistic exercises" 146 Answer: (b) Commentary: Proprioceptive neuromuscular facilitation (P NF) h as the advantage of minimizing resistance to stretch attributed to spinal reflex pathways. Static stretching has low energy cost, low risk of injury, with less chance of producing residual muscle soreness. High energy cost to perform complicated techniques is a disadvantage of active isolated stretching. Ballistic stretching is effective for patients requiring high levels of dynamic flexibility. Reference: Gailey RS, Raya MA. Manual Modalities. In: Go nza le z E G, Myers SJ, Edelstein JE, Lieberman JS, Downey JA, editors. Downey and Darling's Physiological basis of rehabilitation medicine, 3rd ed. Boston: Butterworth Heinemann; 200.; p 767-9.

173. Which is a site where ultrasound should be used with caution? (a) Lumbar laminectomy (b) Osteoporotic hip fracture (c) Lateral epicondyle (d) Patellar bursa

173. (a) Ultrasound should not be used near a pacemaker, near a spinal laminectomy site, near brain, eyes, or reproductive organs, near a malignancy, in areas of skeletal immaturity, or ear arthroplasties, especially in persons using methyl methacrylate.

182. A patient presents with right hemiparesis an d dysarthria but language and sensation are intact. The lesion is most likely in the (a) posterior limb of the internal capsule. (b) left frontoparietal lobe. (c) lateral pons. (d) thalamus.

182. (a) A pure motor stroke (hemiplegia and dysarthria without sensory deficits) is caused by a lesion in the posterior limb of the internal capsule. Ref: Roth E, Harvey R. Rehabilitation of stroke syndromes. In: Braddom RL, Buschbacher RM, editors. Physical medicine and rehabilitation. 2nd ed. Philadelphia: WB Saunders; 2000. p 1131.

19What is the most frequent pulmonary complication of polymyositis and dermatomyositis?a) Aspiration pneumoniab) Interstitial lung diseasec) Pulmonary arterial hypertensiond) Spontaneous pneumothorax

19 Reference(s) Kang E, Song Y. Pulmonary manifestations of inflammatory myopathies. In: Kagen L, editor. The inflammatory myopathies. New York: Humana Press;2 009. p 207-21.

18. Which rehabilitation intervention/consideration does NOT apply to rehabili tation goals f or a pers on with a lung tra nsplant? (a) Addressing psychologic issues of isolation, depression,and anxiety (b) Providing pulmonary toilet and chest physical therapy (c) Educating regarding immunosuppression and rejection(d) Enforcing posttrans plantation mob ilization precauti ons Ref: (a) Palmer SM, Tapson VF. Pulmonary rehabilitation in the surgical pati ent. Lung tran splantati on and lu ng vo lume reduction surgery. Respir Care Clin North Am 1998;4:71-83. (b) Downs AM. Physical therapy in lung transplantation. Phys Ther 1996;76:626-42. Educational Activity 2.10

18 (d) Immobility should be avoided posttransplantation. Peritransplant rehabilitation should start in th e intensive care u nit on day 1 and progress to ambulation as soo n as po ssible. Rehabil it ati on is si mil ar to that used for chronic obstructive pulmonary disease, with focus on strengthening, conditioning, education, pulmonary toilet, and medication and oxygen use.

190. A 60-year-old woman is being discharged to home from your rehabilitation unit fol lowing a righ t m id dle c ereb ral a rt ery i sc hemic stro ke wi th re sultan t l eft he mi paresis. She is ambulatory for short household distances with the use of a hemiwalker on the right. Recommendations for a manual wheelchair for this patient would include (a) hemi-height, with removable swing-away leg rests. (b) hemi-height, with nonremovable swing-away leg rests. (c) standard-height, with removable swing-away leg rests. (d) standard-height, with nonremovable swing-away leg rests.

190 (a) Wheelchair recommendations for a person with hemiparesis and limited ambulatory abilities would include a hemi-height chair which is lower to the groun d than to a stand ard height wheelchair. This configuration provides the patient grea te r adva ntage to propel the wheelc hai r with the unaffected l owe r e xt rem ity . It i s also im port ant fo r t he l eg r est s t o b e rem ov able s o that th ey do no t i nt erf ere w ith the pati ent's ability to propel the wheelchair.

190. What is the minimal number of points of contact that an orthosis must have in order to exert rotational control? (a) One (b) Two (c) Three (d) Four

190 Answer: C Commentary: Rotational control forces or moments across a joint are not effective unless there are at least 3 points of contact between the device and the limb segment. Ref: Shur DJ, Michael JW. Prosthetics and orthotics. 2nd ed. Harrisonburg: Pearson Education; 2002. p 34.

24. Pain associated with bone metastasis is associated with (a) bone marrow in filtration. (b) chronic myositis. (c) bone infarction. (d) involvement of the periosteum. Ref: Posner JB. Neurologic complications of cancer. Philadelphia: FA Davis; 1995. p 111-42 . Educational Activity 3.6

24 (d) Bone marrow is insensitive to pain, and metastasis confined to the marrow is ra rely painful. Pa inf ul metastat ic lesi ons usually invol ve the periosteum, paraverteb ral sof t tissue s, or n er ve roots be for e they become symptomatic.

3. Trials on the use of gluc osam ine a nd chondr oitin for knee and hip osteoarthritis have shown that these compounds (a) reduced subchondral scle rosi s, as ev idence d by x-ray. (b) decreased proteoglycan synt hesis in artic ular cartilage. (c) had a moderate effect on pai n sym pto ms. (d) had an immediate effect on s ympto m s everit y.

3. (c) A meta-analysis of randomized controlled studies on the treatment of knee and hip osteoarthritis with glucosamine and chondroitin found moderate effects on symptoms. These effects take a minimum of 4 weeks. Glucosamine and chondroitin are capable of increasing proteoglycan synthesis in articular cartilage.

50 Which statement is TRUE of the lower trapezius? a) It is innervated by the thoracodorsal nerve. b) It is innervated by the long thoracic nerve. b. c) Contraction of this muscle results in upward rotation of the scapula. c. d) Contraction of this muscle results in abduction of the scapula.

50 Option c is correct.Commentary: The lower trapezius, as well as the upper trapezius and the middle trapezius, is innervated by the spinal accessory nerve (11t h c ranial nerve) and possibly contributions from the ventral rami of C2, C3, and C4. Contraction of the lower trapezius results in scapular depression, adduction, and upward rotation. Reference: Killen SH, Miller JT. Shoulder and arm and upper back. In: Cutter NC, Kevorkian CG, editors. Handbook of manual muscle testing. NewYork: McGra w-Hil l; 1999. p 20-21.

50. Which of the following d isti nguis hes runni ng from walking?(a) Shorter stance phase in walk ing(b) No double support in run ning(c) Longer step length in wa lkin g(d) Shorter stride length in run ning

50. (b) Sixty percent of the complete gait cycle is spent in stance phase while walking. Only 40% of the time is spent in stance phase during running. By definition, running involves little to no heel strike and has no double support. Stride length and step length are much greater in running than in walking.

70. What level of amputation has the highest acceptance rate for an upper extremity prosthesis? (a) Wrist disarticulation (b) Transradial (c) Elbow disarticulation (d) Transhumeral

70 (b) Overall rejection of prosthetic usage occurs in 33%-38% of unilateral upper extremity amputees. The highest acceptance rate is transradial at about 93%, and the lowest is wrist disarticulation at about 6%.

"

70. Double limb stance is what percent of the entire g ait cycle ? (a) 5% (b) 10% (c) 20% (d) 30%" 70 (c) The average double limb support is 20% and single limb support is 40% of the entire gait cycle. Stance phase accounts for 60% of the gait cycle and swing phase accounts for 40%.

71. Which structure is NOT part of the pain pathway for rapid transm ission of immediat e pai n? (a) Rexus lamina 2,3 (b) Lissauer's tract (c) C fibers (d) Periaqueductal gray matter

71. (c) C fibers are unmyelinated fibers that modulate slow pain ("second pain") rather than acute pain. "First pain" is mediated by A-delta nociceptors and is brief and localized.

"63.

A 60-year-old woman with diabetes and hypertension complains of right fo ot swelli ng an d m ild discomfort on ambulation. There is no history of trauma. Examination demonstrates decreased sensation in the foot and a plantar ulcer. There is no sinus tarsi tenderness and no ankle instability. What is the most likely diagnosis? (a) Charcot's joint (b) Lateral ankle sprain (c) Tarsal tunnel syndrome (d) Plantar fasciitis" 63 (a) Charcot's joints often present as painless swelling in patients with diabetes. Other illnesses associated with Charcot's joint include tabes dorsalis and syringomyelia.

"83. Which of the following is the most important lifesty le modifi cati on for prevention of osteoporosis? (a) Avoiding cigarette smoking and high intake of caffei ne (b) Decreasing the intake of alcohol (c) Minimizing the use of nonsteroidal anti-inflammatory medicati ons (d)

Eating a diet high in protein and phosphorus" 83. (a) Factors that impact bone mineral density negatively are smoking and high intake of caffeine, protein, and phosphorus. An active lifestyle with regular weight-bearing exercise is advised. Eliminating fall hazards such as throw rugs throughout the home is also essential.

"71.

The third occipital nerve innervates which structure? (a) C2-3 zygapophysial joint (b) C2-3 intervertebral disc (c) C3-4 zygapophysial joint (d) C3-4 intervertebral disc " 71 Answer:(a) Commentary: The third occipital nerve(TON) innervates the C2-3 zygapophysial joint. The C3-4 zyagpophysial joint is innervated by the C3 and C4 medial branches. Innervation to the cervical discs involves the sinuvertebral nerve, vertebral nerve and sympathetic trunk. Reference: Bogduk N. Practical guidelines: Spinal diagnostic and treatment procedures. San Francisco: International Spine Intervention Society; 2004. p 126.

10. Which knee component is preferred in the prosthetic prescription for an 80 year-ol d debili tated , d ys vascu lar, diab et ic tr an sfemo ral a mp ute e? (a) Single axis (b) Polycentric (c) Pneumatic (d) Manual locking

10 (d) A manual-locking knee is indicated for new unstable amputees and those who need utmost stability because of muscular weakness or poor coordination. The other com ponents are generally used in persons with less risk of falling.

100. What is the arrow pointing to in this upper extremity prosthesi s? (a) An excursion cable (b) An anterior split cable (c) The elbow-lock control cable (d) The elbow flexion cable

100. (c) This is an elbow-lock control cable. Its proximal end originates at the anterior suspension strap and its distal end engages the elbow-locking mechanism. The principal of the elbow-lock mechanism is pull-and-release to lock, pull-and-release to unlock.

110. Which of the following knee types provides good stability in early stance phase and ease of flexionwhile weight bearing during the pre-swing phase (terminal stance) of the gait cycle? (a) Single axis (b) Stance phase control (c) Polycentric (d) Manual locking

110 (c) Many polycentric knees are designed so that the center of rotation moves anteriorly very rapidly during the first few degrees of knee flexion, quickly passing in front of the floor reaction line and facilitating the swing phase. Because the polycentric knee can be flexed under weight bearing during the terminal stance, when properly dynamically aligned it can offer both excellent stance stability and ease of swing-phase flexion. Furthermore, all polycentric knees shorten mechanically to a slight degree during flexion, adding additional toe clearance during midswing.

16 A negative prognostic sign in early multiple sclerosis is young age at onset. a. cerebellar signs. b. optic neuritis. c. monosymptomatic present ation .

16 Option b is correct. Commentary: Early multiple sclerosis shows a more favorable outcome if presentation is at age less than 35 years, monosymptomatic, with optic signs, and of su dde n onset with long remission. Reference: Kraft G, Brown T, Johnson S. Multiple sclerosis. In: Braddom RL, editor. Physical medicine and rehabilitation. 4th ed. Philadelphia: Elsevier- Saund ers ; 2011. p 1238.

169. In which type of cerebral palsy is a seizure disorder most commonly seen?:(a) Tetraplegia:(b) Diplegia:(c) Athetosis:(d) Hemiplegia

169 (d) Approximately 70% of children with hemiplegia have seizures. About 50% of children with tetraplegic cerebral palsy have seizures. Seizures are rare in c hil dren with di plegi a or atheto si s.

18 What is the most frequent pulmonary complication of polymyositis and dermatomyositis? a) Aspiration pneumonia b) Interstitial lung disease c) Pulmonary arterial hypertension d) Spontaneous pneumothorax

18 Reference(s) Kang E, Song Y. Pulmonary manifestations of inflammatory myopathies. In: Kagen L, editor. The inflammatory myopathies. New York: Humana Press;2 009. p 207-21. Option b is correct. Interstitial lung disease occurs in up to 65% of patients with myositis and is a significant contributor to morbidity and mortality. Patients with myositis have further pulmonary complications wit h respect to mu scle weakn ess a nd vascular disease resulting in secondary complications of aspiration pneumonia, infection, respiratory muscle failure, pulmonary edema and pulmonary arterial hypertension. Spontaneous pneumothorax is not commonly associated with myositis.

2. What is the most frequent presenting symptom of brain me ta sta sis? (a) Focal weakness (b) Headache (c) Seizure (d) Visual disturbance

2 Answer: (b) Commentary: Presenting symptoms at the time of diagnosis wi th br ain metastasis, in order of decreasing frequency, are as follows: (patients can have more than a single symptom): headache, 49%; mental disturbance, 32%; focal weakness, 30 %; gait ataxia, 21 %; seizures, 18%; speech difficulty, 12%, visual disturbance, 6%; sensory disturbance, 6%; and limb ataxia, 6%. Reference: Chevelle A. Cancer Rehabilitation. In: Braddo m R L, ed itor. Physical medicine and rehabilitation. 3rd ed. Philadelphia: Elsevier; 2007. p 1376.

22 More than ninety percent of lumbar disc herniations involve which 2 levels? a) L4-L5 and L5-S1 b) L3-L4 and L4-L5 c) L3-L4 and L5-S1 d) L2-L3 and L5-S1

22 Reference(s) (a) Panagos A, Sable AW, Zuhosky JP, Irwin RW, Sullivan WJ, Foye PM. Industrial medicine and acute musculoskeletal rehabilitation. 1. Diagnosti c testing in industrial and acute mu scul oskeletal inj ur ies. Arch Phys Med Rehabil 2007;88(3 Suppl 1):S7. (b) Barr KP, Harrast M Low Back Pain. In: Braddom RL, editor. Physical medicine and rehabilitation. 4th e Philadelphia: Saunders Elsevier; 2011. p895. Option a is correct. Disc herniations at L4-L5 and L5-S1 comprise 98% of all lumbar disc herniations. If radicular symptoms are present then the L5 and S1 nerve roots are most l ikely involve

23. How much knee flexion is required to descend stairs step over step after a total k nee repl aceme nt? (a) 45 degrees (b) 70 degrees c. 90 degrees (d) 110 degrees

23 (d) Descending stairs requires 110° knee flexion.

"

3. The most common clinical manifestation of Ly me disease is (a) monoarticular or oligoarticular arthritis. (b) facial-nerve palsy. (c) atrioventricular block. (d) erythema migrans." 3 (d) Erythema migrans (EM) is a skin lesion that is erythematous, and may be round or oval, flat or raised, and possibly have central clearing. Of persons with untreated EM, sixty percent will have monoarticular or oligoarticular arthritis. Ten percent will have a neurologic presentation, such as facial-nerve palsy. Approximately 5 percent will have a cardiac manifestation such as atrioventricular block. Ref: Wormser GP. Early Lyme disease. N Engl J Med 2006;354:2794-801.

43 What finding is a relative contraindication to therapeutic ultrasound? a) Acute inflammation b) Pain c) Malignancy d) Contracture

43 Reference(s) Weber D, Brown A, Physical agent modalities. In: Braddom RL, editor. Physical medicine and rehabilitation. 2nd e Philadelphia: Elsevier-Saunder s; 2011. p 440-58. Option c is correct. Therapeutic ultrasound is commonly used to decrease pain and muscle soreness through thermal effects. Malignancy is a relative contraindication secondary to concerns of increased tumor growth or h yperemia and he matogenous spre a Relative contraindications also include use near the brain, eyes, or reproductive organs, as well as near pacemakers and laminectomy sites.

43. The most common cause of dis abili ty in the United States is(a) arthritis.(b) carpal tunnel syndrome.(c) coronary artery disease.(d) stroke.

43. (a) Arthritis and other rheumatic conditions are the leading cause of disability in the United States, imparting an aggregate cost of about 1.1% of the gross national product.

46. Which adjuvant analgesic medication acts at ion channels to stab ilize neu ronal cel l mem bra nes? (a) Gabapentin (Neurontin) (b) Amitriptyline (Elavil) (c) Clonidine (Catapres) (d) Baclofen (Lioresal) 49. What percentage of American children with myelomeningocele requi res a shu nt to man age h ydr ocephalus?

46. (a) Gabapentin's therapeutic effects are mediated by binding to a calcium ion channel located on neuronal cell membranes. It may also have GABA agonistic properties, but this remain equivocal.

52. (This question has been eliminated from the exam, therefore, it was not scored. On a pharmacologic basis, which agent used to decrease gastric acid secretion is mo st a pp rop ria te after bra in injur y?:(a) Ranitidine:(b) Famotidine:(c) Omeprazole:(d) Sucralfate

52 (c) (This question has been eliminated from the exam, therefore, it was not scored.) Choices a and b are both histamine type-2 (H2) blockers that are cognitively impairing.

53 A 65-year-old woman with diabetes who will be undergoing an elective transtibial amputation due to a nonhealing wound consults you regarding her upcoming su rgery. Which postoperative dressing woul d you recomme nd to promot e wou nd healing and prevent postoperative complications? a) Elastic bandages b) Nonadhesive gauze c) Stump shrinker compressive bandage d) Removable or nonremovable postoperative cast

53 Reference(s) Huang ME, Miller LA, Lipschutz R, Kuiken T Rehabilitation and prosthetic restoration in lower limb amputation. In: Braddom RL, editor. Physical medicine and rehabilitation. 4th e Philadelphia: Elsevier Saunders; 2011. p 280. Option d is correct. The major tenets of postoperative wound management following transtibial amputation are edema control, wound protection and prevention of knee flexion contr acture. This is best done with a rig id d ressing. Elas ti c bandages and stump shrinkers can help control edema but may be applied inappropriately. They also do not protect the wound from trauma or prevent knee flexion contracture.

6. In order to minimize myocardial oxygen requirements in a patient with known coronar y artery dise ase d uring low inte ns ity r es istiv e exe rc ise it i s best to pr esc ri be (a) isometric supine exercises. (b) isometric standing exercises. (c) isotonic supine exercises. (d) isotonic standing exercises.

6 (d) With low intensity exercise the rate pressure product (RPP), a valid surrogate measure for myocardial oxygen consumption, is higher for supine than sta nding acti vities. At higher exercise intensities the situation is reversed and the R PP is hi gher for standin g ac tiviti es. Isomet ric contractions gr eat er th an 0.15 of m aximum caus e co nt inu ed i ncre ase in th e RPP u ntil f at igue li mit s the du ra tio n of contraction. For patients at risk for cardiac ischemia, low intensity, isotonic, standing exercises should be prescribed to avoid elevation of the RPP.

6. A 65-year-old brain tumor pat ient rec eiving inpatient rehabilitation develops nausea, fever, and headache several hours after radiation therapy. You prescribe (a) ceftriaxone. (b) dexamethasone. (c) nimodipine. (d) sumatriptan.

6. (b) Radiation reactions may occur at any time during or after radiation therapy. Acute reactions that occur within hours after the first dose are caused by edema, and manifested by headache, nausea, vomiting, somnolence and fever. Worsening neurological symptoms may occur with dose fractions greater than 2 Gray. Symptoms are preventable through use of corticosteroids, eg, dexamethasone 2 mg daily or twice daily.

72. Which type of traumatic brain injury resul ts in the most morbidity? (a) Focal cerebral contusion (b) Subarachnoid hemorrhage (c) Epidural hematoma (d) Diffuse axonal injury

72. (d) After a traumatic brain injury, diffuse axonal injury (DAI) is the leading cause of morbidity, this morbidity includes impairments in cognition, behavior, and arousal. Ref: Gennarelli TA. The spectrum of traumatic axonal injury. Neuropathol Appl Neurobiol 1996;22:509-13.

73. In addition to routine weight-bearing exer cis es and calcium supplements, vitamin D is important in persons with osteoporosis because it (a) decreases the amount of calcium supplement ati on needed. (b) enhances muscle strength and reduces the r isk o f falling. (c) decreases bone turnover. (d) improves the mechanism of action of biphos pho na tes.

73. (b) Vitamin D is essential for skeletal maintenance and has been shown to enhance muscle strength and reduce the risk of falling. Ref: Rosen CJ. Postmenopausal osteoporosis. N Engl J Med 2005;353:595-603.

75. What is the most common cause of autonomic dysreflex ia? a) Rectal distention b) Pressure ulcers c) Bladder distention d) Childbirth

75 Answer: (c) Commentary: In a spinal cord injury, the most common cau se of au tonomic dysreflexia is bladder distention. The other answers can also cause autonomic dysreflexia but due to the frequency of bladder distention and potential problems of catheter blockage or bladder distention it is more frequent than the other sources of painful stimulation listed. Reference: (a) Consortium for Spinal Cord Medicine. Acut e m an age ment of autonomic dysreflexia: individuals with spinal cord injury presenting to health-care facilities. 2nd ed. Clinical practice guidelines for healthcare professionals. Washington (DC): Paralyzed Veterans of America; 2001. p 14. (b) Mallory B. Autonomic dysfunction in spinal cord disease. In: Lin VW, editor. Spinal cord medicine principles and practice. New York: Demos; 2010. p 551.

79. Which positive effect of ankle-foot orthotics has been proven beneficial in the tre at men t o f c hildr en w ith cere bra l p alsy?:(a) Improved gait efficiency as measured by gait analysis:(b) Prevention of contractures:(c) Improved knee extensor strength:(d) Decreased plantar flexor posture

79 (a) There are no large, randomized, controlled studies that show the long-term effects of any type of Ankle-Foot Orthosis (AFO) on function or contracture f orm atio n. Sm all stud ies have sh ow n that b oth ri gid and hin ged AFOs imp rove gait eff icie ncy by pr eventing pl antar fle xio n.

92. Characteristic x-ray findings of new bone formation in heterotopic ossification include densities that are (a) noncircumscribed, extra-articular, and extracapsular. (b) noncircumscribed, extra-articular, and intracapsular. (c) circumscribed, extra-articular, and extracapsular. (d) circumscribed, intra-articular, and intracapsular.

92 (a) Typically, heterotopic ossification occurs in the more proximal joints. X-ray findings are a "popcorn" appearance of fluffy (noncircumscribed), immature bone, extracapsular and extraarticular.

"51.

A 35-year-old man with history of psoriat ic ar thritis complains of localized low back pain of insidious onset. The pain is worse in the morning and improves as the day progresses. What is the most likely cause of his back pain? (a) Piriformis strain (b) Sacroiliitis (c) Quadratus lumborum strain (d) Discitis" 51. (b) Spondylonegative spondylarthropathies, such as psoriatic arthritis, are often associated with sacroiliitis. The Gelling phenomenon, characterized by stiffness after prolonged immobility, occurs with many inflammatory arthropathies. Ref: Atkinson K. Psoriatic arthritis. In: Frontera W, Silver J, editors. Essentials of physical medicine and rehabilitation. Philadelphia: Hanley & Belfus; 2002. p 694-8.

"123.

A 40-year-old woman is currently hospitalized for a s ev ere flare of her polymyositis. On consultation, you recommend that while in the hospital she begin (a) passive range-of-motion exercises to prevent contrac tur es . (b) isometric strengthening exercises at the bedside to mai nt ain her strength. (c) ambulation with a walker in the hallways supervised by he r t herapist. (d) strengthening exercises with light hand and ankle weights." 123 Answer: (a) Commentary: Passive range of motion to maintain joint m ove me nt is recommended during periods of acute flares. With resolution of the flare, active-assisted exercises may be started, progressing to strengthening exercises and ambulation. Reference: Guzman J. Rehabilitation of patients with rh eum at ic diseases. In: Braddom RL, editor. Physical medicine and rehabilitation. 3rd ed. Philadelphia: Saunders; 2007. p 789.

"94.

Which myopathy can be e ithe r aut oso mal do minant or autosomal recessive?(a) Facioscapulohumeral dyst roph y(b) Limb-girdle dystrophy(c) Myotonic dystrophy(d) Emery-Dreifuss muscular dyst rophy" 94 (b) Facioscapulohumeral and myotonic dystrophy are usually autosomal recessive. Emery-Dreifuss muscular dystrophy is X-linked recessive. Limb-girdle dystrophy is a group of disorders producing weakness about the hips and shoulders. It can be either autosomal recessive or dominant.

"185. Five weeks after sustaining a T6 complete spinal co rd injury , yo ur patient is noted to have new urinary incontinence with intermittent catheterization volumes of less than 150cc. Work-up is negative for a urinary tract infection. You consider starting

(a) tamsulosin (Flomax). (b) tolterodine (Detrol). (c) terazosin (Hytrin). (d) bethanechol (Urecholine)." 185. (b) The patient is likely developing spontaneous detrusor contractions. You would consider using an anticholinergic agent to decrease detrusor (and hence bladder) pressures. Ideally, you would obtain urodynamic studies to ascertain bladder pressures and detrusor-sphincter coordination and would use these findings to guide treatment.

"166. Standing at ease is equivalent to how many metaboli c equival ents (METs)? (a) 1.5 to 2.0

(b) 2.5 to 3.0 (c) 4.0 to 4.5 (d) 5.0 to 5.5" 166. (a) Lying quietly is 1.0 MET. Light housework is 1.2-3.0 METs. Standing at ease is 1.4-2.0 METs. Walking at 3 miles per hour is equivalent to 4.3 METs.

92. Which Brunnstrom stage of motor recovery i n a s troke patient with a hemiplegic arm is characterized by activating muscles selectively outside the flexor and extensor synergies? (a) Stage 2 (b) Stage 3 (c) Stage 4 (d) Stage 5

92. (c) The Brunnstrom stages of motor recovery can be used to describe motor recovery following stroke. Brunnstrom classification stage 4 is when the patient begins to activate muscles selectively outside of flexor and extensor synergy. Ref: Whyte J, Hart T, Laborde A, Rosenthal M. Rehabilitation of the patient with traumatic brain injury. In: DeLisa JA, Gans BM, Walsh NE, editors. Physical medicine and rehabilitation: principles and practice, 4th ed. Philadelphia: Lippincott Williams & Wilkins; 2005. p 1667

"17.

Workers who participate in a car dio vascul ar training program have been found to(a) communicate with their s uper visor s b etter.(b) be more efficient.(c) have better job performa nce evalu ati ons.(d) report fewer sick days." 17 (d) Workers who participated in a cardiovascular training program were compared to a control group. Those in the training program reported 51% fewer sick days than controls despite no change in their maximum oxygen consumption (VO2max).

"77.

You prescribe a work-hardening program for a 36-year-old assembly-line wor ker. Wor k-harden ing p rog ra ms ar e mo st ef fe ctive w hen (a) focused on upper limb function. (b) the patient's job functions are simulated. (c) combined with a physical therapy program. (d) prescribed by a physiatrist." 77 (b) Work-hardening programs are most effective when essential jo b duties are simulated.

"96. In conducting an exercise tolerance test on a patient with a history of ex ertional angina, a fa ll in syst olic bloo d press ur e at Bruce P rot oco l Stage II sho uld t rigger (a) stabilization at the current exercise level. (b) an increase in exercise intensity. (c)

discontinuation of the test. (d) a reduction in exercise intensity." 96 (c) If systolic blood pressure fails to rise during exercise tes ting with increasing workload, ischemia should be suspected. A fall in systolic blood press ure may re flect s ignificant ischemia and is an indication for aborting the exercise to leranc e test.

"154. A 29-year-old dancer presents with right groin pain. The muscu loskeleta l examina tion is significant for pain to palpation over the right psoas tendon and tightness in the right hip flexor. A muscle energy technique used to reduce pain and improve range of motion of the right hip would include (a) isometric contraction with the hip placed in flexion. (b) isotonic contraction with the hip placed in flexion. (c) isometric contraction with the hip placed in neutral position. ( d)

eccen tric cont racti on with the hip placed in extension." 154 (c) Muscle energy is a therapeutic technique in which the patient produces an isometric muscle contraction, which is followed by passive lengthening of that muscle by the healthcare provider to allow the involved joint to move through a restrictive barrier. In the case described, to lengthen the iliopsoas the hip should be placed in a neutral position and stretched passively into extension as tolerated.

"136. You are formulating a comprehensive rehabilitation program for a cachectic pat ient w ith ch ron ic ob stru cti ve pu lmo nar y diseas e. Su pple men ta l feeding in this patient population has been shown to:(a) improve respiratory muscle endurance (b)

minimize bronchodilator requirements ( c) c ar ry lit tle risk of oxy gen d esa tur ation.: (d) improve functional residual lung capacity.:137. On the stand in a civil litigation case, a physiatrist asked about an event ca usin g an inj ury shou ld:(a) decline to answer when the worker has had no objective testing that proves a di agno si s." 136 (a) Supplemental feeding of malnourished patients leads to improved respiratory muscle endurance and increased respiratory muscle strength. These improvemen ts occu r wit hou t cha nges in ske le tal musc le.

"103. A 70-year-old man comes to your office complaining of posterior neck and bilat eral s hou lde r p ain w orse in the mor nin g. Radio gra ph s of hi s shoulders reveal glenohumeral osteoarthritis, and his erythrocyte sedimentation rate is 70. Your initial management is to:(a) perform bilateral glenohumeral corticosteroid injections.:(b) prescribe Codman exercises.:(c) obtain a cervical spine magnetic resonance image. (d)

prescribe prednisone 20mg per d ay." 103 (d) This patient has polymyalgia rheumatica. When you diagnose polymyalgia rheumatica the patient is placed on daily prednisone and should note dramatic imp rov emen t wit hin a fe w days.

"186. A 10-year-old patient presents with spastic hemiparesis and diminished co gnitive function seve ral y ears foll owing t reatm en t for acut e leu kem ia . His CT sca n r ev eals enhancing bilateral white matter hypodensities. The patient received high-dose systemic chemotherapy with vincristine, prednisone/dexamethasone, and L-asparaginase in conjunction with intrathecal methotrexate. The most effective initial management approach to this patient would be (a) high dose parenteral dexamethasone. (b) resumption of intrathecal methtrexate. (c) hospitalization for broad spectrum antibiotic therapy. (d)

rehabilitation for comp ensatory stra teg ie s." 186 (d) This patient's presentation is typical of diffuse chronic le ukoencephalopathy associated with methotrexate neurotoxicity. Leukoencephalopathy may appea r years af ter com pletion of therapy. Often it is first detected as a learning disabi li ty. Pe diatric l eukemic s tr eated wit h intra thecal methotrex ate an d cra nia l ir ra di ation m ay d emon st rat e a redu cti on in IQ by m ore th an 15 poi nts .

"54. A 17-year-old person presents with frequent falls and no other complaints. On p hysi ca l e xam ina tion you not e unu sua lly high ar che s, dis tal l ower limb strength of 4/5, and decreased reflexes. The most appropriate next step in the diagnosis would be:(a) nerve conduction studies.:(b) muscle biopsy.:(c) magnetic resonance imaging of the lumbosacral spine. (d)

serum heavy metal leve ls." 54 (a) The clinical picture is suggestive of a polyneuropathy. Patients with hereditary polyneuropathies may have feet with high arches. Electrodiagnostic stud ies inc ludin g n erve conduction st udies re mai n t he m ost imp ortant first tests in the eva luatio n o f polyne uro pathy.

"126. When referring to the neurological manifestations of HIV, time locking de scribes (a) the simultaneous occurrence of central and peripheral nervous systems proc esses. ( (a) the simultaneous occurrence of central and peripheral nervous systems proc esses. ( (b)

the o ccurr enc e of mu ltip le pa th ologi ca l pro cesse s wit hin o ne par t o f t he ne uraxis. (b) the o ccurr enc e of mu ltip le pa th ologi ca l pro cesse s wit hin o ne par t o f t he ne uraxis. (c) the tendency for particular neurological processes to occur during certain disease stages. . (c) the tendency for particular neurological processes to occur during certain disease stages. . (d) the capacity of a neurological or medical process to exacerbate a subclini cal neur ological proc ess (d) the capacity of a neurological or medical process to exacerbate a subclini cal neur ological proc ess" 126 (c) Time locking refers to the predictability with which certain neurological manifestations of HIV develop during particular disease stages. For example, AIDS demen tia com plex is a late complication of AIDS which generally occurs when the C D4 cou nt falls below 2 50/m l.

"37. A 37-year-old male pipef itte r has co mplete d physical therapy you prescribed for a C6 radiculopathy. He no longer requires pain medication and is independent in his home exercise program. He complains of some pain and fatigue during physical therapy. His neurologic and strength examination is normal. Your next recommendation is(a) a functional capacity ev alua tion.(b) return to work without r estr ictio ns.(c) vocational rehabilitatio n.(d) exercise program with we ight s at hom e.

" 37. (a) A functional capacity evaluation (FCE) is a comprehensive test with some objective data that tests a person's ability to perform work-related tasks. An FCE helps determine what the worker can do at work on a safe and dependable basis. Testing is usually performed work after the initial rehabilitation program has been completed.

"94.

(This question has been eliminated from the exam, therefore, it was not scored. ) A 45 =-ye ar- old man pres ent s wit h d ipl opia and at ax ia 1 we ek after a viral infection. Physical examination reveals areflexia and ophthalmoplegia. Electrodiagnostic evaluation demonstrates reduced sensory nerve action potential amplitudes. Which course is expected in this disorder?:(a) Need for ventilatory support:(b) Muscle pain:(c) Complete resolution:(d) Improvement with plasma exchange" "94 (c) (This question has been eliminated from the exam, therefore, it was not scored.) The Miller-Fisher variant of Guillain-Barré syndrome consists principal ly of a tria d o f oph thalmoplegi a, ataxia, an d a refl exia . A ltered sensa tion in the h ands and a rou nd the m out h can als o o cc ur. Most patie nt s first not e the s udden on set o f dip lo pia . T his is g ene ral ly a ben ign co ndition and does not require specific immune therapy. Pure sensory and pure motor are other forms of Guillain-Barré syndrome."

102. It is recommended that a patient with a first ische mic strok e wh o is positive for an antiphospholipid antibody be treated with: (a) aspirin, 325mg orally daily. (b) warfarin, with an INR goal of 3.0-3.5. (c) clopidogrel (Plavix), 75mg orally daily. (d) ticlopidine (Ticlid), 250mg orally twice daily.

102. (a) Patients with a first ischemic stroke and a single positive antiphospholipid antibody test result who do not have another indication for anticoagulation may be treated with aspirin (325mg/day) or moderate-intensity warfarin (INR 1.4-2.8).

104. Which statement is correct regarding amyotrophic lateral sclerosis (ALS)? (a) Bowel and bladder dysfunction are common in ALS. (b) ALS is most commonly inherited in an autosomal dominant pattern. (c) Both spasticity and diffuse muscular atrophy and weakness are common in ALS (d) A LS may c ause sev er e car diac cond uc tion ab norma litie s. (d) ALS may cause severe cardiac conduction abnormalities.

104 (c) The vast majority of ALS cases occur sporadically. However, approximately 10% of all ALS cases are familial (FALS), usually inherited as an autosomal d ominant tr ait. Th e disease is typified by both upper and lower motor neuron loss, ca us ing sp asticity and atr ophy .

106. In reviewing the pulmon ary funct ion tests of a patient with Duchenne muscular dystrophy before prescribing an exercise program, relative to a healthy normal person, you would expect the patient to have(a) increased tidal volume a nd i ncrea sed funct ional residual capacity.(b) decreased tidal volume a nd d ecrea sed funct ional residual capacity.(c) increased tidal volume a nd u nchan ged vital capacity.(d) unchanged tidal volume a nd d ecrea sed vital capacity.

106. (b) Duchenne muscular dystrophy, like all myopathies, causes a restrictive pattern of respiratory compromise. Therefore, functional residual capacity, tidal volume, residual capacity, and vital capacity are all reduced relative to age-matched normals.

109. Children with which physical disorder tend to have higher verbal skills compar ed t o ove ral l c ognit ive abi lity?:(a) Muscular dystrophy:(b) Myelodysplasia:(c) Cerebral palsy:(d) Autism

109 (b) Children with myelodysplasia have deceptively good verbal facility that creates the impression of higher intellectual functioning than is found on forma l t esti ng (" coc ktail party synd ro me"). Ch ild ren wit h ce reb ral palsy, a utism, and mu scul ar dys tro phy do n ot typically de mo nst rate this f in ding.

112. What is the most common cau se of tr aumati c brain injury in a child under the age of 1 year?(a) Motor vehicle crash(b) Near drowning in bath(c) Inflicted injuries(d) Fall from changing table

112. (d) For infants, more than two-thirds of all traumatic brain injuries result from falls; only 8% of these result in moderate or severe injuries. For preschool children, falls account for 51% of TBI and motor vehicle crashes for 22%. For children 8 to 9 years of age, etiology of TBI is evenly divided between falls, sports, and recreational activities, and motor vehicle crashes.

12 When is the use of hyperbaric oxygen recommended for the treatment of diabetic foot ulcers? a) As first-line treatment a. b) If there are signs of infection b. c) If standard therapy is ineffective c. d) As prophylaxis after wound has healed

12 Option c is correct. Commentary: Hyperbaric oxygen therapy is a treatment modality that can be considered for non-infected chronic diabetic foot ulcers that have not responde d to oth er therapies. Systematic review did show improved wound healing at 6 weeks with use of hyperbaric oxygen but no differences were noted at 1 year. Reference: (a) Miller AO, Henry M. Update of diagnosis and treatment of diabetic foot infections. Phys Med Rehabil Clin N Am 2009;20:621.(b) Kranke P, Be nnet MH, Martyn-St James M, Schnabel A, Debus SE. Hyperbaric oxygen therapy for chronic wounds. Cochrane Database Syst Rev 2012:4;

150. The proper static alignment of the socket in a transtibial prosthesis wil l place the sock et in sl ig ht (a) flexion. (b) extension. (c) external rotation. (d) abduction.

150 (a) The socket of a transtibial prosthesis is typically slightly flexed during the static prosthesis alignment process. Proper flexion of the socket improv es weight- bearing characteristics and also reduces knee extension forces during mid- st ance a nd the te rminal stan ce pha ses of gai t. The static al ign men t als o p lace s th e socke t in 5° of ad du ct ion wit h t he foot sl ight ly i nset re lat ive t o t he so cket.

16. Amputation rather than limb salvage is recommended for the surgi cal manag ement of high- gra de sarcomas involving the (a) humerus. (b) distal tibia. (c) ilium. (d) distal femur.

16. (b) Limb salvage is almost always preferred for the upper extremity because of the disability associated with upper extremity amputation. Amputation is generally preferred for malignancies of the distal leg and foot in order to ensure adequate tumor resection and because of the excellent functional status of transtibial amputees. Limb salvage is also preferred for sarcomas of the pelvis and thigh.

172. Which characteristic is a risk factor for heterotopic ossification in traumati c br ai n i nju ry?:(a) Male gender:(b) Flaccid tone:(c) Long bone fractures:(d) Older age

172 (c) Risk factors for HO include: long bone fractures, spasticity, prolonged immobilization, and prolonged coma.

50 The most appropriate cervical orthosis for an unstable cervical spine fracture is a) Philadelphia/Miami J orthosis b) Sternal occipital mandibular immobilizer (SOMI) orthosis c) Halo orthosis d) Milwaukee orthosis

50 Reference(s) Moore DP, Tilley E, Sugg P. Spinal orthoses. In: Braddom RL, editor. Physical medicine and rehabilitation. 4th e Philadelphia: Elsevier Saunder s; 2011. p 362-264. Option c is correct. For an unstable fracture, a Halo orthosis provides the best limitation in range of motion to protect the spinal cord from further injury. A Philadelphia/Mia mi J orthosis is not appropriate for uns table fractur es . A SOMI d evice is primarily used in cervical sprains or stable fractures with intact ligaments. A Milwaukee brace/orthosis is a cervicothoracolumbosacral orthosis primarily indicated for scoliotic management of high thoracic curves.

63. Which medication that binds to B-lymphocyte CD20 sur face anti gens (monoclonal antibody) has recently received a new indication for treatment of rheumatoid arthritis in patients who have failed tumor necrosis factor (TNF) alpha antagonists and who are receiving concomitant methotrexate (Trexall)? (a) Etanercept (Enbrel) (b) Abatacept (Orencia) (c) Anakinra (Kineret) (d) Rituximab (Rituxan)

63. (d) Rituximab works by binding to B-lymphocyte CD20 surface antigens (monoclonal antibody) and thereby depleting the B cell population. Its previous indication was for treatment of non-Hodgkin's lymphoma. Etanercept is a TNF alpha antagonist. Abatacept blocks co-stimulatory molecules and T-cell activation. Anakinra inhibits interleukin-1 type receptors.

7. Which finding on your history and physical examination of an injured worker with low b ack pa in would req uir e imm edi ate interve nti on ?:(a) Pain radiating into the posterior thigh:(b) Pain that awakens the worker at night:(c) Paresthesias in the great toe:(d) Ankle weakness only with long-distance walking

7 (b) Pain that awakens the worker at night may indicate a serious etiology including tumor or infection. Radicular pain, paresthesias, and weakness with fati gue may also pr esent with serio us as well as co mmon cau ses such as dis c herniations and spina l s tenosis. Th e latter may h elp dire ct the d iagnosis. H oweve r, pain wo rse a t nig ht sh oul d al ert the tr eatin g p hys ici an to e valuate more serious etiology without delay.

86. A young breast cancer survivor expresses concern over her risk of developing ly mphe de ma. Yo u e xplai n th at an in cre ase d incide nce o f ly mph ed ema is associated with:(a) hypertension.:(b) diabetes mellitus.:(c) osteoarthritis. (d) obesity.

86 (d) Obesity and weight gain has consistently been shown to increase breast cancer survivors' risk of developing lymphedema.

9. In children with spastic cerebral palsy, which intervention strengthens weak mus cles ?:(a) Ankle-foot orthotics:(b) Tendon transfer surgery:(c) Intrathecal baclofen:(d) Functional training program

9 (d) Children with cerebral palsy often have weakness as part of their disorder. Treatments such as bracing, tendon lengthening or transfers, and medications su ch a s bot uli num t oxin or int ra thecal b acl ofe n ad d to th is weakness. Strengthenin g pr ograms or functio nal training pr og ram s can help to strengthen weak m uscles.

"

94. A negative prognosticator for successful surgical nerve rep air after trauma is (a) partial transection of the nerve. (b) distal nerve injury. (c) prior radiation therapy. (d) nerve repair within 4 months of injury." 94 (c) Negative prognosticators for successful nerve repair include advanced age, nerve injury resulting from dislocation (stretch), delay of repair beyond 5 months, prior radiation therapy, nerve discontinuity (gap) exceeding 2.5cm, proximal nerve injury and poor condition of nerve endings.

"79.

Which ethnicity has the highest prevalence of neural tube defects? (a) Eastern European (b) Asian (c) African American (d) Hispanic " 79 Answer: (d) Commentary: Even though neural tube defects have declined across all ethnicities, because of increased folate intake, the disparity between Hispanics and other ethnicities remains. While 50%-70% of neural tube defects can be prevented by adequate folic acid, genetic influences remain. Reference: Pico EL, Wilson PE, Haas R. Spina bifida . In: Alexander MA, Matthews DJ, editors. Pediatric rehabilitation: principles and practice. 4th ed. New York: Demos; 2010. p 199, 201.

143 The use of a magnetic knee wraps in patients suffering with mild to moderate knee osteoarthritis has been shown to (a) decrease edema. (b) increase walking distance. (c) increase isokinetic strength. (d) increase range of motion.

143 Answer: (c) Commentary: The application of magnetic knee wraps has been shown to increase isokinetic strength and improve pain scales. Edema, walking distance and range of motion were not outcome measures. Ref: Chen C-Y, Chen C-L, Hsu S, Chou S, Wang K. Effect of magnetic knee wrap on quadriceps strength in patients with symptomatic knee osteoarthritis. Arch Phys Med Rehabil 2008:89:225864.

183. Non-swimming aquatic therapy in late poliomyelitis patients pro vides whi ch benefi t? (a) Increased peak oxygen uptake (b) Reduction in knee joint effusion (c) Improved muscle strength in affected muscles (d) Reduction of heart rate with same work load

183. (d) After a 5-month non-swimming aquatic therapy program in late poliomyelitis patients, researchers found a reduction of pain, a subjective increase in self perceived fitness level, and a reduction of heart rate with same-work-load lifting. However, they found no changes in peak oxygen uptake or peak work load, and no increase in muscle function.

1. A 61-year-old woman reports tingling in the posterior aspect of her upper arm, f orea rm an d h and . She has no hist ory of trauma and r adio gra ph s are negative. Her physician has ruled out a cervical radiculopathy by electrodiagnostic studies, magnetic resonance imaging, and physical examination. The physician is suspicious that the cause of the patient's paresthesias is myofascial in nature. Which finding would be consistent with myofascial pain?:(a) Positive sharp waves on electromyography at rest in trigger points:(b) Reproduction of symptoms with palpation over the trigger point:(c) Elevated serum creatine phosphokinase associated with prolonged muscle activity:(d) Resolution of symptoms with isokinetic exercise of the affected muscle

1 (b) Moderate, sustained pressure on an irritable trigger point causes symptoms in the reference zone for that muscle. Myofascial trigger points are electric all y si lent and show no resting m uscle ac tiv ity on EMG. No elevation i n CPK is seen wit h this co ndition. Lo cal injec tio n and /or s pray a nd stretch of the mu scle are the treat me nts of cho ice. Is oki netic ex erc ise is not indicated as a treatment for this condition.

176 Ultrasound utilizes the physiologic mechanism of (a) conduction. (b) evaporation. (c) convection. (d) conversion.

176 Answer: D Commentary: Conversion is a process of transforming energy to heat; for example, sound transformation with ultrasound. Conduction is a transfer of thermal energy through direct contact; for example, hot packs. Convection is a process of using a medium to transfer energy; for example, fluidotherapy. Evaporation is a process of transforming a liquid to a gas; for example, vapocoolant sprays. Ref: Weber D, Hoppe K. Physical agent modalities. In: Braddom R, editor. Physical medicine and rehabilitation 3rd. ed. Philadelphia: WB Saunders; 2007. p 459-61.

3 What is the most likely cause of a hemorrhagic stroke in a 14-year-old c hild? a. Congenital heart disease b. Carotid dissection c. Hemophilia d. Sickle cell disease

3 Option c is correct. Hemorrhagic strokes in children can occur as a result of moyamoya diseas e and he mop hi li a. Cau ses of isc hemic str ok e in ch ildr en i nclude cer ebro vascu lar anom ali es, co nge nita l heart disease, carotid dissection, sickle cell disease, inherited disorders of coagulation, and previous infection with varicella zoster. Children have an annual incidence of 2.7 strokes per 100,000.

"122. Which physical diagnosis finding is associated with poor functi onal outc ome follo wing str oke?

(a) Loss of pinprick sensation (b) Prolonged flaccid period (c) Generalized increase in tone (d) Significant shoulder subluxation" 122. (b) Factors associated with poor functional outcome following stroke include a prolonged flaccid period, severe proprioceptive deficits, late return of reflexes, and severe proximal spasticity.

103. A person with Paget's d isea se ty pic ally h as(a) alternating osteoblastic and oste ocl astic phases.(b) equivocal radiographs.(c) evidence of syndesmophyt es.(d) blue sclera.

103. (a) Radiographs will show a mottled appearance. The skull, tibia, pelvis and vertebral bodies are most commonly invloved. Syndesmophytes are seen with spondyloarthropathys. Blue sclera is seen in osteogenesis imperfecta.

122. In patients with a traumatic brain injury, which fa ctor sugg ests a poor prognosis for emergence from unresponsiveness? (a) Decorticate posturing (b) Flaccid muscle tone (c) Conjugate eye movement (d) Reactive pupils

122. (b) After a traumatic brain injury, the following factors are associated with a better prognosis: younger age, reactive pupils, conjugate eye movement, decorticate posturing, early spontaneous eye opening, absence of ventilatory support, and higher Disability Rating Score on admission. Factors associated with poor prognosis include decerebrate posturing and flaccid muscle tone.

122. A 25-year-old woman wit h mu ltipl e s cleros is is considering having a baby, and she wants your advice. You tell her that(a) multiple sclerosis is no t co mpati ble with pregnancy.(b) pregnancy does not influ ence the lon g-term disease course.(c) multiple sclerosis exace rbat ions are incre ased during pregnancy.(d) interferon therapy poses no risks du ring p regnancy.

122. (b) Pregnancy has little impact on the long-term course of multiple sclerosis. The disease tends to be quiescent during pregnancy. Studies have shown an increased number of exacerbations in the postpartum period. The use of interferon therapy during pregnancy is controversial. There have been reports of successful use during pregnancy in other disease processes; however, laboratory data indicate abortifacient properties at higher doses and some degree of organ abnormality in laboratory animals.

135 Which clinical factor offers a favorable prognosis in multiple sclerosis? (a) Male sex (b) Older age at onset (c) Normal MRI at presentation (d) Early development of mild disability

135 Answer: C Commentary: Favorable clinical factors with prognostic value in multiple sclerosis include younger age at onset, female, normal MRI at presentation, complete recovery from first relapse, low relapse rate, long interval to second relapse, and low disability at 2 and 4 years. Ref: Picone MA, Cook SD. Multiple sclerosis. In: Lin VW, editor. Spinal cord medicine: principles and practice. New York: Demos; 2003. p 527-36.

193. A 65-year-old woman with shoulder osteoarthritis asks how to us e ice for her shou lder pai n. For how many minutes per session should she apply ice to her shoulder? (a) 5-10 (b) 10-15 (c) 20-30 (d) 30-40

193. (c) The most effective use of ice for pain control in osteoarthritis is application of ice for 20- to 30minute periods. The ice should not touch the skin, but wrapped in a protective layer or towel to prevent damage to the skin. It should rest over the joint and not be moved around. Frequency should be on an as needed basis, but there should be at least a 30-minute break between sessions.

23. Which factor does NOT contraindicate complete decongestive therapy in a pe rson with lymp hedema? (a) Acute deep venous thrombosis (b) History of radiation therapy to the involved limb (c) Active cellulitis (d) New pathologic humerus fracture Ref: Cohen SR, Payne DK, Tunkel RS. Lymphedema: strategies for management. C ancer 2001;92( 4 Suppl): 980-7. Clinical Activity 3.5

23 (b) Relative contraindications to complete decongestive therapy include significant congestive hear t f ailure, acu te deep venous thrombosi s, acute or untreated infecti on o r i nflammat ion of th e a ffected lim b, and fracture. Lymphatic drainage massage should be avoided over concurrently irradiated soft tissues, but a history of previous radiation therapy is not a contraindication for therapy.

25. Which therapeutic application of functional electric al st imu lation is NOT applicable in the population with spinal cord injury? (a) Lower limb exercise in cauda equina syndrome (b) Ventilatory assistance in a C2 ASIA class A injury (c) Achieving lateral or palmar prehension in a C6 ASIA cla ss A injury (d) Electroejaculation to harvest sperm for assisted rep rod uc tio n techniques

25 Answer: (a) Commentary: Functional electrical stimulation (FES) str ate gi es use applied electrical current to activate weak or denervated muscle. FES is most effective in upper motor neuron injuries with preservation of the anterior horn cells and motor nerve roots. Because of the amount of charge density required to directly depolarize muscle, FES is not effective if large quantities of musculature are denervated. FES can be applied to the skin surface, or by means of implanted electrodes. One application in the population with SCI is its use in conjunction with a bicycle ergometer to improve cardiac capacity. Generally, individuals with cauda equina syndrome will not be good candidates for FES-assisted cycling, due to the extent of denervation associated with this injury level. Phrenic nerve and diaphragmatic pacing have been used to wean standard ventilator dependence in individuals with high tetraplegia and preserved phrenic nerve function. Implanted FES systems have been used to generate hand grasp and release, with or without tendon transplantation. External hand/forearm orthoses have also been developed primarily for therapeutic stimulation, with the hope of developing future neuroprostheses. Patients with intact parasympathetic efferent innervation to the detrusor have improved control of micturition, albeit with the need for sacral deafferentation, resulting in the loss of perineal sensation and reflex erection. Electroejaculation using a rectal probe has been highly successful at producing seminal emission for sperm harvesting for the purpose of assisted reproduction in individuals with SCI. Gorman PH, Ho CH. Functional electrical stimulation. In: Li n VW, editor. Spinal cord medicine: principles and practice. New York: Demos; 2010. p 837-44.

32 Which athlete is LEAST likely to sustain an anterior cru ciate ligament ( ACL) tea r? a. A 30-year-old competitive runner b. A 40-year-old recreational basketball player c. A 25-year-old competitive soccer player d. A 16-year-old high school tennis player

32 Option a is correct. A runner does not typically stress his/her ACL. The hist ory of a non-con tact ACL in ju ry is us ual ly of a movem ent w ith "cu ttin g" a ction, rap id a ccele rat ion o r d ece ler ati on, or twisting.

36. Which chemotherapeutic agent can produce sensory polyneuropathy?:(a) Paclitaxel (Taxol):(b) Doxorubicin (Adriamycin):(c) Cyclophosphamide (Cytoxan):(d) 5-fluorouracil (5-FU)

36 (a) The taxanes, docetaxel (Taxotere) and paclitaxel (Taxol), are being used increasingly as first line chemotherapy for a variety of malignancies including ov aria n, br eas t, an d head and ne ck cance rs. Th ey a re a sso ciated with a high incide nce of sen sor y polyne uro pathy. Fo rtu na tel y, th is oft en resolves o r is si gnifican tly d imini sh ed fol lowi ng d isc ont inuat ion of th e agent . However, some patients continue to experience severe, debilitating neuropathy.

44. A patient ambulates with a T rende len burg g ait. You suspect an injury to the(a) femoral nerve.(b) superior gluteal nerve.(c) obturator nerve.(d) sciatic nerve.

44. (b) Trendelenburg gait is characterized by excessive dropping of the pelvis contralateral to the stancephase leg. It is caused by weakness of the hip abductors, which include the gluteus medius innervated by the superior gluteal nerve.

"112. In multiple sclerosis, which factor is associated with poor pro gnosis?

(a) Female sex (b) Onset before age 30 years (c) Positive Lhermitte sign (d) Cerebellar involvement at onset" 112. (d) Factors associated with poor prognosis in multiple sclerosis are: progressive course at onset, male sex, age at onset >40 years, cerebellar involvement at onset, and multiple system involvement at onset. A positive Lhermitte sign (shocklike sensation down the spine, often into the limbs on neck flexion) is indicative of cervical myelopathy but has no prognostic significance.

1. A 22-year-old runner presents with acute onset of dis tal c alf pain. She is diagnosed with Achilles tendinitis and is referred to physical therapy. Which therapeutic modality is the LEAST beneficial in treating an overuse injury of this sort? (a) Therapeutic ultrasound (b) Iontophoresis (c) Ice massage (d) Neuromuscular electrical stimulation

1 Answer :(d) Commentary: With acute overuse injuries, modalities such as u ltr asound, iontophoresis, and ice massage may decrease pain and facilitate rehabilitation. Electrical stimulation with recruitment of muscle fibers may be contraindicated in treating acute overuse injuries. Reference: Stretanski MF. Achilles tendinitis. Fronter a W R, Si lver JK, Rizzo TD, editors. In: Essentials of physical medicine and rehabilitation. 2nd ed. Philadelphia: Saunders-Elsevier; 2008. p 407-10.

1 The diagnosis of postpolio syndrome requires a confirmed history of poli o wit h r eco ve ry an d a t l eas t 15 years of n euro log ical a nd funct iona l st abili ty follo wed by th e o nset of a. heat intolerance. b. new weakness. c. paresthesias. d. joint contractures.

1 Option b is correct. Postpolio syndrome is a diagnosis of exclusion and requi res a history of poli o w ith r ec ove ry, su bse quen t stab ili ty and pr ogre ss iv e weak ness not expl ain ed by ot her he alt h pr oblems. Two or more of the following problems complete the diagnosis new weakness, fatigue, muscle or joint pain, cold intolerance, decreased function, new atrophy.

2. What magnetic resonance imaging findings would help to distinguish an acute from a chronic intracerebral hemorrhage? (a) T1 decreased, T2 increased (b) T1 decreased, T2 decreased (c) T1 increased, T2 increased (d) T1 increased, T2 decreased

2 (b) The T1 signal in an acute hemorrhagic event would be decreased. Magnetic resonance imaging can be helpful in distinguishing acute from chronic hemorrhagic events. The pathology and subsequent neuroimaging results are based on the stage of hemoglobin molecular breakdown. In acute hemorrhagic states, deoxyhemoglobin predominates and T1/T2 images are both decreased. In chronic hemorrhagic states (more than 2 weeks) methemoglobin predominates (mainly extracellular) and T1/T2 signals are decreased.

2 The risk of an exacerbation of multiple sclerosis is red uced a. during pregnancy. b. in the first 3 months after delivery of a baby. c. in the postoperative period after a surgery. d. during a respiratory infection.

2 Option a is correct. During pregnancy multiple sclerosis (MS) exacerbations d ecrease to about half of wh at t hey wo uld be oth erwise . D ur ing the fir st 3 month s po stpa rtum, th e rel aps e r ate is hig her than normal. The net effect of pregnancy on the course of MS is neutral, and women need not make decisions about pregnancy based on fear that it will worsen their disease. There is no documented effect on MS due to surgery or with a respiratory infection.

"34.

A 19-year-old man with Charcot-Marie-Tooth disease has begun fa lling fre quently a nd yo u n ote 2/5 bilateral ankle dorsiflexion strength. He ambulates with a steppage gait pattern. You would recommend which of the following? (a) Knee-ankle-foot orthosis (b) Ankle-foot orthosis (c) High top shoes with rocker bottom (d) Supramalleolar orthosis" 34 (b) Persons with Charcot-Marie-Tooth (CMT) disease typically have dorsiflexion weakness and require ankle-foot orthoses. Supramalleolar orthoses will not provide enough support to compensate for the weakness in dorsiflexion. A rocker bottom shoe would not be an appropriate option. Patients with CMT disease generally have functional (grade 4/5) strength in their quadriceps, thus a knee-ankle-foot orthosis is not indicated.

156. Cervical traction is a useful modality fo r p at ients with (a) cervical strain. (b) diskitis. (c) acute radiculopathy. (d) vertebral compression fracture.

156. (c) Cervical traction is proven effective for illnesses that involve nerve root irritation or compression of nerve roots. Ref: Atchison JW, Stoll ST, Cotter AC. Manipulation, traction, and massage. In: Braddom RL, editor. Physical medicine and rehabilitation. 2nd ed. Philadelphia: WB Saunders; 2000. p 427.

173. What is the mechanism of action of alendronate (Fosamax) in treating oste oporosis ? (a) Inhibits osteoclasts (b) Binds to estrogen receptors (c) Increases calcium absorption (d) Promotes osteoblasts

173 (d) (a) Fosamax inhibits osteoclasts at doses used for osteoporosis.

10. Which supplement is proven efficacious in lowering lipids? (a) Licorice (b) Omega-3 fatty acids (c) Antioxidants (d) Vitamin E Ref: (a) Chung MK. Vitamins, supplements, herbal medicines, and arrhythmias. Cardiol Rev 2 004;12:73 -84 (b) Ernst E. Cardiovascular adverse effects of herbal medicines: a systemati c review of th e recent literatur e. Ca n J Cardiol 2003;19:818-27. Educational Activity 2.1

10 (b) Beneficial supplements for lowering lipid levels include omega-3 fatty acids an d coenzyme Q-10. Th e roles of antioxi dants and vitamin E have not yet been clarifie d. L ico rice has been i mp lic ated in sev eral negative cardiac effects.

11. Which exercise creates the least stress of the rotator cuff musc ulature? (a) Arms in the scapula plane in internal rotation with weights (b) Arms in the scapula plane in external rotation with weights (c) Prone horizontal shoulder abduction in external rotation (d) Quadruped push-up

11. (d) The quadruped push-up is a closed kinetic chain (CKC) exercise. CKC exercises are the least demanding of the rotator cuff exercises. The axial load with CKC exercises effectively reduces the weight of the arm (ie, load). CKC promote muscle co-activation needed for functional tasks.

113. A 60-year-old man, on postoperative day 4 after total hip arthroplasty, experiences acute pain,redness, and swelling of the first metatarsophalangeal joint. He has had similar episodes in the past. You recommend (a) warfarin. (b) allopurinol. (c) colchicine. (d) dicloxacillin.

113 (c) Postoperative patients are at risk for acute exacerbations of gout. Oral colchicine is the standard therapy for acute gout. Allopurinol is indicated for the prevention of gout flares in patients with frequent (less than every 9 months) attacks and is contraindicated in the acute phase. Low-dose aspirin can precipitate an acute attack. The clinical vignette is not consistent with infection or deep vein thrombosis.

115. Etidronate disodium (Didronel) is used in the manag ement of hete rotopic ossification to (a) improve range of motion. (b) reverse immature ossification. (c) reverse mature ossification. (d) prevent ossification.

115 (d) Etidronate blocks the late phase of bone formation (mineralization), by preventing the conversion of amorphous calcium phosphate to hydroxyapatite. The drug has no effect on the early phase of ossification.

121. When an individual is exposed to a stimulus that ca uses tiss ue d amage, an immediate response occurs that involves withdrawal and/or attempts to escape the stimulus. This reaction is an example of (a) respondent learning. (b) operant learning. (c) cognitive behavioral theory. (d) trial and error.

121. By successfully avoiding pain (ie, "punishment"), the individual achieves a reduction in pain, thus rewarding the avoidance behavior. The acquisition of pain behaviors may be determined initially by the history of learned avoidance behaviors, called operant learning. Respondent learning is when an aversive stimulus is paired with a neutral stimulus and with repeated exposures over time the neutral stimulus will come to elicit an aversive response (ie, fear).

138. In patients with radiculopathy, how long doe s it take for morphologic changes of increased duration and amplitude to occur in the motor unit action potentials? (a) 1 day (b) 1 week (c) 3 weeks (d) 6 weeks

138. (d) Reduced recruitment may be seen day 1. Positive waves may be seen in the paraspinals as early as 1 week, and positive waves and fibrillations may be seen in the extremities by 3-5 weeks, but, because of reinnervation by axon sprouting, it takes at least 6 weeks to see increased amplitude and duration of muscle activity. Ref: Dumitru, D, Zwarts, MJ. Radiculopathies. In Dumitru D, Amato AA, Zwarts, MJ, editors. Electrodiagnostic medicine. 2nd ed. Philadelphia: Hanley & Belfus; 2002. p 732.

152. Regarding subarachnoid hemorrhages,:(a) arteriovenous malformations present with hemorrhage after age 40.:(b) the risk of rebleeding after an unoperated aneurysm is low.:(c) their clinical presentation is nonspecific.:(d) aneurysms usually occur in the anterior region of the circle of Willis.

151 (b) The DeLorme axiom states that high-weight, low-repetition exercises build strength and lowweight, high-repetition exercises build endurance.

"45.

According to data from the Model Spinal Cord Injury Care Sys tem, the leading cause of traumatic spinal cord injury in the United States is (a) motor vehicle accidents. (b) violence. (c) falls. (d) diving accidents." 45 (a) The top three causes of traumatic spinal cord injury in the United States are motor vehicle accidents, falls, and violence.

"57.

If a man injures his low back while on th e j ob and is off work for 6 months, then the chance that he will return to work is (a) 25%. (b) 35%. (c) 50%. (d) 75%." 57 (c) There is about a 50% chance of return to work when a worker who injures his low back on the job is off work for 6 months. The rate drops to 25% when the worker is off for 1 year, and is minimal is he is off for 2 years. Ref: Schuchmann JA. Occupational Rehabilitation. In: Braddom RL, editor. Physical medicine and rehabilitation. 2nd ed. Philadelphia : WB Saunders; 2000. p 994.

"96.

You elect to treat a patient with chronic pain and a seizure di sorder wi th an opi oid. Wh ich medication should be avoided due to potential accumulation of a neurotoxic metabolite? (a) Meperidine (Demerol) (b) Hydromorphone (Dilaudid) (c) Methadone (Dolophine) (d) Fentanyl (Duragesic, Actiq)" 96 (a) Meperidine (Demerol) is converted in the liver to normeperidine. Normeperidine accumulation may cause hyperexcitability of the central nervous system and lower the seizure threshold.

57. A 32-year-old welder suf fere d a b rac hial p lexus injury falling off a scaffold. He is unable to use his right upper extremity because of severe weakness. According to the World Health Organization classification system, the patient's weakness describes his(a) injury.(b) impairment.(c) disability.(d) handicap.

57. (b) Impairment is defined as an alteration of a person's health status, a deviation from normal in a body part or any organ system (any loss or abnormality of psychologic, physiologic, or anatomic structure or function).

"200. What is the average range of motion at the ankle during normal human ambu lation o ver a le vel s urf ac e? (a) 0° dorsiflexion, 10° plantarflexion (b) 30° dorsiflexion, 20° plantarflexion (c)

5° dorsiflexion, 20° plantarflexi on (d) 10° dorsiflexion, 10° plantarflexion End of Examination" 200 (c) The average range of motion at the ankle during normal human ambulation over level surfaces is from 5° of dorsiflexion to 20° of plantarflexion. This r ange of mo tion va ries with ambulation over uneven surfaces and with activities such as stair climbing .

139. A 3-year-old child has a high thoracic sp ina l cord injury. At age 10, which of the following is the most likely complication? (a) Severe lordosis without scoliosis (b) Scoliosis requiring surgical treatment (c) Deep venous thrombosis (d) Heterotopic ossification

139. (b) Children who sustain cervical or high thoracic spinal cord injuries at an early age are at high risk of developing progressive scoliosis that requires surgical management. Ref: Lubicky JP, Betz RR. Spinal deformity in children and adolescents after spinal cord injury. In: Betz RR, Mulcahey MJ, editors. The child with a spinal cord injury. Rosemont (IL): American Academy of Orthopedic Surgeons; 1996. p 363-70.

143. Which type of muscle contraction produces the greatest amount of force generation (torque)? (a) Fast concentric (b) Slow concentric (c) Isometric (d) Fast eccentric

143 Answer: D Commentary: Muscle force generation varies depending on the type of muscle contraction and the speed of the contraction. Eccentric contractions produce greater torque than isometric contractions and isometric contractions produce greater force than concentric muscle contractions. As the speed of contraction increases, eccentric contractions produce greater force. The opposite is true for concentric contractions, which generate greater forces at slower speeds. Ref: De Lateur B. Therapeutic exercise. In: Braddom R, editor. Physical medicine and rehabilitation. 2nd ed. Philadelphia: WB Saunders; 2000. p 397-8.

"160. One advantage of a small diameter caster (front wheelchair wheel) is:(a) greater ability to traverse rough terrain.:(b) better stability on steep inclines.:(c) less frequent maintenance. (d)

greater maneuverability" 160 (d) Small diameter casters on a wheelchair reduce the chair's turning radius, which gives it greater maneuverability. These casters frequently are mounted o n w heel chair s u sed f or sports s uc h as bas ket bal l. S mall er diameter cas ters are less wel l suit ed for outd oor activiti es ov er rough terra in . The size of th e caster d oes n ot af fe ct the mai nten anc e r equir ed and do es not improve wheelchair stability on inclines. Hard, narrow casters are typically recommended for mobility over smooth, level surfaces whereas wider, larger casters are better for mobility over uneven, rough surfaces.

"37 A 68

old woman with well controlled diabetes is asking your opinion on specialized footwear. She has good pedal pulses, intact sensation and no foot deformi ty. She has no prior history of ulce rs. In addition t o frequent f oot i nspections, you recommend a) custom molded diabetic shoes. b) well fitting breathable shoes. c) off-the-shelf diabetic shoes with wide toe box. d) custom insert with metatarsal pa" 37 Reference(s) Howard I . The prevention of foot ulceration in diabetic patients. Phys Med and Rehabil Clin N Am 2009;20:595-609. Option b is correct. Because she has well controlled diabetes (DM), intact pulses, intact sensation and no obvious foot deformity, this patient is at lower risk. Based on her ri sk profile, specialized footwear is not recommende Pa ti ents with DM wi thout risk factors for ulcer could be counseled and educated regarding the use of well fitting tennis shoes and sneakers. Other interventions listed could be considered, if ulcer risk factors change.

"104. A 17-year-old female was involved in a motor vehicl e crash 4 mon ths ago. She sustained a shoulder dislocation. Electromyographic studies have confirmed a brachial plexus injury to her posterior cord and indicate nerve continuity. Although she has completed 4 weeks of occupational therapy, she has had no improvement in her strength from baseline. Your next step would be to (a) reassure the patient and continue to monitor for imp rovement. (b) continue occupational therapy for 4 additional weeks . (c) initiate neuromuscular electrical stimulation to the affected mus cles. (d)

refer her to neurosurgery for exploratory surgery." 104. (d) With closed nerve injury as described, early active and passive range of motion exercise of affected joints is begun. The value of electrical stimulation is uncertain. Surgery is done when there is an incomplete loss of function but no improvement over several weeks or no return of function at 2 months for peripheral nerve and 4 months for a brachial plexus injury. The purpose of surgical repair is to improve peripheral nerve recovery and eventual function. Findings at the time of surgery help establish a prognosis. However, the chances of successful surgical repair begin to decline by 6 months after the injury. By 18 to 24 months, the denervated muscles usually are replaced by fatty connective tissue, making functional recovery impossible.

"125. Treatment is indicated in asymptomatic bacteriuria when an individual wit h a spin al cord injur y h as (a) chronic use of an indwelling Foley catheter. (b) bladder augmentation. (c) pyuria of 10-20 white blood cell count (WBC). (d)

ureteral reflux or hydro nephrosi s." "125 (d) Bacteruria is a common problem in patients with voiding dysf unction. At approximately 1 year postinjury, 66% to 100% of all individuals with spinal cor d injury h ave had at least 1 episode of bacteruria, depending upon their bladder man ag ement program. Asympto mati c bact eru ria has been found to b e p res en t i n 1 %--2 0.05 o f commu nity dwe ll ing p at ient s a nd 0.25 -40% of nur si ng hom e p ati ents old er th an 65 . There is g eneral agreement that asymptomatic bacteruria in a patient using a Foley catheter should not be treated. Attempts should be made to eradicate asymptomatic bacteruria and bacteruria associated with high grade reflux before urologic instrumentation and hydronephrosis, or in the presence of urea splitting agents."

"12. Which of the brain tumors listed is a beni gn tu mor? (a) Medulloblastoma (b) Astrocytoma (c) Glioblastoma (d) Craniopharyngioma

" 12. (d) The only benign brain tumor listed is craniopharyngioma Ref: Gerber LH, Vargo M. Rehabilitation for patients with cancer diagnoses. In: DeLisa JA, Gans BM, Walsh NE, editors. Physical medicine and rehabilitation principles and practice, 4th ed. Philadelphia: Lippincott Williams & Wilkins; 2005. p 1789.

"145. Compared to individuals without spinal co rd in jury, individuals with spinal cord injuries have a (a) lower risk of osteoporosis. (b) higher risk of diabetes. (c) lower rate of dyslipidemia. (d) higher rate of prostate cancer.

" 145 (b) Individuals with spinal cord injury are at an increased risk for carbohydrate intolerance, cardiovascular disease, and dyslipidemia. There does not appear to be an added risk for prostate cancer in men with chronic SCI. Ref

"84. Which presentation of mu ltip le sc ler osis i s most common?(a) Relapsing-remitting(b) Primary progressive(c) Secondary progressive(d) Relapsing-progressive

" 84. (a) Relapsing-remitting multiple sclerosis (MS) is the most common type of presentation— approximately 80%.

21. Hamstring injuries occur most commonly (a) at the proximal attachment of the lateral hamstrings to t he pelvis. (b) during concentric contraction of the medial hamstrin gs. (c) at the distal attachment of the medial hamstrings to th e tib ia. (d) during eccentric contraction of the lateral hamstrin gs.

"21 Answer: (d) Commentary: The majority of hamstring injuries occur fro m i nd ire ct forces during running and sprinting activities. Most injuries occur at the myotendinous junction, not at the osseous attachments, during eccentric contraction of the hamstring. The lateral hamstrings (biceps femoris) are affected more than the medial hamstrings (semitendinous and semimembranosus). Reference: Vetter CS, Hoch AZ. Hamstring strain. In: Fro nte ra WR , Silver JK, Rizzo TD, editors.: Essentials of physical medicine and rehabilitation. 2nd ed. Philadelphia: SaundersElsevier; 2008. p 407-10."

27. Modified radical neck dissection differs from radical neck dissection by s paring the (a) ansa hyp oglossi. (b) larynx. (c) external jugular vein. (d) spinal accessory nerve. Ref: Clinical Activity 3.8. Clinical Activity 3.9

"27 (d) In radical neck dissection, the spinal accessory nerve, sternocleidomastoid mus cle, and interna l j ugular vein are re moved along with the lymphatic chain, ansa hyp oglo ssi , superf icial c er vic al plexu s a nd occasionally other anterior cervical muscles. A modified radical neck dissection spares the spinal accessory nerve, internal jugular nerve, and sternocleidomastoid muscle. Arch Phys Med Rehabil Vol 87, Suppl 1, March 2006 S78 SAE-P ANSWER KEY, Phelan"

"154. Which diagnosis is NOT an indication for tra ns cutaneous electrical neurostimulation (TENS) therapy?

(a) Chronic low back pain (b) Acute surgical pain (c) Urinary urgency (d) Angina" 154. (d) TENS therapy has not been shown to provide benefit for angina. It has been proven to have a beneficial effect in all the other diagnoses. Ref: Basford JR. Therapeutic physical agents. In: DeLisa JA, Gans BM, Walsh NE, editors. Physical medicine and rehabilitation: principles and practice. 4th ed. Philadelphia: Lippincott; 2005. p 263-4.

"26. Osteoblastic lesions are seen in which type of cance r?

(a) Prostate (b) Lung (c) Breast (d) Renal" 26. (a) Bony metastases from prostate cancer usually are blastic, whereas those from breast, lung, and kidney are typically lytic. Knowing whether a metastatic bone lesion is blastic or lytic is important, because lytic lesions have a higher risk of pathologic fracture.

"25. An individual with C7 ASIA D tetraplegia must have

(a) a bulbocavernosus reflex and voluntary sphincter con traction. (b) a muscle grade of 3 or greater in at least half of t he key mu scle s below C7. (c) normal pinprick and light touch sensation through th e sacral derm atomes. (d) normal strength (5/5) in the C7 myotome." 25. (b) A bulbocavernosus reflex does affect American Spinal Injury Association (ASIA) scoring, and voluntary sphincter contraction is not a mandatory component of ASIA C or D. Muscle grade of less than 3 in at least half of the key muscles below C7 would be characterized as ASIA C. Someone with ASIA B through E must have some retained sensation in the sacral segments S4-S5 but that sensation can be normal or impaired. To classify the injury as C7 ASIA D would require a motor score of at least 3 out of 5 in the C7 myotome with normal strength in C6.

"142. Which statement regarding the multifidi muscles of the back is TRUE? (a) Atrophy of the multifidi occurs in patients with low back pain.

(b) Contraction of the multifidi causes gross trunk extension. (c) The mul tifidi mu scles cr oss 4 or more spinal levels. (d) The multifidi have a low composition of muscle spindles." 141. (a) The multifidi are local postural muscles of the lumbar spine. They are length transducers or position sensors of a spinal segment, by way of their rich composition of muscle spindles. The multifidi pass along 2 or 3 spinal levels. They are theorized to work as segmental stabilizers rather than producing gross trunk motion, since they do not have a large moment arm. Researchers have found atrophy of the multifidi in people with low-back pain.

"125. The majority of new spinal cord injuries in the United States a re a resu lt of (a) violence.

(b) falls. (c) motor vehicle accidents. (d) sports." 125. (c) Automobile accidents account for 34.5% of new spinal cord injuries, falls 22.0%, gunshot wounds 17.2%, diving 4.5%, and motorcycle crashes 4.4%. These figures represent all races combined.

"180. The Commission on Accreditation of Rehabilitation Facilities (C ARF) defi nes progr am ev alu ation as a (a) systematic procedure for measuring the outcomes of care.

(b) method of preventing medical complications. (c) routine means of building team relations. (d) procedure to develop new programs for rehabilitation." 180 (a) The Commission on Accreditation of Rehabilitation Facilities (CARF) defines program evaluation as a systematic procedure for measuring the outcomes of care. Program evaluation is 1 method to measure the effectiveness and efficiency of rehabilitation services. The other options listed are not the primary focus of program evaluation.

"112. The most common benign brain tumor in adults is (a) astrocytoma.

(b) oligoblastoma. (c) medulloblastoma. (d) meningioma." 112. (d) Meningiomas are the most common benign brain tumor, comprising about 15% of all primary brain tumors.

"67. According to current guidelines, for the i nju re d factory worker with acute low back pain, what is the recommendation? (a) >6 (b) 4-5 (c) 2-3

(d) <1" 67. (d) In a systematic review of patients with acute low back pain, resting in bed was found to be less effective than staying active. Ref: Hagen KB, Jarntvedt G, Hilde G, Winnem M. The updated Cochrane Review of bed rest, low back pain and sciatica. Spine 2005;30:542-6.

"70. Which form of prosthetic sus pensi on is NOT utilized with the transhumeral amputation?(a) Harness(b) Self-suspension (c) Suc tion

(d) Joint and corset" 70. Traditional suspension systems include harness (figure-of-8 or -9, chest strap, and shoulder saddle), self-suspension (condylar, Muenster, or Northwestern), semisuction (semisuction or hypobaric) and suction (full suction or silicone sock).

"190. Which K level best describes an individua l w ho is able to ambulate within the household, but not out in the community? (a) K 1 (b) K 2 (c) K 3

(d) K 4" 190 (a) The K level of 1 represents that of a household ambulator; K 2 limited community ambulator; K 3 unlimited community ambulator; K4 a very active community ambulator. The household exception FIM score of 5 indicates a "modified independent" ambulator who can handle household distances (i.e., less than 50 feet) inside or out. Ref: Nelson VS, Flood KM, Bryant PR, Huang ME, Pasquina PF, Roberts TL. Limb deficiency and prosthetic management. 1. Decision making in prosthetic prescription and management. Arch Phys Med Rehabil 2006;87(3 Suppl 1):S3-9

"115. In an individual with a C6 complete spinal cord injury, the ability to generat e a "p inc h" is produ ced by:(a) wrist extension. (b) elbow supination (c) wrist flexion.:

(d) elbow pronation." 115 (a) Wrist extension via extensor carpi radialis results in passive shortening of the (finger) flexor tendons. This phenomenon is termed tenodesis.

"135. A 46-year-old man with a 1-year history of C8 ASIA A spinal cord injury presents to your clinic with a 1-month history of increasing bilateral upper extremity weakness and pain. There is no history of trauma. You would (a) observe for 2 to 4 weeks and repeat ASIA exam. (b) perform electrodiagnostic testing to rule out periph eral nerv e co mpression. (c) order a magnetic resonance imaging study to look for posttrau mati c syringomyelia.

(d) initiate a workup for pernicious anemia." 135. (c) Posttraumatic syrinx results in neurologic decline in 3% to 8% of patients with spinal cord injuries and can develop 2 months to 30 years after spinal cord injury. Prompt diagnosis is essential and magnetic resonance imaging is usually definitive for diagnosing posttraumatic syrinx. Surgical treatment is usually indicated when there is clear neurological decline.

"127. Studies have shown that case managers working with injured work ers have (a) interfered with the physician's recommendations. (b) made recommendations that favor the insurer. (c) reduced workers' compensation costs and lost-time cases.

(d) interfere d in the relat ion ship between physician and patient." 127. (c) Case managers reduce workers' compensation costs by 23% and reduce the overall number of lost-time cases. Overall case management can help improve the quality of care, reduce cost, and decrease time loss in the worker's compensation system. Studies show that case managers doe not interfere with the physician-patient relationship or the physician's recommendations, and do not make proposals that favor the insurer.

"25. A 25-year-old man with L5 complete paraplegia is admitted to you r rehabil itation s ervic e 2 weeks after his injury. On admission you note that he is tolerating an oral diet but has not produced a bowel movement for 6 days. At this point, you recommend (a) oxybutynin (Ditropan) 3 times a day. (b) a contact irritant suppository with digital stimulation daily. (c) manual removal of stool from the rectum 1-2 times daily.

(d) nasogastr ic decomp ressi on for a presumed ileus." 25. (c) Individuals with lower lumbar and sacral level injuries usually experience areflexic bowel function. The use of suppositories are usually not useful in these individuals, because of the absence of spinal reflex activity. Manual evacuation is often required for an effective bowel program in a lower motor neuron injury. Anticholinergic medications may lead to constipation.

11. A 38-year-old drywall ha nger pres ent s with shoulder pain after falling onto the tip of his shoulder. He felt immediate pain in the upper part of his shoulder, but no numbness or tingling in his arm. On examination, he has a visible deformity on the superior aspect of his shoulder. He has pain with horizontal adduction of his left arm across his chest and is having difficulty lifting his arm. His passive range of motion is good. The best treatment for this patient would involve (a) use of an arm sling for at l east 4 w eeks. (b) referral to an orthopedi c su rgeon fo r surg ical repair. (c) physical therapy for Cod man exerc ise s. (d) corticosteroid injection after 10 days.

11. (b) This patient has a grade 3 acromioclavicular separation. Grade 1 or 2 separations would not have a visible deformity and would require weighted bilateral shoulder films. A grade 3 separation may have good results with conservative care, but a young manual laborer should be referred for surgical repair to ensure good results. Grade 4-6 separations should be surgically repaired. Patients should only be placed in a sling for a few days until the pain subsides. This will decrease the possibility of losing shoulder range of motion. The shoulder should be given a few days rest, and physical therapy referral is not appropriate at this time. A corticosteroid injection is not the treatment of choice and will not repair the separation.

110. What type of prehension is provided by a wrist-driven tenodesis orthosis? (a) Lateral (b) 3-jaw chuck (c) Cylindrical (d) Tip-to-tip

110 (b) The wrist-driven tenodesis hinge orthosis creates a 3-jaw ch uck prehension by stabilizing the interphalangeal joints of digits 2 and 3 and the interpha langeal an d metac arpophalangeal joints of the thumb. The extensor carpi radialis mus cl e acti vates the wrist exte nsion, wh ich thr ough tenodesis a cti on cr eat es a 3- ja w chuck p rehe nsio n.

110. What is the 5-year mortality rate for per son s with diabetes after sustaining a major lower limb amputation? (a) 0.15 (b) 0.25 (c) 0.33 (d) 0.5

110. (d) At least 50% of persons with diabetes and peripheral arterial disease who undergo major limb amputation will die within 5 years of sustaining major lower limb amputation. Ref: (a) Pandian G, Hameed F, Hammond M. Rehabilitation of the patient with peripheral vascular disease anddiabetic foot problems. In: DeLisa J, Gans B, editors. Physical medicine: principles and practice. 3rd ed. Philadelphia: Lippincott Raven; 1998. p1517-44.(b) Moolik P, Gill G. Mortality in diabetic patients with foot ulcers. Diabetic Foot 2002: Spring.

111. Which maneuver is used to i denti fy sacroi liac joint pathology?(a) Lasegue(b) Gaenslen(c) Ober(d) Thomas

111. (b) The Gaenslen sign is a test to determine sacroiliac pathology. This test is performed by having the patient lie supine. One buttock is extended over the table's edge while the other remains on the table. The ipsilateral leg is allowed to drop below the edge of the table, with the other leg remaining in a flexed position. Pain in the area of the sacroiliac joint on the side of the extended leg represents a positive test. The Thomas test is a test to measure hip flexion contracture. The Ober test is used to identify contracture of the iliotibial band or the tensor fascia lata. The Lasegue sign is the straight leg test, a nerve root stretch test to identify radicular pain.

117. Disablement as defined by the World Health Organization is:(a) all the effects of the injury or illness, impairments, activity limitations and bar ri ers to pa rtici pati on.:(b) inability to advance in the work place due to physical disability or need for w orkp la ce ada pta tions .:(c) recognition that the work injury was the cause of the disability as determined by c iv il lit iga tion.:(d) restrictions in the work site that requires the worker to use special aids.

117 (a) Disablement as defined by the World Health Organization is the term that summarizes all the effects of the injury or illness, impairments, activity limi tat ions , and ba rrier s to partic ip ation ex per ien ced by t he individual. Workers recei ving disab ili ty compe nsa tion have a di sin centi ve to re turn to wor k. Re fo rms have been made t o a llo w wo rker s t o i nvest iga te the ir abil ity to return to the work force without losing benefits. Disablement does not relate to advancement in the workplace.

118. A medical student has been practicing with the electromyography (EMG) machine. You n ote th at the c onve nti onal fil ter setting s h av e be en ch anged. The low frequency filter is now set at 200Hz and the high frequency filter is set at 1,000Hz. This will cause:(a) a decreased common mode rejection ratio.:(b) increased input impedance.:(c) no significant changes.:(d) distortion of recorded potentials.

118 (d) Typical settings are 20Hz for the low frequency filter and 10,000Hz for the high frequency filter. Allowing such a narrow bandwidth will cause distortio n o f th e rec ord ed po tentials. I t would ha ve no effe ct o n t he input imp edance or com mon mode r eje ction ra tio .

118. Which muscle is innerva ted by th e p eronea l division of the sciatic nerve?(a) Adductor magnus-anterior par t(b) Piriformis(c) Semimembranosus(d) Biceps femoris- short he ad

118. (d) The anterior part of adductor magnus is innervated by the obturator nerve. The piriformis receives its own branch off the lumbosacral plexus. The semimembranosus is innervated by the tibial division of the sciatic nerve. Only the short head of the biceps femoris is innervated by the peroneal division of the sciatic nerve.

12. Which factor is a prime dete rmina nt of suc cessful return to work after traumatic brain injury?(a) Presence of associated m uscu loske let al inj uries(b) Glasgow Coma Scale score at 48 ho urs post injury(c) Presence of post-injury depr essio n(d) Pre-injury occupation ty pe

12. (b) There are several determinants to successful return to work for persons with traumatic brain injuries. All studies have identified the severity of head injury as a primary factor in return to work; the Glasgow Coma Scale is one of the robust measures of injury severity. Other factors include preinjury work history, age, cognitive abilities, or motor limitations.

120. In a person with comple te p arapl egi a, whi ch gait has the highest energy expenditure per meter?(a) Swing-through gait in st anda rd kn ee- ankle- foot orthoses(b) Swing-through gait in Sc ott- Craig kn ee-ank le-foot orthoses(c) Reciprocating gait in a reci proca tin g gait orthosis(d) Swing-to gait using a st anda rd wa lke r

120. (c) Energy expenditure in paraplegia is as follows (in order of lowest to highest): normal walking, swing-through gait in a Scott-Craig knee-ankle-foot orthosis (KAFO), swing-through gait in a standard KAFO, reciprocating gait in a reciprocating gait orthosis. Swing-through gait in a reciprocating gait orthosis requires approximately the same energy expenditure as the Scott-Craig KAFO.

123. An elderly person's risk factors for falling include:(a) advanced age.:(b) living alone.:(c) osteoarthritis.:(d) polypharmacy.

123 (d) Risk factors for falling in the elderly include polypharmacy, particularly sedative use, cognitive impairment, lower extremity disability, palmomental r efl ex, foot pro blems , periphera l neuropat hy and poo r ba lan ce.

126. During acute herpes zoster infection, empiric use of which medi cation wa s shown i n a r and omized, blinded trial to reduce the incidence of postherpetic neuralgia? (d) Lamotrigine (Lamictal)

126. (b) In a randomized trial using amitriptyline for preemptive treatment of postherpetic neuralgia, it was found that pain prevalence at 6 months was reduced by greater than 50%. The authors concluded that amitriptyline should be combined with antiviral therapy in the treatment of acute herpes zoster in elderly patients.

138. A 27-year-old woman noted the onset of paresthesias in the lateral 3 digits of the r igh t h and 6 mo nths ag o. In iti all y, these sy mp toms we re constant, lasting for about 1 week, and then resolved spontaneously. Over the 3 weeks prior to this consultation the paresthesias recurred and they are now present in both the right upper and lower extremities. Electromyogram of the right upper extremity is normal. Bilateral median, ulnar, and sural sensory nerve conduction studies were normal. Right peroneal motor nerve conduction study was normal. What electrodiagnostic study would be most appropriate to perform at this time?:(a) Single-fiber electromyography:(b) Somatosensory evoked potentials:(c) Right median F wave:(d) Electromyogram of the right lower extremity

138 (b) With all of the studies being normal a diagnosis of multiple sclerosis should be considered. Of those listed the only study that might provide any usefu l i nfor matio n w ould be the soma to sensory evo ked pot enti als .

138. What parameter of the motor unit action potential ( MUAP) is the most sensitive to the distance between the generator source and recording electrode? (a) Amplitude (b) Phases (c) Firing rate (d) Duration

138. (a) The amplitude is primarily influenced by the distance between the electrode's recording surface and the electrical generator. The duration is a highly stable and reliable parameter of the motor unit. The other parameters are not affected by the distance.

139. The most common etiology of cerebral palsy is (a) premature birth. (b) birth asphyxia. (c) intrauterine stroke. (d) prenatal infection.

139 Answer: (a) Commentary: There is a common misperception that cerebra l p al sy is caused by injury at birth; however, the greatest risk factor for cerebral palsy is prematurity, accounting for nearly half of the cases. Low birth weight, infection and stroke are also risk factors. Birth asphyxia accounts for about 10%. Reference: McMahon M, Pruitt D, Vargus-Adams J. Cerebral pa ls y. In: Alexander MA, Matthews DJ. Pediatric rehabilitation: principles and pr act ic e. 4th ed. New York: Demos;2010. p 244. Matthews DJ. Pediatric rehabilitation: principles and pr act ic e. 4th ed. New York: Demos;2010. p 244.

139. To prevent contractures , wh ich p osi tion i s the correct placement for children with major burns?(a) Shoulder in external rot atio n(b) Wrist in extension(c) Hip in flexion(d) Metacarpophalangeal join ts i n hyp ere xtensi on

139. (b) Children with major burn injuries should be placed in positions that tend to prevent contractures. These include neck extension (no pillows); shoulders at 90° abduction and neutral rotation with elbows, wrists, hips, and knees extended; feet at neutral dorsiflexion, metacarpophalangeal joints at 70° to 90° flexion and finger interphalangeal joints in full extension.

148 Which muscle fiber is characterized by slow-twitch oxidative metabolic properties? (a) Type 2a (b) Type 1 (c) Type 2b (d) Type 3

148 Answer: (b) Commentary: There are 2 primary muscle fiber types in humans. They are categorized according to many different characteristics, including speed of contraction and sources of fuel. Type 1 muscle fibers are slow-twitch with oxidative metabolic pathways. Type 2 muscle fibers are fasttwitch fibers. The type 2 fibers can then be further divided into fast-twitch glyclolytic, and fasttwitch oxidative glycolytic. Ref: DeLateur BJ. Therapeutic exercise. In: Braddom RL, editor. Physical medicine and rehabilitation. 2nd ed. Philadelphia: WB Saunders; 2000. p 398.

15. For persons with spinal cord inju ry who su rvive the first 24 hours, what is the leading cause of death the first year post-injury?(a) Pulmonary embolism(b) Pneumonia(c) Renal insufficiency(d) Nonischemic heart diseas e

15. (b) The leading cause of death for persons with spinal cord injury who survive more than 24 hours is pulmonary dysfunction (pneumonia, adult respiratory distress syndrome) followed by nonischemic heart disease, septicemia and pulmonary embolus.

163. Which condition is a contraindication to high velocity low ampl itude man ipulation of t he cervical spine? (a) Facet arthropathy (b) Vertebral osteomyelitis (c) Discogenic pain (d) History of remote carotid endarterectomy

163. (b) When performing manual medicine, or when referring a patient to a physician for manual medicine, one must be acutely aware of what contraindications apply to the individual one is treating. The absolute contraindication to high velocity low amplitude among the options listed is osteomyelitis.

174. Which symptom of excessive exertion is NOT typical in a person with neuro muscular disease ? (a) Paresthesias (b) Hyperthermia (c) Severe muscle cramping (d) Excessive urine output (diuresis)

174 (d) Anuria or decreased urine output, rather than diuresis, is c ommon following excessive exercise. The other statements are typical of overexertion and ov erwork wea kness.

19. Which tracheostomy tube modification most enhances speech? (a) Open, cuffl ess (b) Large diameter (c) Fenestrated with cuff (d) Valved, cuffless Ref: Dikeman KJ, Kazandjian MS. Communication and swallowing management of t racheostomized and vent ilatordep enden t ad ults. 2nd ed. Clifton Park: Delmar Learning; 2003. Clinical Activity 3.1

19 (d) Tracheostomy patients are usually started with a tube specifically designed for speech with an inf lated cuff; then t hey progress to a fenestrated tube and then to a p hon ation va lve wit h a c uffless tub e. Decreasing the diameter of the tube optimizes speech and respiration.

197. Which statement about the Functional Independence Measure (FIM) is TRUE? (a) The instrument is limited by its lack of evaluation of cognitive skills. (b) It is used by inpatient rehabilitation programs to compare the outcomes of their patients with regional and national outcomes. (c) It uses a 5-point scale to rate the amount of assistance that an individual requires in various functional areas. (d) It is routinely applied only at discharge from an inpatient rehabilitation facility.

197 Answer: B Commentary: The Functional Independence Measure (FIM) is an outcomes measurement tool used by inpatient rehabilitation facilities across the country. It enables inpatient rehabilitation programs to compare their patients' outcomes with regional and national outcomes. The FIM measures an individual's functional abilities and level of assistance required in 18 separate functional areas, including cognition and communication. The FIM instrument uses a 7-point scale to rate the amount of assistance that an individual requires in each of these functional areas. The FIM can be completed at any frequency, but is typically completed at least at the time of admission and at the time of discharge from an inpatient rehabilitation facility. Ref: Christiansen CH. Functional evaluation and management of self-care and other activities of daily living. In: DeLisa JA, Gans BM, editors. Rehabilitation medicine: principles and practice. 4th ed. Philadelphia: Lippincott-Raven; 2005. p 986.

2. A post-coronary artery bypass graft rehabilitation program includes (a) primary prevention education on modifiable risk factors. (b) ambulation on the second postoperative day. (c) upper-extremity aerobic and strengthening exercisesin hospital. (d) initiation of an outpatient aerobic program on discharge. Ref: Bartels MN. Cardiac rehabilitation. In: Grabois M, Garrison SJ, Hart KA , Lehmkuhl LD, editors. Physical medi cine and rehabilitation: the complete approach. Malden: Blackwell Science; 2000. p 1435-56. Clinical Activity 1.3

2 (b) Mobilization after surgery is started and progressed as quickly as possible. Pa tients should be up in a chair the fi rst postoperative day and then are started on limi ted ambulat ion the s eco nd posto per ative day. Patients having already undergone coronary artery bypass graft (CABG) are too late for primary prevention but will benefit from secondary prevention and risk factor modification. Most CABG patients are on sternotomy precautions postoperatively and are therefore highly restricted in upper-extremity activity. Outpatient aerobic programs are usually delayed until patients are completely healed from surgery, which is usually about 6 weeks after CABG.

2 Which medication used in amyotrophic lateral sclerosis has a neuroprotective effect by inhibiting glutaminergic neurotransmission in the spinal cord? a) Riluzole (Rilutek) b) Gabapentin (Neurontin) c) Cannabidiol (Cannabis) d) Prednisone (Deltasone

2 Reference(s) Krivickas LS, Carter GT. Adult motor neuron disease. In: Frontera WR, DeLisa JA, Gans BM, Walsh NE, Robinson LR, editors. DeLisa's physical med icine and rehabilitation: principles and practice. 5t h e Philadel phia: Lippincott-Raven; 2010. p 724. Option a is correct.

2. You are consulted to see a young patient 3 days after the motor vehicle crash in which he sustained a traumatic brain injury. You note that he is not receiving nutritional support. In starting nutrition in this patient, which statement concerning enteral compared to parenteral nutrition is TRUE? (a) Enteral nutrition has a higher incidence of complications. (b) Parenteral nutrition is more likely to cause pneumonia. (c) Enteral access is easier to obtain at a higher cost. (d) No significant difference exists in measured nutritional parameters.

2 Answer: (d) Commentary: Early feeding of a person who has a traumatic brain injury is associated with fewer infections and a trend towards better outcomes in terms of survival and disability. Two trials reported the effect of route of feeding on the incidence of infection of any type, but both trials showed a trend towards more infection with parenteral nutrition (PN) than with enteral nutrition (EN). This difference might reflect catheter related infection with PN. In 3 trials reporting the effect of route of feeding on the occurrence of pneumonia, a trend towards reduced incidence of pneumonia was found in the PN group. Although it is easier to provide PN than it is to obtain adequate EN access, EN has a decreased incidence of complications and lower cost compared to PN, with no significant differences in measured nutritional parameters. Also, providing nutrition to the intestine can stimulate gut immune function and limit deterioration of the intestinal mucosa characteristic of bacterial translocation and its potential for contributing to sepsis. Ref: (a) Perel P, Yanagawa T, Bunn F, Roberts IG, Wentz R. Nutritional support for head-injured patients. Cochrane Database Sys Rev. 2006; (4):CD001530. doi: 10.1002/14651858.CD001530.pub2. (b) Kirby DF, Creasey L, Abou-Assi SG. Gastrointestinal and nutritional issues. In: Zasler ND, Katz DI, Zafonte RD, editors. Brain injury medicine: principles and practice. New York: Demos Medical Publishing; 2007. p 665.

21. Causes of isolated lateral hip pain include (a) hip joint osteoarthritis. (a) piriformis syndrome. (b) sacroiliac joint dysfunction.(c) lumbosacral radiculopathy. Ref: Bozic KJ, Rubash HE. The painful total hip replacement. Clin Orthop Rel at Res 2004;Ma r(420):18 -25 Clinical Activity 3.3

21 (d) Lumbosacral radiculopathy is a common cause of lateral hip pain. Hip joint pain usually radiate s t o the groin . Pirif ormis syndrome an d sacroiliac joint pain usual ly p res ent in t he glut ea l r egion.

24. You have evaluated a 50 year old man for l owe r extremity muscle pain and discomfort. The pain increases with jogging. You have reviewed his medications, which include simvastatin (Zocor). Baseline laboratory studies were normal 6 months ago. The creatine kinase level is mildly elevated at 185 units/L. The next most appropriate step is to (a) check thyroid stimulating hormone levels. (b) order electrodiagnostic study. (c) switch to a different class of lipid lower ing m edications. (d) continue the medication with close monitor ing o f the creatine kinase levels.

24. (d) If a patient on a statin presents with muscle complaints, with or without creatine kinase (CK) elevations, other causes, including strenuous exercise or hypothyroidism, must be considered. If a patient initially has normal or only moderately elevated CK levels, the statin may be continued with close monitoring of symptoms and CK levels; however, if symptoms become intolerable or if the CK level is 10 times the upper limits of normal (ULN) or greater, the statin must be discontinued. If myositis is present or strongly suspected, the statin should be discontinued immediately. Early diagnosis and treatment of symptomatic CK elevations, including cessation of drug therapies potentially related to myopathy, can prevent progression to rhabdomyolysis. Symptoms and CK levels should resolve completely before reinitiating therapy, at a lower dose if possible. Asymptomatic elevation of CK at 10 times the ULN or greater should also prompt discontinuation of the statin. Consideration should also be given to discontinuation of statins before events that may exacerbate muscle injury, such as surgical procedures or extreme physical exertion.Needle electromyography abnormalities are uncommon in statin-induced myopathy. An EMG does not exclude statin-induced myopathy, because it primarily affects type 2 muscle fibers. Electromyography is not routinely performed or recommended unless the clinical presentation does not improve with statin discontinuation or if concern exists about other diagnoses. 25. (c) Heterotopic ossification (HO) may develop as early as 17 days after a neurologic injury. However, it typically takes up to 6 weeks to begin to mineralize and decrease range of motion at the affected joint. Persons with spinal cord injury are prone to develop HO below their level of injury. This patient's progressive loss of range of motion accompanied by a loss of function points toward HO. With no history of trauma, early fracture is unlikely, lack of systemic signs such as fever render an abscess unlikely, and with a deep vein thrombosis (DVT) one would expect edema distal to the clot. Persons with spinal cord injury are at highest risk for DVT within the first 6 to 8 weeks after injury.

24 A patient with cancer receiving which chemotherapeutic agent should be monitored for sensory neuropathy and risk of distal pressure ulcers? a) Daunorubicin b) Bleomycin c) Vincristine d) 5-fluorouracil

24 Reference(s) (a) Posner J. Neurological complications of cancer. Philadelphia: FA Davis; 1995. p 282-310. (b) Cheville AL. Cancer rehabilitation In: Braddom RL, editor. Physical medicine and r ehab ilitation. 4t h e Philadel phia: Elsevier; 2011 p 1385. Option c is correct. Of the chemotherapeutic agents listed, only vincristine can produce neuropathy. Daunorubicin can cause cardiac toxicity and bleomycin can produce pulmonary fibrosis.

29. Which finding is normal in newborn infants?:(a) Extensor tone predominates:(b) Hands are kept fisted:(c) Spine is straight when held in sitting position:(d) Unable to turn head to side in prone position

29 (b) In normal newborn infants flexor tone predominates and hands are kept fisted. In prone position a normal newborn is able to turn the head to either side . T he n ewbor n h as a rounded spi ne when pl ace d i n su ppor ted sitting.

38. Which modification can be used to increase the sensitivity of repetitive testing in neuromuscular junction disorders? (a) Decrease the muscle temperature below 30°C. (b) Allow the muscle to be rested for 6 minutes. (c) Test the most distal muscles in the feet. (d) Test the muscles after inducing ischemia.

38 Answer: (d) Commentary: In postjunctional disorders, such as myasthenia gravis, the proximal muscles seem to be more affected and it is thought that this difference is due to the increased temperature as one gets closer to the core of the body. The higher temperatures potentiate a reduced safety factor. Hence, the sensitivity of detecting a decrement response is higher with proximal muscle testing. Maximal exercise can help in demonstrating postactivation exhaustion. When repetitive stimulation is normal in both proximal and distal muscles, testing under ischemic conditions can demonstrate a decrement in the responses. Ref: Dumitru D, Amato AA. Neuromuscular junction disorders. In: Dumitru D, Amato AA, Zwarts MJ, editors. Electrodiagnostic medicine. 2nd ed. Philadelphia: Hanley & Belfus; 2002. p 1158-62.

39. The most common complication after amputation in the immature child is (a) phantom limb pain. (b) diffuse edema. (c) terminal overgrowth. (d) painful neuroma.

39 (c) Terminal overgrowth at the transected end of a long bone is the most common complication after amputation in the skeletally immature child. It occurs most frequently in the humerus, fibula, tibia, and femur, in that order. The oppositional growth may be so vigorous that the bone pierces the skin. The treatment of choice is surgical revision.

39 A 22-year-old patient presents with left lower limb pain . Plain radiogra phs r eve al a tu mor in th e s haft of th e f em ur. You are c on cerned tha t he will su stain a pat hol ogi c fr acture if a. 25% of the circumference is affected. b. the lesion length is 1 centimeter. c. 50% of the circumference is affected. d. the lesion length is 2 centimeters.

39 Option c is correct. An increased risk of fracture exists when cortical bone destruction affe cts 5 0 or mo re of th e b one 's c ircumf ere nc e, m ore tha n 60 % of i ts d iame ter, mor e tha n 1 .3c m o f t he l ength in the femoral neck, or more than 2.5cm of the lower limb's long bone is involved.

4 Children with burns have good physical and psychological outcomes. Heterotopic ossification is rare in children. Contractures around head, hands, and axilla more commonly require surgical rele ases in children. A pattern of burns on the feet, buttocks and perineum of a 1-year-old child is suggestive of a. Sunburn b. Playing with matches c. A house fire d. Nonaccidental trauma

4 Reference(s) Massagli T, Engel JM. Special consideration for pediatric patients with disability due to traum In: Robinson LR, editor. Trauma rehabilitation. Philadelphia: Lippincott Williams & Wil kins; 2006. p 3 4 Option d is correct.

45. In preparation for discharge from your rehabilitation unit, an i ndividual with T12 ASIA A paraplegia requests information regarding access to a home with 5 steps. You suggest a ramp that, for every 1 inch in elevation, should have a length of (a) 12 inches. (b) 8 inches. (c) 16 inches. (d) 24 inches. 46. Which adjuvant analgesic medication acts at ion channels to stab ilize neu ronal cel l mem bra nes?

45. (a) Ramps must have a 12-inch length for every 1-inch rise in elevation. This ratio is a minimum requirement. A ramp must often be longer for an individual with a higher level spinal cord injury to be able to independently navigate it. When space allows, a longer, less steeply inclined ramp is preferable.

48 What is the most commonly reported health concern of adu lts with cerebra l pal sy? a. Depression b. Seizures c. Falls d. Pain

48 Option d is correct. Pain is the most reported complaint in adults with cereb ral palsy; seizu res a nd fal ls a re les s c omm only repor ted c once rns , fo ll ow ed by cont ract ure a nd osteo art hri tis . D epre ssion is more likely to be a complaint of adults with Down syndrome.

"

48. What is the total duration of the motor uni t action potential shown above? (a) 8 ms (b) 12 ms (c) 18 ms (d) 22 ms" 48. (c) The total duration of a motor unit is measured from the initial deflection from baseline to the final return to baseline. Ref: Wiechers DO. Normal and abnormal motor unit potentials. In: Johnson EW, editor. Practical electromyography. 2nd ed. Baltimore: Lippincott Williams & Wilkins; 1988. p 28-9.

5. Severity of peripheral arterial disease is classified according to (a) frequency and severity of lower-extremity claudication. (b) coexisting coronary artery disease and angina. (c) peripheral edema and pulses.(d) ankle-brachial index. Ref: Clinical Activity 1.5. Arch Phys Med Rehabil Vol 87, Suppl 1, March 2006 Clinical Activity 1.6

5 (d) The ankle-brachial index compares systolic blood pressure (SBP) at the ankle to brachial SBP an d i s used to c lassify the severity of peripheral arterial disease.

56. An HMO case manager ques tion s the po tentia l benefits of inpatient rehabilitation for a patient with metastatic cancer. You explain that the presence of metastases(a) extends inpatient rehabi lita tion sta ys.(b) decreases durable medica l eq uipme nt costs.(c) precludes autonomous mob ilit y and se lf-car e.(d) does not impact achievem ent of re hab ilitat ion goals.

56. (d) In a retrospective chart review it was found that neither the presence of metastatic disease, nor the need for ongoing anticancer therapy (eg, chemo- or radiation therapy delayed the achievement of rehabilitation goals or extended rehabilitation hospital stay.

"

57. The term maximum medical improvement (MMI) indicates that (a) no deterioration of the condition is expected to occur. (b) no further treatment for the condition is required. (c) no further treatment is reasonably expected to improve the condi tion. ( d) physi cian and patient agree that the condition has stabilized." 57 (c) Maximum medical improvement (MMI) is a term used by a physician to indicate that he/she has determined that no further significant recovery of a condition is anticipated to occur. Resolution of the condition may or may not occur. Determining MMI does not indicate that no treatment is indicated.

6. A withdrawal syndrome produced by abrupt medication dose reduction or the administration of an antagonist drug is referred to as (a) tolerance. (b) pseudoaddiction. (c) physical dependence. (d) addiction.

6 (c) Physical dependence is a pharmacologic property of many drugs caused by the body's physiologic acclimation to their presence. In the case of opioids, withdrawal will develop in tolerant patients if the drug is not tapered.

61. Which muscle does NOT depress the scapula?:(a) Serratus anterior:(b) Rhomboid major:(c) Latissimus dorsi:(d) Pectoralis minor

61 (b) The rhomboid major elevates, retracts and causes medial (downward) rotation of the scapula, but it does not depress the scapula. All the other muscles d epr ess the s cap ula.

62 Which finding is frequently associated with sensory neuronopathies? a) Symmetric onset b) Rheumatoid arthritis c) Paraneoplastic syndrome d) Impaired mobility secondary to weakness

62 Reference(s) Martinez A, et al. Sensory neuronopathy and autoimmune disease. Autoimmune Diseases 2012;1-6. Option c is correct. Sensory neuronopathies are uncommon and affect large sensory fibers. The onset is typically asymmetric and gait abnormalities are secondary to impaired prop rioception as opposed to weakness. T he d ifferential d ia gnoses inc lude paraneoplastic syndrome, toxins and Sjögren's syndrome but not rheumatoid arthritis.

63 The mother of a 7-year-old ambulatory child with cerebral palsy (CP) has just visited with the pediatric orthopedic surgeon. He has suggested a single-event multilevel surgery (SEMLS) for her chil This surgery w ould consi st of tendon Achilles lengthenings (TAL), femoral varus derotation osteotomies (VDO) and rectus femoris transfers. She is worried that this is too much to do at once and wants your opinion. Your advice is that? a) he should have serial Tendo Achilles lengthenings (TAL) surgeries, not a singleevent multilevel surgery. b) femoral varus derotation osteotomies (VDO) should not be performed until after puberty. c) single event, multilevel surgery (SEMLS) is the most helpful orthopedic procedure for ambulatory children with CP older than 6 years. d) split posterior tibial tendon transfers (SPLTT) would be the surgery of choice in this case.

63 Reference(s) (a) McGinley, JL, Dobson F, Ganeshalingam R, Shore BJ, Graham HK. Singleevent multilevel surgery for children with cerebral palsy: a systematic review. Dev Med Child Neurol 2012; 54(2 ):117-28. (b) M cMahon M, Pruit t D, Vargus-Adams J. Cerebral palsy. In: Alexander MA, Matthews DJ, editors. Pediatric rehabilitation. 4th e New York: Demos Medical; 2010. p 183. Option c is correct. Multiple procedures during a single surgical event is shown to have less morbidity and better long-term outcomes than single procedures done over multiple, separate, surgical events often span ning many years. Si ngle-event mult ilevel surgery (SEMLS) is defined as 2 or more bony or soft tissue procedures at 2 or more anatomic levels performed during a single surgical procedure. Multiple muscles and joints are targeted when function is the primary goal, because all these structures are interrelated for walking.

64 In a patient with neuromuscular disease, which pulmonary function parameter best represents abdominal and chest wall strength? (a) Tidal volume (b) Maximal inspiratory pressure (c) Peak cough (d) Maximal expiratory pressure

64 Answer: (d) Commentary: The maximal inspiratory pressure reflects diaphragm strength and ventilatory ability. Maximum expiratory pressure is indicative of abdominal and chest wall muscle strength and the ability to cough and clear secretions. The tidal volume represents the normal volume of air displaced between normal inhalation and exhalation when extra effort is not applied. Peak cough flow is a measure of the amount of air flow that a patient can generate during a volitional cough. Ref: (a) Weinberger SE, Rosen IM. Disorders of the respiratory system. Harrison's online [cited May 27, 2009] http://www.accessmedicine.com.ID=2884861&searchStr=pulmonary+function+test#2884861. (b) Panitch, H. The pathophysiology of respiratory impairment in pediatric neuromuscular disease. Pediatrics 2009;123;S215-8.

65. What advantage of a reciprocating gait orthosis applies for a person with T10 A SIA cl ass A spi nal c ord inj ury?:(a) The wearer ambulates with a relatively stable 4-point gait.:(b) It accommodates adductor spasticity through the cable mechanism.:(c) It allows longer steps than with traditional knee-ankle-foot orthoses.:(d) Gait is smooth because the pelvis is never stationary during the gait cycle.

65 (a) The advantages of the reciprocating gait orthosis (RGO) include enabling the wearer to ambulate with a relatively stable 4-point or 2-point crutch gait. Th e ca ble m ech anism prevents t he patient fr om taki ng a n u nduly long s tep that woul d ma ke bal anc e recove ry difficult . T he to rso i s stab il ized with a n ort ho sis, whi ch ma y als o ben efi t pa tien ts who have th ora cic spinal cord injury. Ambulation with RGO is associated with a lower pulse rate and a lower heart rate than with other trunk-hipknee-ankle-foot ortheses (THKAFOs). Velocity of the ambulation is somewhat faster and patients appear to be able to manage longer distances than with other THKAFOs. The disadvantage of the RGO is that donning is time consuming. The gait is slow and is not smooth because the pelvis is stationary momentarily during each gait cycle. The orthosis is heavy. Marked spasticity of the adductors disturbs the operation of the cable mechanism.

65. You are told by a physical therapist that your patient with acut e C5 ASIA A tetrap legia is having difficulty breathing, but only when sitting upright. Appropriate lab tests and radiologic studies are unremarkable. To address the patient's breathing difficulty, you suggest (a) bilateral above-knee compression stockings. (b) a tilt table program. (c) intermittent positive pressure breathing treatments. (d) the use of an abdominal binder.

65. (d) In the acute complete tetraplegic patient there is a lack of abdominal muscle tone. An abdominal binder can help because when the patient using it sits upright the abdominal contents are pulled caudally and also are pushed inward. This action pushes up on the diaphragm and allows it to start in a position of mechanical advantage when inhaling while upright. Compression stockings and tilt table programs can be useful for a decrease in blood pressure that accompanies upright posture. Orthostatic hypotension is associated with lightheadedness, dizziness, nausea, syncope. Intermittent positive pressure breathing can be useful to decrease atelectasis.

66. An elderly patient with breast cancer and widespread osseous metastases has dev elop ed ac ute on set, seve re low t hor aci c back p ain w hile st oo ping forward to lift groceries. In addition to formulating a pain management program and initiating bisphosphonate therapy, what brace do you prescribe for this patient?:(a) Jewett:(b) Knight-Taylor:(c) Williams:(d) Chairback

66 (a) The patient has developed a vertebral compression fracture. An extension brace will minimize her pain and possibly reduce further pathological compressi on frac tures . T he Kn ight-Taylor b race pro vid es thor acol umb ar spine con trol in the s aggi tal an d c oronal p lan es. The W ill ia ms brace is a lumbosacral extension-lateral control orthotic. The chairback brace is an example of a lumbosacral flexion-extension control orthosis.

66. A 42-year-old man with human immunodeficiency virus (HIV) presents with proximal muscle weakness, myalgia, and weight loss. His creatine phosphokinase (CPK) is elevated. What is the most likely cause? (a) HIV myopathy (b) Fibromyalgia (c) Antiretroviral medications (d) Vacuolar myelopathy

66 Answer: A Commentary: HIV myopathy commonly presents with proximal muscle weakness, myalgia (in 25%-50% of cases), and weight loss. Vacuolar myelopathy causes spinal cord dysfunction, such as paraparesis, ataxia, posterior column sensory loss, spasticity, and neurogenic bowel and bladder. CPK would not be elevated in fibromyalgia or as a result of antiretroviral medications. Antiretroviral medications are associated with neuropathies, not myopathies. Ref: Levinson SF, Fine SM. Rehabilitation of the individual with HIV. In: Physical medicine and rehabilitation: principles and practice. 4th ed. DeLisa JA, Gans BM, Walsh NE, editors. Philadelphia: Lippincott Williams and Wilkins; 2005. p 1802-4.

66 The best prognostic indicator for non-invasive ventilation in patients with amyotrophic lateral sclerosis (ALS) is a) functional vital capacity. b) residual volume. c) tidal volume. d) total lung capacity.

66 Reference(s) Gonzalez P, Cuccurullo S. Pulmonary rehabilitation. In: Cuccurullo SJ, editor. Physical medicine and rehabilitation board review. New York: Dem os Medical; 2004. p 594. Option a is correct.Functional vital capacity is the best prognostic indicator for non invasive ventilation in patients with ALS. Patients may lose vital ca pacity at a rate of 1,000 mL or more per year. When v it al capacit y fal ls to 25 mL/kg of body weight, the ability to cough is impaired, increasing the risk of aspiration pneumoni

66. You are conducting an ex erci se to ler ance t est on a patient 2 months after successful heart transplant. Relative to a patient with similar clinical characteristics following coronary artery bypass grafting, you would rely more heavily on monitoring which parameter?(a) Heart rate(b) Perception of pain(c) Intracardiac pressure vi a Sw an Ga nz(d) Electrocardiogram change s

66. (d) The transplanted heart is denervated and therefore cardiac ischemia does not cause pain. Because vagal tone is lost, the resting heart rate following transplant is close to 100 beats per minute. Exercise induced increase in heart rate is blunted and peak heart rates are generally 20% to 25% lower than age-matched controls. Swan Ganz monitoring is not required.

67. A 45-year-old secretary comes in complaining of right hand numbness that began 6 weeks ago, and her symptoms are beginning to bother her at night. After performing a physical exam you diagnose her with carpal tunnel syndrome. Which treatment is shown to improve the symptoms of carpal tunnel syndrome for up to 1 year? (a) Oral corticosteroids (b) Therapeutic ultrasound (c) Wrist/hand splint (d) Tendon glide maneuvers

67 Answer: C Commentary: Using a wrist/hand splint can improve the symptoms of carpal tunnel syndrome for up to 1 year. Therapeutic ultrasound and oral corticosteroids have been shown to provide only short-term relief. Tendon glide maneuvers have not been shown to affect the outcome of carpal tunnel syndrome. Ref:(a) Werner RA, Franzblau A, Gell N. Randomized controlled trial of nocturnal splinting for active workers with symptoms of carpal tunnel syndrome. Arch Phys Med Rehabil 2005;86:17.b) Walker WC, Metzler M, Cifu DX, Swartz Z. Neutral wrist splinting in carpal tunnel syndrome: a comparison of night-only versus full-time wear instructions. Arch Phys Med Rehabil 2000;81:424-9.

68. Which technique may redu ce s timul us artifa ct when performing sensory nerve conduction studies?(a) Increasing the impedance of the r eco rding electrodes(b) Increasing the stimulus dura tion(c) Rotating the anode aroun d th e cat hod e(d) Decreasing the low frequ ency filt er

68. (c) Rotating the anode around the cathode can decrease stimulus artifact. The other choices have no effect, or increase it.

70 In prosthetics, K levels are used to describe or define (a) activity levels. (b) prosthetic feet. (c) funding levels for prosthesis. (d) etiology of amputation.

70 Answer: A Commentary: K levels are used to describe activity levels These K0-K4 designations are guidelines for prosthetic components covered by Medicare. Ref: Gitter,A. Bosker G. Upper and lower extremity prosthetics. In: Physical Medicine and Rehabilitation. ed De Lisa, J et al. 2004 p 1330-1.

76. Which is the most common neuropsychological dysfunction after a liver transplant? (a) Seizures (b) Encephalopathy (c) Stroke (d) Depression

76 Answer: B Commentary: In a study by Ghaus et al, 62% of liver transplant patients developed encephalopathy. Seizures occurred in 11% and stroke in 9%. In another study by Rothenhausler, 3% of transplant patients had depression. Ref: Zafonte RD, Pippin B, Munin M, Thai N. Transplant medicine: a rehabilitation perspective. In: Physical medicine and rehabilitation: principles and practice. 4th ed. DeLisa JA, Gans BM, Walsh NE, editors. Philadelphia: Lippincott Williams and Wilkins; 2005. p 1922.

77 The current workers' compensation system in the United States is a "no fault" system. This means that the (a) employee does not have to prove that the employer is at fault for the injury. (b) employer does not have to prove they are at fault for the employee's injury. (c) employee and employer do not have to prove that the other is at fault for the injury. (d) employer does not have to prove that the employee is at fault for the employee's injury.

77 Answer: (c) Commentary: In the United States workers' compensation system the injured worker does not have to prove that the employer is at fault for the employee's injury. Similarly, the employer does not have to prove that the injured worker is at fault for his/her injury. If the injury occurred at work, the medical costs and partial payment of lost income are covered. Ref: Foley BS, Buschbacher RM. Occupational rehabilitation. In: Braddom RL, editor. Physical medicine and rehabilitation. 3rd ed. New York: Saunders Elsevier; 2007. p 1048.

78. During the electromyographic evaluation of a patient, you note discharges consistent with myotonia and small motor units in the distal muscles of the upper and lower extremities. The most likely diagnosis is (a) paramyotonia congenita. (b) myotonic dystrophy. (c) myotonia congenita. (d) hyperkalemic periodic paralysis.

78 (b) Although myotonic discharges may be seen in all of these disorders, myotonic dystrophy is the only one with low-amplitude motor units in the distal muscles.

79. Your 14-year-old patient with spastic diplegic cerebral palsy has increasi ng probl ems with spas tic it y. He wal ks wi th ankl e- foot ortho se s ( AFO s) and c rut che s a nd is independent in his activities of daily living. Which medication would reduce his spasticity while minimizing undesirable side effects? (a) Diazepam (Valium) (b) Baclofen (Lioresal) (c) Dantrolene (Dantrium) (d) Oxybutynin (Ditropan)

79 (b) Diazepam has lethargy and sleepiness as major side effects. Dantrolene works at the level of the muscle and often causes weakness, which can interfere with funct ion. Ox ybutynin relaxes the muscles of the bladder, not skeletal muscles.

79. The family of your 15-year-old patient who ha d a severe traumatic brain injury 6 weeks ago asks you if they may feed their son. You observe that the patient is agitated at times, has a hoarse voice, and drools. You try to feed him applesauce and notice that he seems to swallow part of it and does not cough. The most likely finding on the videofluoroscopic swallowing study will be (a) Silent aspiration (b) Reflux (c) Coughing and gagging (d) Normal swallow

79. (a) The lack of coughing in a patient with neurologic impairment when presented with food may mean a normal swallow, but is more likely to mean silent aspiration. A normal videofluoroscopic swallowing study is unlikely in a patient with a TBI who is drooling and hoarse. Hoarseness may be a sign of reflux, but in a child with a TBI is more likely to mean vocal cord abnormality. Ref: Smith C, Hill J. Language development and disorders of communication and oral motor function. In: Molnar GE, Alexander MA, editors. Pediatric rehabilitation. 3rd ed. Philadelphia: Hanley & Belfus; 1999.p 72-8.

8. One advantage of a concentric needle compared to a monopolar needle is its (a) higher amplitude of motor unit action potentials(MUAPS). (b) decreased likelihood of electrical interference. (c) ability to vary the recording surface size. (d) longer duration of the MUAPs.

8 Answer: (b) Commentary: The shaft of a concentric needle serves as the reference electrode, whereas an additional electrode (typically a surface electrode) is needed as a reference when using a monopolar needle. MUAPs recorded from monopolar needles are slightly longer in duration and have higher amplitude, since they record from the entire area around the needle tip rather than only from the fibers facing the bevel. Because the concentric needle shaft serves as the reference electrode, the recording surface size is fixed and interference from surrounding muscles is minimized. Reference: (a) Daube JR, Rubin DI. AANEM Monograph #11: needle electromyography. Muscle Nerve. 2009;39:249. (b) Preston DC, Shapiro BE. basic overview of electromyography. In: Preston DC, Shapiro BE, editors. Electromyography and neuromuscular disorders. 2nd ed. Philadelphia: Butterworth-Heinemann; 2005. p 164.

8. Which safety practice is the most appropriate when performing an electrodiagnostic study on a patient in a hospital bed? (a) The device should be turned on after the placement of electrodes on the patient. (b) An insulated extension cord should be used to connect the power line. (c) More than 1 ground electrode should be attached to the patient. (d) All electrical devices in contact with the patient should share a common ground.

8 Answer: (d) Commentary: It is important to have all the electrical devices that are in contact with the patient plugged into the same outlet to share a common ground. Similarly, only 1 ground electrode should be used on the patient. To avoid power surges, the device should be turned on prior to the application of any electrodes to the patient and turned off after the removal of electrodes. Extension cords can increase leakage currents and should be avoided. Ref: Dumitru D, Zwarts MJ. Instrumentation. In: Dumitru D, Amato AA, Zwarts MJ, editors. Electrodiagnostic medicine. 2nd ed. Philadelphia: Hanley & Belfus; 2002. p 956.

80. Which muscle is primarily responsible for clearance of the leg d uring swi ng phase? (a) Iliopsoas (b) Hamstrings (c) Quadriceps (d) Tibialis anterior

80. (d) Midswing is the continuation of the passive pendulum action of the leg. Foot clearance is maintained by activity of the tibialis anterior.

9 A 28-year-old male firefighter sustained deep dermal burns across his lower face, neck, anterior chest, and shoulders. To help manage the formation of hy pertr oph ic scars, you recommend a) corticosteroid injections directly into localized, early hypertrophic scars. a. b) compression garments to be worn 12 hours a day. b. c) topical silicone to large areas of hypertrophic scar. c. d) ultrasound treatments with passive stretching.

9 Option a is correct. Commentary: Corticosteroid injections directly into localized, early hypertrophic scars can be useful, especially in highly cosmetic locations (face or n eck) or in scars that are very pruritic. Compression garments should be worn 23 hours a day until wound erythema begins to abate, usually about 12-18 months after injury. Topical silicone, applied as a sheet, is effective in the management of small areas of hypertrophic scar. In a prospective randomized double-blind study, the effectiveness of ultrasound with passive stretching versus placebo ultrasound with passive stretching showed no difference in joint range of motion or perceived pain between the 2 treatment groups. This finding suggests that, although widely used, ultrasound may not have a beneficial effect on contractures that form secondary to hypertrophic scarring. Reference: (a) Ahn ST, Monafo WW, Mustoe TA. Topical silicone gel for the prevention and treatment of hypertrophic scar. Arch Surg 1991;126(4):499-504. ( b) Es sel man PC, Thombs BD, Magyar-Russell G. Burn rehabilitation: state of the science. Am J Phys Med Rehabil. 2006;85(4):383-413. (c) Ward RS, Hayes-Lundy C, Reddy R, et al. Evaluation of topical therapeutic ultrasound to improve response to physical therapy and lessen scar contracture after burn injury. J Burn Care Rehabil 1994;15:74-9. (d) Helm PA, Kowalske K, Head M. Burn rehabilitation In: DeLisa JA, Gans BM, Walsh NE, editors. Physical medicine and rehabilitation: principles and practice. 4th ed. Philadelphia: Lippincott Williams & Wilkins; 2005 p 1884-5

91. Regarding anterior shoulder dislocation, which statement is the most correct?:(a) The most common neural injury is a lower trunk brachial plexus injury.:(b) Return to full contact sports is allowed only after isokinetic values are 100% of n or mal .:(c) To prevent capsular adhesions, Codman exercises are allowed starting 48 hours p osti nj ury . ( d) Initi al r eha bilit ati on (1-3 wee ks) s houl d a vo id active external rotation past 45°.

91 (d) Following anterior shoulder dislocation, the most common neural injury is damage to the axillary nerve. Gentle pendular exercises (Codman exercises) are no t al lowed un til w eeks 2-3 po st injury. Ear ly post redu cti on rehabilit ation should avoi d acti ve external ro tation pa st 45 ° f or at least 3 weeks and usual ly up to 6 week s. Ex te rna l r otat ion wit h a bduct ion is no t allow ed until 8 weeks postinjury. Return to contact sports is not allowed until isokinetic values reach 85% of normal.

91. A 32-year-old male runner presents with le ft fo ot pain 3 days after completing a marathon. If the patient has exquisite tenderness to palpation at the location of the examiner's right thumb, pictured above, what is your next step? (a) Order physical therapy for closed chain ex erc is es. (b) Order a plantarflexion night splint. (c) Implement strict non-weight bearing. (d) Prescribe oral steroids.

91. (c) The patient has a navicular stress fracture until proven otherwise. It is among the most common stress fractures in athletes. Physical examination reveals tenderness over the so-called "N" spot between anterior tibialis and extensor hallicus longus tendons as well as pain with weight bearing and hopping. Strict non-weight bearing cast immobilization for 6 weeks should be implemented. Magnetic resonance imagining or bone scan can confirm the diagnosis. Ref: Brukner P, Khan K. Clinical sports medicine. 2nd ed. Roseville, NSW, Australia. New York:McGraw-Hill; 2002. p 588.

92. A patient with focal right upper extremity spasticity initially demonstrated a good r esp ons e t o ele ctro myo graph ica lly =-guided bot ul inum to xi n injections, as measured functionally and on the Modified Ashworth Scale. He received 2 sets of follow-up injections, each 6 months apart, when spasticity returned. He returns 4 weeks after his most recent injection, complaining that he has not seen any effect. Of the options given, what is the most likely explanation for this lack of effect?:(a) Diffusion characteristics of botulinum toxin change with repeated administratio n.:(b) It becomes very difficult to localize spastic muscles with repeated administrat ion.:(c) Antibodies have developed to botulinum toxin, neutralizing it.:(d) The hepatic enzymes that metabolize botulinum toxin have increased with repeate d ad mi nis tra tio n.

92 (c) Diffusion characteristics do not change. Electromyographic guidance helps to ensure that the botulinum toxin is injected into the targeted muscle. Local iza tion does no t bec ome more di ff icult wi th rep eate d in jec tions when s pasticity ret urns . Botu lin um toxin is not meta bol iz ed by th e live r. The develo pment o f neutra lizin g ant ib odi es is t he p rim ary reas on for lo ss of e fficacy with repeated injections.

"43.

A 72-year-old woman underwent right total knee arthroplasty 2 days ago. When you see her in consultation, she tells you that she has numbness along the lateral portion of the incision site. What is the most likely cause? (a) Femoral or peroneal nerve injury (b) Deep vein thrombosis (c) Cutaneous nerve injury (d) Temporary side effect from anesthesia " 43 Answer: (c) Commentary: Cutaneous sensory loss is a very common complication following primary total knee arthroplasty. One study from 1995 found that 100% of patients had lateral skin flap numbness, and more recent studies in 2004 and 2009 found 81%-86% of patients had lateral skin flap numbness. In most cases, the numbness does improve with time (50% recovered in 2 years in the 2009 study). Deep vein thrombosis (DVT) and common peroneal nerve palsy are other known complications of total knee arthroplasty. References: (a) Borley NR, Edwards D, Villar RN. Lateral skin flap numbness after total knee arthroplasty. J Arthroplasty. 1995Feb;10(1):13-4. (b) Hopton BP, Tommichan MC, Howell FR. Reducing lateral skin flap numbness after total knee arthroplasty. Knee. 2004 Aug;11(4):289-91. (c) Subramanian S, Lateef H, Massraf A. Cutaneous sensory loss following primary total knee arthroplasty: a two years follow-up study. Acta Orthop Belg. 2009 Oct; 75(5):649-53.

"53.

A forty-year-old woman with rheumatoid ar thr it is (RA) complains of right wrist pain that limits her ability to use her computer and phone at work as a computer analyst. On exam, she has metacarpal phalangeal ulnar deviation, wrist radial deviation, and several boutonniere deformities in her fingers. There is no active synovitis. You suggest occupational therapy and (a) oral prednisone. (b) short forearm cast. (c) a resting wrist orthotic. (d) finger splints." 53. (c) Resting wrist splints provide light support for a painful joint and are well tolerated. They are the most commonly prescribed orthotic in RA. Ref: Biundo JJ, Rush PJ. Rehabilitation of patients with rheumatic disease. In: Ruddy S, Harris ED, Sledge CB, editors. Kelley's textbook of rheumatology. 6th ed. Philadelphia: W.B. Saunders Company; 2001. p 769.

"125. Individuals with spinal cord injury who are at the highest risk of developing blad de r c anc er have as a ri sk fa cto r:(a) multiple urinary tract infections.:(b) indwelling Foley catheter.:(c) history of bladder calculi. (d)

ASIA class A." 125 (b) Bladder cancer is the fifth most common neoplasm and the twelfth leading cause of cancer mortality in the United States. Known risk factors for bladder can cer inclu de male gender, smo ki ng, occu pat ion al e xpos ure to aromatic amines and s chis tosomi asi s. In st udi es of spi nal c ord inju ry fea tu ring age-ma tched a nd gende r-adj usted s tan dar dize d da ta, ho wever , b lad der cancer has generally been found to be far more prevalent. When looking at independent variables, which point to a higher risk of bladder cancer, only bladder management method and age at spinal cord injury significantly predicted bladder cancer. ASIA classification, level of spinal cord injury and a history of bladder calculi did not contribute significant ly. Ris k of bla dder cancer is t he highest in in divi dual s w ho have used an indwellin g ca theter s f or longe r t han 10 ye ars . The rela tive r is k of bladde r can ce r from i ndwel ling ca the ter use is rel ati vely unc han ged when a djusted for smoking status. Multiple urinary tract infections is not a risk factor.

"12.

An 18-year-old man was in a high-speed motor vehicle collision 24 hours ag o. He re quired a prol ong ed extr icat ion f ro m his v ehicl e and l ost co ns ciousn ess at th e scene of the accident. Head computed tomography (CT) scan was notable for a small subarachnoid hemorrhage. He has had several episodes of hypotension and hypoxemia since admission. What information in this clinical case makes diffuse axonal injury highly likely? (a) High-speed motor vehicle collision (b) Subarachnoid hemorrhage on head CT scan (c) Episodes of hypoxia and hypotension (d) Prolonged extrication from vehicle" 12 (a) Diffuse axonal injury is most commonly seen after high-speed motor vehicle collisions, particularly when immediate loss of consciousness occur.

"41.

An 8-year-old boy presents with an insidious onset of diffuse r ight elbo w pain. Physi cal examination reveals loss of full right-elbow extension. Radiographs reveal bone fragment contiguous with the capitellum. Your advice to the patient and his parents is (a) reassurance that this condition is self-limiting. (b) referral to an orthopedic surgeon for surgery. (c) start a weight-training program of his upper body. (d) short-arm casting of the elbow for 4 to 6 weeks." 41 (a) This patient likely has Panner's disease, or developmental osteochondrosis of the capitellum. This is a self-limiting condition that does not need surgery or casting. Weight-training in children should not be commenced until, at least, reaching puberty.

"96.

Cooling can produce physiological changes in the body. One of these changes is an increase in (a) nerve conduction rate. (b) stretch receptor sensitivity. (c) elasticity of connective tissue. (d) general sympathetic activity. " 96 Answer: (d) Commentary: Nerve conduction rate is slowed by cooling. The stretch receptor sensitivity in muscles and tendons is reduced, and the elasticity of connective tissue diminishes with cooling. The general sympathetic activity is increased with cooling of the body, and this may affect the responses of the stretch receptors in a beneficial way. Reference: Lemons DE, Riedel G, Downey JA. Thermoregulation and the effects of thermoregulation. In: Gonzalez EG, Myers SJ, Edelstein JE, Lieberman JS, Downey JA, editors. Downey and Darling's physiological basis of rehabilitation medicine. 3rd ed. Boston: Butterworth Heinemann; 2001. p 518.

"90.

In a transtibial amputee, ambulation with a prosthe sis, inst ead of unilateral non-weight bearing (with crutches) results in (a) higher rate of energy expenditure. (b) lower heart rate. (c) higher respiratory exchange rate. (d) equivalent amounts of energy to walk the same distan ce." 90 (b) Transtibial amputees have a lower rate of energy expenditure, heart rate and oxygen consumption when using a prosthesis (vs. non-weight bearing crutch gait). The cardiovascular demand of crutch walking is high, with increased rate of oxygen consumption, increased heart rate, increased energy costs, and respiratory exchange rate in the anaerobic range.

"182. Which statement is TRUE about the relative response s of the brai n and the spinal cord after concussive trauma? (a) The brain is more sensitive to trauma than the spina l cord. ( b)

The spinal cord is more sensitive to trauma than the brain. (c) The brain and the spinal cord are equally sensitive to trauma . (d) The brain's neurologic recovery is less predictable than the spin al cord's in its response to a given amount of trauma." 182. (b) Concussive injuries of the spinal cord are more varied in gradation than injuries to the brain. Seemingly mild spinal concussions, seen most frequently in cervical hyperextension, may lead to complete tetraplegia, even in the absence of penetration of the spinal canal or even vertebral fracture. Mild concussive trauma to the brain results in a more mild brain injury and a more severe concussive trauma to the brain results in a more severe neurologic dysfunction.

"83. Regarding the mechanism of nonsteroidal anti-inflammatory medications, which st atem en t i s TRUE?:(a) They are required for the synthesis of prostaglandins.:(b) Cyclooxygenase 2 protects the gastric mucosa.:(c) Cyclooxygenase 2 is primarily induced by inflammatory stimuli. (d)

They inhibit syn th esi s o f a rachi doni c a cid." 83 (c) Nonsteroidal anti-inflammatory medications inhibit the cyclooxygenase enzyme (COX) which is necessary for prostaglandin and thromboxane synthesis. At le ast two isof orm s of cyclooxygen as e have b een id enti fied , n amed COX 1 a nd COX 2. COX 1 i s in t he gastric muc osa where th e pro stagl andin sy nthesis has a pr ot ective e ffect ; COX 2 is pr imar ily ind uce d by inf lam mat ory sti muli and is the target of the new COX 2 inhibitors, which have less gastric toxicity.

"30.

What acquired upper extremity amputation is most common in adults? (a) Dominant extremity at the transradial level (b) Dominant extremity at the transhumeral level (c) Non-dominant extremity at the transradial level (d) Non-dominant extremity at the transhumeral level" 30 (a) Acquired upper limb amputations in adults occur most commonl y in males between the ages of 21 and 64 years. These amputations result frequently from wo rk-related accide nts or trauma and are most common in the dominant limb at the trans ra dial l evel. In contras t, c ongeni tal upper limb deficiencie s o ccu r mos t c ommo nl y on the left sid e at th e tran sra dia l l evel.

"42.

Which is the most significant risk factor for a stroke? (a) Smoking (b) Hypertension (c) Age (d) Diabetes " 42 Answer: (c) Commentary: Age is the single most important risk factor for stroke, worldwide. The incidence of stroke for both males and females doubles for each decade after age 55. Stroke is more prevalent in men than women, except for the age cohort of 35-44 (a finding considered to be due to the use of oral contraceptives and pregnancy) and among persons over age 85. Hypertension is the most important modifiable risk factor for both ischemic and hemorrhagic stroke regardless of age. A family history of stroke increases the risk of stroke by about 30%. Cigarette smoking is an important risk factor and doubles one's risk of ischemic stroke and triples the risk of subarachnoid hemorrhage. Other well-documented risk factors include diabetes, dyslipidemia, and atrial fibrillation. Reference: a)Goldstein LB, Bushnell CD, Adams RJ, Appel LJ, Braun LT, Chaturvedi S, et al. Guidelines for the primary prevention of stroke: A guideline for healthcare professionals from the American Heart Association/American Stroke Association. Stroke 2011;42:517-584. b) Brandstater ME. Stroke rehabilitation. In: DeLisa JA, Gans BM, Walsh NE, editors. Physical medicine and rehabilitation: principles and practice. 4th ed. Philadelphia: Lippincott-Raven; 2005. p 1657-1659. c) Zorowitz R, Baerga E, Cuccurullo S. In: Cuccurullo S, editor. Physical medicine and rehabilitation board review. New York: Demos Medical; 2004. p 1.

"49.

Your 6-month-old patient had burns to his head and both arms in a house fi re. What approxi mate per ce nt of his tota l body su rface area ( TBS A) wa s burn ed? (a) 37 (b) 18 c. 27 (d) 49" 49 (a) An infant's head is approximately 19% and each arm constitut es 9% of the total body surface area (TBSA). In adults and older children the head is appro ximately 9 % of th e TBSA.

"178. The potential above is (a) a fibrillation. (b) an end plate spike. (c) a voluntary motor unit. (d)

a fasciculation." 178 (d) The potential is of a duration consistent with a motor unit and fires only once over a 500-ms period. The firing rate would be inconsistent with a volu ntary moto r unit but is typical of a fasciculation. The amplitude and duration of th e potent ial would also p recl ude an en d plate spike or fibril lat ion p ote nti al. Fu rt hermore , a fibr il lat io n pote nti al wou ld ha ve an in it ial pos iti ve de fle ct ion .

"170. Type 1 (alpha) error is best described as oc cu rring in research when (a) the study finds a positive benefit from th e i nt ervention, but no benefit really exists. (b) the study finds no benefit from the interv ent io n, but a benefit really exists. (c) study results are biased by subject select ion c riteria. (d) study results are not generalizable outsid e t he research setting.

" 170. (a) Type 1 (alpha) error is best described as occurring in research when the study finds a positive benefit from the intervention, but no benefit really exists. In this case, the study conclusions would be falsely positive. Ref: Fletcher RH, Fletcher SW, Wagner EH. Clinical epidemiology: the essentials. 3rd ed. Williams & Wilkins; 1996. p 186-8.

149. A 16-year-old girl who had a severe traumatic brain injury 4 weeks ago with le ft f ro nta l c ont usion and le ft ba sil ar skull fr act ur e de mon st rates worse auditory than visual attention. Which diagnostic test would be most likely to explain this finding?:(a) Magnetic resonance imaging:(b) Electroencephalogram:(c) Audiogram:(d) Visual evoked response

"148 (a) The presence of an insensate thumb and index finger with a normal median sensory nerve action potential (SNAP) recorded from the index finger is indicat ive of a les ion prox imal to the d orsal ro ot gan glio n, s uch as root avu lsion. Positi ve w aves i n t he paras pin als are u sua ll y s een 1 week po st onset. W hile th e full E MG fi nding s may no t be app are nt for 3 to 4 wee ks, the data presented are sufficient to draw conclusions."

179. A 1-year-old child with a midlength transfemoral limb deficiency presents for pros th eti c m ana gemen t. W hic h com pon ent should be in the pr os thetic prescription?:(a) Socket with a growth liner:(b) Single action knee joint:(c) Vertical shock pylon:(d) Dynamic foot

"179. (a) The 1-year-old child should be fit with a simple prosthesis which suspends securely and allows for growth. The knee joint should be added betwe en 3 an d 5 y ear s. Th e SACH foot i s most c omm onl y pr escr ibe d because of its simple d esig n and dur ability. Al l prosthe ses f or growi ng chi ld ren should incor po rate a r emova ble g ro wth li ner in t he soc ket."

180. In which case would a wrist-hand-finger orthosis utilizing dynamic metacarpoph alan ge al ext ens ion a ssis tan ce be mo st indicate d?:(a) Complete radial nerve injury at the level of the mid-humerus:(b) Partial ulnar nerve injury at the level of the elbow:(c) Complete median nerve injury in the forearm:(d) C5 level ASIA class A spinal cord injury

"180. (a) With a radial nerve injury at the level described, there is loss of wrist and metacarpal extension because of denervation of the forearm extens or musc les. A d orsal wrist-hand =-f inger or tho sis tha t ho lds the wrist, fingers, and thum b in e xte nsion an d p ermits fl exi on of the digits w ith a low-p rofil e outrigge r is used to su bst itut e fo r t his loss of fu nct ion. Th is type of orthosis would be inappropriate for the other conditions described."

"12. Which diagnosis is associated with an increased risk of post-tra umatic hy drocephal us? (a) Diffuse axonal injury

(b) Subdural hematoma (c) Epidural hematoma (d) Subarachnoid hemorrhage" 12. (d) Hydrocephalus is a well-recognized complication of subarachnoid hemorrhage. The fundamental abnormality in post-traumatic hydrocephalus is an imbalance in the production and absorption of cerebral spinal fluid (CSF). As the blood in the subarachnoid space obstructs the arachnoid villi it impairs absorption of CSF, thus causing hydrocephalus.

"72. A 26-year-old woman is admitted to the inpatient rehabilitation unit foll owing a t rauma tic brain injury. She is confused, inappropriate, agitated, and requires a padded floor bed. What is her Rancho Los Amigos level? (a) IV (b) V

(c) VI (d) VII" 72. (a) The Rancho Los Amigos level IV applies to persons who are confused and agitated. The need of a padded floor bed indicates that the patient is restless and agitated.

"44. A 68-year-old man with diabetes-related peripheral neuropathy pr esents wi th a pain ful f ore foot. Physical examination reveals a pes cavus foot with hammertoe deformity. Your initial recommendation is (a) surgical referral for correction of deformity. (b) physical therapy focusing on plantar fascia lengthening and intr insic foo t muscle stren gth ening. (c) custom-made, total-contact foot orthotic with a built-up arch an d metatar sal bar.

(d) b otulinum toxin injection of the foot intrinsic muscles." 44. (c) A well built, custom foot orthotic with a built-up arch and metatarsal bar will support and help correct the foot deformity. Due to the patient's insensate skin, the orthosis must be custom fit with even pressure distribution. The skin should be checked daily for pressure areas. Although corrective foot surgery may improve function, the initial treatment should be an orthosis. In addition to the built-up arch, the metatarsal bar is needed to help relieve the hammertoe deformity.

1 A patient with Parkinson disease is referred to you for rehabilitation recommendations, but you question the diagnosis. Which clinical feature is typical of parkinsonism and therefore increase s th e likelihood of a correct diag nosis? a) Poor vertical eye movement b) Asymmetric onset of tremor c) Rapid progression of symptoms d) Hyperreflexia

1 Reference(s) Hirschberg R, Sharma N, Scarborough DM. Rehabilitation of persons with Parkinson's disease and other movement disorders. In: Frontera WR, edito r. Physical medicine and rehabilitat ion: principles a nd practice. 5th e Philadelphia: Lippincott Williams & Wilkins; 2010. p 645-657. Option b is correct. The typical individual afflicted with Parkinson disease presents with symptoms on 1 side of the body, a pattern referred to as asymmetric onset. Progressive supranuclear palsy is characterized by the inability t o exercise volu ntary movement of the eyes. Parkinson-plus syndromes include several diseases, including multiple system atrophy (MSA), progressive supranuclear palsy (PSP) and diffuse Lewy body disease (DLBD). The symptoms of Parkinson-plus syndromes progress within the first 3 to 5 years, while it usually takes 10 to 15 years to see these symptoms in Parkinson disease. People with Parkinson-plus syndromes also have a poor response to dopamine replacement. Rigidity but not hyperreflexia is seen in Parkinson disease.

1. A 32-year-old male runner pre sents to your office with foot pain for the last 3 weeks. He reports severe pain on the bottom of his foot, which is worse with the first few steps in the morning after getting out of bed. He has no history of trauma and previously ran up to 12 miles daily. His running has been severely limited since this pain began. What is the most likely diagnosis? (a) Morton's neuroma (b) Plantar fasciitis (c) Tarsal tunnel syndrome (d) Stress fracture

1. (b) Plantar fasciitis is classically most painful upon arising first thing in the morning, and is aggravated by overuse or change in footwear. An S1 radiculopathy often presents with numbness and tingling and has associated reflex changes and possibly weakness in the plantar flexors. Tarsal tunnel syndrome is caused by compression of the posterior tibial nerve inferior to the medial malleolus. A Morton's neuroma causes plantar pain in the forefoot and is aggravated by wearing tight, restrictive shoes.

10. Comparing lower limb amputations to upper limb amputations in the United States, lower limb amputations are (a) most often due to trauma. (b) expected to significantly increase over the next 20 years due to increasing rates of diabetes mellitus. (c) less common than upper extremity amputations. (d) expected to decrease over time due to improved prenatal care leading to less congenital malformations.

10 Answer (b) Commentary: The prevalence of diabetes mellitus continues to increase in the United States and this trend is expected to cause increasing rates of lower extremity amputation. Lower extremity amputations are more common than upper extremity amputations and are more likely to be related to dysvascular causes. Despite improvements in prenatal care enabling more births, rates of amputations due to congenital defects have not changed significantly. The most common cause of upper extremity limb loss is trauma-related injury. Reference: a) Sheehan TP. Rehabilitation and prosthetic restoration in upper limb amputation. In: Braddom RL, editor. Physical Medicine and Rehabilitation. 4th ed. Philadelphia: Elsevier Saunders, 2011:257-276. b)Huang ME, Miller LA, Lipschutz R, Kuiken TA. Rehabilitation and prosthetic restoration in lower limb amputation. In: Braddom RL, editor. Physical Medicine and Rehabilitation. 4th ed. Philadelphia: Elsevier Saunders; 2011:277-316.

10 You are asked to consult on a 60-year-old cancer patient with an acute deep vein thrombosis (DVT) in the right upper limb, secondary to a long-standing c entra l v enous catheter. What therapy restrictions would you recommend for the patient? a) Bed rest for 10-12 days to allow for clot maturation. a. b) No activity restrictions, since upper limb DVTs have a low likelihood of causing a pulmonary embolus. b. c) Begin resistive exercises 12-24 hours after the patient is therapeutic on an anticoagulant. c. d) Limit therapy to ambulation, balance, and ADL training if anticoagulation is medically contraindicated.

10 Option d is correct. Commentary: Because patients with acute upper limb DVT who cannot safely be anticoagulated are at high risk for pulmonary emboli and death, their physica l, oc cup ational, or lymphedema therapy should be functional in nature (ie, ambulation, balance, ADL training). Resistive exercises should be deferred until 48 - 72 hours after a patient is therapeutic on an anticoagulant (low molecular weight heparin, unfractionated heparin, or Coumadin). Prolonged bed rest for clot maturation is no longer supported within the medical literature for lower extremity DVTs, since the initial recommendation was based on a single limited study. Although the timing of mobilization following an acute upper limb DVT and institution of therapy has not yet been defined in the literature, bed rest for 10-12 days is overly restrictive. However, placing no activity restrictions on the patient is potentially dangerous, since an immature clot may break off and embolize to the lungs. Reference: (a) Stubblefield MD and O'Dell MW, editors. Cancer rehabilitation: principles and practice. New York: Demos Medical Publishing; 2009. (b) Stubblefield MD, Pearce CK. Rehabilitation of the cancer patient: identification, evaluation, and rehabilitation of p atien ts with complications of cancer and its treatment from impending fracture to hematologic abnormalities. Paper presented at: American Academy of Physical Medicine and Rehabilitation Annual Assembly; 2011. (c) Kiser TS, Stefans VA. Pulmonary embolism in rehabilitation patients: relation to time before return to physical therapy after diagnosis of deep vein thrombosis. Arch Phys Med Rehabil 1997;78(9):942-5.

173. The presence of which factor would give a patient a poor prognosis for home di scha rg e a fte r h ip fr actu re?:(a) Being 85 years old:(b) Having coronary artery disease:(c) Being of male gender:(d) Having a nonhealing wound

173 (a) Factors associated with permanent institutionalization after hip fractures are age greater than 80 years, lack of involvement by family members, insuffi cie nt p hysic al thera py at a ski ll ed nursi ng fac ilit y, a nd pre-existing dementia.

196. What is the effect of treadmill training in Parkinson patients? (a) It has no effect on fall risk. (b) It improves quality of life. (c) It produces no change in gait impairments. (d) It reduces tremor.

196 Answer: B Commentary: A 6-week treadmill training program in patients with Parkinson disease showed that the program decreased fall risk, improved quality of life, and improved gait impairments. There was no indication of a reduction of tremors. Ref: Haerman T, Giladi N, et al. Six weeks of intensive treadmill training improves gait and quality of life in patients with Parkinson's disease: a pilot study. Arch Phys Med Rehabil 2007:88:1154-8.

197. In 2003, which diagnosis-related group (D RG) h ad the most admissions to inpatient rehabilitation facilities? (a) Stroke (b) Unilateral joint replacement in a lower ex tre mi ty (c) Amputation for circulatory disorders excep t u pp er limb and toe (d) Hip or pelvis fracture

197 (b) In fiscal year 2003, the number of admissions to an inpatient rehabilitation facility with the diagnosis related group unilateral joint replacement in a lower extremity was 124,754, stroke was 54,433, amputation for circulatory disorders except upper limb and toe was 7,200, and hip or pelvis fracture was 5,863. Ref: US Government Accountability Office. Medicare: more specific diagnoses needed to classify inpatient rehabilitation facilities. April 2005. Report GAO-05-366. Available at: http://www.gao.gov/new.items/d05366.pdf. Accessed July 4, 2007.

197. A 39-year-old factory line worker admits to not wanting to return to work . Which job char acter ist ic is a ssoc iated w ith a d elaye d ret ur n t o w or k? (a) Evening shift (b) Job monotony (c) Autonomy (d) Flexible hours

197 (b) Several job characteristics are associated with delayed retu rn to work. These characteristics include: low pay, low challenge, low control, monotony, low task enjoyment, and conflicts with a supervisor.

24. What is the major factor limiting ambulation in Duchenne muscula r dystrop hy? (a) Joint contracture (b) Weakness (c) Scoliosis (d) Restrictive lung disease

24. (b) Weakness, not contracture formation, is the major cause of loss of ambulation in Duchenne's muscular dystrophy. Contracture formation may actually assist with ambulation.

28. Which electrodiagnostic finding is more common in radiation plexopathy than in neop la sti c p lex opath y?:(a) Myokymic discharges:(b) Fibrillations:(c) Decreased motor unit recruitment:(d) Decreased amplitude of the sensory nerve action potential

28 (a) Myokymia is present in 50% of patients who have radiation plexopathy, but is rarely seen in neoplastic plexopathy. The other findings noted in both ple xop athi es, b ut predo minate in n ei ther.

3. The New York Heart Association classification of congestive heart failure ( CHF) classifie s CHF sev erity bas ed on oxy gen consumption and (a) left ventricular ejection fraction. (b) pulmonary and peripheral edema. (c) symptomatic functional level.(d) etiology and age of onset. Ref: Clinical Activity 1.3.

3 (c) The New York Heart Association classification of congestive heart failure ranks severity from c las s 1 through class 4 based on oxygen consumption and functional l evel at which s ymptoms a re present.

30. What is the most common tumor associated with LambertEaton myasthenic synd rome? (a) Small-cell lung cancer (b) Breast cancer(c) Lymphoma (d) Ovarian cancer SAE-P, Phelan

30. (a) Although many cancers have been associated with Lambert-Eaton myastheni c syndrome, by f ar the most co mmon im plicated tumor is small-cell lung cancer.

32. According to the Hunt and Hess Scale, which grade of subarachnoid hemorrhage wo uld ap ply to a patie nt w ho prese nts wi th moder ate ly sev ere h eadache, meningismus, and cranial nerve deficit?:(a) 0 (b) 1:(c) 2:(d) 3

32 (c) Grade 2 of the Hunt and Hess Scale is moderately severe headache/meningismus, no neurologic deficit, except cranial nerve palsy.

34. What is the most common initial manifestation in Parkinson's disease?:(a) Fatigue:(b) Gait difficulty:(c) Hypophonia:(d) Resting tremor

34 (d) Resting tremor and bradykinesia are the most common initial manifestations of Parkinson's disease. The other signs are less likely to be presenting comp lai nts in pa rki nsoni sm.

37. A human resource firm working with a manufacturing company inquires about personal factors and low back injuries. It is interested in matching individuals with work stations. Which of the following was found to be most important in predicting injuries? (a) Age greater than 40 (b) Obesity (c) Psychologic factors (d) Poor physical fitness

37 (c) Other than a history of previous back injury, psychologic factors were found to be more important than physical factors in predicting injuries.

39. Which sign is associated with central dysautonomia f oll ow ing severe traumatic brain injury? (a) Flaccidity (b) Hyperthermia (c) Hypotension (d) Bradycardia

39 Answer (b) Commentary: Central dysautonomia can occur acutely after se ve re traumatic brain injury. It has also been called diencephalic seizures, autonomic or neuro storming or hypothalamic dysregulation syndrome. Signs include elevated temperature with a normal fever work up, tachycardia, elevated blood pressure, rapid respiratory rate and posturing. Facial flushing and diaphoresis may also be seen. Reference: Krach LE, Gormley ME, Ward M. Traumatic brain in ju ry. In: Alexander MA, Matthews DJ. Pediatric rehabilitation: principles and practice. 4th ed. New York: Demos;2010. p 244.

5. According to the Consortium for Spinal Cord Medicine's Clinical P ractice G uidelines for the Prevention of Thromboembolism in Spinal Cord Injury, individuals with motor complete (ASIA A or B) spinal cord injury should receive (a) warfarin (Coumadin), international normalized ratio target: 2-3. (b) low-molecular-weight heparin or adjusted-dose unfractionated hep arin. (c) inferior vena cava filter. (d) unfractionated heparin, 5000 units every 12 hours.

5. (b) Clinical practice guidelines for spinal cord injury (SCI) have been established for the prevention of thromboembolism. Anticoagulant prophylaxis either with low-molecular-weight heparin or adjusted-dose unfractionated heparin should be initiated within 72 hours after SCI. Inferior vena cava filter placement is indicated in SCI patients who have failed to respond to or have contraindications to anticoagulation. Filter placement is not considered a substitute for thromboprophylaxis.

62. Which type of stroke typically results in the best overall outcome? (a) Pontine hemorrhage (b) Embolic cortical infarction (c) Anterior cerebral artery aneurysm rupture (d) Internal capsule lacune

62 (d) Lacunar strokes are typically small and very localized and, in general, have the best prognosis.

83. Which agent acts on osteoblasts to increase bone formation? (a) Calcium (b) Calcitonin (c) Vitamin D (d) Estrogen

83. (c) Bone remodeling has 5 phases; activation, resorption, reversal, formation, quiescence. Vitamin D will increase this process and acts on the osteoblasts, whereas the other choices act on the osteoclasts to stabilize bone.

"166.

A 45-year-old breast cancer pati en t c omp lai ns of dim ini shed exe rti onal tol era nc e. S he co mpleted adjuvant chemotherapy 1 year ago. Chest x-ray reveals evidence of congestive heart failure. Which agents could have caused this patient's symptoms?:(a) Cyclophosphamide (Cytoxan) and methotrexate (Rheumatrix):(b) Docetaxel (Taxotere) and bleomycin (Blenoxane):(c) Cisplatinum (Platinal AQ) and 5-fluorouracil (5-FU):(d) Doxorubicin (Adriamycin) and herceptin (Trastuzumab)" 166 (d) Doxorubicin (Adriamycin) and herceptin (Trastizumab) are increasingly common antineoplastics used in primary breast cancer treatment. Both are associate d w ith cardi ac toxic ity. Patien ts who rec eiv e e ithe r ag ent have a mult igated angiog ram (MUGA) sc an prior to the init iat io n o f che mother ap y to establ ish b as eline ca rdiac func ti on. Si gns or s ymp tom s sug ges tiv e o f cardi ac failure indicate the need for a repeat MUGA scan and referral to a cardiologist.

"69.

A 9-year-old girl with C5 ASIA A spinal cord injury sustained 2 years ag o is eval uated fo r upper extremity splinting. Which statement regarding this scenario is TRUE? (a) A resting hand splint should be prescribed for daytime use to pr eserve fu nction. (b) A wrist extension splint would be contraindicated for daytime us e because it would inte rfe re with function. (c) A mobile arm support or balanced forearm orthosis could be presc ribed to make self feed ing possible. (d) A short hand splint should be prescribed to strengthen wrist ext ensor mus cles." 69 (c) A mobile arm support or balanced forearm orthosis would enable the child with C5 ASIA A spinal cord injury (SCI) and weak arm muscles to move the arm through useful active range of motion and to position the hand for function. In patients with C5 SCI, these orthoses are typically combined with wrist extension splints or a universal or palmar cuff for feeding. A resting hand splint may be used at night to improve or maintain range of motion, but would interfere with daytime function. A short hand splint would not strengthen wrist extensor muscles, but may be useful to improve function.

"46.

A patient with advanced ankylosing spondylitis complains of increasing dys pnea. Yo u order pulmo nar y funct ion tests . Which p arame ter d o you an ti cipate wi ll dev ia te the most from normal, age-adjusted values? (a) Functional residual capacity (b) Expiratory reserve volume (c) Vital capacity (d) Tidal volume" 46 (c) Spinal flexion and extension are necessary for full thoracic expansion. Limited spinal mobility, as occurs in diffuse skeletal hyperostosis and ankylos ing spondy litis, will directly affect full respiratory capability. Functional residu al capac ity, resi dual vo lume , and tid al volu me may be decrea sed . H ow eve r, vita l ca pacity is r elat ed to i ns pira tor y c apa city an d wi ll t here for e b e mor e s ig nif icant ly affected by reduced spinal mobility.

"104.

A significant improvement in quality of life for advanced ALS patients is attributed to (a) having advanced directives. (b) invasive ventilation to prolong life. (c) earlier hospice transition. (d) placement in long-term care. " 104 Answer:(c) Commentary: Improved quality-of-life for advanced ALS patients is attributed to optimizing inhome care and early hospice transition. Hospice can ease the burden of care by the family members. Hospice organizations have guidelines for early entry into hospice during advanced stages. Most patients who undergo invasive mechanical ventilation do so emergently and often against their wishes. Invasive ventilation is not preferred in over 92% of ALS patients surveyed. Often, there is a lack of advanced directives to guide the treating team. Lack of physician communication with the patient about advanced directives is the major barrier to patients developing their advanced directives. Reference: Joyce N, Carter G. In: Braddom RL. Physical medicine and rehabilitation. 4th ed. Philadelphia: Elsevier-Saunders; 2011. p1056-7.

"9.

According to national databases of spinal cord injur y (SCI), chil dren under the age of 6 years are more likely to have which epidemiologic pattern of spinal cord injury? (a) high tetraplegia, motor incomplete, occurred in moto r vehicle acc ident (b) paraplegia, complete, occurred in motor vehicle acci dent (c) high tetraplegia, complete, caused by medical/surgic al compli cati ons (d) paraplegia, motor incomplete, caused by medical/surg ical comp lica tions" 9 (b) According to databases of the Model SCI Systems and Shriner's Hospital for Children, children under age 6 years are more likely than teenagers to sustain SCI in a motor vehicle accident. Their injuries are more likely to be T1 and lower, and they are more likely to have complete injuries.

"59.

Acquired subluxation or dislocation of th e h ip s in spastic cerebral palsy is usually due to muscular imbalance with excessive tone of which muscles? (a) Hip flexors and tensor fascia lata (b) Hip flexors and hip adductors (c) Rectus femoris and hip abductors (d) Tensor fascia lata and hip extensors" 59 (b) Progressive changes associated with hip subluxation in patients with cerebral palsy result from the effects of neuromuscular imbalance on the growth and development of the hip joint. The primary problem is spasticity and muscular imbalance, and the musculoskeletal manifestations are secondary. Soft tissue abnormalities include a muscular imbalance between the stronger flexors and adductors, and the weaker extensors and abductors. A flexion-adduction contracture also shifts the center of rotation of the hip from the femoral head to the lesser trochanter, and the proximal femur is gradually displaced upward and outward. Ref: Spiegel DA, Flynn JM. Evaluation and treatment of hip dysplasia in cerebral palsy. Orthopedic Clinics of North America 2006;37(2):185-96.

"41. A 42-year-old amateur te nnis play er compla ins of severe right elbow pain for 6 months. He has tried heat, ice, and compression wrap without relief. He denies numbness, but does report weakness in his grip, especially with his backhand. On examination, the patient has severe pain with palpation just inferior to the lateral epicondyle. Which finding will most likely be present on further examination?(a) Pain will be increased w ith ulnar de viatio n of the wrist with resisted flexion.(b) The patient's brachiorad iali s ref lex will be significantly diminished or absent.(c) Resisted wrist extension wit h a s tra ighten ed elbow will reproduce the patient's pain. (d)

An audible click will be heard with active supination of the forearm." 41. (c) This patient has lateral epicondylitis or "tennis elbow," a condition brought on by repetitive flexionextension or pronation-supination of the forearm. The pain will be increased by resisted wrist extension with the elbow at 180°. The reflexes will not be affected, nor will atrophy be noted. This is not a neurologic condition, but a myofascial one. No audible click will be heard. This might occur if the radial head is subluxing, but not in lateral epicondylitis.

"1.

An overhead athlete with shoulder pain is found to have shoulder posterio r capsule tigh tne ss. A biomechanical consequence of this tightness is (a) posterior displacement of the humeral head relative to the gleno id when t he should er is fl exing. (b) glenohumeral internal rotation deficit with range of motion less than 15° from neu tral. (c) impingement of the humeral head onto the superior glenoid labrum when the shoulder is a bdu cted less than 90°. (d) posterior tilting of the scapula in the static position and with shoulder abductio n." 1 (b) A tight posterior capsule is a common phenomenon in overhead athletes. Often rotator cuff tendinopathy results from impingement and scapular dyskinesis. Glenohumeral internal rotation deficit (GIRD) defined as internal rotation less than 15° or a greater than 25° side to side difference, is due to tight posterior capsule, teres minor, and infraspinatus. Further, Matsen and Harriman showed that a tight posterior capsule drives the humeral head anteriorly when flexing the shoulder, thus predisposing to further impingement at the coracoacromial arch. Scapular dyskinesis, due to weak scapular stabilizer muscle weakness, is typically in a position of anterior tilting rather than posterior tilting.

"85.

Based on the revised edition of the American Spinal Injury Asso ciation ( ASIA) Imp airme nt Scale, published in the year 2000, which condition would be sufficient to categorize a spinal cord injury as motor incomplete? (a) Some motor function more than 2 levels below the motor level (b) Voluntary anal sphincter contraction (c) A well-defined zone of partial preservation (d) An anterior spinal artery syndrome" 85 (b) For an individual to receive an ASIA classification of motor incomplete injury (ASIA C or D), he/she must have either voluntary anal sphincter contraction or sensory sacral sparing with sparing of motor function more than 3 levels below the motor level. The zone of partial preservation is used only in complete injuries. Individuals with anterior spinal artery syndrome are often motor complete.

"177. What is the most common cause of ulnar neuropathy in computer users?:(a) Repetitive forearm supination from manipulation the mouse (b)

Compression of th e ul na r n erv e a t the elb ow by th e a rm rest:(c) Excessive elbow extension due to a mouse placed out of reach:(d) Repetitive forearm pronation with elbow extension from mispositioned keyboard" 177 (b) Compression of the ulnar nerve at the elbow is a common occurrence among computer users. Etiologies include compression of the nerve against the chair a rmr est or co mbi ned e lbow flexio n with pro nat ion . El bow ext ension and s upination red uce tensio n o n the ul nar nerve at th e elb ow.

"166.

Delerium commonly complicates the rehabilitation of patients with advance d cancer . The mo st co mmo n cause of deler iu m in th is pa tient p opu lat io n is (a) organ failure. (b) drugs. (c) hypoxia. (d) brain lesions." 166 (b) In a series of 140 cancer patients with delerium, 59% were f ound to have some degree of phyarmacological contribution. Organ failure, fluid/electrolyte imbalance , and i nfection were also important contributing factors.

"52.

During the initial, acute evaluation of a yo un g spinal cord injury patient, which factor would make you suspicious of a concomitant brain injury? (a) Fall as the mechanism of injury (b) Female patient (c) Higher level spinal cord injury (d) African-American patient" 52. (c) The following factors, evidenced at the time of a spinal cord injury, place an individual at higher risk for a concomitant traumatic brain injury: Male sex and a higher level of spinal cord injury. Up to the age of 74 years-old, a transportation accident is the major source of traumatic brain injury (TBI) and not falls. Studies have shown a potential relationship between race and the incidence of TBI, but there are too many confounding variables and no study has shown a clear evidence of a relationship. Ref: a) Kirschblum S. Rehabilitation of Spinal Cord Injury. In: DeLisa JA, Gans BM, Walsh NE, editors. Physical medicine and rehabilitation: principles and practice. 4th ed. Philadelphia: Lippincott-Williams & Wilkins; 2005. p 1742-3. b) Zafonte RD, Elovic E. Dual Diagnosis: Traumatic Brain Injury in a person with Spinal Cord Injury. In: Kirschblum S. Campagnolo DI. DeLisa JA. Editors. Spinal Cord Medicine. Philadelphia: Lippincott Williams & Wilkins; 2002. p 261-71.

"109. You are asked to evaluate an 8-month-old child with developmental delay. On exam you find low ton e, but bris k dee p tendo n refle xes a t the kn ee s and bic eps , f ul l passive range of motion, and poor head and trunk control. This child's diagnosis is likely (a) myotonic dystrophy. (b) cerebral palsy. (c) spinal muscular atrophy. (d)

Hunter's syndrome." 109 (b) This patient presents with hypotonia, weakness, and hyperref lexia, a combination most commonly seen in central nervous system lesions such as cerebral palsy. A c hild wi th a neuromuscular disorder would not have hyperreflexia with the h yp otonia and weak ness. I n se vere c ere bral pa lsy it is common to se e ear ly hypo to ni a with bris k re fl exe s th at c han ges to hype rt onia a s the chi ld gets old er .

"27.

For injured workers with chronic low back pain, wh ich o utc ome is associated with better performance during a functional capacity evaluation (FCE)? (a) Shorter usage of temporary disability benefits (b) Lower subjective reports of perceived disability (c) Higher likelihood of sustainable work tolerance (d) Fewer recurrences of low back pain over the next 12 mon th s " 27 Answer: (a) Commentary: Functional capacity evaluations (FCEs) are c omm on ly used to determine readiness for return to work. These evaluations measure the injured worker's functional abilities relative to the physical demands required by the job. The clinical assumption is that workers who perform better during FCEs will have a lower risk of reinjures and less pain exacerbation upon return to work. One-year follow-up of patients with chronic low back pain whose FCE demonstrated performance that met or exceeded physical job requirements did not demonstrate a reduction of recurrent low back pain, improved occupational sustainability, or improved perception of disability. Better FCE performance was mildly associated with faster return to work and shorter duration of temporary disability benefits. Reference: Gross DP, Battie MC. Functional capacity eval uat io n p erformance does not predict sustained return to work in claimants with chronic back pain. J Occup Rehabil 2005;15(3):28594.

"73.

How does a weighted kypho-orthosis (Posture Training Support®) improve fun ction in patient s wit h c hr onic thor acic ky phosi s? (a) Improves posture (b) Strengthens spinal extensors (c) Promotes bone formation (d) Helps activate rectus abdominus" 73 (a) A weighted kypho-orthosis improves posture without any effec t on pain.

"114. Of the 4 disorders described below, which definitions is NOT correct?:(a) Hyperesthesia is an exaggeration of any sensory modality response.:(b) Allodynia is the experience of pain in response to a normally nonpainful stimul us.:(c) Paresthesia is a perversion of sensation, producing a perception that is abnorm al i n cha rac ter . (d)

Hyp est hesia is a diminuti on to pai nfu l stimuli." 114 (d) All the definitions are correct except for the definition of hypesthesia, which is not only diminution to painful stimuli but also diminished sensation to any senso ry modal ity. Parest he sias can al so be d escr ibe d as abnorma l sensations in t he abs enc e of sti mul ation.

"161.

In 1997, the Ameri can Ac ade my of Neuro logy (A AN) p ubl ish ed its r epo rt on man ag ement of concussion in sports. They made recommendations regarding return to play after a single sportsrelated concussion, after the patient is asymptomatic at rest and with exertion. Which recommendation is appropriate according to the AAN report defining concussion grades?:(a) Grade 1 concussion, the participant may return to play after 15 minutes.:(b) Grade 2 concussion, the participant may return to play after 2 weeks. (c) Grade 3 c on cus sio n, the p arti cip ant m ay ret urn to p lay a fter 3 we eks. (d) Grade 4 concussion, the participant may not return to play at all." 161 (a) A Grade 1 concussion has no loss of consciousness (LOC) and symptoms which resolve in less than 15 minutes. Participants may return to play only after b ein g as ympto mat ic wi th normal n eu rologic exa min atio n at re st and with exercise afte r 15 minut es provided th ere has b een o nly 1 in sult. A Grade 2 con cussi on has no LOC, but s ym pto ms last mor e t han 15 m inu tes . T hese pa rticipants may return to play after 1 week after a single insult (after 2 weeks for multiple insults). A Grade 3 concussion is characterized by LOC of any duration, either for seconds (brief) or minutes

"140.

In adult patients, what advantage does a knee disarticulation amputation have ove r a tran sfemo ral a mputa tion ? (a) Better cosmetic appearance with prosthetic fitting (b) Reduced risk of phantom pain (c) Increased ability to bear weight on the distal residual limb (d) Improved surgical wound healing" 140 (c) Compared with a transfemoral level amputation, a knee disart iculation provides increased ability to bear weight on the distal residual limb. There is n o evidence that k nee disarticulation amputations have a reduced risk of phantom pain o r have improved surgic al w ound h eal ing. Am putations at the tr ans fe mor al leve l ty pically hav e a be tte r co smet ic app ear ance wi th p ro st heti c f itt ings bec au se the d istal prosth etic socket can be tapered to be more symmetric with the opposite side. Knee disarticulation amputations also have a prosthetic knee axis of rotation that is distal compared to the intact knee.

"47.

In assisting patients returning to their pre vi ous level of work, work hardening programs can achieve return-to-work rates of (a) less than 25%. (b) 25%--49%. (c) 50%--75%. (d) more than 75%." 47. (d) Return-to-work rates of 77% can be achieved with work hardening programs. Poor outcome was associated with an increased number of treatments before the program, an increased length of time off from work; the patient's having lower satisfaction with the program, and a lawyer being involved in the case. Ref: Foye PM, Stitik TP, Marquardt CA, Cianca JC, Prather H. Industrial medicine and acute musculoskeletal rehabilitation. 5. Effective medical management of industrial injuries: from causality to case closure. Arch Phys Med Rehabil 2002;83(Suppl 1):S20.

"91.

In athletes, the etiology of thoracic outlet syndro me is mo st likely due to a) clavicle fractures. b) anatomic variations. c) repetitive overhead activity. d) hyperextension injuries of the neck." 91 Answer: (c) Commentary: The most common etiology of thoracic outlet sy nd rom e(TOS) in sports is likely related to repetitive overhead motion. Hypertrophy of sport-specific musculature may predispose to TOS. The role of anatomic variations is uncertain in TOS and may be common in patients with and without this disorder. Fractures of the clavicle and hyperextension injuries of the neck may be causes of TOS in the setting of trauma. Reference: Laker S, Sullivan WJ, Whitehill TA. Thoracic out le t s yndrome. In: Akuthota V, Herring SA, editors. Nerve and vascular injuries in sports medicine. New York: Springer; 2009. p 113-26.

"39.

In which activity should a 16-year-old girl with C5 ASIA class A spinal co rd injur y be ind epend ent w ith t he u se of a ssist iv e dev ices? (a) Self catheterization (b) Transfers to level surfaces (c) Self feeding (d) Bathing" 39 (c) While boys with C5 spinal cord injury (SCI) may learn to per form bladder self-catheterization with assistive devices, girls do not. Level transfers req uire activ e elbow and wrist extension, which would not be present in a person with C 5 SCI. S elf-feedi ng with ass istive de vices s uch as a palmar ban d c an us ual ly b e do ne by p erso ns w it h C 5 te trap leg ia.

"155.

Indications for bulle t re mo val af ter a gu nsho t w ound inj ury to the spi na l co rd in clude:(a) cauda equina location.:(b) the need to reduce the risk of lead toxicity.:(c) pain reduction.:(d) the need to reduce risk of infection." 154 (a) The patient's main weakness is proximal and she has sufficient ankle dorsiflexion and knee extension strength. Hence she would not benefit just with kne e-a nkle =-foot or those s. Reciproc al gait or tho ses are des ign ed to help c hildren with acti ve hip fl exion bu t n o hip ext ens io n. Furth ermore , both these optio ns would b e too bulk y for su ch a pat ien t t o han dle ef fic iently. A standard walker would result in a bent forward posture and would limit the patient's gait speed.

"103. What scapular motions should a patient avoid after a shoulder replacement? ? (a) Internal rotation and adduction (b)

Internal rotation and abduction (c) External rotation and adduction (d) External rotation and abduction 104. Which statement is correct regarding amyotrophic lateral sclerosis (ALS)?" 103 (d) External rotation and abduction increase risk of dislocation because of anterior instability.

"141. If the L3 and L4 medial branches of the dorsal rami are abla ted, the patient will experience blocked afferents from the (a) L5-S1 facet joint. (b) L4-5 facet joint. (c)

L3-4 facet joint. (d) L2-3 f acet join t." 141. (b) The medial branches of the dorsal rami supply innervation to the facet joints and the deep paraspinals, namely the segmental multifidi and rotators. The sacral multifidi are innervated by the sacral (rather than the lumbar) dorsal rami. Each lumbar medial branch innervates the facet joint at and below its derivation. The L4-5 facet joint is innervated by the L3 and L4 medial branches, derived from the L3 and L4 nerve roots.

"39. The family of your 10-year-old patient who had a severe traumatic brain injury You try to feed him applesauce and notice that he seems to swallow part of it and d 6 we ek s a go ask s you if the y may fe ed their so n. Yo u ob ser ve that he is agitated at times, has a hoarse voice, and drools.:(a) Silent aspiration.:(b) Reflux.:(c) Coughing and gagging. (d)

Normal swallow." 39 (a) The lack of coughing in a patient with neurologic impairment when he/she is presented with food may mean a normal swallow, but is more likely to mean si len t as pirat ion . A n ormal videofluoroscopic swallowing study is unlikely in a patient with a traumatic brain injury (TBI) who is drooling and hoarse. Hoarseness may be a sign of ref lux, but in a chi ld with a T BI it is m ore li kely to mea n vocal cord abnormality.

"40. A patient with a left transfemoral amputation demons tra te s a lateral trunk lean towards his prosthetic side. What is the most likely cause? (a) Prosthesis too long (b)

Long residual limb (c) Prosthesis aligned in adduction (d) Hip abduction contracture" 40 Answer: (d) Commentary: Causes of lateral trunk lean towards the pro sth et ic side include: prosthesis too short, hip abduction contracture, prosthesis lined in abduction, and short residual limb. Reference: Kuiken TA, Miller L, Lipschutz R, Huang ME. Reh ab ili tation of people with lower limb amputation. In: Braddom RL, editor. Physical medicine and rehabilitation. 3rd ed. Philadelphia: Elsevier; 2007 p 314.

"82. A 20-year-old man sustained a severe traumatic brain in ju ry and a femur fracture 1 week ago. Magnetic resonance imaging reveals a diffuse axonal injury with no evidence of hemorrhage or a hematoma. His condition is stable 1 day after open reduction, internal fixation of the femur fracture and he is nonweight bearing on that leg. What is the appropriate recommendation for deep venous thrombosis prophylaxis in this patient? (a) Placement of a vena cava filter (b) Sequential compression devices (c) Graded compression stockings (d)

Low molecular weig ht he par in sodium" 82 Answer: (d) Commentary: Prophylaxis for deep vein thrombosis (DVT) s hou ld be considered in all patients with a traumatic brain injury after acute admission to the hospital. Graded compression stockings are of little benefit. Thigh high intermittent compression devices help reduce DVT risk but are not an appropriate primary prophylaxis. A vena cava filter is not appropriate prophylaxis and chemical prophylaxis is needed as soon as feasible. In patients who are not fully ambulatory in 24 hours unfractionated heparin sodium is adequate and can be used 12 hours after surgery. However, in all patients who have long-bone fractures, prior DVT, or more than 4 total risk factors, low molecular weight heparin sodium should be used until the patient is fully mobilized. Reference: Cifu DX, Kreuzer JS, Slater DN, Taylor L. Re hab il ita tion after traumatic brain injury. In: Braddom RL, editor. Physical medicine and rehabilitation. 3rd ed. Philadelphia: Elsevier; 2007. p 1141.

"167. What ergonomic recommendation is appropriate for a computer user who has exces sive w ris t e xte nsion at the keyb oar d?:(a) Adjust the chair height and support the forearms horizontal to the floor.:(b) Raise the height of the mouse and position it on a mouse pad.:(c) Adjust the chair height to angle the forearms towards the keyboard. (d)

Lower t he c ha ir hei ght and use an angle d f oot support ." 167 (a) The worker at the computer keyboard may need several adjustments to avoid excessive wrist extension. Raising the chair height to allow the forearm to re mai n ho rizon tal to t he floor al lo ws the wrist to remain in a neutral position. Increasing the height of the mouse, angling the forearms towards the keyboard, and lowering the chair will all in

"62. A 19 year-old male is seen after a traumat ic br ain injury. The patient's mother is at the bedside and is asking you questions about the patient's prognosis for recovery. As you consider your response, which statement is TRUE? (a) Severe disability is unlikely if the lengt h o f coma is less than 1 month. (b) Good recovery is unlikely if posttraumatic am ne sia (PTA) lasts longer than 3 months. (c) An initial Glasgow Coma Scale score of les s t ha n 8 is associated with a poor outcome. (d)

Neuroimaging studies are not helpful to determine a patient's prognosis." 62. (b) Multiple studies have shown that age, initial Glasgow Coma Scale (GCS) score, duration of coma, duration of posttraumatic amnesia (PTA), and neuroimaging findings are correlated with outcome. All provide valuable information that the clinician can use to mark milestones, and help with prognosis, but the most powerful of these is the duration of PTA. The longer the duration of the PTA, the worse the outcome. It is unlikely for a person with PTA lasting less than 2 months to have a serious disability; however, the likelihood of a good recovery is poor if the PTA extends beyond 3 months. Length of coma is determined by the time from coma onset to the time when the patient can follow commands. On average only 7%--8% will make a good recovery if the coma lasts longer than 4 weeks, and severe disability is unlikely if the coma lasts less than 2 weeks. Although the GCS score provides a general idea about the severity of the injury, it does not by itself yield a definitive prognosis. Ref: Kothari S. Prognosis after severe TBI: a practical, evidence-based approach. In: Zasler ND, Katz DI, Zafonte RD, editors. Brain injury medicine: principals and practice. New York: Demos; 2007. p 169-99.

"16. A 50-year-old man has obstructive sleep apnea (OSA). He i s m orbidly obese and has a body mass index (BMI) of 39 kg/m². He is also complaining of chronic low back pain, which he claims limits his mobility. Which approach would best benefit him? (a) Prescribe a motorized wheelchair. (b) Prescribe modafinil (Provigil) for daytime sleepines s. (c) Schedule opioid analgesics for pain control. (d)

Or der s urg ical referral for a tracheostomy." 16 Answer: (b) Commentary: Obstructive sleep apnea (OSA) is characteriz ed by sn oring, arousals, and daytime sleepiness. Most patients with OSA are male, middle-aged, with an average BMI of 32.5 +/- 9.0kg/m2. Wheelchairs should be used only in cases of compromised mobility and powered mobility used only when no other options exist. Modafinil can be used as adjunct therapy for daytime sleepiness. Narcotic analgesics should be prescribed with caution because of depression of central respiratory drive. Positive airway pressure (PAP) delivered with continuous (CPAP) or bilevel (BiPAP) pressures can correct upper airway obstruction. If the noninvasive approach is not effective, tracheostomy may be necessary. Reference: Alba AS, Kim H, Whiteson JH, Bartels MN. Card iop ul mon ary rehabilitation and cancer rehabilitation. 2. Pulmonary rehabilitation review. Arch Phys Med Rehabil 2006;87(Suppl 1):S58.

"92.

Personality changes and/or aphasia are typical of which dementia? (a) Alzheimer (b) Frontotemporal (c) Parkinson's disease with dementia (d) Vascular " 92 Answer: (b) Commentary: Frontotemporal dementia is a neurodegenerative disease of unknown etiology with atrophy and neuronal loss in the frontal and temporal lobes of the brain resulting in a gradual and progressive decline in behavior and/or language. Overuse of stock phrases, lack of conversational initiation and echolalia are more common in frontotemporal dementia. Alzheimer disease is primarily associated with memory and visuospatial loss of function, and speech is more fluent than in persons with frontotemporal dementia. Parkinson disease with dementia is associated with symptoms of memory loss, fluctuating cognition, and visual hallucinations with spontaneous parkinsonism motor features. Persons with vascular dementia usually have a history of stroke or have focal neurologic deficits, early gait disturbance, changes in personality and mood and a history of frequent falls or unsteadiness. Reference: (a) Cardarelli R, Kertesz A, Knebl JA. Frontal dementia: a review for primary care physicians. Am Fam Physician. 2010;82(11):1372-1377.(b) Miller RM, Groher ME, Yorkston KM, Rees TS, Palmer JB. Speech, language, swallowing and auditory rehabilitation. In: DeLisa JA, Gans BM, Walsh NE, editors. Physical medicine and rehabilitation: principles and practice. 4th ed. Philadelphia: Lippincott-Raven; 2005. p 1036.

"156. A patient presents with Stage I upper extremity lymphedema related to pri mary bre ast canc er th era py . She is inter es ted i n pharm aceut ic al and d ietary tr eat men t options. Which of the following would be appropriate in the management of this patient? (a) Reduced protein consumption (b) Treatment with a diuretic (c)

Reduced salt consumption (d) Treatment with a benzopyrone." 156 (c) Moderation in salt consumption will avoid increased intravas cular volume and capillary ultrafiltration thereby reducing the likelihood of lymphedema pr ogession. Diureti cs should not be used in the management of uncomplicated lymphedema . Reduce d protein consum ptio n will po tential ly lead to hypoa lbu min em ia and red uc ed colloi d os moti c pre ss ur e of th e p las ma. T hi s wi ll i ncre ase ul trafi ltr at ion , wor sening lymph edema. Benzopyrone therapy has not been shown to benefit lymphedema related to breast cancer, and is associated with hepatotoxicity.

26. A patient with a history of cancer treated wi th chemotherapy complains that her feet feel swollen, cold, and painful. The pain is described as shooting and is rated 10/10. On examination, there is no swelling and no temperature changes, but there is hypesthesia and dysesthesia. Of the following choices, which is the most appropriate pain management for this patient? (a) MS Contin (extended release morphine sul fat e) 15 mg every 12 hours (b) Prednisone taper starting at 60 mg daily (c) Neurontin (gabapentin) 300 mg 3 times a da y (d) Naprosyn (naproxen) 500 mg twice daily

Ref: (a) Gillis TA, Garden FH. Principles of cancer rehabilitation. In: Braddom RL, editor. Physical medicine and rehabilitation. 2nd ed. Philadelphia: WB Saunders; 2000. p 1311. (b) Vargo M, Gerber LH. Rehabilitation for patients with cancer diagnoses. In: DeLisa JA, Gans BM, Walsh NE, editors. Physical medicine and rehabilitation: principles and practice. 4th ed. Philadelphia: Lippincott Williams &Wilkins; 2005. p 1774-6. (c) Dworkin RH, Backonja M, Rowbotham MC, Allen RR, Argoff CR, Bennett GJ, et al. Advances in neuropathic pain: diagnosis, mechanisms, and treatment recommendations. Arch Neurol. 2003;60(11):1524-34.

"11.

Shoulder pain in the hemiparetic shoulder due to cerebrovascular accident:(a) is commonly due to formation of heterotopic ossification in the glenohumeral jo int.:(b) occurs more frequently in flaccid hemiplegia than in spastic hemiplegia.:(c) results in complex regional pain syndrome in up to 10%-15% of patients. (d) is best t rea ted wi th re st a nd immob ili zat ion in a sl in g wh ile i n bed." 11 (c) Complex regional pain syndrome is present in up to 10%-15% of patients with stroke and shoulder pain. This condition has also been called shoulder-hand syn drom e and re flex sympathetic d ystrophy . H ete roto pic oss ification ma y occur in th e el bow or sh oulder j oin t followi ng CV A, but i t is u nc ommon. Shou lder pa in assoc iated with h emi par esis or hem ipl egia due to st roke is more often associated with spastic hemiparesis than with flaccid hemiparesis. Gentle, passive range of motion exercises should be started early. Proper positioning should begin early as well. Abduction and external rotation is the position of choice while the patient is in bed. A sling should not be used in bed.

"102. Neuromuscular electrical stimulation to treat shoul der s ubl uxation after stroke should be applied to which muscles? (a) Deltoid and supraspinatus (b) Supraspinatus and infraspinatus (c) Deltoid and trapezius (d)

Subscapularis and infrasp ina tu s" 102 Answer: (a) Commentary: Neuromuscular electrical stimulation (NMES) to th e d eltoid (mainly posterior) and the supraspinatus can decrease subluxation and reduce shoulder pain. It is required for several hours daily over several weeks to achieve clinical benefits. Reference: Harvey RL, Roth EJ, Yu D. Rehabilitation i n s tr oke syndromes. In: Braddom RL, editor. Physical medicine and rehabilitation. 3rd ed. Philadelphia: Elsevier; 2007. p 1193-4.

"60. What is the shortest functional level for a transtibial amputation?:(a) Just proximal to the tibial tuberosity:(b) Just distal to the tibial tuberosity:(c) Six centimeters distal to the tibial tuberosity (d)

Ten centimeters distal to t he t ib ial tu ber osity" 60 (b) The shortest functional amputation level for a transtibial amputation is just distal to the tibial tuberosity. Knee flexion and extension can occur with th is l evel of amput ation becau se the pat ell a t endo n an d h amstring ten don attachmen ts a re sti ll present. Co ntrol of kne e fle xion and ex te nsion of th e kne e is lost with amput at ion s p roxi mal to the tibi al tub ero sity.

"62. [ITEM WAS NOT SCORED ON 2007 SAE-R]

The largest change in bone mineral density in a hemipl egic pati ent 1 year after a stroke occurs in the (a) humerus on the paretic side. (b) proximal femur on the paretic side. (c) distal radius on the paretic side." 62. [ITEM WAS NOT SCORED ON 2007 SAE-R]

"19.

The most common spinal problem seen with achondroplasia during childhood i s (a) kyphosis. (b) scoliosis. (c) spinal stenosis. (d) low back pain." 19 (a) While scoliosis may occur in children with achondroplasia, i t is less common than kyphosis, which begins in infancy. Spinal stenosis occurs frequently in individ uals wi th achondroplasia, with 38 years being the average age of symptom o ns et. Lo w back pa in is e xtre mely f req uent in adults with ach ond rop la sia , b ut r ar e in chil dren . Pr og res si ve kyp hos is tha t occ ur s in i nf ants an d y oung chi ld ren with achondropla sia is treated with a spinal orthosis.

"132. Which statement is TRUE regarding post-stroke centr al pain? (a) Damage to the thalamus plays a central role in the p athogenes is o f central pain. (b) Amitriptyline is the drug of first choice to treat c entral pa in. (c) 80% of stroke patients with central pain develop the pain wit hin a month of their stroke. (d)

The pain usually resolves spontaneously and does not require medication." 132. The onset of central pain following a stroke occurs more than 1 month after the stroke in 40% to 60% of all patients. The pathogenesis of central pain is still largely a matter of conjecture and hypothesis. It is generally believed that damage to the spinothalamicocortical sensory pathways plays a significant role in the pathogenesis, but central pain can occur with lesions in any part of the brain. Treatment options are limited and at present amitriptyline is the drug of first choice, other drugs, including antidepressants, anticonvulsants, antiarrhythmics, and opioids may provide relief for some patients who do not respond to amitriptyline.

"40.

What is the primary benefit of using a postoperative, rigid, non-removable dressing in a new transtibial amputee? (a) Improved monitoring of postoperative wounds (b) Protection of the wound and edema control (c) Prevention of hip flexion contractures (d) Improved strength in the residual limb " 40 Answer (b) Commentary: The primary benefits of a rigid dressing include wound protection, edema control and prevention of knee flexion contractures (not hip flexion contractures). Monitoring the wound may be more difficult with a non-removable rigid dressing. The dressing should be removed for wound check regularly and if there is a concern for infection. Type of postoperative dressing has no effect on residual limb strength. Reference: Huang ME, Miller LA, Lipschutz R, Kuiken TA. Rehabilitation and prosthetic restoration in lower limb amputation. In: Braddom RL, editor. Physical Medicine and Rehabilitation. 4th ed. Philadelphia: Elsevier Saunders; 2011:277-316.

"99.

What neurological level of spina bifida is associated with active plantar flexion, cavus foot deformities and neurogenic bowel and bladder? (a) thoracic (T2-12) (b) upper lumbar (L1-3) (c) lower lumbar (L4-5) (d) sacral (S1-2) " 99 Answer: (d) Commentary: Bowel and bladder involvement is common at all levels, even in sacral lesions. Plantar flexion and inversion causes development of the cavus foot in sacral lesions. Lower lumbar lesions develop unopposed ankle dorsiflexion leading to a calcaneous foot. Upper lumbar and thoracic lesions develop ankle plantar flexion contractures due to the inability to move the ankle at all. Reference: Pico EL, Wilson PE, Haas R. Spina bifida. In: Alexander MA, Matthews DJ, editors. Pediatric rehabilitation: principles and practice. 4th ed. New York: Demos; 2010. p 204-205.

"59.

Which joints are most commonly involved in juvenile rheumatoid arthritis? (a) Shoulder, hip, fingers (b) Atlantoaxial, costomanubrum, hip (c) Sternomanubrum, shoulder, sacroiliac (d) Elbow, hip, temporomandibular" 59 (d) The elbow is involved 90% of the time in juvenile rheumatoid arthritis (JRA), the temporomandibular and hip 50% each. The shoulder is involved about 8% of the ti me in e arly JRA and about 33% later.

"134.

Which pulmonary function parameters would you expect to increase in a child with advanced neuromuscular disease? (a) Total lung capacity (b) Vital capacity (c) Residual volume (d) Expiratory reserve volume " 134 Answer:(c) Commentary: Patients with neuromuscular diseases (NMDs) demonstrate a restrictive pattern when fractional lung volumes are measured. There is a reduction in total lung capacity, vital capacity, and the expiratory reserve volume. In contrast, residual volume (the volume of air that remains in the lungs after a maximal, complete expiratory maneuver) can actually be elevated when the respiratory muscles are too weak to deform the chest wall inward to deflate the lungs fully. These patterns will be exaggerated in children with NMDs who also develop scoliosis. Forced expiratory flows are typically reduced in proportion to lung volume so that the ratio of the forced expiratory volume in the first second (FEV1) to FVC is normal or high. Reference: Panitch H. The pathophysiology of respiratory impairment in pediatric neuromuscular disease. Pediatrics. 2009;123(suppl):S215-S218.

"24. [ITEM WAS NOT SCORED ON 2007 SAE-R]

You have evaluated a 50-year-old man for lower extremi ty muscle pai n and discomfort. The pain increases with jogging. You have reviewed his medications, which include simvastatin (Zocor). Baseline laboratory studies were normal 6 months ago. The creatine kinase level is mildly elevated at 185 units/L. The next most appropriate step is to (b) order electrodiagnostic study. (c) switch to a different class of lipid lowering medica tions. (d) continue the medication with close monitoring of the creatine kin ase levels." 24. [ITEM WAS NOT SCORED ON 2007 SAE-R] Asymptomatic elevation of CK at 10 times the ULN or greater should also prompt discontinuation of the statin. Consideration should also be given to discontinuation of statins before events that may exacerbate muscle injury, such as surgical procedures or extreme physical exertion. EMG does not exclude statin-induced myopathy, because it primarily affects type 2 muscle fibers. Electromyography is not routinely performed or recommended unless the clinical presentation does not improve with statin discontinuation or if concern exists about other diagnoses.

"107. A 52-year-old house painter has completed physical therapy for shoulder p ain due to rotat or cu ff te ndono path y. He i s ind ep enden t wit h his ho me exerc ise pr ogr am and pain free with his usual activities. He has been off work for 10 weeks. You next recommend (a) return to work without restrictions. (b) a functional capacity evaluation. (c) physical therapy for a total of 14 weeks. (d)

an aerobic conditioning program" 107 (b) A functional capacity evaluation will determine if the worke r can return to his work duties on a safe and dependable basis. A painter's work responsibilities requ ire significant repetitive upper extremity activities not necessarily challenged with thera rcise. Although aerobic conditioning is important, simulation of work duti es shoul d be done before thi s pati ent return s to work. A functional capacity evaluation will determine if the worke r can return to his work duties on a safe

"51. A 14-year-old soccer player seen on the day of injury is unable to bear weight on h er ri ght fo ot. O n ex ami natio n, she has sig nif ic ant swe ll ing and mild ecchymosis laterally. The patient is tender over the lateral side of her ankle. X-rays are negative. Your initial plan of management should include:(a) a walking cast for 4-6 weeks.:(b) nonsteroidal anti-inflammatory drugs and contrast baths for 48 hours.:(c) an air stirrup for 2 weeks with weight bearing as tolerated. (d)

an ankle brace and n on- wei ght bear ing for 3-4 wee ks." 51 (c) This patient has suffered a fairly significant ankle inversion sprain. The lateral collateral ligaments are injured 85% of the time with this type of in jur y. T his p ati ent s hould be pl ac ed in an ai r s tirr up f or 2 weeks, wei ght bearing a s to lerate d, using cr utc hes as ne ede d. A cast or cas t boot is acc eptab le for 2 w eeks but u su all y i s no t re qui red . Con tra st bat hs are not indicated before 48 hours following injury.

"6. You are consulting on a 28-year-old woman w ith m etastatic cervical cancer. She is married with one young child. At this time, she requires minimum to moderate assistance with her mobility and activities of daily living. The oncology service is debating whether to discharge the patient to home with hospice care or to give her inpatient rehabilitation. You inform them that acute inpatient rehabilitation (a) improves function and quality of life desp ite t he patient being

at the end of her life. (b) is too much of a physical demand for her a nd ag ree with hospice care. (c) takes time away from the patient being wit h h er family, so hospice is preferable. (d) will help the patient to some extent, but not a s much as a patient without cancer." 6. (a) When consulted on a patient with cancer, the physiatrist must balance the need to maximize the patient's independence through rehabilitation with the desire to have the patient return home as soon as possible. Inpatient rehabilitation is useful to improve the patient's quality of life. Functional gains have been demonstrated to be significant and comparable to those gained by patients without cancer. The presence of metastatic disease does not influence functional outcome and should not preclude participation. Ref: (a) Marciniak CM, Sliwa JA, Spill G, Heinemann AW, Semik PE. Functional outcome following rehabilitation of the cancer patient. Arch Phys Med Rehabil 1996; 77(1):54-7.(b) Vargo M, Gerber LH. Rehabilitation for patients with cancer diagnoses. In: DeLisa JA, Gans BM, Walsh NE, editors. Physical medicine and rehabilitation: principles and practice. 4th ed. Philadelphia: Lippincott Williams & Wilkins; 2005. p 1771-3.

"93. A 24-year-old college student reports low bac k and lower limb joint pain for the past several months. His heels are especially painful, which makes it difficult for him to walk or stand for prolonged periods of time. He recalls an episode of gastroenteritis requiring hospitalization 6 months ago. Upon further questioning, he admits to some mild dysuria. His neurologic exam is normal. Radiographs of his ankles will most likely demonstrate (a) a normal joint. (b) osteophytes and subchondral cysts in the t ibi ot alar joint. (c) periosteal reaction and bony erosions of t he ca lcaneus. (d)

avascular necrosis of the talus." 93. (c) The diagnosis is reactive arthritis / seronegative spondyloarthropathy that develops after certain genitourinary or gastrointestinal infections, most commonly in young men. Non-gonococcal urethritis and conjunctivitis is the remainder of the clinical triad. Heel pain is one of the most frequent and distinctive clinical features, along with low back pain radiating into the buttocks. Periostitis and erosions occur in the ankle joint in individuals with a several month history of heel pain. Osteophytes and subchondral cysts are typical of osteoarthritis. Avascular necrosis is not typical. Ref: Arnett FC. Seronegative spondyloarthropathies: reactive arthritis and enteropathic arthritis. In: Klippel JH, editor. Primer on rheumatic disease. 12th ed. Atlanta (GA): Arthritis Foundation; 2001. p 245-9.

"140. An advantage of a knee disarticulation co mpa re d to a transfemoral amputation is that the knee disarticulation offers (a) more options for a prosthetic knee. (b) enhanced ability to create power during am bul at ion or running. (c) better soft tissue coverage within the zon e o f injury. (d)

better prosthetic cosmesis." 140. (b) Disarticulation results in a bulbus distal residual limb, which may complicate prosthetic fitting. Choice of prosthetic knee options for a person with a knee disarticulation, therefore, is limited and potentially excludes the newer, more advanced knee-joint designs. Benefits of a knee disarticulation over a transfemoral approach include greater tolerance to distal limb weight bearing, a longer lever arm to create power during ambulation and running, and improved sitting balance. Of note, functional outcome studies of trauma-related lower extremity amputees concluded that persons with through knee amputations had significantly poorer outcomes. These poorer outcomes are attributed to complications arising from soft tissue failure within the zone of injury. Ref: Pasquini PS, Bryant PR, Huang ME, Roberts TL, Nelson VS, Flood KM. Advances in amputee care. Arch Phys Med Rehabil 2006:87(3 Suppl1);S34-43. 141. (d) Risk factors for chronic whiplash-associated pain include presence of preexisting degenerative disc disease, preexisting loss of cervical lordosis, female gender, awkward head position at time of impact, presence of radiating pain into upper limbs, and prior history of headache. A famous Lithuanian study showed no incidence of long-term whiplash pain in a country that had no compensation system for whiplash.

"126. A 45-year-old woman with advanced acquired immunode ficiency synd rome (AIDS) presents with a gradual onset of forgetfulness and inattention without other focal neurologic deficits. Review of systems is negative for headache and fever. The most likely diagnosis is (a) toxoplasmosis. (b) cryptococcal meningitis. (c) human immunodeficiency virus (HIV) encephalopathy. ( d)

centr al n ervous system (CNS) lymphoma." 126. (c) Also known as AIDS-dementia complex, human immunodeficiency virus (HIV) encephalopathy is usually seen late in the disease course. HIV encephalopathy develops in weeks to months, whereas symptoms of toxoplasmosis and central nervous system (CNS) lymphoma are seen in days to weeks. Fever and headache, along with mental status changes, would be seen in cryptococcal meningitis. Headaches, seizures, and fatigue are commonly seen in toxoplasmosis, along with focal or non-focal neurologic signs. In CNS lymphoma, headache, confusion, memory loss, or focal neurologic signs are typically present.

"137. On the stand in a civil litigation case, a physiatrist asked about an event ca usin g an inj ury shou ld:(a) decline to answer when the worker has had no objective testing that proves a di agno si s.:(b) answer probable if he/she believes there is more than a 50% chance the event ca used t he inj ury .:(c) answer probable if he/she believes there is less than a 50% chance the event ca used t he inj ury . (d)

dec lin e to ans wer if he/s he is not th e treating physician." 137 (b) During civil litigation, the physiatrist may be asked to comment on causality of an injury. The determination that 1 condition caused another condition sho uld be de ter mined on a "reas on able deg ree of med ical ce rtainty." In this setting "pr obable " i ndicates th at the ph ysi ci an belie ves th er e is a grea ter t ha n 50% ch ance that co ndi tio n A caus ed con ditio n B . T he term "p ossible" indicates that the physician believes there is a less than 50% chance that 1 event caused another condition.

"105. A 23-year-old man with C8 tetraplegia req ues ts your opinion regarding routine urologic

evaluations after spinal cord injury. You advise that (a) an intravenous pyelogram (IVP) should be p erf or med every 1 to 2 years. (b) annual abdominal plain films are sufficien t t o detect early hydronephrosis. (c) renal ultrasound should be performed every 5 ye ars. (d) it is reasonable to wait 10 years before g ett in g his first cystoscopy." 105. (d) Renal ultrasound should be included in the annual assessment of renal function and is more sensitive for detecting early hydronephrosis than are plain films. An IVP is not required on a regular basis unless a specific indication exists, such as localizing a renal stone. Patients with indwelling catheters should have a cystoscopy after the first 10 years postinjury. Ref: Schmitt JK, James J, Midha M, Armstrong B, McGurl J. Primary care for persons with spinal cord injury. In: Lin VW, editor. Spinal cord medicine principles and practice. New York: Demos; 2003. p 237-45.

"160. A 30-year-old man who is otherwise healthy presents following traumatic b ilateral transfe moral am pu tatio ns. Based o n you r under stand in g o f t he incre ase d m eta bo lic demands associated with prosthetic ambulation, you conclude that (a) he should not be considered a prosthetic candidate. (b) he will have reduced ability to ambulate for long distances (c) cardiac stress testing is recommended prior to consideration for prostheti c fittin g. (d)

h e wil l b e able to u se pr os theti c devic es fo r tra nsf er s and sta ndi ng ac tivities only." 160 (b) The metabolic cost of ambulation (mL oxygen consumed per bod y weight per meter) is a measure of energy consumption per distance. The metabolic cost of ambulation increa ses following a lower limb amputation and the amount of increase is r elated to the l evel of the amput ati on as w ell as the amput ati on' s eti olo gy. Be ca use of this inc re ase d me tabo lic co st, pers on s wi ll h ave red uce d end ura nc e f or am bulation fol lowing an amputation. This would be especially true for an individual with bilateral transfemoral amputation(s). The person described, however, should be considered a candidate for prosthetic ambulation and, since he has no prior cardiac history or significant risk factors, cardiac stress testing would not be indicated.

"57. The presence of nonorganic physical signs in low-back pain patients as des cribed b y Gordon Waddell is intended to (a) alert the examiner that psychological issues may be contributing to pain. (b) identify patients and workers that are malingerers. (c) determine which workers would benefit from low back and abdominal exercise s. (d)

i ndicate which pa ti ents requ ire e xa minat io n by a psy ch iat ris t. (d) i ndicate which pa ti ents requ ire e xa minat io n by a psy ch iat ris t." 57 (a) Nonorganic physical signs in patients with low back pain was described by Gordon Waddell. His original paper describes the physical signs that correlat ed with ps ycholog ical data and were distinguishable from standard clinical signs of ph ysical patholog y. Wadd ell' s sign s s hould b e used to determ ine if n ono rga nic is su es may be c ontr ib uti ng t o th e p ati ent 's co mp lain ts . Not all pa tient s w it h n onorg anic physica l signs are malingerers or require psychiatric evaluation. The signs are not indicators for therapeutic modalities.

"116. A patient with advanced chr onic obs tructi ve pulmonary disease (COPD), coronary artery disease, and copious airway secretions has begun a pulmonary rehabilitation program. The program involves progressive aerobic conditioning on a treadmill. In order to enhance the efficacy of the program, you recommend all the following EXCEPT(a) nutritional counseling a nd i ncrea sed carbo hydrate intake.(b) application of positive airw ay pr ess ure br eathing.(c) low flow supplemental in tran asal oxy gen. ( d)

inspiratory resistive loading." 116. (a) Optimizing the nutritional status of patients undergoing pulmonary rehabilitation is critical for treatment success. However, inappropriately increasing carbohydrate consumption can aggravate hypercapnia.

"169. Relevant to investigational studies, the pri nc iple of justice requires that (a) harm be minimized and benefits maximized. (b) informed consent includes full disclosure of ri sks and benefits. (c) research subjects be treated fairly and se lec te d equitably. (d)

investigators respect the privacy of research subjects." 169. (c) The principle of justice requires subjects to be treated fairly and selected equitably. The principle of beneficence ensures that researchers minimize harm and maximize benefits associated with research involvement. The principle of autonomy ensures that informed consent includes full disclosure of risks and benefits and that the privacy of subjects is respected. Ref: (a) Haas JF. Ethical issues in rehabilitation medicine. In: DeLisa JA, Gans BM, editors. Rehabilitation medicine: principles and practice. 3rd ed. Philadelphia: Lippincott-Raven; 1998. p 345.(b) Lo B, Feigal D, Cummins S, Hulley SB. Addressing ethical issues. In:Hulley SB, Cummings SR, editors. Designing clinical research. Baltimore: Williams & Wilkins; 1988. p 151.

"147. Maximum medical improvement is defined as the time when the worker:(a) has completed 3 months of physical therapy.:(b) is not able to perform his/her job because of pain.:(c) is not likely to achieve further improvement of the injury. (d)

is able to retu rn t o his /he r p revio us j ob." 147 (c) Maximum medical improvement is defined as the time when no improvement is likely to further be achieved after a work-related injury. This definition is exc lusi ve of th e amo unt of time s pent in phy sic al t hera py, the worker' s subjective comp laints , o r whethe r o r not the wo rk er is ab le to re turn to his /her pr evious j ob. A work er ma y r each max imu m m edica l i mpr ove ment an d continue to have impairments that prevent return to his/her former job.

"25. Regarding spinal shock in acute spinal cord injury,:(a) duration of spinal shock is correlated with long term outcome.:(b) reflex activity typically returns over the course of days.:(c) a reliable ASIA classification can be performed during spinal shock. (d)

it is more c omm on in tetra pleg ia than in par aplegia." 25 (c) Spinal shock is a condition in which upper motor neuron sensory motor loss is associated with areflexia below the level of injury. It is a poorly define d p heno menon . R eflex activity c an often b e d ete cted by ele ctrophysiolo gic study whe n it is no t c linicall y a pparent. Ref le x a ctivi ty typ ic ally return s ove r the cour se of week s or mon ths. The pr ese nce o f s pin al shock i s of marginal prognostic significance. A reliable ASIA classification can be carried out when spinal shock is present.

"102. For a patient with hemiplegia who prefers to u se his legs and push his wheelchair backwards, the wheelchair should be configured with (a) the back edge of the seat lower than the fro nt edge. (b) a single arm drive mechanism on the non-he mip le gic side. (c) the large wheel axle plate moved to a more an te rior position. (d)

large wheels in the front and casters in the back." 102. (d) The casters should lead the rear wheels for the most common direction of travel. This will help reduce the possibility of the user flipping over when hitting an obstacle and will make the chair more directionally stable. Ref: Cooper, RA. Wheelchair selection and configuration. New York: Demos; 1998. p 204.

"63. A 60-year-old woman with rheumatoid arthritis is con cer ne d a bout her fingers being crooked. The ulnar deviation of her fingers at the metacarpal phalangeal joints is due to the rupture of the (a)

lateral retinaculum of the extensor tendon sheath. (b) central slip of the extensor tendon. (c) radial retinaculum. (d) ulnar collateral ligament." 63 Answer: (c) Commentary: Rupture of the radial retinaculum produces uln ar su bluxation of the metacarpal phalangeal (MCP) joints. Rupture of the lateral retinaculum of the extensor tendon sheath at the proximal interphalangeal joints produces swan-neck deformities. Rupture of the central slip of the extensor tendon produces boutonniere deformities. Rupture of the ulnar collateral ligament would result in radial deviation of the fingers. Reference: Guzman J. Rehabilitation of patients with rhe uma ti c d iseases. In: Braddom RL, editor. Physical medicine and rehabilitation. 3rd ed. Philadelphia: Saunders; 2007. p 787.

"162. Regarding post-traumatic seizures following brain injury,:(a) by definition, early seizures occur in the first 24 hours after an injury.:(b) prophylaxis beyond the first week postinjury does not prevent late seizures.:(c) an association exists between late post-traumatic seizures and alcohol use. (d)

mos t sei zur es are o f th e g enera liz ed tonicocl oni c type ." 162 (b) Early seizures occur within the first week. Late seizures are either simple partial or complex partial. Alcohol is not a risk factor for developing late po st-t rauma tic seiz ures.

"24. A 17-year-old boy from India presents with a longstanding history of areflexia and as ymm etr ic muscu lar atr ophy aft er a febril e i ll ness as a child. The likeliest site of neurologic pathology is:(a) myelin sheath of peripheral nerves.:(b) axons of peripheral nerves.:(c) anterior horn cells. (d)

muscle membrane." 24 (c) Poliomyelitis involves the anterior horn cells. During an acute infection, the virus is transported to the anterior horn cells followed by inflammation and los s of spi nal a nd bulbar m ot or neuro ns.

"38. In patients with steroid myopathy, the needle electromyographic study usually reveals (a) small motor unit action potentials (MUAPs) with early recruitment. (b) small MUAPs with reduced recruitment. (c) positive waves and fibrillation potentials in proximal muscles. (d)

normal MUAPs and normal recruitment. " 38 Answer: (d) Commentary: Needle examination in patients with steroid myopathy usually reveals normal insertional activity and no abnormal spontaneous activity. Motor unit potential morphology and recruitment do not reveal any abnormalities. This combination occurs because in steroid myopathy the type 2 muscle fibers are preferentially affected, in contrast to the first-recruited type 1 fibers. Reference: Dumitru D, Amato AA. Acquired myopathies. In: Dumitru D, Amato AA, Zwarts MJ, editors. Electrodiagnostic medicine. 2nd ed. Philadelphia: Hanley & Belfus; 2002. p 1397-8.

"16. A pulmonary rehabilitation patient has a temperature of 101.5° and is brea thing at a rate of 10 br ea ths p er m inute . In or de r to optim iz e h is pe rcent hem ogl obi n saturation at a given O2 partial pressure, it would be best to (a) administer an antipyretic and encourage him to breathe more rapidly. (b) not treat his fever, but encourage him to breathe more rapidly. (c) administer an antipyretic and encourage him to maintain his current respiratory rate. atory ra (d)

not t rea t his f ever and en coura ge him to ma in tai n h is curre nt res pir at ory rate. (d) not rea t his f ever and en coura ge him to ma in tai n h is curre nt res pir at ory rate." 16 (a) A number of different factors have the capacity to shift the hemoglobin-oxygen dissociation curve. Acidosis, elevated temperature, and increased PCO2 a ll cause t he curv e to shift to the right. Thus, controlling this patients fever and en courag ing him t o expir e mo re rap idl y there by reducing PCO2 an d a ci dos is and wo ul d incre ase the de gre e of hem ogl obi n s atura ti on a t a give n O 0.583333333333333 artia l p re ssu re.

"21 A 22

old female gymnast presents to your clinic after a patellar dislocation during practice. She was treated in the emergency room with reduction of the pate lla and immobilization. Radiographs and magnetic resonance imaging of the knee are negative for fracture or evidence of osteochondral lesions. You choose to treat her with immobilization for 2 weeks and then begin physical therapy. The most appropriate therapy recommendation is to focus on improving a) flexibility of gastrocnemius-soleus complex. b) strength of the iliopsoas. b. c) flexibility of the biceps femoris. c. d) strength of the vastus medialis." 21 Option d is correct. Commentary: Physical therapy in this patient should focus on strengthening of her medial quadriceps muscles and restoration of normal patellar motion. Su rgery in select instances addresses realignment of the patella by a lateral retinacular release and/or medial retinaculum repair when torn. Reference: Diduch D, Scuderi GR, Scott WN. Knee injuries. In: Scuderi GR, McCann PD, editors. Sports medicine: a comprehensive approach. 2nd ed. Philadel phia: El sevier; 2005. p 376-7.

"41 A 40

old painter presents to your office 2 weeks after falling off scaffolding at work. He complains of knee pain, swelling, occasional locking and give-way sensation, although he has not fall en s ince the acci de nt. The ca se ma nager asks if the painter can return to work. Your recommendation includes: a) return to sedentary work while diagnostic tests are done. b) return to work if the magnetic resonance image shows a normal anterior cruciate ligament. c) transfer to the emergency room to be assessed for a septic knee. d) return to light duty that includes ladder climbing." 41 Reference(s) (a) Nadler SF, Stitik T, Malanga G Optimizing outcome in the injured worker with low back pain. Crit Rev Phys Med Rehabil 1999;11:139- 69. (b) Foley BS. Occupational rehabilitation. In: Braddom RL, editor. Physical medicine and rehabilitation. 4th e Philadelphia: Saunders Elsevier; 2011. p 1034 Option a is correct. The painter's history implies a meniscus tear. Because you know the painter must climb ladders, and stand and bend for long periods of time, further evaluat ion is in order to assess the best t reat ment options. T he patient 's me chanism of injury and symptoms do not suggest an infectious etiology, making a septic joint unlikely. Return to a modified job may facilitate healing while still encouraging the patient to continue to work.

"32. A 30-year-old man with a recent traumatic brain injury has frequent episod es of em esis wit h gas tro st omy t ube bolus f eedin gs desp ite r ec eiv ing a gents to fac ili ta te gastric emptying. The most appropriate next course of action is to (a) switch the tube feeding formula. (b) switch to continuous tube feedings. (c) order a gastric endoscopy. (d)

place a jejunostomy tube." 32 (b) Intolerance to feeding can be related to increased gastric d istention, and adjusting from bolus to a slower rate with longer feeding time may provide r elief. Con verting to a jejunostomy is appropriate if simpler measures fail.

"104. A 17 year old woman was involved in a mot or ve hicle crash 4 months ago. She suffered a shoulder dislocation. Electromyographic studies have confirmed a brachial plexus injury to her posterior cord and indicate nerve continuity (Sunderland 2 injury). Although she has completed 4 weeks of occupational therapy, she has had no improvement in her strength from baseline. Your next step would be to (a) reassure the patient and continue to monit or fo r improvement. (b) continue occupational therapy for 4 additi ona l weeks. (c) initiate neuromuscular electrical stimulat ion t o the affected muscles. (d)

refer the patient to neurosurgery for exploratory surgery." 104. (d) With closed nerve injury as described, early active and passive range of motion (ROM) therapy of the affected joints is begun. The value of electrical stimulation is uncertain. The purpose of surgical repair is to improve peripheral nerve recovery and eventual function. Therefore, surgery is done when the patient has an incomplete loss of function but shows no improvement over several weeks, or no return of function at 2 months for peripheral nerve and 4 months for a brachial plexus injury. Findings at the time of surgery help establish a prognosis. However, the chances of successful surgical repair begin to decline by 6 months after the injury. By 18--24 months, the denervated muscles usually are replaced by fatty connective tissue, making functional recovery impossible. Ref: Thomas MA, Felsenthal G, Fast A, Young M. Peripheral neuropathy. In: DeLisa JA, Gans BM, Walsh, NE, editors. Physical medicine and rehabilitation: principles and practice. 4th ed. Philadelphia: Lippincott Williams & Wilkins; 2005. p 908-9.

"147. A 62-year-old ice cream truck driver fell getting out of his truck and su ffered a medial menis cal t ear. Afte r 4 w ee ks of p hysic al th er apy , h e has 95 ° o f k nee f lexion, and the knee effusion has resolved. He is able to perform his activities of daily living with minimal pain. A functional capacity evaluation determines that he is able to perform duties at a medium level category. His job requires him to lift 75 pounds at each delivery site. You recommend (a) return to work with a lifting restriction of 20 pounds. (b) continue in physical therapy to increase knee flexion to 115°. (c) undergo a work hardening program with re-evaluation in 3 weeks. (d)

return to work with a derot ati on knee bra ce." 147 (c) The functional capacity evaluation indicated he can lift up to 50 pounds safely but his job requires that he lift 75 pounds so further intervention bef ore return to wor k is indicated. A work hardening program is an interdisciplinary pr og ram th at will i ncorpor ate work-s pec ific ac tivities and con dit ion in g t o t rans it io n the w orke r fr om ac ut e care to a saf e ret ur n to w or k. A lth oug h gai nin g kne e fle xion may be beneficial, it is not the major focus for return to work.

"154. A 37-year-old woman with polymyositis complains of difficulty walking and occa sion al fa lli ng. On p hysi cal exam ina tio n, she h as mu scle st re ngth: iliopsoas 2/5, gluteus maximus 4/5, quadriceps 4+/5, hamstrings 3+/5, anterior tibialis 4/5, gastrocnemius/soleus 4/5. Range of motion is normal. You would prescribe:(a) bilateral forearm crutches.:(b) knee-ankle-foot orthoses.:(c) reciprocal gait orthosis. (d)

standard adult walker" 153 (a) The World Health Organization defines osteoporosis as bone mass 2.5 standard deviations below normals. A history of fracture is not part of the definiti on.

"145. A 27-year-old man with a T12 ASIA A spinal cord injury for 10 years presents w ith ri ght sh oul der p ain tha t is wor se with use , p ar ticu lar ly when reaching and doing transfers. He plays basketball twice weekly. Recommendations should include:(a) no wheeling or transfers for 2 weeks.:(b) immobilization of the elbow and shoulder.:(c) electrodiagnostic study of the upper extremity. (d)

strengthening of the scapul ar s ta bil ize rs." 145 (d) Shoulder and neck pain are common following spinal cord injury (SCI). The pain may arise from the neck, shoulder, girdle, or the glenohumeral joint. Pai n m ay b e a s ymp tom o f post-trau ma tic syri ngo mye lia or a ma nifestation of cervical d isc degene rat ion. The pr evalence of sh oul der p ain in p ersons with SCI is estimat ed to be 3 0 to 50 %. R otat or cuf f tea r, bur sit is, ten donitis and impingement have all been reported. While the diagnosis of these disorders is similar to that in the able-bodied population, the treatment is not. In a person with SCI and upper limb pain, rest is often not possible. Pain is often related to overall posture and poor biomechanics. Strengthening of scapular stabilizers can help to correct this imbalance. Immobilization should be avoided. Pain relief is the focus, and may include: relative rest (not to interfere with a person's independence), medications, injections, icing, ultrasound, transcutaneous electrical nerve stimulation, and/or accupuncture.

"77. A 55-year-old long-distance truck driver is recoveri ng from a wor k related low back injury that occurred during lifting. The worker has completed 2 weeks of physical therapy and continues to have low back pain, lower extremity pain, and paresthesias. The employer calls you and is upset that you have restricted the worker from truck driving during the treatment phase, citing that "driving is sedentary work." You recommend that the driver refrain from truck driving because (a) a minimum of 4 weeks of physical therapy will be ne cessary t o fa cilitate recovery. (b) low back pain has been found to be more frequent in people ex pose d to whole body vibration. (c) workers with low back pain should not sit while sym ptoms of radi culopathy are present. (d)

the employer is unlikely to follow the restrictions you recommend." 77. (b) Whole body-vibration is associated with increased frequency of low back pain. Some studies have found a correlation between increased frequency of disc protrusion and occupational driving. The exposure to vibration will likely facilitate continued symptoms in this worker, and relative rest is indicated during the initial stages of recovery. There is no predetermined length of physical therapy that is associated with recovery. Workers with low back pain and leg pain must learn to sit without increasing symptoms. Complete avoidance will not necessarily improve recovery and is not practical. The driver can likely perform some duties with restrictions. The employer has the responsibility to provide a job that meets the restrictions set by the physician. If the employer is unable to provide a job with these restrictions then the employee must remain off work.

"82. You are seeing a 56-year-old male patient in consult ation 3 d ays after a severe stroke. He is medically stable and has flaccid hemiplegia with poor sitting balance. He is sitting up in a chair for 2 hours twice daily and has just started bedside physical therapy (PT) and occupational therapy (OT). You recommend (a) continued bedside therapy with OT and PT, focusing o n sitting bal ance, followed by transfer to your inpatient rehabilitation unit when he can sit and stand with minimum assistance. (b)

transfer to your inpatient rehabilitation unit to start aggressive PT and OT. (c) transfer to a subacute rehabilitation center to allo w the pat ient time to improve with less intensive therapy. (d) that his OT start functional electrical stimulation to the fl acci d arm to enhance neurologic recovery." 82. (b) Early and aggressive therapy addressing the higher level skills of gait, higher order functional skills, and problem solving were associated with better outcomes in a multi-center observational study.

"175. You are performing urodynamic studies on an individual with a T10 ASIA class A spi na l c ord in jury sust ain ed 2 yea rs previous ly. H e pe rfo rm s intermittent catheterization every 6 hours and reports no episodes of urinary incontinence between catheterizations. You find that the filling pressure or leak point pressure is 20cm of water. At this time, you recommend:(a) oxybutinin (Ditropan).:(b) initiation of Credé maneuver.:(c) continuation of current bladder program. (d)

urecholine (Bethanechol)." 175 (c) The primary risk factor for serious urologic complications such as vesicoureteral reflux and deterioration of renal function in persons with detrusor ex ter nal sphin cte r dys synergia is t hat of e lev ate d in trav esi cal pressure . For this re ason , it i s e ssential to determin e u ri nar y sto rage a nd leak point pres su res. In gener al, d am age to the upp er uri nary tra ct can be avo ided if voiding pressure is maintained less than 60cm of water while the maximum filling pressure or leak point pressure should be less than 40cm of water. For this patient who is having no episodes of leaking between his every 6-hour catheterizations, continuation of current program is reasonable. If on follow-up, his leak point pressure exceeds 40cm of water, conversion to a low pressure system via anticholinergics would be reasonable. The use of a cholinergic agonist (Urecholine) would be contraindicated, since this agent may increase the intravesical pressure.

"45. Your patient has a C6 ASIA class A spinal cord injury which he sustained 8 week s ag o. He ha s b een n onco mpl iant abo ut attendin g t he rapy . T od ay he refuses to participate in therapy because he states he has a headache. The nurses report poor urine output from the Foley catheter in the last 3 hours. You order:(a) intravenous bolus of normal saline.:(b) push oral fluids and go to therapy.:(c) replacement of catheter. (d)

visit from peer mentor." 45 (c) Autonomic dysreflexia must be ruled out. A Foley kink, or plugged catheter can distend the bladder, causing autonomic dysreflexia with headache (and, ul tim atel y, hy per tensi on, piloere ct ion and flu shi ng). The ca theter shoul d be checked for twists an d kinks and be flush ed. I f u rine/ flush re turn is poo r, th e catheter shou ld be c han ged .

"139. A 3-year-old boy with upper lumbar spina bifida has bilaterally dislocated hips on x-ray during a routine follow up visit. You advise his parents that he (a) needs surgery to relocate both his hips. (b) should have at least one hip relocated, surgically. (c) can still stand and walk despite the hip dislocations. (d)

will not be able to sit comfortably in a wheelchair. " 139 Answer: (c) Commentary: Due to the imbalance between hip flexors/adductors and hip extensor/abductors, hip dislocation occurs in 30%-36% of children with upper lumbar spina bifida. Unilateral hip dislocations tend to cause pelvic obliquity, difficulty with wheelchair positioning and standing, and surgery is advocated. However, bilateral hip dislocations do not generally require surgical intervention, because a level pelvis and good range of motion are more important for ambulation than located hips. Reference: Pico EL, Wilson PE, Haas R. Spina bifida . In: Alexander MA, Matthews DJ, editors. Pediatric rehabilitation: principles and practice. 4th ed. New York: Demos; 2010. p 215-216.

41. Which injury level is the most common location for a n osteopo roti c vertebral compression fracture? (a) Upper thoracic spine (b) Middle thoracic spine (c) Thoracolumbar junction (d) Middle lumbar spine

41. (b) The most common location for vertebral compression fractures due to osteoporosis is the midthoracic spine, followed by the thoracolumbar junction. If fractures are seen at other levels, a higher degree of suspicion for a pathologic (due to cancer) fracture should be raised.

"104. When using ultrasound, the production in a sound fi eld o f g as bubblesthat grow and collapse producing high temperatures and tissue damage is called (a) standing waves. (b) oscillatory movement. (c) acoustic streaming. (d) unstable cavitation.

" 104 Answer: (d) Commentary: Unstable cavitation refers to bubbles that c ont in ue to grow in size and then collapse. The high temperatures and pressures generated by this can produce platelet aggregation, localize tissue damage and cause cell death. The physiologic effects of ultrasound can be divided into thermal and nonthermal effects. Nonthermal effects include cavitation, media motion (acoustic streaming, microstreaming) and standing waves. Reference:Weber DC, Hoppee KM. Physical agent modalities . In : B raddom R L, editor. Physical medicine and rehabilitation. 3rd ed. Philadelph ia: S aun ders-Elsevier; 2007. p 466.

"54. Your patient is having difficulty writing or performing tasks in certain positions. He has an asymmetric tremor that is present only with movement. You suspect which type of tremor? (a) Essential (b) Primary orthostatic (c) Cerebellar (d) Dystonic

" 54 Answer: (d) Commentary: Suspect a dystonic tremor if it is very asymmetrical and shows postural dependence. There may be associated dystonia elsewhere in the body. The tremor may have task specificity, such as writing or postural dependence. A cerebellar tremor will often have other cerebellar signs present and often improves if the patient performs movement with eyes closed. Essential tremor is symmetrical and best seen with an outstretch hand. Primary orthostatic tremor presents as unsteadiness or tremor in the legs that increases with prolonged standing, and the symptoms are relieved by sitting or walking. Reference: Harris MK, Shneyder N, Borazanci A, Korniychuk E, Kelley RE, Minagar A. Med Clin North Am. 2009;93(2):371-388.

86. A 41- year-old African-American man had an orthotopi c h ea rt transplant 2 months ago. He has started outpatient cardiac rehabilitation, 3 times a week. Compared to an age-matched individual with a normal heart, which finding do you expect in this patient when he exercises? a) Lower resting heart rate b) Higher oxygen consumption c) Slower ability to reach maximal heart rate d) Higher peak heart rate during maximal exercise

"86 Answer: (c) Commentary: A transplanted heart is denervated, and has a hi ghe r than normal resting heart rate due to loss of vagal tone. It also has lower oxygen consumption during submaximal exercise than that of the normal heart. It achieves a maximal heart rate more slowly than a normal heart, and the peak heart rate achieved during maximal exercise is considerably lower in cardiac transplant recipients than in age-matched controls. Reference: Young MA, Stiens SA. Organ transplantation an d r eh abi litation. In: Braddom RL, editor. Physical medicine and rehabilitation, 3rd ed. New York: WB Saunders; 2007. p 1438-41."

124. What is the median post-diagnosis survival in amyotrophic lateral scleros is? (a) 1 year (b) 2.5 years (c) 5 years (d) Too variable too predict

124 (b) Survival rates vary depending on the patient's decision to u se mechanical ventilation and a feeding tube. Nonetheless, the median 50% survival rate is 2.5 years and by 5 years postdiagnosis the overall survival rate is only 28%.

"66. A 52-year-old woman with a history of non- alc oh olic steatohepatitis underwent a recent liver transplant. Her protein and albumin levels are very low and, on exam, she has anasarca. Your inpatient rehabilitation admission orders should include (a) referral for paracentesis. (b) nursing orders to avoid use of an abdomina l b in der. (c) high protein diet with high protein oral s upp le ments. (d) oxandrolone and monitoring of liver enzymes.

" 66. (c) Malnutrition is significant in patients with liver disease. Ascites can promote excessive protein loss. Patients should receive a high protein diet with high protein oral supplements when they are in rehabilitation. Paracentesis would be required only if the patient was having symptoms from the ascites and would probably not be appropriate in the admission orders. Oxandrolone carries a risk of liver damage and therefore should not be prescribed in this patient. Abdominal binders may be used to help with ascites, particularly if the patient has an umbilical hernia from it. Ref: (a) Young MA, Stiens SA. Rehabilitation aspects of organ transplantation. In: Braddom RL, editor. Physical medicine and rehabilitation. 2nd ed. Philadelphia: WB Saunders; 2000. p 1397. (b) Zafonte RD, Pippin B, Munin M, Thai N. Transplantation medicine: a rehabilitation perspective. In: DeLisa JA, Gans BM, Walsh NE, editors. Physical medicine and rehabilitation: principles and practice. 4th ed. Philadelphia: Lippincott-Williams & Wilkins; 2005. p 1921-2.(c) Cortazzo MH, Helkowski W, Pippin B, Boninger ML, Zafonte R. Acute inpatient rehabilitation of 55 patients after liver transplantation. Am J Phys Med Rehabil 2005; Nov 84(11):880-4. (d) Available at: http://www.merck.com/mmpe/lexicomp/oxandrolone.html. Accessed 10/18/07(e) Available at: http://www.emedicine.com/med/topic3183.htm. Accessed 10/18/07Wolf DC. Cirrhosis

"85. On hospital rounds, you note that yo ur pat ient, who has a T10 ASIA B spinal cord injury is now using a rigid frame wheelchair in the therapy gym. In his attempt to show off as he propels toward you, he suddenly flips over backward. What is the most likely problem?(a) The rear axles are locat ed d irect ly under his center of gravity.(b) The rolling resistance i s in creas ed.(c) There is too much caster flu tter.(d) There is asymmetry in th e ch air's ca mber a ngle from side to side.

" 85. (a) The center of gravity for a hypothetical wheelchair rider is typically located slightly forward of the rear axle. Moving the rear axle directly under the wheelchair user makes the person and the chair more likely to flip backwards (wheelie). However, the advantages to having the center of gravity near the rear axles include decreased tendency for caster flutter, decreased rolling resistance, since most of the weight is borne by the larger rear wheels, and minimization of the turning torque.

105. What is the most appropriate treatment of an acute multiple sclerosis relapse? (a) Glucocorticosteroids (b) Mitoxantrone (c) Interferon (d) Methotrexate

"105 (a) Most significant multiple sclerosis (MS) relapses should be acutely treated with Gluco corticosteroids. A significant relapse in MS is defined by visua l or motor change s, pain, or incoordination. Such a relapse requires acute glucocort ic ostero id therap y. Alth ough there is no evi dence that stero id the ra py alt ers th e long-te rm c ours e of MS , clin ica l e xpe rienc e show s th at m ost re lapse s r es olv e mor e quickly an d possibly more completely with adequate glucocorticosteroids. Immunomodulating agents are used to reduce relapse frequency. These agents include Glatiramer acetate (Copaxone), which when used long term and continuous ly, reduces the occurrence of relapses and lowers or delays the risk of fixed disability. Interferons are presumed to act by way of antiproliferating and immunomodulating effects. Interferons should be used in MS patients in hopes of reducin g their re lapse r ate and their risk of increasing disability. Adverse affects of int er ferons include persist ent flu-li ke symptom s. Methotrexate was or ig ina lly con si de red a p oten tial t rea tm en t fo r M S b eca use o f some s im ilar iti es betwe en rh eum atoid arthritis a nd MS patients' immune alterations and relapsing clinical courses. Methotrexate may offer a relatively well-tolerated treatment option for patients with chronic, progressive MS. Mitoxantrone, an antineoplastic agent, exerts a potent immun omodulating effect that suppresses immunity and enhances suppressor function. Further study of this t herapy for patients with the progressive forms of MS is needed, particular ly in li ght of it s cardi otox ic eff ect s. Most significant multiple sclerosis (MS) relapses should be acutely treated with Gluco corticosteroids. A significant relapse in MS is defined by visua l or motor change s, pain, or incoordination. Such a relapse requires acute glucocort ic ostero id therap y. Alth ough there is no evi dence that stero id the ra py alt ers th e long-te rm c ours e of MS , clin ica l e xpe rienc e show s th at m ost re lapse s r es olv e mor e quickly an"

106. A 36-year-old man has a known history of human immu nod ef ici ency virus (HIV). His family has observed worsening confusion and memory loss. He later develops progressive paraparesis, ataxia, posterior column sensory loss, and neurogenic bowel and bladder. The most likely diagnosis is (a) viral myelitis. (b) multiple sclerosis. (c) cytomegalovirus (CMV) polyradiculopathy. (d) vacuolar myelopathy.

"106 Answer: (d) Commentary: Vacuolar myelopathy is the most common caus e o f spi nal cord dysfunction in human immunodeficiency virus (HIV) patients, being found in 11% to 22% of acquired immunodeficiency disease (AIDS) cases, and demonstrable in as many as 40% of cases at autopsy. It is strongly associated with HIV dementia, and shares a virtually identical histopathology. The other diagnoses are less common, and can be ruled out or in with imaging, laboratory and electrodiagnostic studies. Reference: Levinson SF, Fine SM. Rehabilitation of the i ndi vi dua l with HIV. In: Delisa JA, Gans BM, Walsh NE, editors. Physical medicine and rehabilitation: principles and practice. 4th ed. Philadelphia: Lippincott Williams & Wilkins; 2005. p 1804."

12. Which cardiopulmonary complication is NOT associated with obstructive slee p apnea? (a) V entricula r hypertr ophy (b) Pulmonary hypertension (c) Left ventricular failure (d) Alveolar hypoventilation Ref: Flemons WW, Douglas NJ, Kuna ST, Rodenstein DO, Wheatley J. Access to d iagnosis and t reatment of patien ts wi th s uspected sleep apnea. Am J Respir Crit Care Med 2004;169: 668-72. S73 Educational Activity 2.3

"12 (c) Cardiopulmonary complications of obstructive sleep apnea include right ventricu lar failure, ven tri cular hyper trophy, pulmonary and sy stemic hypertension, and alve olar hy poventil ation. Arch Phys Med Rehabil Vol 87, Suppl 1, March 2006 SAE-P ANSWER KEY, Phelan S77"

61. A 55-year-old overweight man presents to clinic with co mp lai nts of numbness in his left leg. He reports that he does not exercise and has an office job. He is diagnosed with meralgia paresthetica. Which of the following is consistent with this diagnosis? (a) Peroneal motor F-wave study is abnormal. (b) Sensory nerve conduction studies reveal decreased am pli tu de on the affected side. (c) Electromyography findings show denervation in the va stu s lat eralis. (d) Exam reveals decreased sensation in the medial thigh .

"61 Answer: (b) Commentary: Meralgia paresthetica presents with paresthesias in the lat er al thigh. Sensory nerve conduction studies of the lateral femoral cutaneous nerve may show a drop in the sensory nerve action potential (SNAP) amplitude on the affected side compared to the asymptomatic, contralateral study. Symptoms are confined to below the inguinal ligament and above the knee. Peroneal motor nerve conduction studies and F waves should be normal and needle electromyography should not show acute or chronic axonal motor loss, because the lateral femoral cutaneous nerve is purely sensory. Reference: Craig EJ, Clinchot DM. Lateral femoral cutane ous n eur opathy. In: Frontera WR, Silver JK, Rizzo TD, editors. Essentials of physical medicine and rehabilitation. 2nd ed. Philadelphia: Saunders-Elsevier; 2008. p 283-9."

"65.

(This question has been eli minat ed from t he exam, therefore, it was not scored.) Which statement is TRUE re gard ing p ers ons wi th complete spinal cord injury with concurrent posterior rhizotomy who receive functional neuromuscular stimulation via an implanted device to restore bowel and bladder function?(a) Stimulation of the poste rior S2, S3, S4 ne rve roots will produce micturition.(b) Stimulation will enhance ref lex v oid ing.(c) Stimulation will improve ref lexog eni c erec tion.(d) Stimulation will increas e bl adder ca pacity ." 65. (d) Because electrical stimulation for bladder and bowel function depends on the ability to activate intact motor neurons from the sacral segments of the cord, it is at this time limited to persons with suprasacral lesions. Micturition is produced by stimulation of the anterior (motor) S2, S3, and S4 nerve roots. Continence has been greatly improved by concurrent posterior rhizotomy of the (sensory) sacral nerve roots. The advantages of posterior rhizotomy include increasing bladder capacity and abolishing reflex voiding, reducing dyssynergia and abolishing episodes of autonomic dysreflexia. The primary disadvantage of posterior rhizotomy is the loss of reflex erection and reflex ejaculation (if these are present). The hardware cost is approximately $40,000 with the projection that after factoring in the cost of medications, supplies, medical procedures, durable medical equipment, and attendant care, the device pays for itself in 5 to 7 years.

"140. Bracing for curves deve lopi ng af ter age 3 , but before puberty onset (juvenile idiopathic scoliosis) should commence when the curve reaches approximately how many degrees?

(a) 15° (b) 25° (c) 40° (d) 60°" 140. (b) Unlike infantile idiopathic scoliosis, the juvenile type almost never spontaneously resolves, and owing to the many years of growth during which progression can take place, extremely severe curves can develop. Because of the very poor prognosis of this scoliosis, and the great desire to avoid fusion at a young age, bracing becomes an extremely important method of management. Therefore, the standard of care is to begin bracing when the curve reaches approximately 25°. It is not necessary to brace curves less than 20°, and curves as high as 60° can still respond to a brace. This is a much higher value than for successful bracing of adolescent idiopathic scoliosis where the upper limit is 40° to 45°.

"45. Which statement is correct regarding the m ana ge ment of labor and delivery for women with cervical spinal cord injuries?

(a) Pre-eclampsia is 3 times more likely to oc cur t han in able-bodied women. (b) Vaginal delivery is contraindicated. (c) Autonomic dysreflexia occurs 60%--80% of t he ti me. (d) Spinal and epidural anesthesia are contrai ndi ca ted" 45. (c) Women with paraplegia or tetraplegia can give birth vaginally and caesarean delivery is rarely necessary. Patients with neurologic levels above T6 are at risk for autonomic dysreflexia during pregnancy, labor, and delivery. Autonomic dysreflexia is reported to occur in 60% to 80% of women with SCI with lesions above T6. Preeclampsia occurs with the same frequency in able-bodied women and women with disabilities. Complications from autonomic dysreflexia may be severe and include encephalopathy, cerebrovascular accidents, death of the mother, and severe fetal asphyxia. Spinal or epidural anesthesia extending to the T10 level is the treatment of choice and the most reliable method of preventing and treating autonomic dysreflexia during labor and delivery. Ref: Jackson A. Women's health challenges after spinal cord injury. In: Lin VW, editor. Spinal cord medicine principles and practice. New York: Demos; 2003. p 842-5.

"65. A 21-year-old man is evaluated in your spinal cord i njury cli nic 12 months after a C2 complete spinal cord injury requiring full-time mechanical ventilation. You recommend

(a) avoiding a breath control system for his power wheel chair. (b) aggressive diaphragmatic strengthening exercises. (c) initiating a weaning protocol by slowly decreasing t idal volu me. (d) an electrodiagnostic study to evaluate for a phrenic nerve pa cema ker." 65. (d) It is unlikely that an individual will be able to wean from a ventilator if he is still completely dependent on mechanical ventilation 12 months after a C2 complete injury, so a weaning protocol and diaphragmatic strengthening are not indicated. An individual who requires mechanical ventilation can use a breath control system effectively. If electrodiagnostic testing indicate that the phrenic nerves are intact, then a phrenic pacemaker could be implanted, which would significantly reduce the need for mechanical ventilation.

"184. Electromyographic biofeedback for stroke patients i s most be nefi cial when

(a) proprioception is preserved. (b) used in the upper limb. (c) the patient is young. (d) the patient has flaccid paralysis." 184. (a) Hemiplegic stroke patients engaged in electromyography biofeedback training have a better functional outcome with lower extremity training than with upper extremity training. Further, their age and the duration of their hemiplegia have no effect on training outcome. Proprioceptive loss of the upper limb decreases the probability of making functional gains. Motivation by the patient is a necessity and is most beneficial when some voluntary activity is present.

"82. The mechanism of action of a phenol nerve block is

(a) reduction of calcium release from the sarcoplasmic reticulum. (b) agonist action at alpha-adrenergic receptor sites. (c) denaturation of protein in myelin sheaths and axons. (d) inhibiton of presynaptic acetylcholine release." 82. (c) Phenol acts as a neurolytic agent that denatures protein in myelin sheaths and axons. Dantrolene sodium (Dantrium) reduces calcium release from the sarcoplasmic reticulum. Tizanidine (Zanaflex) is an alpha-2 agonist. Botulinum toxin (Botox) inhibits acetylcholine release.

"131. Which term describes a maladaptive pattern of drug use marke d by increasing doses to achieve a similar pain relieving effect and a withdrawal syndrome? (a) Dependence

(b) Addiction (c) Craving (d) Tolerance" 131. (a) Dependence is a maladaptive pattern of drug use marked by tolerance and a drug-class-specific withdrawal syndrome that can be produced by abrupt cessation, rapid dose reduction, decreasing blood levels of drug, or administration of an antagonist. Tolerance is a state of adaptation in which exposure to a drug induces changes that result in diminution of 1 or more of the drug's effects over time. Addiction is a chronic biopsychosocial disease characterized by impaired control over drug use, compulsive use, continued use despite harm, and craving.

"141. Which factor increases the risk for long- ter m symptoms after a whiplash-type injury? (a) Male gender

(b) Eastern European descent (c) Preexisting hyperlordosis of cervical spin e (d) Presence of radiating pain into the limb" 141. (d) Risk factors for chronic whiplash-associated pain include presence of preexisting degenerative disc disease, preexisting loss of cervical lordosis, female gender, awkward head position at time of impact, presence of radiating pain into upper limbs, and prior history of headache. A famous Lithuanian study showed no incidence of long-term whiplash pain in a country that had no compensation system for whiplash. Ref: Seroussi RE, Ferrari R. Curve/countercurve: Whiplash. SpineLine 2001:12-9.

"18. Which statement is TRUE about F-wave and H-reflex responses? (a) They both are obtained with supramaximal stimulation.

(b) F waves can only be obtained from posterior tibial and median ne rves. (c) They both involve conduction along motor and sensory fibers. ( d) They both can be he lpf ul in the diagnosis of S1 radiculopathy." 18. (d) The F-wave response is a pure motor response that is obtained with supramaximal stimulation and can be obtained from any motor nerve in an adult. The H reflex is usually obtained with submaximal stimulation and involves both sensory and motor fibers. Tibial and peroneal nerve F waves can be abnormal in L5-S1 radiculopathies and tibial nerve H reflex can be abnormal in S1 radiculopathies.

"7. Which group would NOT be incl uded on a list of occupations with the largest incidences of low back injuries that receive workman's compensation? (a) Truck drivers

(b) House painters (c) Machine operators (d) Nurses" 7. (b) Alhough, house painters may be at risk for injury, machine operators, truck drivers, and nurses have the greatest incidence in compensated low back pain injuries.

"178. You are conducting a research study, and you w ant to use Functional Independence Measure (FIM) scores as 1 of your outcome measures. Because FIM information is scored on a 1 to 7 scale to describe the level of assistance an individual requires to perform a functional task, these scores represent which category of research data? (a) Nominal

(b) Ordinal (c) Ratio (d) Interval" 178. (b) FIM scores would be classified as ordinal data, because in an ordinal scale consecutive values are rankordered, but not equally spaced. For example, although there is an order to the ranking in the FIM scale, the difference between a 2 and a 3 may not be equal to the difference between a 6 and a 7. Nominal data refers to data with discrete values (yes/no; alive/dead). With interval data, there are equal intervals between consecutive values. An example of interval data is temperature in degrees Fahrenheit. Ratio data are interval data with equal intervals between consecutive values, but with an absolute zero point. Ref: Katz RT, Priebe MM, Campagnolo DI. Research in physical medicine and rehabilitation. In: Braddom RL, editor. Physical medicine and rehabilitation. 2nd ed. Philadelphia: WB Saunders; 2000. p 170.

"24. Which deep heat method recommends the use of protective eyewear to prevent the formation of cataracts? (a) Ultrasound

(b) Short wave diathermy (c) Fluidotherapy (d) Microwave diathermy " 24 Answer:(d) Commentary: Microwave diathermy is another form of electromagnetic energy that uses conversion as its primary form of heat production. Temperature distribution in a particular tissue is affected largely by its water content. In general, tissues with high water content absorb greater amounts of energy and are selectively heated. General heat precautions should be observed with microwave procedures. Metal implants, pacemakers, sites of skeletal immaturity, reproductive organs and brain, and fluid-filled cavities (eye, bullae, effusions, etc.) should be avoided. Microwaves can cause cataracts and protective eye wear should be worn by both patient and therapist to reduce risk. Reference: Basford J. Therapeutic physical agents. In: DeLisa JA, Gans BM, Walsh NE, editors. Physical medicine and rehabilitation: principles and practice. 4th ed. Philadelphia: LippincottRaven; 2005. p 260.

55. Regarding the American Spinal Injury Association (ASIA) classification in progn osti ca tin g r eco very,:(a) ASIA class A has a reasonable probability of improvement if there is no concurr ent br ain in jur y.:(b) preservation of pinprick in ASIA class B carries a better potential for ambulat ion th an pre ser vatio n of li ght t ouc h s ensation .:(c) recovery statistics for ASIA class C do not include the central-cord syndrome.:(d) Brown-Séquard's syndrome has the worst potential for ambulation in ASIA class D .

(b) The presence of sensation in the sacral (S3-S5) dermatomes in patients with motor complete injury indicates a favorable prognosis in terms of motor reco ver y, w ith p inp rick sparing hav in g the cl ose st corr elat ion for motor r ecovery. Moto r se gments in the zon e o f injury in pa tie nts w ith co mp lete injury and an initial stre ngth of 0/ 5 w ere more li kel y to rec ove ry strengt h of 3/5 or more at 1 year if the sensation in the corresponding dermatomes was intact. Most patients originally categorized as ASIA (or Frankel) class A who progressed to ASIA class D or E had sustained traumatic brain injury with cognitive impairment and were incorrectly diagno sed ini tiall y a s cla ss A.

"40. When should upper extremity prosthesis fitting be in itiated i n th e adult? (a) Within the first month after amputation

(b) When residual limb strength is full. (c) When the patient requests a prosthesis (d) When residual limb volume has stabilized" 40. (a) The first month after upper limb amputation is the optimal period for prosthesis fitting. Fitting should be initiated during this time to maximize the level of acceptance and use of the prosthesis.

"44. Cognitive deficits are common in (a) Duchenne muscular dystrophy.

(b) inclusion body myositis. (c) fascioscapulohumeral dystrophy. (d) Becker muscular dystrophy." 44. (a) A brain isoform of dystrophin exists and there are documented mildly decreased Intelligence Quotient scores in people with Duchenne muscular dystrophy. These lower scores may be specific to deficits with tasks requiring attention to complex verbal information. Ref: Kilmer DD. Myopathy. In: DeLisa JA, Gans BM, Walsh, NE, editors. Physical medicine and rehabilitation: principles and practice. 4th ed. Philadelphia: Lippincott Williams & Wilkins; 2005. p 916-21

"195. Trauma to the sacral roots would most lik ely r esult in (a) vesicoureteral reflux.

(b) incontinence. (c) detrusor hyperreflexia. (d) small bladder capacity. " 195 (b) Damage to the sacral roots usually results in a flaccid bladder. Incontinence often occurs due to a weak sphincter mechanism, particularly if the patient has increased bladder volume or an increase in intraabdominal pressure. However, the external sphincter may not always be affected to the same degree as the detrusor. This imbalance results in bladder overdistension and the possibility of upper tract deterioration. Ref: Linsenmeyer TA. Neurogenic bladder following spinal cord injury. In: Kirshblum S, Campagnola D, DeLisa J, editors. Spinal cord medicine. Philadelphia: Williams & Wilkins; 2002. p 198-200.

"27. A 42-year-old jackhammer ope rator pr esents with low back and left posterior-lateral thigh pain and numbness that began at the end of his shift 1 week ago. He has been unable to return to his job since the onset of pain. His physical examination reveals normal reflexes, strength, and sensory function upon examination of both lower extremities. His pain is reproduced with forward flexion of the lumbar spine and left straight leg raise. Your recommendations include(a) ice, muscle relaxants, x =-ray s, 10 da ys of bedrest.

(b) nonsteroidal anti-inflam mato ry dr ugs , musc le relaxants, lumbar corset, return to work.(c) nonsteroidal anti-inflam mato ry dr ugs , educ ation in positions of comfort, physical therapy. (d) narcotic pain medication s, l umbar co rset, lumbar discography." 27. (c) Nonsteroidal medications, education in lumbar positions of comfort, and physical therapy constitute the standard of care in conservative management of lumbar radiculopathy. Ten days of bedrest is no longer recommended, since the effects of immobilization can further impair recovery. Limited or relative rest can help relieve repetitive trauma while acute pain management interventions are underway. A lumbar corset may help with pain in the first few days but immediate return to work while relying on a lumbar corset does aid in recovery. Jackhammer operators are exposed to a great deal of vibration, which increases an individual's risk of disc injury. When a disc injury is suspected in this population, return to a modified work description avoiding lifting, bending, twisting, and vibration should be recommended. Lumbar discography should be reserved for individuals who have exhausted conservative management and are contemplating a spine procedure.

"42. A 67-year-old man with Parkinson disease i s e xp eriencing more falls. These falls usually occur shortly after getting up in the morning, or after a large meal. You suspect his falls are due to (a) vestibular dysfunction.

(b) orthostatic hypotension. (c) increased lower extremity weakness. (d) increased rigidity." 42. (b) The majority of patients with Parkinson disease experience orthostatic hypotension (OH) as the disease progresses. The patient's history suggests falls related to postural changes and situations that lower blood pressure. Educating your patient to avoid or mitigate these situations (slow postural changes, small meals, and avoidance of high heat exposure and alcohol) is the best initial treatment. Ref: Senard JM et al. Orthostatic hypotension in patients with Parkinson's disease: pathophysiology and management. Drugs Aging 2001;18:495-505.

"117. You are being deposed r egar ding a 4 2-year old factory worker you have diagnosed and treated with carpal tunnel syndrome. Objective testing that confirms your diagnosis includes(a) nerve conduction studies and elec tro myogra phy.

(b) x-rays of the wrist.(c) positive Tinel's test at the wris t.(d) paresthesias in the thum b an d ind ex finger ." 117. (a) Electrodiagnostic testing supplies objective measurement of peripheral nerve compression. Though x-rays are an objective test, a diagnosis of carpal tunnel syndrome cannot be made by x-rays. A positive Tinel's test and symptoms of paresthesias in the thumb and index finger are commonly found in patients diagnosed with carpal tunnel syndrome but are not objective findings.

"17. A construction company manager is concerned about hi rin g emp loyees over the age of 40, citing lower productivity because of lower endurance compared to younger workers. You tell him that the average decline in physical work capacity between the ages of 40 and 60 is (a) 5% (b) 20% (c) 35%

(d) 50%" 17 Answer: (b) Commentary: While variation exists, an average decline o f 2 0 in physical work capacity has been reported between the ages of 40 and 60 years, due to decreases in aerobic and musculoskeletal capacity. However, differences in habitual physical activity will influence the variability seen in individual physical work capacity and its components. Reference: Kenny GP, Yardley JE, Martineau L, Jay O. Phy sic al wo rk capacity in older adults: implications for the aging worker. Am J Ind Med 2008;51:610-25.

"136. A 58-year-old man sustained a myocardial infarction 1 week a go. He is undergoing phase 1 of cardiac rehabilitation. His activity level should be limited to how many metabolic equivalents (METs)? (a) 1 (b) 3 (c) 5

(d) 7" 136. (c) After a myocardial infarction, exercise intensity should start at 2 metabolic equivalents (METs) and gradually progress to a maximum of 5 METs. Patients should await myocardial infarct healing before vigorous exercise greater than 5 METs is performed, usually within 4 to 6 weeks post infarctioin.

"135. Which statement is TRUE regarding spasticity in the individual with spin al cord i njury ? (a) Most antispasticity medications can completely eliminate spastic ity. (b) The incidence of spasticity is higher in individuals with lower thoracic spinal co rd in jur y than in those with cervical spinal cord injuries. (c) Spasticity can offer a functional benefit to some individuals wi th spinal cord inj uries .

(d) Clonus is an example of a tonic stretch reflex." 135 (c) No single medication for spasticity is universally beneficial and reduction of spasticity, rather than elimination of it, is the more likely outcome. The incidence of spasticity is higher in individuals with cervical and upper thoracic injury than in those with lower thoracic injury. Lower extremity tone may be helpul for activities such as transfers, standing, and ambulation. Spasticity is often characterized as either phasic or tonic. Tonic spasticity is seen as increased tone. Phasic spasticity is usually seen in hyperactive tendon jerks.

"171. A far lateral L4-5 disc herniation pictur ed ab ove will impinge on which nerve root? (a) L3 (b) L4 (c) L5

(d) S1" 171. (a) Since it occurs lateral to the intervertebral disc, a far lateral disc herniation is a relatively unusual location for disc herniation. A far lateral disc herniation can actually impinge the nerve root exiting above that intervertebral level. Ref: In Interventional Spine: An Algorithmic Approach. Ed: Slipman CW, Derby R, Simeone FA, Mayer T. Philadelphia: Elsevier. 2008

"81. A 70-year-old man with severe bilateral knee osteoarthritis stat es that o ver the c ounte r n onsteroidal anti-inflammatory drugs and aceteminophen are not controlling his knee pain. He inquires about a more potent anti-inflammatory medication he saw on a television commercial. Once treatment has been initiated, which recommendation would be most appropriate? (a) Check kidney function and a hematocrit for anemia every 6 months . (b) Start concomitant anti-hypertensives if the patient develops hig h blood p ressure. (c) Switch to a cyclooxygenase-2 inhibitor if the patient is on aspi rin for c ardioprot ectiv e e ffects.

(d) Start concomitant proton pump inhibitor or histamine H2-receptor antagonists." 81. (a) Older patients on nonsteroidal anti-inflammatory drugs are at a high risk for medication-induced side effects. Kidney function and gastrointestinal (GI) bleeds (causing anemia) should be monitored every 6 months. The purported gastroproctective effects of cyclooxygenase-2 inhibitors may be lost with concomitant aspirin use. Gastritis side effects, especially in the elderly, should be worked up for a GI bleed prior to instituting histamine H2-receptor antagonists or a proton pump inhibitor.

"7. The purpose of the Health Insurance Portability and Accountabilit y Act (HI PAA) is t o (a) ensure that a patient's medical record is available only to heal th care p roviders as di rec ted by the patient. (b) make sure that only qualified physicians have access to a patien t's medic al record . (c) allow a lawyer access to a medical record only if litigation is pending.

(d) al low o nly the treating physician to know the patient's diagnosis." 7. (a) The purpose of the Health Insurance Portability and Accountability Act (HIPAA) is to ensure that a patient's medical record remains private. A non-treating physician, lawyer, insurance company, etc. can have access so long as the patient directs it, with written authorization. There are no stipulations about a physician's qualifications with regards to medical information access.

"187. You are asked to make specific exercise recommendations regardi ng how to reduce l ow ba ck injuries in a factory's workers. As part of your program, you recommend that workers (a) be fitted for a lumbar corsets. (b) with heavy lifting work requirements avoid participation in spor ts. (c) with a history of low back pain be moved to more sedentary, seat ed jobs.

(d) be inst ruc ted to reduce the amount of early morning flexion." 187 (d) Patients instructed to limit early morning flexion have shown reduced pain intensity, compared to controls. This finding is related to disc hydration that occurs during sleep. No studies have shown that all workers wearing lumbar corsets have reduction in injuries. Athletic activities outside of the work environment often help improve endurance, which is protective for low back pain. Sedentary seated jobs over-promote disc loading, resulting in increased pain.

"127. The activity established as most predictive of deve loping a low back disorder is (a) carrying an object at an increased horizontal distan ce from t he b ody. (b) lifting an object repeatedly at 20% less than the in dividual' s ma ximum lift capacity. (c) repetitive sit-to-stand transitions with a weighted back pack .

(d) bending at knees rather than at the waist to lift an object." 127. (a) The work by Marras and colleagues showed that increasing the horizontal distance from the trunk of an object being carried increased the risk of developing a low back disorder. This increase in distance increased the forces consistently on the anterior column of the spine. Although the other options can all place the worker at risk for a low back injury, only the increased carrying distance from the trunk has been shown to be the most predictive of a low back injury.

"17. The Americans with Disabilities Act (ADA) was passed in order to (a) require employers to hire individuals with disabilities. (b) protect individuals with disabilities from having to work. (c) ensure that the pay of workers with disabilities is equal to tha t of nond isabled w orker s.

(d) ensure that workers with disabilities have equal access in the work environment." 17. (d) The Americans with Disabilities Act (ADA) provides that workers with disabilities be offered equal access in the work environment. The Act requires the employer to provide accessible environment for the workers. Dimensions for doorways, hallways, ramps and elevator lifts are specified to allow wheelchair access. The ADA does not make stipulations with regards to pay, or that an employer is forced to hire a disabled worker. The ADA does not make any determinations regarding a disabled person's ability to work.

"183. Vapocoolant spray produces its cooling ef fec ts through (a) conduction. (b) convection. (c) conversion .

(d) evaporation." 183. (d) Evaporation is a process of transforming a liquid into a gas and requires thermal energy, as in vapocoolant spray. Convection is a process of using a medium to transport energy, for example husks during fluidotherapy and water during whirlpool therapy. Conduction is a process of transferring thermal energy to bodies that are in direct contact, for instance cold packs applied to skin. Conversion is a process of transforming energy into heat, as occurs with an ultrasound device. Ref: Weber DC, Brown AW. Physical agent modalities. In: Braddom R, editor. Physical medicine and rehabilitation. 2nd ed. Philadelphia: WB Saunders; 2000. p 442-3, 451.

"77. What percentage of patients with whiplash- ass oc iated disorders develop chronic symptoms? (a) less than 25% (b) 25%--49% (c) 50%--75%

(d) more than 75%" 77. (b) Up to 33% of individuals with symptoms from whiplash-associated disorders have chronic symptoms. Symptoms associated with whiplash-associated disorders include neck pain, arm pain, paresthesias, temporomandibular joint dysfunction, headache, dizziness, visual disturbances, and difficulty with memory and concentration. Ref: Panagos A, Sable AW, Zuhosky JP, Irwin RW, Sullivan WJ, and Foye PM. Industrial medicine and acute musculoskeletal rehabilitation. 1. Diagnostic testing in industrial and acute musculoskeletal injuries. Arch Phys Med Rehabil 2007;88(3 Suppl):S5.

"183. A 70-year-old woman presents with a cemented right total hip art hroplasty. She is partial weight bearing and struggling with physical therapy. The therapist asks to use ultrasound to the right hip to help with bone healing and ultimately progress the patient to weight bearing as tolerated. You advise (a) yes, because ultrasound helps with bone healing. (b) no, because ultrasound near arthroplasties is contra indicated . (c) yes, because the heat may help with pain management.

(d) n o, b ecause ultrasound is expensive to use." 183. (b) Ultrasound is typically an inexpensive treatment that may help with pain and bone maturation, however, it is contraindicated near arthroplasties and therefore not a good treatment in this case. Further contraindications include use of ultrasound: near pacemaker, near spine or laminectomy site, near brain, eyes, or reproductive organs, is someone with malignancy or skeletal immaturity, or near sites where methyl methacrylate was applied.

1. You are called onto a football field immediately afte r a defen sive player involved in a spearheading tackle complains of neck pain and right greater than left arm tingling. What should be the next step? (a) Call for an ambulance and stabilize the neck. (b) Remove the athlete's football helmet and palpate for any neck ten derness. (c) Return the athlete to the game if his strength exam is normal . (d) Walk the athlete to the locker room and perform a th orough ne urol ogic examination.

1 (a) A telltale sign of cervical cord involvement is bilateral symptoms. In this case, the athlete should be treated as having a potential spinal cord injury and should have his cervical spine immobilized. The football helmet should not be removed, since the cervical spine may fall into extension in the act of removing the helmet. If the airway needs to be accessed, then the face guards should be removed using special equipment. If the athlete suffered and recovered from a temporary "stinger," involving 1 limb, he may return to play as long as his neurologic examination is normal.

1. Which statement concerning cardiac rehabilitation in women is TRUE? (a) Women are referred to cardiac rehabilitation less frequently and attend it less frequent ly than d o men. (b) Women have a lower dropout rate than men. (c) Women have even greater benefit from cardiac rehabilitation than do men. (d) Nonmedical factors such as transportation and psychosocial impairment tend to be less of an issue for wome n tha n me n. Ref: Gallagher R, McKinley S, Dracup K. Predictors of women's attendance at cardiac rehabi litation programs. Prog Car diovasc Nurs 2003;18:121-6. Clinical Activity 1.2

1 (a) Women are referred less frequently to cardiac rehabilitation and attend it less frequently than do men. Women have a higher dropout r ate compared with men because of tra nsportat ion, me di cal comorbi dit ies, and psychosocial impairment. The benefit women derive from cardiac rehabilitation is equal to that of men in improving aerobic capacity.

1. 20-year-old football player reports anterior shoulder pain during a game. He completes the game, but radiographs after the game revealed a type 2 acromioclavicular (AC) joint sprain. How is a type 2 acromioclavicular (AC) joint injury defined? (a) Acromioclavicular and coracoclavicular ligaments are both disrupted. (b) Acromioclavicular and coracoclavicular ligaments are both intact. (c) Acromioclavicular ligament is disrupted, but the coracoclavicular ligament is intact. (d) Acromioclavicular ligament is intact, and the coracoclavicular ligament is disrupted.

1 Answer: (c) Commentary: Acromioclavicular joint injuries are classified into 6 types according Rockwood classification. A type 1 injury describes a mild injury to the AC joint without disruption of either the acromioclavicular or the coracoclavicular ligaments. A type 2 injury describes disruption of the acromioclavicular ligament, but the coracoclavicular ligament remains intact. A type 3 injury describes disruption of both ligaments whereas a type 4 injury entails complete disruption of both ligaments with posterior displacement of the distal clavicle into the trapezius muscle. Ref: Finnoff J. Musculoskeletal problems of the upper limb. In: Braddom R. Physical medicine and rehabilitation. Philadelphia: Elsevier; 2007. p 828.

1 A 67-year-old man presents to your clinic with weakness and frequent falls. You suspect cervical stenosis. Calculating anteroposterior (AP) ratios to other anatomical structures, which ratio would enable you to assess for bony cervical spinal stenosis on lateral radiographs? (a) AP diameter of the vertebral body to the height of vertebral body. (b) AP diameter of the vertebral canal to the AP diameter of the vertebral body at the same level. (c) Vertebral height to the AP of the vertebral canal at the same level. (d) Distance from the anterior border of the vertebral body to the tip of the spinous process.

1 Answer: B Commentary: Assessment of cervical spinal stenosis on lateral radiograph can be made by calculating the ratio of the anteroposterior (AP) diameter of the vertebral canal to the AP diameter of the vertebral body at the same level. This ratio is called the Pavlov ratio. A normal ratio is 1.0 with less than 0.82 indicating stenosis. The Torg ratio is the same as Pavlov ratio. Ref: Kawaguchi Y, Rydevik B. Cervical myelopathy. In: Slipman CW, Derby R, Simeone FA, Mayer TG, editors. Interventional spine: an algorithmic approach. 1st ed. Philadelphia:Elsevier; 2008. P 558.

112. Which statement about primary cerebral ly mph om a is TRUE? (a) It has an increased incidence in patients wit h (HIV) infection. (b) It usually presents as a solitary tumor. (c) It is treated surgically for improved outc ome . (d) It has a median survival of approximately 2 y ea rs.

112. (a) Primary cerebral lymphoma presents as multiple tumor deposits in the brain and has an increased incidence in patients infected with human immunodeficiency virus (HIV). Surgical removal does not improve outcome. Ref: Gerber LH, Vargo M. Rehabilitation for patients with cancer diagnoses. In: DeLisa JA, Gans BM, Walsh NE, editors. Physical medicine and rehabilitation: principles and practice. 4th ed. Philadelphia: Lippincott Williams & Wilkins; 2005. p 1789.

1 A 57-year-old man with chronic low back pain is referred to see you. He has commercial medical insurance. The patient leaves radiologic films and medical reco rds for review and also asks you to complete disability paperwork. That evening you review the films and records, fill out the paperwork, and call the referring physician. In addition to the charges for a New Patient Consultation, what charges may you submit to the commercial medical insurance for these services? a) No additional charges, since all your services are covered by the New Patient Consultation charge a. b) No additional charges to the commercial insurance, although you may charge the disability insurance for completing the disability paperwork b. c) One additional charge for Prolonged Services, up to 60 minutes for the time needed to complete all the additional services c. d) Two additional charges, one for radiology review and another for Prolonged Services, up to 60 minutes for the time needed to complete the other servic es

1 Option a is correct. Commentary: For outpatient services, Evaluation and Management (E/M) coding allows providers to bill based on time spent, as long as that time is spent f ace-t o-f ace, and at least 50% of that time is used for patient counseling and education. Provider services that occur outside of the actual face-to-face encounter are assumed as part of the work necessary to complete the evaluation and management of the patient and may not be billed separately as "Prolonged Services." Commercial and disability insurance companies are not obligated to reimburse providers to complete paperwork, and providers may opt to charge patients directly for completion of disability paperwork. Had this patient presented through the Workers' Compensation (WC) system, some of these additional services may have been reimbursable, depending on the specific WC laws that are stipulated by each st ate. Reference: American Medical Association. Current procedural terminology (CPT) 2012 professional edition. Chicago (IL) American Medical Association; 2011. P 8.

113 Which factor is a criterion for hip osteoarthritis? (a) Periarticularosteopenia (b) Femoral head erosions with sclerosis (c) Acetabular osteophytes (d) Erythrocyte sedimentation rate above20mm/hr

113 Answer: C Commentary: The American College of Rheumatology states that the criteria for osteoarthritis of the hip are hip pain along with 2 of the three findings: erythrocyte sedimentation rate less than 20mm/hr, radiographic evidence of femoral/acetabular osteophytes, radiographic evidence jointspace narrowing. Ref: Lane NE. Osteoarthritis of the hip. N Engl J Med 2007;357:1414

137. What is the major deter mina nt of su ccessf ul return to work after a work related injury?(a) Amount of lost time(b) Type of injury sustained(c) Type of job(d) Surgical intervention wa s re quire d

137. (a) The amount of lost time is a major determinant of return to the work place. Although more extensive injuries may more easily deter the employee from returning to work, the extent or type of injury has not been found to be the major determinant in all worker's compensation cases. Neither the type of job or the type of treatment required, such as surgery, are major determinants of successful return to work. Several studies have shown that the longer the worker is out of work related to the injury, the more unlikely it is that he/she will return to work successfully.

148. In evaluating a hypotonic infant with electromyography you find low-amplitude, short-durationmotor units with early recruitment. Based on these findings, the LEAST likely diagnosis would be (a) central core disease. (b) nemaline myopathy. (c) type II glycogenosis (acid maltase deficiency). (d) infantile spinal muscular atrophy.

148 (d) The motor unit changes noted are typically seen in myopathies. Spinal muscular atrophy is an anterior horn cell disease.

109. Which of the following is NOT a feature of central autonomic dys function in traumatic brain injury in children? (a) Hypertension (b) Tachypnea (c) Rigidity (d) Hypothermia

109. (d) Central autonomic dysfunction occurs in some children following severe brain injury. It is characterized by hypertension, hyperpyrexia, rigidity, tachypnea, tachycardia, and diaphoresis. Various medications are used to treat this dysfunction, but no studies prove the value of one medication over another.

10 Your 67-year-old patient with ankylosing spondylitis com es for an urgent appo int men t af ter fa lli ng 3 we eks ag o. He com pla ins of a heada che at t he ba ck of hi s h ead , a lon g wi th progressive weakness, and numbness in his hands and feet. You suspect a. atlantoaxial subluxation. b. epidural hematoma. c. Guillain-Barré syndrome. d. occipital neuralgia.

10 Option a is correct. Spontaneous atlantoaxial subluxation may be present in i ndividuals with ankyl osi ng sp on dyl iti s. Min or t rauma, su ch as a f all, m ay compl icat e a spont ane ous s ubl uxa tio n b y in creasing pressure on the spinal cord and producing long-tract signs along with an occipital headache. Although epidural hematoma also typically occurs after a fall, symptoms are more acute and usually associated with altered consciousness and bruising or skull fracture. Guillain-Barré syndrome, or acute inflammatory polyneuropathy, is not associated with fall or headache. Occipital neuralgia does not produce generalized weakness.

10 Aseptic loosening is the most predominant cause of long-term failure of hip arthroplasty at 10 years or longer after surgery. Infection in the joint within a year of the operation is due to di rect contaminatio n at the tim e of surgery; afterwards it is due to hematogenous seeding. Heterotopic ossification, if it occurs, does not change after 6 to 12 weeks postoperatively even though the bone continues to mature. Low bone density does not happen as a result of hip arthroplasty. A 48-year-old man with C6 tetraplegia from a skiing accident 23 years ago presents for his annual evaluation. He has noticed weakness in bilateral wrist ext ension over the last 3 months. In ad diti on to confirm in g his new weakn ess on exam, you note a loss of his left biceps reflexes since last year. To confirm your diagnosis you order a) computed tomography (CT) of his hea b) electromyography (EMG) of his upper extremities. c) lumbar puncture for cerebral spinal fluid (CSF) evaluation. d) magnetic resonance imaging (MRI) of his cervical spine.

10 Reference(s) Bryce TN, Ragnarsson KT, Stein AD, et al. Spinal cord injury. In: Braddom RL, editor. Physical medicine and rehabilitation. 4th e Philadelphia: Elsevier; 2011. p 1320. Option d is correct.

10. Individuals with diabetes are at high risk of a mputation despite ankle pressures greater than 55 mmHg because (a) the ankle brachial pressure index must be gre at er than or equal to 0.3 to prevent limb threatening ischemia. (b) ankle pressures seldom correlate with seve rit y of symptoms and are unreliable. (c) calcification of the arterial media result s i n a spuriously high pressure. (d) transcutaneous oxygen partial pressures an d n ot ankle pressures correlate with ischemia.

10. (c) In patients with diabetes, amputation is a strong possibility, even with ankle pressures higher than 55 mmHg because spuriously high pressures can be present in these patients as a result of calcification of the arterial media. The ankle brachial pressure index (ABPI) is the patient's brachial pressure compared to the ankle pressure. A resting ABPI greater than 1.0 is considered normal. Patients with intermittent claudication have an ABPI in the range of 0.5 to 0.7, and patients with rest pain or other symptoms of severe ischemia have an ABPI of less than or equal to 0.3. A pressure less than 50 mmHg at the ankle is associated with limb threatening ischemia. Ref: McCollum PT, Raza Z. Vascular disease: Limb salvage vs. amputation. In: Smith DG, Michael JW, Bowker JH, editors. Atlas of amputations and limb deficiencies. 3rd ed. Rosemont (IL): American Academy of Orthopedic Surgeons; 2004. p 38-9.

100. Which spinal orthosis is used to prevent thoracic spinal flexion by providing 3-po in t p res sur e ove r th e s ternu m a nd pubis an ter io rly and t he upper lumbar spine posteriorly?:(a) Custom molded, plastic thoracolumbosacral orthosis:(b) Lumbosacral corset with posterior metal stays:(c) Jewett orthosis:(d) Taylor orthosis

100 (c) Several different types of thoracolumbosacral (TLSO) orthoses are available to control segmental spine motion in this region. A custom molded plastic TL SO prov ides alm ost t otal contac t support for un ifor m pr ess ure distribu tion and cont rol of mot ion in all pla ins. A lu mbo sa cra l cor set wi th metal stay s pro vi des supp ort c ircum fe ren tia lly and hel ps reduc e s pin e m otion p rimarily in the lumbosacral region. A Taylor orthosis also provides circumferential support with the addition of axillary straps. The Taylor orthosis is primarily designed to resist flexion and extension. A Jewett brace is designed to limit thoracic spine flexion by providing 3-point pressure over the sterum and pubis anteriorally and the upper lumbar spine posteriorally. This type of brace is used most commonly for individuals with thoracic spine anterior compression fractures.

100. You are performing a consultation on a 58-year-old man with a history of diabetes and peripheral vascular disease who presents with a non-healing foot ulcer. You are concerned that he is at risk for amputation because his (a) ankle brachial index (ABI) is 0.8. (b) ABI is 0.4. (c) transcutaneous oxygen pressure (TcPO2) is 80mmHg. (d) TcPO2 is 40mmHg.

100 Answer: (b) Commentary: ABI is a noninvasive technique that is used in the assessment of arterial occlusive disease. The ABI is the ratio between the ankle and the brachial systolic pressure. Normal ABI is defined as values greater than 0.9. An ABI below 0.4 tends to carry a poor prognosis. TcPO2 is defined as transcutaneous oxygen, which is in essence a "blood gas" of the skin. Normal TcPO2 is greater than 50mmHg. Values of more than 40mmHg are associated with healing. Ischemia is defined as periwound TcPO2 < 20mmHg. Ref: (a) Goldman R, Popescu A, Hess CT, Salcido R. Prevention and management of chronic wounds. In: Braddom RL, editor. Physical medicine and rehabilitation. 3rd ed. Philadelphia: Elsevier; 2007 p 688-9, 700. (b) Spires MC, Henke PK. Lower limb peripheral vascular disease. In: Braddom RL, editor. Physical medicine and rehabilitation. 3rd ed. Philadelphia: Elsevier; 2007 p 1352.

100. Your patient demonstrates ipsilateral pel vic d rop during gait. What is the most likely cause? (a) Scoliosis (b) Short contralateral limb (c) Hip adductor weakness (d) Weak hip extensors

100. (a) Deformity in the spine presents with malalignment of in the pelvis as either contralateral or ipsilateral drop. Two other causes of ipsilateral pelvic drop are contralateral hip abductor weakness and short ipsilateral limb. Weak hip extensors are a cause of backward lean. In stance, a backward lean of the trunk substitutes for weak hip extensors. Ref: Perry J. Gait analysis, normal and pathologic function. Soack; 1992. p 269-73.

100. A medical student asks you to sh ow her ho w to assess the fit of a traditional quadrilateral socket. You explain that the best evaluation is carried out by placing the examiner's finger between the ischial tuberosity (IT) and the ischial seat (IS). You then tell that during full weight bearing(a) the IT slides down into the socke t w hen pr operly fit.(b) there should be 1 finger brea th be twe en the IT and the IS during knee extension.(c) the best position for de term ining th e rela tionship of the IT and the IS is knee flexion.(d) there is no room to plac e a finge r b etween the IT and the IS.

100. (d) During full weight bearing the ischial tuberosity sits on the ischial seat (the wide flat posterior brim). Only with ischial containment sockets does it slide down into the socket when properly fit. There should be room for the finger between the ischial tuberosity and the ischial seat during knee flexion when weight bearing is decreased. The best position for determining the relationship of the ischial tuberosity and the ischial seat is knee extension. If there is no room for the finger between the ischial tuberosity and the ischial seat during full weight bearing, the socket fits correctly with regard to the proximal landmarks.

101. 25-year-old man presents to clinic with an insidious onset of low back pain over the past 6 months. He denies any trauma, but is quite active running and biking. He does not report any leg symptoms. His pain is worse in the morning, but improves with activity and with antiinflammatory medication. What additional information would be most helpful in making the diagnosis? (a) Blood work revealing elevated erythrocyte sedimentation rate (ESR) (b) Magnetic resonance imaging revealing degenerative disc disease (c) Plain radiograph revealing sacroiliitis (d) Physical examination revealing an absent Achilles deep tendon reflex (DTR)

101 Answer: (c) Commentary: This patient presents with a clinical history consistent with ankylosing spondylitis (AS).This spondyloarthropathy is more common in men in their late teenage years to early twenties. It generally presents with morning stiffness in the low back and/or buttocks. Criteria for diagnosis (modified New York classification) include the presence of sacroiliitis on x-ray and 1 of the following: history of inflammatory back, decreased range of motion of spine, and limited chest expansion. Ref: Barr KP, Harrast MA. Low back pain. In: Braddom R. Physical medicine and rehabilitation. Philadelphia: Elsevier; 2007. p 916-7.

101. A woman in her third trimester of pregnan cy pr esents with severe burning pain in her right anterior thigh for the last 2 months. The pain is aggravated by prolonged standing. On examination of the right lower limb, there is no evidence of discoloration, edema, tenderness to palpation, or motor deficit. She has decreased sensation to pinprick over the lateral aspect of her distal thigh. What is the most likely diagnosis? (a) Trochanteric bursitis (b) Sacroiliitis (c) Femoral neuropathy (d) Meralgia paresthetica

101. (d) This history and examination are consistent with a lateral femoral cutaneous neuropathy, known as "meralgia paresthetica." It is seen with pregnancy, obesity, pressure from tight clothing, trauma and seatbelt use. The cause is usually idiopathic, and spontaneous recovery is usually the rule. This problem may be difficult to treat and may be recurrent for years. Anti-inflammatory medications and neuropathic pain medications may be tried, as well as attempting to remove the causative factors. This is a sensory nerve and should not result in motor deficit. Sacroiliac pain is associated with pregnancy but will not present with this pattern. A femoral neuropathy would likely have motor findings, and would have numbness in the anterior portion of the thigh. An L3 radiculopathy might mimic this condition. Trochanteric bursitis would cause this patient to be tender to palpation over the greater trochanter. Patients are usually unable to lie on the affected hip. Ref: Dumitru D. Electrodiagnostic medicine. Philadelphia: Hanley & Belfus; 1995. p 671-3

102. Which of the following is an appropriate compensatory technique for managing d ysph ag ia?:(a) Tilting the head to the weaker side:(b) Glossopharyngeal breathing:(c) Chin tuck:(d) Turning the head to the stronger side

102 (c) Tilting the head to the stronger side and turning the head to the weaker side (but not vice versa) are appropriate compensatory techniques. Glossopharyn gea l br eathi ng is us ed in pulmo na ry rehab ili tat ion and has no value in dysphagia ma nage ment.

102. A 28-year-old PM&R resi dent had a m ild tr aumatic brain injury. He is now having problems organizing and leading team meeting, although his ability to perform other aspects of his job as an individual are unimpaired. Of the following, the most useful neuropsychologic test for assessing his deficits is the(a) Galveston Orientation an d Am nesia Te st.(b) Wechsler Adult Intellige nce Scale -Re vised.(c) Wechsler Memory Scale.(d) Trails A and B test.

102. From the details given in the case, this resident appears to be having problems in mental flexibility and paying attention to multiple stimuli. The Trails A and B test examines simple and alternating attention. The Galveston Orientation and Amnesia Test, which is used to measure the presence of post-traumatic amnesia, would not be helpful. The Wechsler Adult Intelligence Scale-Revised and Wechsler Memory Scale, while necessary in a complete neuropsychological examination, do not directly measure attentional abilities.

103. In Lyme disease, beyond the initial erythema migran s l es ion from infection with the spirochete Borrelia burgdorfer what other findings may be seen later on? (a) Facial nerve palsy (b) Renal insufficiency (c) Pleural effusion (d) Cardiomyopathy

103 Answer: (a) Commentary: Lyme disease is the result of a bite from a ti ck in fected with Borrelia burgdorferi. Erythema migrans lesion is typically the initial skin le sio n see n. Other findings from systemic infection include mono-/polyarticular arthritis, facial ner ve pa lsy, aseptic meningitis, radiculopathy, or heart block. Renal insufficiency and pleural effusion are not seen. Reference: Wormser GP. Early Lyme disease. N Engl J Med 20 6 ;35 4:2794-801.

103. A 40-year-old man with psoriatic arthritis consults you rega rdin g his hand pain. On examination, you notice that his left index finger is noticeably shorter than all of his other fingers and has extra folds of skin. The most likely diagnosis is (a) arthritis mutilans. (b) Auspitz's sign. (c) dactylitis. (d) Jaccoud's arthritis.

103. (a) Arthritis mutilans is osteolysis of the phalanges and metacarpals, which results in telescoping, or shortening, of the involved digit. It is a highly characteristic feature of psoriatic arthritis. Auspitz's sign is pinpoint bleeding after scraping a psoriatic plaque. Dactylitis, or "sausage digits," is a combination of tenosynovitis and arthritis of the distal or proximal interphalangeal joint. Jaccoud's arthritis is a non-erosive deforming arthritis in systemic lupus erythematosus.

104. Muscle function changes after polio consist of all of the following EXCEPT:(a) muscle fiber hypertrophy.:(b) increase in motor unit size.:(c) decreased jitter on single fiber electromyography.:(d) increased fiber density on single fiber electromyography.

104 (c) Polio affects the anterior horn cells, resulting in motor axon dropout. Compensatory effects include individual muscle fiber hypertrophy of the survivin g m uscl e fib ers and increase in m otor uni t s ize due to spr outing and i nnervation of som e of t he surround ing denervat ed mu scl es. T here i s also an inc rease i n fiber densi ty as a re sul t of re- inn erv ation as we ll as atro phy and replacement of denervated muscles. This can be assessed by single fiber electromyography. Jitter also can be increased because of the re-innervated muscle fibers' instability.

104. Which statement is TRUE for children with acute inflammatory demyelinating polyneuropathy compared to adults with that disease? (a) Both recover at the same rate. (b) Disease course is more benign. (c) Residual weakness is more common. (d) Respiratory failure rates are equal.

104 Answer: B Commentary: The natural history of acute inflammatory demyelinating polyneuropathy (AIDP) in children is more benign than AIDP in adults. Children usually recover more quickly by 3 months on some occasions. Residual weakness is not as common for adults. The best prognostic indicator is the degree of disability at the peak of illness. Ref: Thomas MA, Felsenthal G, Fast A, Young M. Peripheral neuropathy. In: DeLisa JA, Gans BM, Walsh, NE, editors. Physical medicine and rehabilitation: principles and practice. 4th edition. Philadelphia: Lippincott Williams & Wilkins; 2005. p 898-9.

104. A 60-year-old woman presents with severe bulbar involvement at the time of initia l dia gno sis with amyotrophic lateral sclerosis. Regarding her prognosis, which statement is TRUE? (a) Riluzole (Rilutek) will improve life expectancy by 3 years. (b) Her prognosis is quite poor and her life expectancy is less than 5 years. (c) With aggressive speech therapy her prognosis will significantly improve. (d) Bulbar symptoms at the time of initial diagnosis have a positive influenc e on prog nosis .

104. (b) Bulbar palsy associated with amyotrophic lateral sclerosis (ALS), particularly in women, is known to be prognostic of a poor outcome. While it may help palliate symptoms and improve ability to communicate, speech therapy has not been shown to modify disease progression in ALS. In clinical trials, riluzole improved the life expectancy of ALS patients by approximately 3 months over an 18month period.

105. What is the most common level of occult spine fracture after trauma that is mi ssed b y p lai n r adiog raph s?:(a) C7/T1:(b) T5/T6:(c) T12/L1:(d) L4/L5

105 (a) Occult cervical fractures are most often seen at the C1 and C7 levels. By adding computed tomography (CT) scanning to the evaluation of trauma patients, a sign ifica nt numbe r of occult c ervical fra ctu res can be diagnosed. O f spinal frac ture s, 5%- 0.3 are mul tip le and ma y a pp ear at n oncont ig uous levels . Thu s, radiogr aphic eval ua tio n o f th e en tir e s pinal ax is is necessa ry whenever injury at 1 region of the spine is detected.

105. Which respiratory measure declines when a patient w ith t etr aplegia moves from a supine to seated position? (a) Total lung capacity (b) Functional residual capacity (c) Vital capacity (d) Residual volume

105 Answer: (c) Commentary: With the exception of vital capacity (VC), the d ire ction of change in total lung capacity and functional residual capacity decrease in the supine position and increase in the seated position, similar to an individual without a spinal cord injury. In contrast, patients with tetraplegia or high paraplegia have a decrease in the VC in the seated position, which is the result of an increase in the residual volume (RV) caused by the effect of gravity on the abdominal contents, causing the diaphragm to move down into a less efficient position and increasing the RV. Reference: Baydur A, Sassoon CSH. Respiratory dysfunctio n i n spi nal cord disorders. In: Lin VW, editor. Spinal cord medicine principles and practice. New York: Demos; 2010. p 216.

105. What function would be expected in a 24-year-old healthy woman with C7 ASIA A tetraplegia? (a) Requires minimal assistance for level transfers (b) Requires minimal assistance for side-side weight shifts (c) Independent manual wheelchair use on uneven terrain (d) Independent dressing and bathing with adaptive equipment

105 Answer: D Commentary: The C7 level is considered the key level for becoming independent in most activities at a wheelchair level. Persons with a C7 motor level who are in good health are usually independent for weight shifts, transfers between level surfaces, feeding, grooming, and upper body dressing. Some assistance may be required for wheelchair propulsion on uneven terrain. Bathing can be performed independently with the appropriate adaptive equipment. Ref: Kirshblum S, Ho CH, House JG, Druin E, Nead C, Drastal S. Rehabilitation of spinal cord injury. In: Kirshblum S, Campagnola DI, DeLisa JA, editors. Spinal cord medicine. Philadelphia: Lippincott Williams & Wilkins; 2002. p 275-98.

105. Compared with able-bodi ed i ndivi dua ls, pe rsons with spinal cord injury are likely to have(a) equivalent percentage of reg ional an d tota l body lean tissue.(b) higher testosterone leve ls.(c) equivalent incidence of dysl ipide mia .(d) a lower resting metaboli c ra te.

105. (d) In persons with spinal cord injury, there is an initial dramatic loss of muscle mass after the acute paralysis. However, even decades after injury, there is continuous loss of lean body tissue compared to that observed in an able-bodied person. It is of particular interest that the arms of persons with paraplegia have significantly less percent lean tissue compared with controls. No differences in the cross sectional rate of loss of lean body mass is noted between persons with tetraplegia and paraplegia. Men with spinal cord injury can be expected to lose about 3.2% per decade of the total lean body tissue vs. 1% per decade in able-bodied males. Individuals with spinal cord injury have a pattern of metabolic alteration that is atherogenic with dyslipidemia, glucose intolerance, insulin resistance, and reduction in metabolic rate. Although the literature in persons with spinal cord injury is conflicting regarding anabolic hormonal changes in persons with spinal cord injury, there are subsets of individuals with relative androgen deficiency states. The etiology of a relative deficiency of testosterone in persons with spinal cord injury has not yet been established. However, it is conceivable that prolonged sitting and euthermia of the scrotal sack and testes may itself have a deleterious local effect on testosterone production.

11. An 11-year-old baseball player presents to yo ur clinic complaining of elbow pain. X-rays of the affected side reveal an 8-mm separation of the medial epicondyle. What should be the next step in management? (a) Relative rest for at least 6 weeks (b) Long arm cast for at least 4 weeks (c) Refer to pediatric orthopedic surgeon (d) Physical therapy for strengthening

11. (c) Little league elbow, seen in throwing athletes with immature skeletons, is a conglomeration of different diagnostic entities caused by valgus and extension-overload. Medial epicondylar avulsion can frequently occur. Separation from 3--5mm can be managed nonsurgically. However, separations greater 5mm usually require surgery. Ref: Rudzki JR, Paletta GA. Clin Sports Med 2004;23:581-608.

106. A leukemic patient undergoing allogeniec bone marrow transplant has recov ered fro m a pain ful v ari ce lla z oste r out br eak. He cont inues t o c omp la in of dys est het ic pain in the affected dermatome. Hoping to reduce to patient's risk of developing chronic post-herpetic neural gia, you initiat e the rap y with (a) oxycodone (Roxicodone). (b) ibuprofen (Motrin). (c) carbamazepine (Tegretol). (d) amitriptyline (Elavil).

106 (d) Of the medications listed, only amitriptyline has been demon strated to reduce the incidence and severity of post-herpetic neuralgia. While carbamazepin e has well demons trated analgesic effects, it would be a poor choice in a bone marro w transp lant pati ent giv en t he ris k o f aggra vating leukopeni a. Sim il arl y, the an ti platel et e ffec ts of i bu prof en are un desir ab le i n th is c lin ica l con tex t. Wh ile o pioids have been shown to be effective in controlling neuropathic pain, adjuvant analgesics generally remain first-line therapy. Opioids have not been shown to influence the incidence of post-herpetic neuralgia.

106. After cardiac rehabilitation, patients with coronary artery disease remain asy mpto ma tic at ex ertio nal lev els t hat pr eviously pr ov oked an gi na. This change occurs primarily because of increased:(a) coronary collateralization and neoangiogenesis.:(b) serum hemoglobin concentrations.:(c) myocardial mitochondrial enzyme concentrations.:(d) oxidative enzyme concentrations in skeletal muscle.

106 (d) Peripheral training effects are largely responsible for the enhanced functional status of patients who undergo cardiac rehabilitation. Increased oxygen ext ract ion a nd a wid er arteriov en ous oxyg en dif fere nce hav e been descr ibed. Improve d ut ilizat ion of oxyg en by active mu sc les , bec ause o f an increase in o xi dative e nzyme s, is a n i mpo rtan t fa cto r.

106 Supplemental oxygen therapy in patients with chronic obstructive pulmonary disease (COPD) has been shown to (a) improve walking endurance. (b) increase blood pressures. (c) maximize work rate. (d) produce polycythemia.

106 Answer: (a) Commentary: Supplemental oxygen therapy is indicated in patients with arterial partial pressure of oxygen (PO2) continuously less than 55-60mmHg. Home oxygen therapy can decrease pulmonary hypertension, polycythemia, blood pressure, and pulse. In patients with mild hypoxemia and exercise desaturation, supplemental oxygen by nasal prongs did not influence maximum work rate, but did increase mean walking endurance time and exercise tolerance. Ref: Bach JR. Rehabilitation of the patient with respiratory dysfunction. In: DeLisa JA, Gans BM, Walsh, NE, editors. Physical medicine and rehabilitation: principles and practice. 4th ed. Philadelphia: Lippincott Williams & Wilkins; 2005. p 1850.

106. What is the target exercise intensity for optimal a erobic tr aini ng in a healthy, young individual? (a) Borg rating of perceived exertion 6. (b) 70%-80% of maximum heart rate. (c) Borg rating of perceived exertion 20. (d) 50%-60% of maximum heart rate.

106. (b) The target for this individual is 70% to 80% of maximum heart rate. One may use either this value or the Borg scale. The Borg scale ranges from 6 to 20. This person's goal is Borg 13 (somewhat hard), in the range of 11 to 15 (fairly light to hard).

106. A 47-year-old man with human immunodefici enc y virus (HIV) presents with fever, headache, and memory loss. The most likely diagnosis is (a) progressive multifocal leukoencephalopathy (P ML ). (b) HIV encephalopathy. (c) cryptococcal meningitis. (d) central nervous system (CNS) lymphoma.

106. (c) The patient most likely has cryptococcal meningitis. Fever would not be present in PML, HIV encephalopathy, or CNS lymphoma. In addition, headache is typically not a feature of PML or HIV encephalopathy. Ref: Levinson SF, Fine SM. Rehabilitation of the individual with HIV. In: DeLisa JA, Gans BM, Walsh NE, editors. Physical medicine and rehabilitation: principles and practice. 4th ed. Philadelphia: Lippincott Williams & Wilkins; 2005. p 1803.

107. A data entry operator presents to your office complaining of a 2-week hi story of bilat era l elbow pain that increases in severity the longer she types. She denies sensory changes or weakness. She has not had treatment. The only significant exam finding is pain on palpation of the proximal extensor forearm muscles just distal to the lateral epicondyles. You recommend (a) ice, physical therapy, evaluation of keyboard placement. (b) heat, extensor strengthening exercises, raising of keyboard heig ht. (c) ultrasound, elbow brace to limit flexion, raising of monitor hei ght. (d) iontophoresis, wrist splint to limit extension, lowering of moni tor heigh t.

107 (a) The most likely diagnosis in this case is lateral epicondylitis. In acute overuse injuries ice should be used first to reduce inflammation. Physical therapy may be used to facilitate a home exercise program for flexibility and strengthening exercises and to ensure appropriate posture adaptations. The computer set-up—including chair height and positioning of the monitor, keyboard, and mouse—will help resolve current complaints and reduce risk of re-injury. This ergonomic evaluation is multifactorial.

107. Cumulative trauma disorders:(a) develop as a result of repetitive macrotrauma.:(b) are most common in workers who work at a keyboard.:(c) occur after ergonomic recommendations have been implemented.:(d) result from repetitive microtrauma in the setting of poor ergonomics.

107 (d) Cumulative trauma disorders (CTDs) develop as a result of repetitive microtrauma to tissue in the setting of poor ergonomics. Any worker who performs a tas k re quiri ng repet itive loadi ng or moti on is at r isk. Th is includes keyboard oper ator s but is not excl usi ve to the m. Th e m ost c ommon oc cupational hazar d has not been deter mi ned . O ften , im ple men ting erg ono mic recomm endations can help reduce CTDs.

152. What is the most common medical complication during postacute stroke rehabilitation? (a) Venous thromboembolism (b) Falls (c) Depression (d) Pulmonary aspiration, pneumonia

152 Answer: D Commentary: Of the complications listed, aspiration/pneumonia is seen in about 40%, while venous thromboembolism is seen in 6%; falls occur in 16%, musculoskeletal complications in 5%, and reflex sympathetic dystrophy (RSD) in 30 %. Depression affects 30%. Urinary tract infection is just as frequent at 40%, but is not listed. Ref: Brandstater ME. Stroke rehabilitation. In: DeLisa JA, Gans BM, Walsh NE, editors. Physical medicine and rehabilitation: principles and practice. 4th ed. Philadelphia: Lippincott Williams & Wilkins; 2005. p 1671.

107. A firefighter who is now 5 days postsurgery for a r ota to r c uff and labral tear is in significant pain, but is concerned about opioid use for pain control. He is concerned about becoming "addicted to the pain killers." In educating the patient about opioids and the issues of addiction, dependence and tolerance, which statement is correct? (a) While all 3 terms have subtle differences, they are ess en tia lly identical in meaning and can be used interchangeably. (b) Since he is a firefighter, he should avoid use of an y o pi oid s at all times since he is subject to toxicology screening. (c) Addiction is predictable and avoidable, and since he al re ady concerned about it, he is unlikely to have problems with addiction. (d) Addiction is characterized by behavioral issues, whe rea s dep endence and tolerance are characterized by physiologic adaptation.

107 Answer: (d) Commentary: Physical dependence, tolerance, and addictio n a re di screte and different phenomena that are often confused. Addiction is characterized by behaviors that include one or more of the following: impaired control over drug use, compulsive use, continued use despite harm, and craving. Addiction is not a predictable drug effect, but represents an idiosyncratic adverse reaction in biologically and psychosocially vulnerable individuals. Physical dependence is a state of adaptation characterized by specific withdrawal symptoms that can be produced by abrupt cessation, rapid dose reduction, and/or administration of an antagonist. Tolerance is a state of adaptation that results in a decreased effect of a drug over time. Reference: Savage S, Covington EC, Heit HA, Hunt J, Jora nso n D, Schnoll SH. Definitions related to the use of opioids for the treatment of pain. American Academy of Pain Medicine, American Pain Society, American Society of Addiction Medicine consensus document; 2001. <http://www.ampainsoc.org/advocacy/opioids2.htm>

107. A data entry clerk pres ents with co mplain ts of right wrist and hand pain. She attributes the problem to prolonged use of the computer mouse. Which intervention should be included in the initial treatment plan?(a) Eccentric strengthening of t he wr ist exten sors with hand weights(b) Biweekly steroid injecti ons to mi nim ize sy mptoms(c) Wrist splint to minimize sym ptoms(d) Moving the mouse away fr om t he ke ybo ard

107. (c) Repetitive injuries from keyboard occupations are well recognized and require active rehabilitation for restorative function. Eccentric exercises will increase repetitive stress to the injured site and should not be used in the initial treatment recommendations. The prescription of biweekly steroid injections is excessive and may promote musculotendon atrophy and susceptibility to further injury. Moving the mouse further away from the keyboard will likely increase symptoms and dysfunctions. Wrist splints can be used during the initial treatment program to provide relative rest, reduce inflammation, and to provide comfort.

109. The most common movement disorder associated with c ere br al palsy is (a) spasticity. (b) dystonia. (c) athetosis. (d) rigidity.

109 Answer: (a) Commentary: While all of the listed movement disorders c an be se en in cerebral palsy, spasticity -- defined as velocity-dependent resistance to passive movement -- is the most common. Dystonia is described as involuntary sustained muscle contractions that result in twisting and abnormal posturing of the extremities. Athetosis is a slow, nearly continuous writhing movement, once a common result of kernicterus, coupled with chorea. Rigidity is resistance to movement of a joint because agonist and antagonist muscles are both contracting. It is not velocity dependent and is rarely found in cerebral palsy. Reference: Murphy N, Such-Neibar T. Cerebral palsy diagn osi s and management: the state of the art. Curr Probl Pediatrc Adolesc Health Care 2003;33:146-69.

109 A 14-year-old girl with spastic quadriplegic cerebral palsy (CP) has driven a power wheelchair as her primary means of mobility since age 4 years. She recently sustained a fracture of her tibia with no known significant trauma. You anticipate that the bone mineral density (BMD) z score for her distal femur as measured on a dual-energy x-ray absorptiometry (DEXA) scan will be (a) increased relative to peers without spasticity. (b) increased relative to younger nonambulatory children with CP. (c) similar to younger ambulatory children with CP. (d) decreased relative to age-matched peers without CP.

109 Answer: (d) Commentary: The common root among the affected muscles is L5. It is often difficult to narrow the involvement to a single root level. Ref: Dumitru D, Zwarts MJ. Radiculopathies. In: Dumitru D, Amato AA, Zwarts MJ, editors. Electrodiagnostic medicine. 2nd ed. Philadelphia: Hanley & Belfus; 2002. p 722, 752-3.

109. A 6-month-old child with L4 spina bifida presents to your clinic. He also has shunted hydrocephalus. Other than repair of his back and shunt placement, his past medical history has been negative. On examination, you find that he has full hip flexion against gravity and knee extension strength is at least 4/5. The infant has no movement around the ankle. Feet are in neutral position. Hip examination is symmetric. Which prediction is most accurate in this patient? (a) The child is likely to be a functional community ambulator by age 5 years. (b) The child is likely to be only a household ambulator. (c) The child is likely to learn to crutch walk by 18 months. (d) The child is likely to be only a wheelchair user.

109 Answer: A Commentary: This child has a strong quadriceps muscle and no deformities noted at 6 months of age. He is reported to be healthy. The best early predictor of ambulation in children with spina bifida is a strong quadriceps muscle. Negative predictors are spine and lower extremity deformities and obesity. Children do not typically learn to use crutches until 3 to 5 years of age or older. Ref: Molnar GE, Murphy KP. Spina bifida. In: Molnar GE, Alexander MA, editors. Pediatric rehabilitation. 3rd ed. Philadelphia: Hanley & Belfus; 1999. p 237-8.

109. Which statement is TRUE about swallowing in infants? (a) Sucking and swallowing are well-coordinated for oral intake by 3 4 weeks o f gestati on. (b) The infant's larynx is low, about the level of the sixth cervica l vertebr a. (c) The infant's tongue fills less of the oral cavity than the adult 's tongue . (d) Oral breathing occurs at birth and may interfere with sucking.

109. (a) Sucking and swallowing are well-coordinated at 34 weeks' gestation. Because the infant's tongue is more anterior than the adult's, the tongue fills more of the oral cavity than in the adult. The larynx in the infant is high, about at the C2-3 level. Newborn infants are obligate nose breathers and oral breathing is not observed until 3 to 4 months of age.

109. A 6-month-old child pre sent s in you r offi ce for rehabilitation assessment. She was born at full term. There was mild transient respiratory distress at birth. The patient was noted to be diffusely hypotonic at birth except for normal cranial nerves. There were no feeding issues once the respiratory distress resolved within 24 hours. The baby has remained relatively hypotonic since birth. However, she has become very socially alert and aware and attempts to use her arms to reach for toys and pick up lightweight objects. She doesn't roll. She cannot sit except very briefly when propped and bearing weight through both arms with elbows extended. On examination, head circumference is normal, length is normal, as is weight. There is a pronounced head lag. Arms, while in the supine position, maintain a "jug-handle" posture. Reflexes are present but diminished. There is no spasticity. The cranial nerves are normal except for fine fasciculations of the tongue. The most likely diagnosis is(a) myotonic muscular dystro phy.(b) cerebral palsy.(c) infantile botulism.(d) spinal muscular atrophy.

109. (d) Spinal muscular atrophy (SMA)is a term used to describe a group of inherited disorders characterized by weakness and muscle wasting due to degeneration of anterior horn cells of the spinal cord and brainstem motor nuclei. Three subtypes of autosomal recessive predominantly proximal SMA have been linked to chromosome 5q. The majority of cases of SMA type I present within the first 2 months of life with generalized hypotonia and symmetric weakness. Children typically sit only with support. Tongue fasciculations have been reported in 56%-61% of patients. Proximal muscles are weaker than distal.

11. Respiratory parameters for ventilator weaning include (a) rapid shallow breathing test result greater than 105 breaths per min/L. (b) minute volume greater than 10L/min. (c) static compliance less than 35cmH2O. (d) negative inspiratory pressure greater than 20cmH2O. Ref: Vecchione JJ, Sansone GR, Jawed I, Esan O, Ghumman C, Frengley JD. Crit eria for weani ng from p rolonged venti lato r support: how well do they work? [abstract]. Crit Care Med 2003;31(Suppl):A99. Educational Activity 2.2

11 (d) The shallow breathing test, static compliance, and negative inspiratory pressur e are the most i mpo rtant predi ctors o f patients likely to fail weaning. The shallow bre ath ing test or fre qu enc y/tidal vol ume should be less than 105 breaths per min/L. Static compliance should be greater than 35cmH2O and negative inspiratory pressure should be greater than 20cmH2O. Other weaning parameters include minute volume less than 10L/min, ability to trigger the ventilator, dynamic compliance greater than 25cmH2O, and respiratory rate below 25/min.

11. When applying cryotherapy in the treatment of musculoskeletal disorders, which of the following events is NOT a contraindication to its use? (a) Paroxysmal cold hemoglobinuria (b) Impaired sensation (c) Arterial insufficiency (d) Spasticity

11 Answer : (d) Commentary: Contraindications for the use of cryotherapy include paroxysmal cold hemoglobinuria, impaired sensation and arterial insufficiency. Other contraindications are cold hypersensitivity, cryopathies, cold intolerance, cryotherapy-induced neurapraxia, and Raynaud disease. Spasticity is one of the general uses of cryotherapy in addition to musculoskeletal injuries and pain syndromes, postoperative conditions and emergency treatment of minor burns. Reference: Weber DC, Hoppe KM. Physical agent modalities. In: Braddom RL, editor. Physical medicine and rehabilitation. 4th ed. Philadelphia: Elsevier-Saunders; 2011. p 458.

11 A 40-year-old woman presents with shoulder and neck pain . Her exam is un remar kab le ex ce pt for di scr ete, focal , h yp erir rit able s po ts loc ated in taut ban ds of sk ele tal mu scle . An appropriate treatment would be to a. inject lidocaine into skeletal muscle over area of ma ximal tenderness in a fa nwi se m ann er. b. inject corticosteroid into skeletal muscle over area of maximal tende rness in a fa nw ise ma nne r. c. inject lidocaine and corticosteroid mixture into acro mioclavicular jo int s pac e. d. inject corticosteroid into cervical epidural space un der fluoroscopy.

11 Option a is correct. This woman has myofascial dysfunction with one or more t rigger points. T rigge r p oin ts a re tre ate d w ith dry ne edl in g, a nes thet ic , or bot ulin um t oxin inj ectio ns. Co rti cos tero id injection into skeletal muscle can cause atrophy and necrosis. There is no indication for a joint or epidural injection. Findings more specific for acromioclavicular joint, glenohumeral joint, rotator cuff pathology, and stenosis are not noted.

11 A 25-year-old man comes to your office for evaluation of low back pain. As part of the physical examination, you mark a point at the L5 vertebral body an d ano the r point midline 10 cm above. You ask him to flex forward maximally while keeping his knees extended and measure the distance between the two points. This distance is 13.5 cm. You suspect he may have what diagnosis? a) Lumbar spondylolisthesis a. b) Scheuermann disease b. c) Lumbar herniated disc c. d) Ankylosing spondylitis

11 Option d is correct. Commentary: The Schober test is used to assess restricted range of motion seen in ankylosing spondylitis as the disease progresses. The distance between the 2 po ints with forward flexion exceeds 15 cm in normal individuals. This clinical test is not used for the assessment of lumbar spondylolisthesis, Scheuermann disease, or lumbar herniated discs. Reference: Borg-Stein J, Bermas B. Spondyloarthropathies. In: Slipman CW, Derby R, Simeone FA, Mayer TG, editors. Interventional spine: an algorithmic approach. 1st ed. Philadelphia: Elsevier; 2008.

11 New neurologic deficits in the upper extremities are not uncommon in patients with traumatic injury to the cervical spinal cor In this case, given the bilat eral and ascending nature of the sym ptom s, the most l ik ely diagno sis i s syrinx, which would be diagnosed with MRI. Ulnar and median nerve entrapments are the most common cause of upper extremity neurologic deficits, which can be confirmed by EMG. This patient, however, has a loss of wrist extension, which would be caused by damage to the radial nerve, not the ulnar or median. Head CT and lumbar puncture are least likely to provide diagnostic information. A 55-year-old woman with breast cancer consults you regarding the effects of exercise training on cancer-related fatigue. You state that the benefits of exe rcise on fatigue are greater in pati ents who a. have solid tumors, rather than hematological malignancies. b. are enrolled in progressive resistance exercise programs, rather than aerobic conditioning programs. c. are adult survivors of childhood cancers. d. have metastatic disease, rather than primary tumors.

11 Reference(s) (a) Cramp F, Byron-Daniel J. Exercise for the management of cancer-related fatigue in adults. Cochrane Database Syst Rev 2012 Nov 14;11. (b) Mi ller AM, Lopez-Mitnik G, Somarriba G , et al. Exercise c apacity in long term survivors of pediatric cancer: an analysis from The Cardiac Risk Factors in Childhood cancer survivors study. Pediatr Blood Cancer 2013 Apr;60(4):663-8. (c) Cheville AL, Kollasch J, Vandenberg J, et al. A home-based exercise program to improve function, fatigue, and sleep quality in patients with stage IV lung and colo rectal cancer : a randomiz ed co ntrolled trial. J Pain Symptom Manage 2013.;45(5):811-21. Option a is correct. A meta-analysis incorporating 1640 patients with cancer-related fatigue found that the benefits of exercise on fatigue were observed for interventions deliv ered during or postadjuvant cancer t hera py. Patients wi th solid t umors (breast, prostate) benefited more from exercise interventions than did patients with hematological malignancies. Aerobic exercise significantly reduced fatigue but resistance training and alternative forms of exercise failed to reach statistical significance. The data on exercise prescription for adult survivors of childhood cancers is scarce. These patients, in particular, should undergo cardiac screening before engaging in an exercise program because this group has a higher incidence of left ventricular dysfunction. Data are emerging on the potential of exercise intervention in patients with late stage colorectal and lung cancers for improving certain health-related quality-of-life variables, such as mobility, sleep quality and fatigue

157. Which factor contributes most to poor worker motivation during rehabilita tion? (a) Loss of worker identity (b) Boredom while away from work (c) Good evaluations by a supervisor prior to injury (d) Worker's perception of severity of pain

157 (a) Loss of worker identity caused by depression and anxiety is associated with poor motivation during the rehabilitation process. Other associated factors include d eterior ating financial status, change in family roles, loss of control reg ar ding t he future , and d eter iorati ng employe r relationships.

110. For which individual would a high-profile, dynamic elastic response (energy-st orin g) pr ost het ic fo ot a nd ankle me cha nism be mos t indi cat ed ?:(a) 75-year-old man with a transtibial amputation who lives in a nursing home:(b) 53-year-old woman with a transfemoral amputation who is a recreational swimmer:(c) 60-year-old man with a transtibial amputation who enjoys jogging:(d) 43-year-old man with a transfemoral amputation who farms

110 (c) High-profile, dynamic elastic response prosthetic feet and ankle components such as the Flex-Foot and Springlite are primarily indicated for individuals wh o ar e exp ect ed to be communi ty ambulat ors an d ar e ab le to ambulate at variable c aden ces. T his class o f p rosthetic co mp one nts i s also u sed for ind ividu al s partic ipati ng in r unn ing and spo rts ac tivit ies . T her efore, this class of components would be most indicated for the individual with a transtibial amputation interested in returning to running. An individual who farms would likely benefit from a prosthetic foot and ankle system which is more accommodative over uneven surfaces.

110. Which wheelchair component is appropriate for a pat ien t wit h T10 spinal cord injury? (a) Quick release axle (b) Projection rims (c) Arm trough (d) Tilt-in-space system

110 Answer: (a) Commentary: Quick release axles allow persons with spina l c or d i njury who drive to load their wheelchairs into the car more easily. Projection rims assist with wheelchair propulsion in patients who have insufficient hand function. The tilt-in-space recline system offers independent pressure relief in patients with tetraparesis. Arm troughs support the arms and forearms of persons with limited upper limb strength. A patient with T10 spinal cord injury has sufficient upper limb and trunk control so that projection rims, arm trough, and tilt-in-space features are not necessary. Reference: Furumasu J, Gilinsky G, Krapfl BJ. Positioni ng an d w heeled moblity for children and adults with disabilities. In: Goldberg B, Hsu JD, editors. Atlas of orthoses and assistive devices. 3rd ed. St. Louis: Mosby; 1997. p 588, 595, 597, 600.

110. The primary advantage of a soft insert fitted into the socket of a transtibial prosthesis is that it is (a) perspiration resistant. (b) easy to keep clean. (c) easily modified. (d) very durable.

110 Answer: C Commentary: Soft inserts are fabricated to fit inside the socket. They are recommended for patients with thin, sensitive, or scarred skin, or peripheral vascular disease (PVD). They are easily modified. Hard sockets also have their advantages. They are perspiration resistant, less bulky than sockets fitted with a soft insert, easy to keep clean, and durable. Further, reliefs or modifications can be located with precision in the hard socket. Ref: Kapp SL, Fergason JR. Transtibial amputation: prosthetic management. In: Smith GD, Michael JW, Bowker JH, editors. Atlas amputation and limb deficiency. 3rd ed. Rosemont: AAOS; 2004. p 508.

110. The primary advantage of a 4-point crutch gait over a 2-point c rutch gai t is (a) stability. (b) speed. (c) weight-bearing relief. (d) efficiency of gait.

110. (a) The 4-point crutch gait has stability as its primary advantage. At least 3 points are always in contact with the ground. It is more difficult to learn than the other gait patterns and is a relatively slow form of ambulation. The 3-point crutch gait is used by patients with lower limb fractures, amputations, or toe-touch weight-bearing. The 4-point gait pattern enables the crutch user to eliminate all the weight-bearing on the affected lower limb. The 2-point crutch gait is much faster than the 4-point gait and yet still provides some weight-bearing relief to both lower limbs.

110. A patient presents for eval uatio n o f his new ankle-foot orthosis (AFO). During ambulation you notice that he has reduced hip extension and stride length, and a slowed gait. At the initiation of stance phase, heel strike is not consistently present. You conclude that the abnormal gait is due to excessive(a) knee extension at heel s trik e and re commen d setting the AFO in greater dorsiflexion. (b) hip flexion contracture and recommend setting the AFO in greater dorsiflexion.(c) knee flexion at heel str ike and r eco mmend setting the AFO in greater plantar flexion.(d) dorsiflexion of the ankl e an d rec omm end se tting the AFO in plantar flexion.

110. (b) Excessive knee extension at heel strike is frequently seen with hamstring weakness or gastrocnemius muscle spasticity. It causes the patient to walk on the heel, with external rotation of the leg and no knee flexion at heel strike. Excessive knee flexion at heel strike is frequently seen with an ankle-foot orthosis (AFO) that is set in too much dorsiflexion. This setting causes the midstance period to be reduced and the push-off effect diminished, causing the knee to be excessively flexed and thereby slowing the gait. If the tip of the shoe on the AFO side is raised too high at heel strike, the AFO is set in too much dorsiflexion. Only answer b addresses all of the abnormalities seen in this individual.

110. The primary goal of a knee orthosis is to (a) prevent knee injury in athletes. (b) control knee instability in the anterior direction. (c) prevent recurvatum. (d) decrease the quadriceps force across the knee.

110. (c) Knee orthoses are prescribed to prevent genu recurvatum and provide mediolateral stability. They may be used during sports and other activities to provide functional support for an unstable knee or during the rehabilitation phase following injury or surgery on the knee. The use of knee orthoses for the prevention of knee injury in athletes is controversial. The Swedish knee cage prevents recurvatum but permits flexion. The three way knee stabilizer gives good control of structural knee instability in the lateral, medial, and posterior directions.

111. A 5-year-old boy is brought to your office with pain in his left groin for the las t 2 m ont hs. He h as a n a ntalg ic gai t and fa vor s his lef t leg. He has been unable to play with his siblings because of pain and reports increased severity of the pain at night. There is no history of trauma. Passive range of motion of the left hip is extremely painful. Which of the following is most likely?:(a) The child is not telling the truth about the trauma for fear of being punished by h is pa ren ts.:(b) A simple radiograph will demonstrate this problem to be a benign syndrome that only r equ ire s m onito ring .:(c) This child is at risk for permanent disability and treatment requires the use o f an a bdu cti on brace .:(d) You are very concerned about physical abuse by the parents, since the child is very q uie t w hen the pare nts are in the examina tio n room .

111 (c) This child has Legg-Calvé-Perthes disease. This is avascular necrosis of the femoral head (also known as idiopathic osteonecrosis of the femoral head) a nd typi cally af fects children b et ween the ag es of 4 and 8 years. Boys are affected 4 ti mes mo re than gir ls are, and it is un ilate ral in 9 0% of child ren. It is unco mmon in Af ri can Am eric an c hil dre n. Th e b one di es and loses its structural integrity, leading to collapse of the femoral head with deformity and arthritis. An abduction brace is worn all day in an attempt to position the femoral head in the acetabulum. This will hopefully lead to a more spherical head of the femur. The femoral head can revascularize and remodel, but this occurs over several months. Osteotomy is usually reserved for older children.

111. A 35-year-old man presents to your clinic with a 3-month history of groin pain exacerbated by activity. He is an avid skier and runner. He has been taking anti-inflammatories with minimal relief. Anteroposterior films of the hip were normal. The magnetic resonance imaging of the hip reported a bony prominence at the femoral head-neck junction. What clinical exam finding is most likely to correlate with these radiographic "abnormalities" ? (a) Pain with resisted straight leg raise (b) Pain with hip flexion, external rotation, and abduction (c) Pain with sacral thrust (d) Pain with hip flexion, internal rotation, and adduction

111 Answer:D Commentary: This patient has radiographic evidence of femoroacetabular impingement. Two types have been described, cam impingement and pincer impingement. Cam impingement is described more commonly in active males and describes a non-spherical femoral head or osseous abnormalities of the femoral head-neck junction. These bony abnormalities have abnormal contact with the acetabulum in hip flexion, adduction, and internal rotation. Pincer impingement describes abnormal contact between the femur and the acetabulum due to overcoverage of the femoral head from an abnormally deep or retroverted acetabulum. Ref: Standart C, Manner PA, Herring SA. Expert opinion and controversies in musculoskeletal and sports medicine: femoroacetabular impingement. Arch Phys Med Rehabil 2008;89:890-3. 112. The usual time of onset of diabetes insipidus in patients with traumatic brain injury is

111. Which nerve does NOT innervate the outer ann ul us of the lumbar intervertebral disc? (a) sinuvertebral nerve (b) lumbar medial branches of dorsal rami (c) grey rami communicantes (d) lumbar ventral rami

111. (b) The lumbar medial branches of the dorsal rami supply the facet joints as well as the deep paraspinals, such as the rotators and multifidi. The sinuvertebral nerve, also termed the recurrent meningeal nerve is the primary source of nerve supply to the lumbar intervertebral disc. It is derived from portions of the ventral rami and grey rami communicantes (sympathetic input). Accordingly, the referral pattern seen with intrinsic disc pain is vague and diffuse. Ref: ISIS Practice Guidelines for Spinal Diagnostics and Treatment Procedures, 2005; p 22.

111. A 28-year-old non-pregnant woman presents with persistent lanci nating pa in along her r igh t zygomatic arch. The pain is intermittent and often triggered by chewing. What is the initial management? (a) Transcutaneous electrical nerve stimulation (b) Carbamazepine (Tegretol) (c) Botulinum toxin (Botox) (d) Cervical epidural injection

111. (b) The patient suffers from trigeminal neuralgia (tic douloureux). This spasm is thought to be due to intracranial vascular compression of cranial nerve V. Often the pain is triggered by non-noxious stimuli and from an area separate from the area of pain. Trigeminal neuralgia is paroxysmal and often responsive to anticonvulsants. Carbamazepine appears to be particularly effective. Capsaicin and gabapentin have proven benefit for postherpetic neuralgia but not trigeminal neuralgia.

111. Which statement describes the chronic-pain concept of "centr al s ensitization"? (a) The evoked response of A-delta fibers to subsequent input is ampl ified. (b) The influx of sodium is fundamental to electrical si gnaling a nd s ubsequent generation of action potentials and excitatory postsynaptic potentials. (c) A complex set of activation-dependent post-translati onal chan ges occurs at the dorsal horn, brainstem, and higher cerebral sites. (d) The so-called "inflammatory soup," rich in algesic s ubstances , ca uses a lowering of threshold for activation and subsequent evoked pain.

111. (c) Central sensitization is a complex set of activation dependent post-translational changes occurring at the dorsal horn, brainstem, and higher cerebral sites that sensitizes the central nervous system to further perception of pain. Wind-up is an amplified evoked response to repeated afferent inputs at the level of the dorsal horn.

112. Which measure will ensure the best outcome for a 68-year-old man who is in the cri ti cal ca re unit with se vere tra uma tic brai n i nj ury?:(a) Minimizing cerebral perfusion pressure:(b) Minimizing early hypoxia:(c) Avoiding hypertension:(d) Inducing hypothermia

112 (b) Hypoxia in the setting of brain injury is associated with poor outcome. Maintaining perfusion pressure and avoiding hypotension are important critical c are mea sures to avoi d secondary c omplicat ion s i n br ain inj ury. Inducin g hypothermia has not b een found t o i mprove ou tco me in pati ents w it h brain inj ury.

113. A 50-year-old man complains of malaise, f ati gu e, and hand arthralgias for the past several months. He was recently diagnosed with diabetes mellitus. On exam, he has mild tenderness to palpation in his bilateral second and third metacarpophalangeal (MCP) joints with erosive lesions on radiographs. He also has a generalized bronzing of his skin. What is the most appropriate initial test to order? (a) Complete blood count (CBC) (b) Iron studies (c) Erythrocyte sedimentation rate (ESR) (d) Adrenocorticotropic hormone (ACTH) stimula tio n test

113. (b) Hemochromatosis is the diagnosis. It is a commonly inherited, autosomal recessive disorder (5 in 1000 persons) affecting Caucasians of European descent typically in the fourth and fifth decade of life. In hemochromatosis, arthralgias may be the first symptom and are classically in the second and third MCP and proximal interphalangeal (PIP) joints, resembling osteoarthritis (OA) on radiographs. However, OA typically affects the distal interphalangeal joints. The CBC, ESR, and ACTH tests are normal in hemochromatosis. Ref: (a) Pietrangelo A. Hereditary hemochromatosis - A new look at an old disease. N Engl J Med 2004;350:2383-97.(b) Gordon DA. Storage and deposition disease. In: Klippel JH, editor. Primer on rheumatic disease. 12th ed. Atlanta (GA): Arthritis Foundation; 2001. p 459-60.

113. A 66-year-old woman with rheumatoid arthritis is ad mitted to inp atient rehabilitation following a right total knee arthroplasty (TKA). On her initial day of therapy, she had difficulty walking with her physical therapist. Her medications included methotrexate (Trexall), etanercept (Enbrel), ezetimibe (Zetia), multi-vitamin, calcium with vitamin D, alendronate (Fosamax), acetaminophen/hydrocodone (Norco), and warfarin (Coumadin). Re-examination shows hip flexion 5/5, knee extension 4/5, knee flexion 4/4, ankle dorsiflexion 1-2/5, ankle plantar flexion 5/5. You suspect (a) sciatic nerve stretch injury. (b) posterior tibialis tendon rupture. (c) inadequate pain control. (d) peroneal nerve injury.

113. (d) Temporary weaknesses of peri-articular muscles typically occurs after knee arthroplasty along with loss of full flexion and extension due to pain, edema, and the procedure itself. Ankle dorsiflexion is not typically weak following TKA and therefore peroneal nerve injury due to a hematoma would be suspected, especially since the patient is on warfarin. This injury requires surgical exploration and decompression. Sciatic nerve stretch injury, posterior tibialis tendon rupture, and inadequate pain control would not present as ankle dorsiflexion weakness.

113. For the past 2 months, a 75 year ol d man has had low back pain that radiates in a posterolateral distribution down his right leg when he stands or ambulates. The pain is relieved by pushing a cart in the store. He has a non-focal neurologic examination and an x-ray rules out metastatic disease. Your initial management plan should include(a) neurosurgical evaluation .(b) lumbosacral corset.(c) epidural steroid injecti on.(d) flexion based exercises.

113. (d) This patient describes classic neurogenic claudication due to spinal stenosis. He has a non-focal examination and should be managed conservatively initially with flexion based exercises. Epidural steroid injections may be warranted if the patient does not respond to initial therapeutic exercise.

114. A 55-year-old man with limb girdle muscular dystrophy has sever e lordosi s and chr onic low back pain. What is your initial recommendation? (a) Thoracolumbosacral orthosis (b) Aquatic therapy program (c) Lumbar facet intra-articular steroid injections (d) Core strengthening exercises

114. (b) People with limb girdle muscular dystrophy have significant weakness in the abdominal muscles and the spinal extensors, leading to lordosis. Strengthening exercises will not likely help, nor will facet injections. Aquatic therapy will help unload his spine and will help maintain mobility. Putting the patient into a thoracolumbosacral orthosis will force his center of gravity too far forward, making him fall over.

114. Which muscles are usual ly i nnerv ate d by t he median nerve?(a) Third and fourth lumbric als(b) Third and fourth dorsal inte rosse i(c) First and second lumbric als(d) First and second dorsal inte rosse i

114. (c) The ulnar nerve innervates all the dorsal interossei muscles and the third and fourth lumbricals. The median nerve innervates the first dorsal interosseus muscle in about 1% of the individuals. Rarely, the radial nerve innervates the first dorsal interosseus. The first and second lumbricals are innervated by the median nerve.

115. The management of acute spinal cord compression due to tumor includes (a) nerve growth factor. (b) thoracolumbosacral orthosis (TLSO). (c) corticosteroids. (d) urecholine.

115 (c) Corticosteroids are indicated in acute spinal cord compressi on in an effort to reduce the tumorrelated inflammatory changes and prostaglandin productio n. The dos e, howe ver, is controversial. Radiation therapy is indicated for patients wi th spi nal cord compres sion due t o s oft tis sue encroachment . I t c an be em ploy ed a s monot hera py i n cas es o f sp ina l s tab ility , with o r with out ne urolo gic c han ges, or as an adj unctive therapy to surgery for patients with spinal instability. The major complications of radiation treatment include the development of radiation myelopathy, radiation plexopathy, and tumor recurrence. However, radiation therapy is indicated for these individuals to provide decompression of neural structures and cytoreduction of the tumor, prevention of neurologic progression and prevention of local recurrence, and for relief of pain. Most surgeons will not consider surgical intervention if the patient has a prognosis of less than 3 months' survival when faced with the question of surgical management of tumors. With either a primary or metastatic spine tumor, commonly a corpectomy will be performed. In this procedure most of the involved vertebral body and the intervertebral disk above and below the involved vertebra are excised. Since the majority of corpectomies for spine tumors involve metastatic disease, the goal of surgery is palliation rather than sur gical cure . Corticosteroids are indicated in acute spinal cord compressi on in an effort to reduce the tumorrelated inflammatory changes and prostaglandin productio n. The dos e, howe ver, is controversial. Radiation therapy is indicated for patients wi th spi nal cord compres sion due t o s oft tis sue encroachment . I t c an be em ploy ed a s monot hera py i n cas es o f sp ina l s tab ility , with o r with out ne urolo gic c han ges, or as an adj

115. Which statement is TRUE regarding treatment of spasticity in in dividuals with spi nal c ord injuries? (a) Over 75% of individuals require treatment for their spasticity a t the tim e of disc harge fr om rehabilitation. (b) The use of a muscle relaxant such as carisoprodol (Soma) is effe ctive in spasticit y of spi nal origin. (c) Botulinum toxin injections are effective because of their long ( 6-12 mont h) durati on of ef fect. (d) Intrathecal baclofen delivery often eliminates the need for oral antispas ticity me dicat ion .

115. (d) Less than 40% of individuals with spinal cord injuries require treatment for their spinal cord injury at the time of discharge from acute rehabilitation. There is no evidence to support the use of carisoprodol in spasticity of spinal origin. The average duration of effect for botulinum toxin is 3 to 6 months.

116. At the time of seroconversion, an HIV-infected patient treated prophylact ically w ith azat hiopr ine ( AZT) for 4 mon th s dev el ops r apidl y pro gre ss ive we akn ess of a ll extremities over several days with sensory sparing. Distal and proximal weakness are comparable. The most likely cause of weakness is (a) Chronic demyelinating polyneuropathy. (b) Acute demyelinating polyneuropathy. (c) HIV myopathy. (d) AZT myopathy.

116 (b) AIDP generally develops over days in HIV patients early in t he course of disease. Often AIDP presents at the time of seroconversion. While CIDP may als o develop early i n the disease course, it usually develops gradually, over weeks. My op athy d oes not o ccur at a p articu lar stage in the disease. Sim ila r to mos t my op at hic pro cess es, HI V m yo pa thy aff ect s p roxim al mus cl es to a g rea ter e xte nt . A ZT my opathy prese nts after 9 or more months of drug use.

116. For optimal sensitivity, cardiac stress testing conducted 4 to 8 weeks followi ng m yo car dia l i nfarc tion mu st pr oce ed to what per ce ntag e o f a patient's age-adjusted maximal heart rate?:(a) 60%:(b) 75%:(c) 85% :(d) 90%

116 (c) For both functional and diagnostic testing, patients must proceed to 85% of their age-adjusted maximal heart rate. Otherwise half of the abnormalities w ill be misse d. Patie nts on beta b lockers sho uld be test ed to a work lo ad that would inc ur oxy gen consump tio n of 80% of ma xim al ox ygen c on suption (V0 2.max ).

117 A recent study of individuals undergoing a single knee or hip replacement surgery who were treated at an inpatient rehabilitation facility (IRF) compared to those treated at a skilled nursing facility (SNF) found that those treated in an IRF were more likely to (a) need the use of a walker to ambulate. (b) require home care services. (c) be discharged home. (d) ambulate a shorter distance.

117 Answer: C Commentary: Patients undergoing single knee or hip replacement surgery who were treated at an IRF were more likely to be discharged home, less likely to require home care services upon discharge, and were able to ambulate farther distances compared to those treated in an SNF. Ref: Walsh MB, Herbold J. Outcome following rehabilitation for total joint replacement at IRF and SNF: A case-controlled comparison. Am J Phys Med Rehabil 2006;85:1-5.

117. A factory owner consults you to make recommendations on how to reduce wo rkers' co mpens ati on claims. You recommend evaluation of workers' ergonomics and equipment. Educating the employer regarding psychosocial factors is important because workers (a) who do not enjoy their job are more likely to report a back inju ry. (b) will claim low back injuries for attention. (c) employed in factories often have a psychiatric history. (d) claiming injuries after ergonomic evaluation are malingerers.

117. (a) Bigos's study of over 3,000 aircraft employees identified risk factors for low back injury that are outside of the physical factors. Workers who hardly ever enjoyed their job were 2.5 times more likely to report a back injury than those who "almost always" enjoyed their job. This study suggested that identifying factors outside of the physical and ergonomic model was important in determining risk of injury. No studies demonstrate that factory workers often have mental illness.

118. When performing needle electromyography in someone the ra peu tically anticoagulated with warfarin, which muscle would be the safest to examine? (a) Tibialis posterior (b) Paraspinal muscles (c) Deltoid (d) Flexor pollicis longus

118 Answer (c) Commentary: Although needle EMG is generally safe in pa tie nt s t aking warfarin, whenever bleeding risk is increased it is best to limit the needle examination to a few superficial muscles, where prolonged compression can be performed if necessary. The pronator teres and tibialis posterior are deep muscles in which, theoretically, a hematoma could develop. It is also best to avoid needle insertion into areas where possible hematoma could compress a nerve or artery, such as exiting spinal nerves near paraspinals or the radial artery near the flexor pollicis longus. Reference: Preston DC, Shapiro BE. Electrical safety an d i at rog enic complications of electrodiagnostic studies. In: Preston DC, Shapiro BE, editors. Electromyography and neuromuscular disorders: clinical-electrophysiologic correlations, 2nd ed. Philadelphia: Butterworth-Heinemann; 2005. p 658-9.

119. Intrathecal baclofen decreases spasticity by what m ech an ism ? (a) It blocks Ia afferent signals through the dorsal roo t g an gli a. (b) It acts as a GABA agonist to inhibit gamma motor neu ron a cti vity and decrease muscle spindle sensitivity to spinal reflexes. (c) It blocks acetylcholine release from neurons at the per ip her al neuromuscular junction. (d) It inhibits calcium release from the sarcoplasmic re tic ul um during muscle contraction.

119 Answer: (b) Commentary: Intrathecal baclofen acts as a GABA agonist inh ib iti ng the spinal reflex arc stimulated by intrafusal muscle fiber stretch. Selective dorsal rhizotomy seeks to diminish spasticity by decreasing the afferent signal by cutting rootlets in the dorsal root ganglia. Botulinum toxin blocks acetylcholine release at the neuromuscular junction. Dantrolene sodium decreases muscle contraction by inhibiting calcium release from the sarcoplasmic reticulum. All these therapeutics can be useful in the treatment of spasticity. Reference: (a) Grabb PA, Doyle JS: The contemporary surg ica l man agement of spasticity in children. Phys Med Rehabil Clin N Am 2001;12(4):907-22. (b)Yablon S. Botulinum neurotoxin intramuscular chemoden erv at ion : role in the management of spastic hypertonia and related motor disorders. Phys Med Rehabil Clin N Am 2001;12(4):83374. (c) Hinderer SR, Dixon K. Physiologic and clinical monitoring of spastic hypertonia. Phys Med Rehabil Clin N Am 2001;12(4):733-46.

119. You suspect that a 10-month-old boy's subdural hematoma may be the result of child abuse. Which action is LEAST likely to be helpful in further evaluation? (a) Skeletal survey (b) Physical examination for cutaneous injuries (c) Hematology consultation (d) Ophthalmology consultation

119 Answer: (c) Commentary: The most common cause of serious brain injury in children younger than 1 year of age is abuse. In very young children subdural hematoma, subarachnoid hemorrhage, retinal hemorrhages and associated cutaneous, skeletal and visceral injuries are significantly more common among those with inflicted brain injury than in children with unintentional injury. Ref: Reece RM, Sege R. Childhood head injuries: accidental or inflicted? Arch Pediatr Adolesc Med 2000;154:11-5.

119 Which statement is true about the asymmetric tonic neck reflex (ATNR) or the symmetric tonic neck reflex (STNR)? (a) The STNR is present at birth and fades away by 1 year. (b) The STNR provides postural stability as the child goes from crawling to standing. (c) The ATNR appears about 6 months of age and fades away by 1 year. (d) The ATNR is obligatory in all children at certain ages.

119 Answer: B Commentary: The ATNR is also known as the fencer position and is a neonatal reflex that disappears by 6 months of age. It is never obligatory in normal children. The STNR appears about 6 months and disappears by 1 year. It provides postural stability as the child makes the transition from crawling to standing. Ref: Pidcock FS, Christensen JR, Trovato MK. General concepts of pediatric rehabilitation. In: O'Young BJ, Young MA, Stiens SA, editors. Physical medicine and rehabilitation secrets. 3rd ed,Philadelphia, Mosby Elsevier; 2007. p 657.

12. What is the greatest risk factor for late post-traum atic seiz ures in patients with a traumatic brain injury? (a) Multiple subcortical contusions (b) Subdural hematoma with evacuation (c) Midline shift greater than 5mm (d) Bilateral parietal contusions

12 (d) In a 4-site Model System Center observational study, the highest risk factors for late post-traumatic seizures were found to be bilateral parietal contusion (66%), penetration of the dura (62.5%), and multiple intracranial operations (36.5%), multiple subcortical contusions (33.4%), subdural hematoma with evacuation (27.8%), and midline shift greater than 5mm (25.8%).

12 Your patient has a tremor of 5-8Hz, which is made worse with activity, and there is little or no tremor at rest. This finding best describes which type of tremor? (a) Parkinsonian (b) Essential (c) Enhanced physiological (d) Cerebellar

12 Answer: B Commentary: An essential tremor is more prominent with activity (5-8Hz) and is diminished at rest. Stress exacerbates the tremor. The essential tremor can be confused with parkinsonian tremor. Essential tremor is more prominent with activity, while parkinsonian tremor is more prominent at rest, with a 4-5Hz frequency. There is also an absence of the other symptoms of parkinsonism such as loss of postural reflexes, rigidity and bradykinesia. Enhanced physiologic tremor is a high-frequency tremor that is most prominent with posture and action. It is exacerbated by anxiety, fatigue and many drugs. It can be seen with alcohol withdrawal. The cerebellar tremor has a frequency of about 3Hz and is mainly in a horizontal plane. It is most prominent with fine repetitive action of the extremities and is associated with other signs of cerebellar ataxia. Ref: (a) Louis ED. Essential tremor. In: Rowland, LP, editor. Merritt's neurology. 11th ed. Philadelphia: Lippincott Williams & Wilkins; 2005. p 827-8. (b) Misulis KE. Neurologic localization and diagnosis. Boston: Butterworth-Heinemann; 1996. p 57-8.

121 Which clinical scenario is most consistent with a L4-5 foraminal disc herniation? (a) Weakness of the extensor hallicus longus, decreased sensation of the web space between the first and second toes, absent hamstring reflex (b) Weakness of the gastrocnemius, decreased sensation of lateral foot, absent Achilles reflex (c) Weakness of the anterior tibialis, decreased sensation of the web space between the first and second toes, absent hamstring reflex (d) Weakness of quadriceps and anterior tibialis, decreased sensation of medial lower leg, absent patellar reflex

121 Answer: (d) Commentary: A foraminal disc herniation at L4-5 level would most likely affect the exiting L4 nerve root. A nerve root lesion could result in muscle weakness in the affected myotomes, sensation loss in the affected dermatomes, and deep tendon reflex changes. The physical examination findings most consistent with a lesion to the L4 nerve root would be weakness of the quadriceps (L2-4), decreased sensation in L4 dermatomes, and decreased or absent patellar deep tendon reflex (L4). Ref: Ellenberg M, Honet JC. Lumbar radiculopathy. In: Frontera W, Silver JK, Rizzo, TD. Essentials of physical medicine and rehabilitation. Philadelphia: Elsevier; 2008. p 241-6.

12 Full neuropsychological assessment to test complex menta l abilities afte r a m ode rat e to se ver e t rau mati c brai n i nj ury sho uld be c omplet ed a. before discharge from the acute rehabilitation hospit al, to establish a ba sel ine o f neu rol ogi c f unct ion pr ior t o tr ans itio n ba ck int o th e co mmuni ty. b. during the initial stages of emergence from coma, to aid the rehabili tatio n s taf f in ma xim izi ng reco very a nd ne w le arn ing. c. before discharge from the acute rehabilitation hospit al, to document progr ess th ro ug h p ost tra uma tic amnesi a a nd hel p d eter mi ne the p rogn osis of r eco very. d. when the mental endurance and ability of the patient to complete the testi ng pro ce ss is ad equ ate , an d it i s n ee ded to answ er q uestio ns r elat ed to co mmuni ty act ivi tie s or treatment plan.

12 Option d is correct. Full neuropsychologic test batteries are often not admin istered for seve ral m ont hs af te r m ode rat e t o se vere t rau ma tic bra in i nj ur y (TBI ). T he t estin g m ay be us ed to ans wer questions related to return to school, work, driving or other activities, as well as to assess ongoing recovery. After a traumatic brain injury the recovery of physical abilities and functional skills such as mobility occurs rapidly, often within 3 months after injury. However, the recovery of more complex mental abilities, as assessed by neuropsychological measures, appears to be more variable. This aspect of recovery has been studied in large samples in the Traumatic Coma Data Bank study and in the TBI Model Systems Project. Impairments of attention, information processing speed, memory, and executive function have been shown to be persistent up to 10 years postinjury.

12 Non-accidental traumatic brain injury in young children is characterized by the clinical triad of subdural hematoma, retinal hemorrhages, and encephalopathy . As the child's history is taken, w hich finding is c la ssically n oted? a) The history given is consistent with the severity of injuries. b) The injuries are reported as unwitnesse c) Multiple care providers were present at time of injury. d) Medical attention was sought promptly after the injury.

12 Reference(s) Krach LE, Gormley ME, Ward M. Traumatic brain injury. In: Alexander MA, Matthews DJ, editors. Pediatric rehabilitation. 4th e New York: Demos M edical; 2010. p 233. Option b is correct.

121. A 43-year-old alcoholic man complains of left hip pain that is increased with wei ght bear ing. He ha s had pro gress iv e sym pt oms f or ab ou t 3 mo nt hs. Ad dit ion al me dical history reveals no trauma, but the patient has oxygen dependent chronic obstructive pulmonary disease (COPD). On examination, internal and external rotation of both hips is painful. The patient ambulates with a lateral lean to the left. He uses a walking stick, which helps his pain. What radiologic evaluation would be most useful in diagnosing this patient's problem? (a) Ultrasound of the left hip (b) Magnetic resonance imaging of both hips (c) Arthrography of the left hip (d) Tagged white blood cell scan of the hip

121 (b) This patient has multiple risk factors for osteonecrosis of the hip (avascular necrosis). Risk factors for this condition include steroid use, alcohol use, traum a, rheu matoid arthritis, sickle cell disease, radiation, chronic pancreati ti s, sys temic lup us eryt hema tosus, Ga ucher d isease, and Croh n's di se ase . I t ty pi ca lly occ urs in t he th ir d to f ift h d eca des a nd aff ec ts 10, 0 to 20,0 0 ne w p atien ts per year in the US. This condition is often bilateral. Radiologic evaluation should include plain anterior-posterior pelvis and frog lateral views of the hip. If collapse of the femoral head is seen, no additional radiographic evaluation is necessary. If these are negative, magnetic resonance imaging of the hip may be more sensitive and should be performed on both hips. Ultrasound and arthrography are not indicated for this condition. Tagged white blood cell scan would evaluate for infection of the hip.

124. A 25-year-old man presents with a 3-week history of left arm weakness after a fall f rom a mot orcyc le. He has d ecr eas ed pinpr ick s ensa tio n over the deltoid area, the lateral forearm, and the thumb and index finger. Which test is most useful in prognostication?:(a) Electrodiagnostic testing:(b) Magnetic resonance imaging of the cervical spine:(c) Magnetic resonance imaging of the shoulder:(d) Computed tomography myelogram

124 (a) Electrodiagnostic testing at this point would be an appropriate test, given that the patient is 3 weeks postinjury. It could help differentiate a pre- v ers us p ostdo rsa l roo t ganglion le sion. Al tho ugh rad iolo gic studies can assess for p ossi ble ro ot avulsion th ey do not as se ss the p hysiol og ic function of t he periphe ral n eural e lem ent s.

121 A 55-year-old woman presents to the clinic with a 6-week history of right wrist pain. She is an administrative assistant and has been working extra hours for the past 3 months. She has been taking anti-inflammatory medications without relief. You diagnose her with de Quervain tenosynovitis. What is the next most appropriate step in treatment? (a) Trial of a higher dose of anti-inflammatory medication (b) Surgical consultation (c) Corticosteroid injection (d) Splinting the wrist

121 Answer: C Commentary: Corticosteroid injection for de Quervain tenosynovitis has been shown to be more effective treatment than splinting and anti-inflammatory medications. Ref: Foye PM, Sullivan WJ, Panagos A, Zuhosky JP, Sable AW, Irwin RW. Industrial medicine and acute musculoskeletal rehabilitation. 6. Upper- and lower-limb injections for acute musculoskeletal injuries and injured workers. Arch Phys Med Rehabil 2007;88(Suppl):S30.

121. A 19-year-old male comp etit ive s wim mer co mplains of medial knee pain for the last 6 weeks. He specializes in the breaststroke, and his current practice regimen consists of sprinting 1 hour per day 4 days per week. On exam, he has no effusion or crepitus. His Lachman, McMurray and pivot shift tests are all negative, and there is no significant laxity. His Q angle is 24° with his knee extended. His strength is normal. Which action should be recommended?(a) Strengthen his quadricep s, c oncen tra ting o n the last 30° of extension.(b) Modify his swimming prog ram to 3 hou rs per day of endurance training 6 days per week.(c) Quit swimming, as he doe s no t hav e t he kne e joint architecture for the sport.(d) Refer for arthroscopic e valu ation fo r inte rnal derangement.

121. (a) The breaststroke has an associated "whip kick" which puts stress on the medial knee. For proper propulsion, the knee must be in a significant valgus position during leg extension. This can lead to overuse of the adductors and the quadriceps and a condition known as "breaststroker's knee." Individuals who have patellar tracking problems or a high Q angle (greater than 18° in men) with the knee extended will be prone to patellofemoral pain. You should prescribe that this young man work on stretching and strengthening of his adductors and quadriceps. The quadriceps strengthening should focus on the last 30° of knee extension, which selectively strengthens the vastus medialis. This patient has negative tests for internal derangement, such as anterior cruciate ligament tear or meniscus tear. No laxity was noted, and referral for arthroscopy is not indicated.

121. During a physical examination, which structure CANNOT be effect ively ass essed by the m odi fied Thomas test? (a) Iliotibial band (b) Hip flexors (c) Knee extensors (d) Hamstrings

121. (d) The modified Thomas test is performed with the patient supine; with the tested leg in maximal passive hip extension and knee flexion and the contralateral knee to the chest. The test position causes the pelvis to be placed in posterior pelvic tilt and reduces any lumbar lordosis. Tightness of hip flexors is measured by the degrees of hip flexion from neutral. Tightness of the knee extensors is measured by the degrees of knee flexion from neutral. Tightness of the iliotibial band is measured by the degrees of hip and knee external rotation.

122. In a patient with traumatic brain injury and vertigo, which physical exam ination finding from a D ix Hall pike test w ould en coura ge yo u to per fo rm an Epl ey man eu ver? (a) Persistence of responses to repeated provocation (b) Fixed direction of nystagmus with rotary component (c) Nonconcordant or divergent gaze (d) Persistence of nystagmus, 30 seconds for each provocation

122 (b) A positive Dix-Hallpike test definitively establishes the di agnosis of benign paroxysmal positional vertigo (BPPV). The Dix-Hallpike maneuver is perfor med by qui ckly dr opping a patient backwards from a sitting position so that the head i s rota ted 45 de grees a nd h angs o ver the ed ge of a mat. Cha rac ter is tic s o f BP PV a re mixe d to rsio na l a nd v erti cal ny sta gmus, w hich l as ts f or 10- 20 se con ds , a ssoci ated with a sensation of rotational vertigo. Repetition of the test results in a reduction in the intensity of vertigo and nystagmus. The Epley maneuver, which is performed at the bedside, relocates debris from the posterior semicircular canal into the vestibule of the vestibular labyrinth.

123. A 45-year-old woman pre sent s wit h p atello femoral pain syndrome. Which physical finding is likely?(a) Positive anterior drawer(b) Tight lateral thigh stru ctur es th at cross the joint(c) Q angle less than 15°(d) Medial patellar tilt

123. (b) Anterior knee pain syndrome is associated with atrophy of the quadriceps, a large (more than 15°) Q angle, tight lateral thigh structures, positive posterior drawer, and lateral patellar tilt.

123. When using local steroid injections in patients wit h tendino path ies (a) injection into the tendon substance is optimal. (b) minimum interval between injections is 2 weeks. (c) select the finest needle that will reach the area. (d) early postinjection local anesthesia is a complicati on.

123. (c) It is advisable to select the finest needle that will reach the area. The injection should be peritendinous with avoidance of the tendon to prevent rupture. The minimum interval between injections should be at least 6 weeks. Early postinjection local anesthesia is not a complication of steroids, but it will occur if local anesthetic is mixed with the steroid.

123. A 25-year-old nulliparous woman with a history of psoriasis com plains of low-back pain an d occasional groin discomfort. There is no history of recent trauma. Which finding would you expect on pelvic radiographs? (a) Avulsion fracture at the pubic tubercle (b) Osteitis condensans ilii (c) Traction apophysitis (d) Sacroiliitis

123. (d) Psoriatic arthritis is a seronegative spondyloarthropathy. Sacroilitis must be ruled out in these patients when they complain of back pain. Osteitis condensans ilii occurs in women with pelvic pain after pregnancy, avulsion fracture would typically be seen after a traumatic or high-impact event, and apophysitis is seen in the adolescent before full maturation of the end plates.

124. A patient with neuromuscular disease complains of morning headache and excessive daytime fatigue. What is your initial diagnostic evaluation? (a) Order chest radiographs. (b) Order pulmonary function tests. (c) Monitor end-tidal carbon dioxide levels. (d) Measure assisted-cough peak flows.

124 Answer: (c) Commentary: Patients with neuromuscular disease (NMD) are often sent for pulmonary function tests designed for patients with lung disease. Patients with NMD often do not have a history of asthma or cigarette smoking and most of these tests are unnecessary, except for spirometry. Because underventilation often begins during sleep, spirometry or simple determination of vital capacity is best done in the supine position. The patient's carbon dioxide level will provide insight into hypoventilation and should be monitored, especially when the patient complains of excess fatigue and headaches. Ref: Bach, John. Conventional end-stage management of NMD. In: Bach, JA. Management of patients with neuromuscular disease. Philadelphia: Hanley & Belfus; 2004. p 161.

124. Which factor makes it most probable that a patient is at risk for nerve damage? (a) Bone fracture without dislocation (b) Hematoma (c) Open fracture (d) High velocity trauma

124 Answer: B Commentary: A hematoma places the nerve at risk for injury by 400%, since the expanding fluid will lead to an acute compression neuropathy. Ref:Thomas MA, Felsenthal G, Fast A, Young M. Peripheral neuropathy. In: DeLisa JA, Gans BM, Walsh, NE, editors. Physical medicine and rehabilitation: principles and practice. 4th ed. Philadelphia: Lippincott Williams & Wilkins; 2005. p 906.

124. A 56 year old woman with myasthenia gravi s i s in the intensive care unit with urosepsis. Which antibiotic should be avoided in this patient? (a) Aztreonam (Azactam) (b) Gentamicin (Garamycin) (c) Ceftriaxone (Rocephin) (d) Ciprofloxacin (Cipro)

124. (b) For hospitalized patients, therapy consists of parenteral (or oral once the oral route is available) ceftriaxone, quinolone, gentamicin (plus ampicillin), or aztreonam until defervescence. Then, an oral quinolone, cephalosporin, or trimethoprim-sulfamethoxazole for 14 days may be added to complete treatment. The aminoglycoside class of antibiotics is contraindicated in patients with myasthenia and other neuromuscular junction disorders. Most aminoglycosides exert their effect through reducing the number of acetylcholine quanta released. Use may lead to a myasthenic exacerbation. Ref: (a) Middleton LT. Disorders of the neuromuscular junction. In: Schapira AH, Griggs RC. Blue books of neurology: muscle diseases. Boston: Butterworth-Heinemann; 1999. p 277-8 (b) Gonzalez G. Proteus infections. eMedicine [on line].2006 March; Available at: http://www.emedicine.com/MED/topic1929.htm. Accessed May 6, 2007.

124. A 56-year-old woman with myasthenia gravis is in th e intensi ve c are unit with urosepsis. Which antibiotic should be avoided in this patient? (a) Aztreonam (Azactam) (b) Gentamicin (Garamycin) (c) Ceftriaxone (Rocephin) (d) Ciprofloxacin (Cipro)

124. (b) The aminoglycoside class of antibiotics is contraindicated in patients with myasthenia and other neuromuscular junction disorders. Most aminoglycosides exert their effect through reducing the number of acetacholine quanta released. Use may lead to a myasthenic exacerbation. Acute uncomplicated pyelonephritis in women can be treated with oral quinolones for 7 to 14 days, single-dose ceftriaxone or gentamicin followed by trimethoprim-sulfamethoxazole, or an oral cephalosporin or quinolone for 14 days as outpatient therapy. For hospitalized patients, therapy consists of parenteral (or oral once the oral route is available) ceftriaxone, quinolone, gentamicin (plus ampicillin), or aztreonam until defervescence. Then, an oral quinolone, cephalosporin, or trimethoprim-sulfamethoxazole for 14 days may be added to complete treatment.

124. A 40-year-old construct ion worke r p resent s with the gradual onset of aching pain in the forearm and hand. He reports numbness of the thumb and index fingers and drops objects at work. On exam, weakness is present in thumb opposition, wrist flexion, and finger flexion. Reflexes at the biceps and triceps are intact. The most likely diagnosis is(a) C7 radiculopathy.(b) upper trunk plexopathy ( Erb' s pal sy) .(c) median neuropathy at the elb ow (p ron ator s yndrome).(d) carpal tunnel syndrome.

124. (c) The pattern of weakness and numbness is consistent with a median neuropathy in the forearm. Pronator syndrome is an entrapment of the median nerve as it passes through the pronator teres muscle. Depending on severity, neurologic deficits may include weakness in wrist flexion (flexor carpi radialis), finger flexion (flexor pollicis longus, flexor digitorum superficialis, flexor digitorum profundus digit 2/3), intrinsic hand muscles (abductor pollicis brevis, opponens pollicis), and sensory abnormalities in a median nerve distribution. Reflexes are normal.

125 An individual with T4 American Spinal Injury Association Impairment Scale (AIS) A paraplegia is 2 months postinjury and acutely develops pounding headache, flushing of the face and upper trunk, anxiety and piloerection of the lower body. Blood pressure is 120/80 with a usual blood pressure of 100/60. After loosening all tight garments, what should be the next intervention? (a) Assess bladder for distention. (b) Check bowel for impaction. (c) Apply topical nitroglycerin immediately. (d) Lay the patient supine immediately.

125 Answer: (a) Commentary: The scenario depicts a typical presentation for autonomic dysreflexia (AD). Treatment should consist of checking the blood pressure, elevating the head, loosening tight clothing or garments, and proceeding with systematic investigation and elimination of causative factors. Because bladder distension is the most common stimulus for AD, the algorithm proposed in the clinical practice guideline begins with assessment for bladder-related causes and treatment of any distension. Because bowel obstruction or distension is the second most common stimulus and it, therefore, should be evaluated next if urinary evaluation fails to reveal the cause. The guideline recommends consideration for antihypertensive pharmacotherapy if the individual's systolic blood pressure is above 150. Ref :Consortium for Spinal Cord Medicine Clinical Practice Guidelines. Acute management of autonomic dysreflexia: individuals with spinal cord injury presenting to health-care facilities. 2nd ed. DC: Paralyzed Veterans of America; 2001.

14. A 59-year-old man admitted with atypical chest pain underwent an angiogram 2 da ys a go . H e i s n ow ha ving di fficu lty wa lking. P hys ic al e xam in ation demonstrates 2/5 strength in right hip flexion and knee extension. Muscle strength is otherwise 5/5. Sensation is decreased over the medial distal right leg. What initial diagnostic test would be most appropriate?:(a) Electromyogram and nerve conduction velocities:(b) Magnetic resonance imaging of the lumbosacral spine:(c) Computed tomography scan of the pelvis:(d) Ultrasound of the abdomen

14 (c) The most likely pathology is the involvement of the femoral nerve secondary to hemorrhage. In this situation imaging studies should be done promptly to ass ess for a he matom a or a mass . Electrod iag nos tic stud ies are useful to confirm th e pr esence of femoral ne rve invol vem en t, but o ne sho ul d wait 3-4 weeks p ostinjur y.

125. Which of the following is t rue r ega rding first-year costs for persons with complete spinal cord injury?(a) Average hospital charges for tetr apl egia a re almost double those for paraplegia.(b) Average costs for medica tion s and su pplies are equivalent to those for paraplegics and tetraplegics.(c) Average costs for home m odif icati ons equal those for the acute rehabilitation hospitalization.(d) Annual recurring costs f or m edica l c are an d treatment of secondary complications approach those for that of initial injury costs.

125. (a) Initial acute care and rehabilitation costs average $223,261 per person. Acute care charges are higher for persons with tetraplegia compared with persons who have paraplegia at the equivalent severity of injury. Charges approach $157,000 for ASIA A, B, or C tetraplegia and vary from $69,000 to $ 87,000 for other persons with spinal cord injury. Hospital costs for rehabilitation are more than twice as high for persons with severe tetraplegia compared with severe paraplegia. Average annual medical costs (excluding medications, supplies, and physician costs) are just over $9,000 per year. Given a prevalence of 180,000 persons with spinal cord injury beyond their first year of injury, this approaches $1.65 billion for the spinal cord injury population. Costs for supplies and medications are 30% greater for tetraplegics than for paraplegics ($3,308 vs. $2,470). Home modifications to the residence of a person with spinal cord injury is more than $15,000, and the cost to modify other homes owned by the person, family, or friends is an additional $5,000.

125. Baclofen is thought to reduce spasticity by (a) preventing the release of calcium from the sa rc oplasmic reticulum. (b) blocking sodium and potassium channels. (c) depressing of brainstem neuronal activity. (d) acting as a gamma-aminobutyric acid agonis t.

125. (d) Baclofen is an analog of gamma-aminobutyric acid (GABA), a neurotransmitter that exerts inhibitory activity on monosynaptic and polysynaptic reflexes. Dantrolene prevents the release of calcium from the sarcoplasmic reticulum. Ref: Priebe MM, Goetz LL, Wuermser LA. Spasticity following spinal cord injury. In: Kirshblum S, Campagnola D, DeLisa J, editors. Spinal cord medicine. Philadelphia: Lippincott-Williams & Wilkins; 2002. p 221-33.

126. You are consulted on a 25-year-old patient with cardiomyopathy who has express ed i nt ere st in cardi ac r eha bilit ati on. In revi ewi ng her ec ho cardiogram, you note that her left ventricular ejection fraction (LVEF) is 20%. During your discussion with the house staff you maintain that:(a) exercise capacity is directly correlated with LVEF in patients with heart failu re.:(b) an LVEF less than or equal to 20% is a relative contraindication to cardiac reh abil it ati on.:(c) patients with LVEF less than 30% may attain work capacities of 100 watts.:(d) cardiac rehabilitation may proceed, but should be confined to resistive exercise.

126 (c) In a large series, cardiac patients with LVEFs less than 30% had normal hemodynamics and 28% attained a work capacity of 100 watts. Thirty-three percent of pat ients wi th LV EFs greater t han 45% had ab norm al h emo dynamic resp onses and low er w ork ca pac ities. T hes e data in dic at e t hat p erfoma nc e capacity is la rg ely depe ndent on p er iph era l ad apta tio ns and t hat LV EF has lim ited predictive capacity.

126. Because urinary tract infections (UTIs) are the mos t w id esp read bacterial infection and the most common source of bacteremia in older adults, treatment for bacteriuria greater than 10,000 CFU/ml in this population is indicated for an older patient with (a) vaginal atrophy. (b) a chronic indwelling Foley catheter. (c) a neurogenic bladder. (d) increased incontinence.

126 Answer: (d) Commentary: Bacteriuria is defined as a quantitative cou nt of 10 ,000 CFU/ml or more of 1 or more organisms found in a patient's urine culture, in the absence of clinical signs or symptoms of UTI in the host. Vaginal atrophy, neurogenic bladder, and chronic use of urethral or condom catheters are risk factors for UTIs. However, treatment in the elderly is indicated only if systemic signs and symptoms -- such as low-grade fever, increased confusion, incontinence, anorexia and functional decline -- are present. Reference: Nicolle LE. Urinary tract infections: asympto mat ic ba cteriuria in the elderly. Infect Dis Clin North Am 1997;11:647-62.

126. A 70-year-old man underwent a 2-vessel co ron ar y artery bypass graft and mechanical mitral valve replacement five days ago. You note that he is presently taking Coumadin (warfarin). The primary reason to put this patient on Coumadin after this procedure is to prevent (a) deep vein thrombosis. (b) embolic stroke. (c) coronary artery occlusion. (d) valvular adhesion.

126. (b) Patients are anticoagulated following mechanical valve replacements to prevent thromboembolic strokes. Anticoagulation will also prevent deep vein thromboses, but this is not the primary reason why it is prescribed. Ref: (a) Ezekowitz MD. Anticoagulation management of valve replacement patients. J Heart Valve Dis 2002;11(Suppl 1):S56-60. (b) Stein PD, Alpert JS, Bussey HI, Dalen JE, Turpie AG. Antithrombotic therapy in patients with mechanical and biological prosthetic heart valves. Chest 2001;119(1 Suppl):220S-7.

126. Regarding exercise in s ever e hea rt failur e, which of the following is true?(a) Strengthening should be done isom etr ically .(b) Exercise heart rate shou ld b e at 0.8 -90% o f estimated maximum heart rate.(c) Rapid hemodynamic change s ma y occ ur during warm-water aquatic therapy.(d) Telemetry is never neces sary .

126. (c) One limitation to the use of aquatic therapy in severe heart failure is its potential to produce rapid hemodynamic changes. Isometric strengthening can result in increased afterload, with the possibility of deleterious effects on ventricular function. In severe heart failure, it is generally recommended that the exercise heart rate be kept at least 10 beats per minute below the arrhythmia/severe dyspnea level. Telemetry is recommended in this population (at least at the initiation of their exercise program), as they are usually at the highest level of risk stratification.

14. Which risk factor does NOT typically predispose a person with chronic obst ructive pulmon ary disea se to ost eopen ia a nd osteoporosis? (a) Smoking (b) Decreased physical activity (c) Corticosteroid therapy (d) Increased body mass index Ref: Sin DD, Man JP, Man SF. The risk of osteoporosis in Caucasian men and w omen with obst ructive a irways di sease . Am J Med 2003;114:10-4. Educational Activity 2.6

14 (d) Risk factors that predispose chronic obstructive pulmonary disease patients to accelerated bone lo ss and oste oporosi s are reduced phy sical activity, poor nutritio n, l ow body mas s index , smo king, de cre ased exposure to sunlight, nutritional deficiencies, hypercatabolic effects of ongoing inflammatory processes, and corticosteroid therapy.

127. A warehouse manager asks you about prescribing lumb ar su ppo rts to help his workers prevent and treat lumbar strains. You inform him that evidence-based medicine shows that lumbar supports (a) prevent excessive spinal motion by providing sensory fe ed bac k. (b) demonstrate low compliance by workers for both treat men t and prevention of lumbar strain. (c) increase intra-abdominal pressure without increasing ab do min al muscle activity. (d) are superior to other available treatments in provid ing l umb ar pain relief..

127 Answer: (b) Commentary: While theories have been proposed regarding how l umb ar supports may treat and prevent lumbar strains, evidence-based medicine does not demonstrate consistent findings to support their use. Studies have shown that there is very low compliance with consistent use of lumbar supports. There have been no consistent findings showing that lumbar supports prevent excessive spinal motion or increase intra-abdominal pressure without increasing abdominal muscle activity. Outcomes studies do not demonstrate superior outcomes with use of lumbar supports compared to other available treatments. Reference: Barr KP, Harrast MA. Low Back Pain. In: Bradd om RL , e ditor. Physical medicine and rehabilitation. 3rd ed. Philadelphia: Saunders - Elsevier; 2007. p 909.

127. Which structure is required by the American with Disabilities Act (ADA) to have adequate accessibility for individuals with disabilities, so long as the modifications to it are readily achievable? (a) House of worship (b) Physician's office within a private residence (c) Commercial airplane (d) Residential private apartments

127 Answer: B Commentary: If a publicly accessible office is present within a single family home, it is required to be accessible under the ADA, so long as the necessary modifications are readily achievable. Accessibility of commercial airplanes is covered under the Air Carrier Access Act, not the ADA. Access to houses of worship or strictly residential private apartments is not required under the ADA. Ref:Americans with Disability Act Website. Questions and Answers. http://www.usdoj.gov/crt/ada/q%26aeng02.htm. Accessed May 26, 2008.

127. According to the Joint Commission on Accr edi ta tion of Hospital Organizations (JCAHO), what is the minimum number of patient identifiers needed before medications, blood products, or other treatments or procedures may be administered? (a) 1 (b) 2 (c) 3 (d) 4

127. (b) While more than 2 patient identifiers may be used, a minimum of 2 is required: first, a marker to identify the individual as the person for whom the service or treatment is intended; second, an identifier to match the service or treatment to that individual. Ref: Joint Commission on Accreditation of Hospital Organizations. JCAHO 2007 National Patient Safety Goals. JCAHO Web site. Available at: http://www.jointcommission.org/NR/rdonlyres/98572685815E-4AF3-B1C4-C31B6ED22E8E/0/07_hap_npsgs.pdf. Date accessed July 4, 2007.

128. In a patient with early generalized myasthenia gravis which electromyographic abno rm ali ty is the m ost lik ely?:(a) Fibrillation potentials:(b) Fasciculation potentials:(c) Amplitude variability in a single motor unit:(d) Myokymic discharges

128 (c) This can be noted using a trigger and delay line and observing a single motor unit firing repetitively. The other abnormalities are not seen in myasthen ia grav is.

128. Which electrodiagnostic finding is most consistent with neurogenic thoracic outlet syndrome? (a) Small median motor response from the thenar muscles (b) Abnormal response of the lateral antebrachial cutaneous nerve (c) Abnormal median sensory responses (d) Abnormal spontaneous activity in the pronator teres muscle

128 Answer: B Commentary: Neurogenic thoracic outlet syndrome involves the lower trunk of the brachial plexus; hence, sensory and motor loss develops in the C8-T1 distribution. Thumb abduction is often affected. Sensory changes are usually in the distribution of the ulnar and medial antebrachial cutaneous nerves. Ref:Preston DC, Shapiro BE. Brachial plexopathy. In: Preston DC, Shapiro BE, editors. Electromyography and neuromuscular disorders: clinical-electrophysiologic correlations. Boston: Butterworth-Heinemann; 1998. p 438.

128. Surface electrodes for reco rding an tidrom ic sural nerve conduction studies are best placed(a) posterior to the medial mall eolus .(b) posterior to the lateral mal leolu s.(c) anterior to the medial m alle olus.(d) anterior to the lateral mall eolus .

128. (b) The sural nerve travels posterior to the lateral malleolus and is best recorded over this area.

129. You are performing a consult on an 8-year-old child who has sus tained a traumatic brai n i njury. The child has hyperthermia, hypertension, tachycardia, and rigidity. The best management for this child would be (a) Propranolol (Inderal). (b) Baclofen (Lioresal). (c) Nonsteroidal anti-inflammatory drugs (NSAIDs). (d) Amantadine (Symmetrel).

129 (a) Fever in a child with a severe traumatic brain injury should be investigated and infections treated with appropriate antibiotics. In the absence of infection, the fever in central autonomic dysfunction is poorly responsive to nonsteroidal anti-inflammatory drugs. Baclofen may help to control the spasticity, but propranolol is more effective in controlling the hypertension, tachycardia, and hyperthermia.

129. Your 10-year old patient with T6 ASIA class B paraplegia complains of right kn ee p ai n. On exa minat ion the re is no sw elling o f t he kne e o r leg. Knee examination is normal. The right leg appears shorter when the hips and knees are flexed. What is the most likely cause of these findings?:(a) Knee sprain:(b) Right hip subluxation:(c) Hip adductor spasticity:(d) Dysesthetic pain

129 (b) Hip subluxation is the most likely cause of knee pain in a child with T6 ASIA B SCI. Pain from hip pathology is often referred to the knee in children. Whi le h ip ad duc tor s pasticity c on tributes to su blux atio n o r dislocatio n, the spasti city itsel f i s not pa inf ul. Dyses the ti c p ain i s usua ll y generaliz ed. A n abnormal knee exam in ati on is u sual ly fou nd in a kne e i njury t hat causes pain.

129. Which endocrine abnormality is most likely to occur 5 years after severe traumati c brain injur y i n a 2-y ear- old g ir l? (a) Diabetes insipidus (b) Precocious puberty (c) Hypothyroidism (d) Hyperparathyroidism

129 (b) Precocious puberty occurs in up to 50% of girls who sustain a severe traumatic brain injury (TBI) in early childhood. Diabetes insipidus is an early co mplication of TBI . While other endocrine abnormalities may occur, they are less comm on .

17. The owner of a landscape business is interested in implementing a program to reduc e low ba ck in jur ie s in his worke rs . Whi ch ther apeut ic ex erc is e prog ram ha s b ee n shown to reduce the risk of low back injuries? (a) Lumbar extension exercises (b) Lumbar flexion exercises (c) Lumbar spine stabilization exercises (d) No specific group of exercises reduces risk

17 (d) No specific lumbar spine strengthening program has been show n to prevent low back pain in the work place. Spine strength is not a predictor of reduced risk for o nset of low back pain.

129. Your 3-year-old patient with cerebral palsy has a Gross Motor Function Classification System (GMFCS) Level IV. Family is asking you what to expect she will be able to do when she is a teenager. Which activity is the highest level she is most likely to attain as a teenager? (a) Independent ambulation in the household (b) Independent ambulation in the community (c) No independent mobility in the household (d) Wheelchair use in the community

129 Answer: D Commentary: The Gross Motor Function Classification System (GMFCS) classifies mobility of people with cerebral palsy from I to V. Level I is independent ambulation indoors and outdoors with no assistive device. Level IV requires wheelchair for household and community mobility. Ref: (a) Hurvitz EA, Ayyangar RN, Aisen M. Cerebral palsy: diagnosis and treatment. In: O'Young BJ, Young MA, Stiens SA, editors. Physical medicine and rehabilitation secrets. 3rd ed. Philadelphia, Mosby Elsevier; 2007. p 669.(b) Palisano R, Rosenbaum P, Bartlett D, Livingston M. Gross Motor Function Classification System: expanded and revised (GMFCS - E & R); 2007.Available at: http://www.canchild.ca/Portals/0/outcomes/pdf/GMFCS-ER.pdf.accessed August 8, 2008.

129. Secondary injury in ped iatr ic br ain traum a is caused by(a) hypotension, hypoxia, an d hy droce pha lus.(b) growing skull fractures.(c) coup and contrecoup cere bral cont usi ons.(d) diffuse axonal injuries and punct ate hemor rhages.

129. (a) Any disorder that interferes with cerebral perfusion or oxygenation can cause further damage following traumatic brain injury. This includes hypotension, hypoxia, increased intracranial pressure because of cerebral edema, acute hydrocephalus, or space-occupying lesions. Midline shift or herniation may lead to infarction because of pressure or traction on cerebral vessels. Therefore, efforts are made to control intracranial pressure through fluid and electrolyte management, hyperventilation, and maintenance of normal blood pressure and oxygenation. Growing skull fractures result from the arachnoid protruding through a dural tear, producing a cyst that can contribute to a widening skull deficit, which usually requires operative repair. This is a complication of traumatic brain injury but not a secondary injury. Coup and contrecoup cerebral contusions and diffuse axonal injuries are examples of primary injury.

13. What is the most common form of arthritis in adults?:(a) Crystal arthritis:(b) Septic arthritis:(c) Osteoarthritis:(d) Rheumatoid arthritis

13 (c) Osteoarthritis is the most common form of arthritis.

13. In which case would bulbar involvement and acute respiratory failure NOT c ommonly be see n? (a) Am yotrophic late ral sclerosis (b) Parkinson's disease (c) Spinal cord injury (d) Multiple sclerosis Ref: Harrop JS, Sharan AD, Scheid EH Jr, Vaccaro AR, Przybylski GJ. Tracheos tomy placement in patie nts with compl ete cervical spinal cord injuries: American Spinal Injury Association Grade A. J Neurosurg 2004;100(1 Suppl Spine): 20-3. Educational Activity 2.4

13 (c) Respiratory failure in spinal cord injury is not associated with bulbar involve ment. Along with th ose listed, respir atory failure wit h bulbar involvement may also be see n with G uillain arr syndro me, myasthenia gravis, and muscular dystrophy.

13 The use of an abdominal binder in a sitting patient who has tetraplegia due t o a sp in al co rd inj ury wil l a. decrease the risk of autonomic dysreflexia. b. eliminate the need for lateral wheelchair supports. c. result in a need for more frequent bladder catheteriz ations. d. improve pulmonary function by elevating the diaphragm .

13 Option d is correct. When sitting, the abdominal contents pull down on the di aphragm, placing the mus cle i n a m ech ani cal ly d isadva nta ge d po sit ion an d limiti ng t he a bilit y t o tak e a de ep bre ath. The abdominal binder will help to restore the diaphragm to a position more functionally conducive to maintaining vital capacity. The binder can be used to help reduce orthostatic hypotension and it does not appear to alter bladder capacity. An abdominal binder will not decrease the risk of autonomic dysreflexia or eliminate the need for lateral wheelchair supports.

13 The classic history for nonaccidental traumatic brain injury (shaken baby syndrome) is that of a history incompatible with the severity of injuries, an unwi tnessed event, so detail is missing, and the injury i s inflicted by a solitary care provider. There is often a delay in seeking medical attention. Recent studies indicate that the child usually experiences a physical impact, in addition to the shaking. In contrast to adults with rheumatoid arthritis, which joints are more commonly involved in children with juvenile idiopathic arthritis? a. Metacarpophalangeal b. Shoulder c. Interphalangeal d. Temporomandibular

13 Reference(s) Murphy KP, Wunderlich CA, Pico EL, Driscoll SW, Moberg-Wolff E, Rak M, Nelson MR. Orthopedics and musculoskeletal conditions. In: Alexander MA, Matthews DJ editors. Pediatric reha bili tation. 4th e N ew York: D emos Medical; 2010. p 371. Option d is correct.

13. A 70-year-old woman comp lain s of acu te loc alized mid back pain. She has a non-focal neurologic examination. An anteroposterior and lateral thoracic spine x-ray confirms your clinical suspicion of an acute T8 compression fracture. Which recommendation would best help her to reduce her risk of future fractures?(a) Swimming laps 20-30 minu tes daily(b) Isotonic abdominal stren gthe ning pro gram(c) A weight reduction diet(d) Avoidance of tobacco use

13. (d) The National Osteoporosis Foundation (NOF) established guidelines to reduce risk of osteoporotic fractures. These recommendations include, participating in weight bearing exercise, ingesting adequate calcium (1200mg/day) and vitamin D (400-800IU), and avoiding tobacco use.

130. In a transfemoral amputee, a circumducted gait pattern, on the prosthetic side, could be caused by which factor? (a) Insufficient prosthetic knee friction (b) Long prosthetic limb (c) Hip flexion contracture (d) Poor balance

130 Answer (b) Commentary: When observing gait deviations in an amputee, one should consider both the prosthetic issues and amputee compensatory maneuvers as a potential cause for the deviation. A circumducted gait pattern can have various causes, including a long prosthetic limb, excessive prosthetic knee friction (making it difficult to bend the knee), and hip abduction contracture. Poor balance is usually associated with excessive lateral trunk bending, uneven arm swing, and short stance phase on the prosthetic side. Reference: Huang ME, Miller LA, Lipschutz R, Kuiken TA. Rehabilitation and prosthetic restoration in lower limb amputation. In: Braddom RL, editor. Physical Medicine and Rehabilitation. 4th ed. Philadelphia: Elsevier Saunders: 2011:307-308.

130. How are mobility devices paid for through Medicare? (a) The patient must make a 50% down payment, with the rest covered by Medicare upon delivery of the device. (b) Medicare part A pays 80% of the allowed purchase price and Medicare part B pays the remaining 20%. (c) Medicare will pay for purchase but not rental of mobility devices. (d) Medicare part B pays 80% of the allowed purchase price in one lump sum.

130 Answer: D Commentary: Medicare Part B pays 80% of the allowed purchase price in one lump sum payment if the patient chooses to purchase the device. The patient is required to pay 20% of the allowed purchase price. If the patient chooses to rent a wheelchair, Medicare part B will pay 80% of the allowed rental price for months 1 through 10 and the patient will pay 20% of the allowed rental charge. Ref: www.cms.hhs.gov/apps/media/press/release.asp Accessed May 15, 2008.

130. A potential benefit of osseointegration ( the d irect skeletal attachment of the prosthesis to bone) is (a) elimination of poor prosthetic socket fit. (b) ability to return to running activities. (c) early prosthetic fitting. (d) ability to perform heavy manual work.

130. (a) The primary benefits of attaching a prosthesis directly to the skeleton are comfort, elimination of poor prosthetic socket fit, and elimination of skin problems. Recipients report improved sensory feedback from the skeletally attached limb. Limitations include a 2-stage procedure, which results in an extended time of non-weight bearing, and extended rehabilitation (up to 2 years). The procedure poses a significant risk of infection, and the recipient must limit running, jumping, and heavy manual work in order to minimize loosening of the prosthesis. Ref: Pasquini PS, Bryant PR, Huang NE, Robert TL, Nelson VS, Flood KM. Advances in amputee care. Arch Phys Med Rehabil 2006:87(3 Suppl1);S34-43

130. In a metal ankle joint (dou ble a cti on joi nt or Klenzak ankle joint) used in ankle-foot orthoses, which component assists dorsiflexion?(a) Anterior spring(b) Anterior rod(c) Posterior spring(d) Posterior rod

130. (c) An anterior spring assists plantar flexion and has no specific clinical indications. An anterior rod limits dorsiflexion and is used for weak plantar flexors, weak knee extensors, and pain with ankle motion. A posterior spring assists dorsiflexion and is used for flaccid footdrop and knee hyperextension. A posterior rod limits plantar flexion and is used for plantar spasticity, toe drag, and pain with ankle motion.

131. A 38-year-old laborer presents with shoulder pain after falling on to the tip of h is le ft sho ulder . He fe lt im med iat e pain i n t he upp er pa rt of his shoulder, but no numbness or tingling in his arm. On examination, he is noted to have a deformity on the superior aspect of his shoulder. He has pain with horizontal adduction of his left arm across his chest and is having difficulty lifting his left arm. His passive range of motion is good. Which x-rays with the patient holding and not holding weight(s) should you order?:(a) A left shoulder anteroposterior radiograph:(b) A scapular Y view radiograph of the left shoulder:(c) Bilateral anteroposterior shoulder radiographs:(d) A left shoulder axillary view radiograph

131 (c) The first step would be to make sure the patient is neurovascularly intact. The next step would be to obtain bilateral shoulder films with the patient h old ing and n ot holdi ng 10-pound w eights t o a cce ntua te a ny laxity prese nt. Bilateral fil ms are ne cessary as the patie nt ma y h ave l ax lig am ents in bot h sho ul ders. A scapu lar Y v iew is use d fo r s hou lder dis loc ati ons. An axillary radiograph is used for shoulder dislocations and will also evaluate the glenoid.

131 Which therapeutic modality delivers medication to the site of pathology by promoting the movement of charged particles through the skin under an imposed electrical field? (a) Phonophoresis (b) Low energy laser (c) Ultraviolet radiation (d) Iontophoresis

131 Answer: (d) Commentary: Iontophoresis is a physical medicine modality that delivers topical medicine, such as corticosteroids, through the skin and into soft tissues. A current is created to direct a particular solution away from the electrode and into the surrounding tissue. Phonophoresis utilizes ultrasound rather than current to deliver the topical medication. Low energy laser and ultraviolet radiation are not used to deliver topical medications. Ref: Weber DC, Hoppe KM. Physical agent modalities. In: Braddom R. Physical medicine and rehabilitation. Philadelphia: Elsevier; 2007. p 472.

131. 22-year-old female volleyball player fell on an outstretched right hand 3 weeks ago and complains of continued wrist pain. On examination, she has minimal swelling of the distal limb and is tender to palpation distal to the ulnar styloid between the flexor carpi ulnaris and extensor carpi ulnaris tendons. A plain radiograph was normal except for an ulnar plus variant. She failed conservative treatment with splinting and activity modification. The most appropriate imaging study to obtain would be (a) computed tomography scan of the wrist. (b) repeat plain films in 10 days. (c) triple phase bone scan. (d) magnetic resonance imaging with arthrogram.

131 Answer: D Commentary: This patient sustained an injury to her triangular fibrocartilage complex. This structure is a stabilizer of the distal radioulnar joint and is composed of an avascular articular disc and radioulnar ligament complex. It is often injured with repetitive wrist activities or compressive loads. Tears to it are best imaged by MRI arthrogram. Injury to this complex would not be optimally evaluated on plain films, bone scan, or computed tomography scan. Ref: Finnoff JT. Musculoskeletal problems of the upper limb. In: Braddom RL,. editor. Physical medicine and rehabilitation.3rd ed. Philadelphia: Elsevier; 2007. p 846.

142. Which genetic factor may link Alzheimer disease and chronic residual deficits in t ra uma tic br ain i njur y?:(a) Apolipoprotein-?4:(b) Human leukocyte antigen B27:(c) Mitochondrial protein C450:(d) Platelet aggregation factor

142 (a) The apolipoprotein-?4 has been linked both to Alzheimer disease and to chronic effects of traumatic brain injury.

131. Which statement is TRUE reg ardin g c ortico steroid injection for severe carpal tunnel syndrome?(a) Axonal injury can be suc cess fully re versed with this procedure.(b) To recognize nerve injec tion , do not dilut e the corticosteroid with anesthetics.(c) New pain and numbness la stin g for mo re tha n 48 hours are normal.(d) Local tenderness and sup erfi cial hem atomas are rare and indicate a complication.

131. (b) Anesthetics mixed with corticosteroid can mask the pain associated with needle placement into the nerve and should not be used. The risk of intraneural injection is real, but in experienced hands this injection is safe. Numbness is anticipated with this injection without use of anesthetics, and helps to confirm proper placement. Local tenderness and hematomas are common with this injection and do not represent a complication. Persistent or worsening pain, or swelling lasting more than 48 hours, are signs of nerve injection or neurotoxic injury. Severe carpal tunnel syndrome with axonal loss is not reversed with this procedure.

131. A 22-year-old female gymnast with chronic lo w back pain is diagnosed with spondylolysis of the right L5 pars interarticularis. Spondylolisthesis is not identified on plain x-rays. What is the best test to determine fracture healing? (a) magnetic resonance imaging (b) single photon emission computed tomography (c) computed tomography scan (d) flexion and extension lateral x-rays

131. (c) Computed tomography (CT) scans with thin cuts through the area of the pars interarticularis can identify the healing pattern of a pars stress fracture. Ref: Standaert CJ, Herring SA. Expert opinion and controversies in sports and musculoskeletal medicine: the diagnosis and treatment of spondylolysis in adolescent athletes. Arch Phys Med Rehabil. 2007. Apr;88(4):537-40.

133. A 40-year-old woman with history of Hodgkin's lymphoma was trea ted with radiation 20 y ear s ago. She complains of left groin and thigh pain aggravated by weight-bearing. Symptoms have been insidious in onset and there is no evidence of recurrent cancer. She has limited active hip flexion, abduction, and external rotation. She has no back pain with passive hip extension and knee flexion in the prone position. What is the most likely diagnosis? (a) Avascular necrosis of the femoral head (b) Herniated nucleus pulposus at L3/L4 (c) Iliotibial band friction syndrome (d) Snapping hip syndrome

133 (a) The patient has avascular necrosis. Radiation, alcohol use, trauma, infections, and pancreatitis increase the risk of developing avascular necrosis. With a normal neurologic examination and a negative femoral stretch as described, a herniated disc is less likely.

133. Painful, arthritic hip disease is associated with the following gait abno rmality. (a) Prolonged stance phase of the affected limb (b) Limb circumduction during swing (c) Lateral trunk shift over the affected joint during stance (d) Shortest step length of the affected limb

133 (c) The patient will lean over the affected hip (compensated Tre ndelenburg gait) in an attempt to minimize painful contractions of the hip abductors. In ad dition, th e chara cteristics of an antalgic gait include shorter stance phase on the af fected leg (as the pat ient attem pts to rem ove weight quick ly fro m the af fect ed l eg), de crea sed sw ing p ha se o f t he uni nvolv ed leg , an d a sho rte r ste p l en gth on t he uninvolve d side. Limb circumduction is characteristic of increased limb length or abductor muscle shortening.

133 A 45-year-old woman is currently hospitalized for an acute flare of her dermatomyositis. On consultation, you recommend (a) passive range-of-motion exercises. (b) isometric strengthening exercises at the bedside. (c) ambulation with a walker in the hallways. (d) aquatic therapy.

133 Answer: A Commentary: Passive range of motion to maintain joint movement is recommended during periods of acute flares. With resolution of the flare, active-assisted exercises may be started, progressing to strengthening exercises. Ref: Guzman J. Rehabilitation of patients with rheumatic diseases. In: Braddom RL, editor. Physical medicine and rehabilitation. 3rd ed. Philadelphia: Saunders; 2007. p 789.

134. Which process results primarily in an axonal, mixed motor, and sensory periphe ral ne uro pat hy?:(a) Alcoholic neuropathy:(b) Diabetic neuropathy:(c) Guillain-Barré syndrome:(d) Lead neuropathy

134 (a) Alcohol-induced neuropathy involves both motor and sensory fibers and is typically axonal in nature rather than demyelinating. Diabetic neuropathy is us ual ly a mixe d m otor/ sensory and m ixed axo nal /de myel inat ing process. Gu illain-Barré synd rome i s p rimarily de myelinati ng. L ead neur opathy a ffects prim arily m otor fib ers a nd is a xon al in n atur e.

134. A 12-year-old girl with limb girdle muscular dystrophy has trouble keepin g up in school b ecaus e o f slow gait and wr iting s peed. What i s t he pr obable ca use of h er clinical problems? (a) Skeletal muscle weakness (b) Spasticity (c) Cardiomyopathy (d) Joint contracture

134 (a) The majority of clinical problems encountered in neuromuscul ar disease can be directly linked to skeletal muscle weakness. Studies indicate that what c auses most of the functional problems and impairs quality of life for people with ne ur omuscu lar disea se is m uscl e weak nes s.

134. A 29-year-old man retur ned from his Hawai ian honeymoon complaining of hand weakness. On examination, you find the following on muscle testing: shoulder abduction 5/5, elbow flexion 5/5, elbow extension 5/5, wrist flexion 4+/5, wrist extension 3-/5, finger flexion 4+/5, finger extension 3/5, thumb adduction 5/5, thumb opposition 5/5. What nerve is most likely affected?(a) Axillary nerve at the hu mera l hea d(b) Radial nerve at the spir al g roove(c) Ulnar nerve at the cubit al t unnel(d) Posterior interosseous n erve

134. (b) The radial nerve is the most likely nerve to be affected at the level of the spiral groove of the humerus. This can either be due to fractures or compressive lesions (Saturday night palsy). These patients usually have involvement of the radial innervated muscles except the triceps and anconeus. Abductor pollicis longus is innervated by the radial nerve.

135. Which factor is associated with successful phrenic nerve pacing for indep endent r espirati on in pa ti ents with tetr ap legia ? (a) Initiation of pacing shortly after injury (b) Location of injury at or above C2 (c) The presence of central sleep apnea (d) Ability to breathe on t-piece for 15 minutes

135 (b) Electrophrenic respiration or phrenic nerve pacing should no t be performed until at least several months after injury, since some patients recover diap hragmatic functio n over a period of several months. Criteria for use of phrenic nerv e pacing include an inju ry a t or a bov e C2, w ith intact phren ic ner ve s. Mos t pa ti en ts inju red at C 3, C4 , or C5 sho uld be able t o we an f rom the ve ntila tor . Som e ris k exists wit h phrenic pacing because the electric transmitter can fail. Additionally, patients may not be able to develop a deep enough breath with this technique to prevent atelectasis. Electrophrenic pacing is expensive and requires extensive training of personnel who will care for patients. There must be a backup ventilator in the event of electrophrenic failure.

135. In spinal cord injury, increased reports of pain are seen more commonly in pat ient s wit h:(a) complete spinal cord injury.:(b) hypercalcemia.:(c) gunshot wound.:(d) surgical stabilization.

135 (c) The prevalence of pain in spinal cord injury (SCI) has been reported to be as high as 94%. It is reported to interfere with activities of daily living i n 5 % to 0.45 of patie nts with sp in al cord inj ury . Th e on set of SCI pain typically oc curs withi n t he first ye ar of inj ury i n t he ma jority o f patients. Pain i s report ed to decr ea se in inte nsit y a nd frequ enc y o ver time. Factors related to self-reported pain include patients with older age, incomplete spinal cord injury, cauda equina injuries, cervical spinal cord injury, central cord syndrome, gunshot wound injury, and syringomyelia.

135. In a 22-year-old man wh o in curre d a n acut e C5-6 fracture-subluxation (complete C5 tetraplegia), from diving with an initial restoration of arm function includes(a) upper extremity tendon t rans fers as early as possible to enhance goals for acute rehabilitation.(b) splinting in a flat hand pos ition to avoid tightening of the flexor tendons.(c) a short opponens orthosi s or uten sil cuff to initiate self-care activities.(d) exclusive use of a manua l wh eelch air to en hance upper extremity muscle strength.

135. (c) For persons with tetraplegia, proper hand position is maintained by resting hand splints that allow tightening of the flexor tendons; this tightening promotes the use of tenodesis for hand function. Functional activities improve significantly with the addition of wrist extensor muscles at the C6 level. Active wrist extensor result in tenodesis of the hand. With wrist control, patients can use a short opponens orthosis or utensil cuff to feed themselves. While patients with tetraplegia usually benefit from a lightweight, manual wheelchair, these patients are often appropriate for powered mobility. The energy saved from pushing the wheelchair can be used for transfers, weight shifts, and other activities, reducing the wear and tear on joints and soft tissues. Tendon transfers and upper limb reconstructive surgery are considered 1 year postinjury, keeping in mind that upper limb muscle recovery can occur over the course of up to 2 years.

136. The bulk of personal long-term care for most older ind iv idu als in the United States is provided by (a) paid home health aides. (b) extended care facilities. (c) government agencies. (d) family members.

136 Answer: (d) Commentary: In the United States, immediate and extended fa mi lie s provide the bulk (up to 90%) of personalized long-term care for their elderly disabled relatives. This includes personal care, nursing care, meals, housekeeping, transportation and shopping. Outside or alternative support systems (friends and neighbors, government and agencies) supplement this care, and can become increasingly important with advancing age. Reference: Clark GS, Siebens HC. Geriatric rehabilitatio n. In : D elisa JA, Gans BM, Walsh NE, editors. Physical medicine and rehabilitation: Principles and practice. 4th ed. Philadelphia: Lippincott Williams & Wilkins; 2005. p 1541.

136. In patients with Duchen ne m uscul ar dystro phy, decline in vital capacity tends to coincide with the onset of(a) limb contractures.(b) use of orthotic/assistiv e de vices fo r ambu lation.(c) wheelchair dependence. ( d) d yspha gia .

136. (c) Scoliosis often becomes evident or exacerbated during this stage. Customized seating and orthopedic interventions can minimize loss of vital capacity.

137. At the time of the initial evaluation of an injured worker, the physiatr ist shoul d (a) define the anticipated time frame that pain medications will be used. (b) ensure that the worker understands narcotic medications will be used if n ecessary up un til time to return to work. (c) recommend that only non-narcotic pain medications will be used d uring reh abilitati on. (d) recommend patient-directed use of pain medications during rehabi litation and retur n to wor k.

137 (a) Narcotics and non-narcotic medications often may be necessary to manage pain in the injured worker. It is important to set expectations of how these medications will be used. Expected length of time of narcotic usage is especially important. The goal should be to return the worker to his/her job without medication. Simply cutting off medications may be inappropriate. As pain improves, and strength and function improve, medications should be tapered.

137. A 47-year-old woman develops complex regional pain syn dr ome (CRPS) type I following a fall at work which resulted in a distal radius fracture. Although no established gold-standard treatment for CRPS currently exists, which option has be en st udi ed in multiple, large-scale randomized trials? (a) Bisphosphonates (b) Gabapentin (c) Stellate/lumbar sympathetic blocks (d) Calcitonin

137 Answer: (a) Commentary: While all of the listed options have been us ed fo r t he treatment of CRPS, only bisphosphonates have been investigated in multiple, large-scale randomized trials. Clear benefits have not been reported with gabapentin or stellate/lumbar sympathetic blocks. Available evidence does not support the use of calcitonin. Reference: (a) Tran DQH, Duong S, Bertini P, Finlayson R J. Tr eat ment of complex regional pain syndrome: a review of the evidence. Can J Anesth 2010;57:149-66. (b) Hsu ES. Practical management of complex regional pai n s yn dro me. Am J Ther 2009;16(2):147-54. (c) Mackey S, Feinberg S. Pharmacologic therapies for co mpl ex re gional pain syndrome. Curr Pain Headache Rep 2007;11(1):38-43.

142. Which factor does NOT increase a person's risk for hydrocephalus followin g trauma tic brai n inj ury ? (a) subarachnoid hemorrhage (b) post traumatic seizures (c) intracranial infections (d) penetrating injury

142 (b) In communicating post-traumatic hydrocephalus, cerebral spin al fluid (CSF) absorption by the villi is impaired. This occurs with inflammation or subara chnoid hem orrhage .

137. According to the Joint Commission on Accr edi ta tion of Hospital Organizations (JCAHO) which abbreviation may be used when writing a prescription? (a) U for units (b) QD for once daily (c) 2 mg for 2 milligrams (d) MSO4 for morphine sulfate

137. (c) The only listed expression that may be written on a prescription is 2 mg for 2 milligrams. JCAHO expects that the other abbreviations will not be used in writing drug prescriptions, since they can lead to errors. Davis' Medical Abbreviations cites U as "the most dangerous abbreviation" and says spell out unit. The expression QD is too easily read as 4 times daily. Regarding MSO4, Davis also calls this as "a dangerous abbreviation." Ref: 2007 National Patient Safety Goals. Joint Commissions Resources Web site. Available at: http://www.jcrinc.com/13469. Accessed July 4, 2007. 138. (d) Reduced recruitment may be seen day 1. Positive waves may be seen in the paraspinals as early as 1 week, and positive waves and fibrillations may be seen in the extremities by 3-5 weeks, but, because of reinnervation by axon sprouting, it takes at least 6 weeks to see increased amplitude and duration of muscle activity.

138. A man presents to your clinic complaining of buttoc k p ai n t hat radiates posteriorly down the thigh. On exam you note that he has a leg length discrepancy, symptoms are provoked by placing the affected limb in the FAIR position (hip in flexion, adduction and internal rotation), and he has a positive straight leg raise test. He has normal nerve conduction studies and a normal needle electromyography test. Treatments that may be beneficial include (a) stretching exercises of the iliotibial band and cort ico st ero id injection of the greater trochanteric bursae. (b) a lumbar stabilization exercise program and coricost ero id lu mbar epidural spinal injection. (c) a lumbar stabilization exercise program and botulinu m t ox in injection of the lumbar paraspinals. (d) stretching exercises in the FAIR position and botuli num t oxi n injection to the piriformis.

138 Answer (d) Commentary: This is a description of piriformis syndrom e. Al tho ugh some positive findings on needle examination may be seen with piriformis syndrome, electrodiagnostic studies are often normal. On the other hand, positive findings are expected in cases of lumbar radiculopathy. Conservative treatment of piriformis syndrome begins with piriformis stretching (FAIR position is a good position for this) and nonsteroidal anti-inflammatory drugs (NSAIDs), followed by lumbosacral stabilization, hip strengthening, and myofascial release. Botulinum toxin relieves pain via multiple mechanisms and is increasingly used in the treatment of myofascial dysfunction. A lumbar stabilization exercise program and botulinum toxin injection of the lumbar paraspinals may help relieve some of this patient's pain if he also has low back pain, but would not address the main issue, piriformis syndrome. Stretching exercises of the iliotibial band and corticosteroid injection of the greater trochanteric bursae would be the treatment for greater trochanteric bursitis. A lumbar stabilization exercise program and corticosteroid lumbar epidural spinal injection would treat a lumbar radiculopathy. Reference: Kirschner JS, Foye PM, Cole JL. Piriformis s ynd ro me, diagnosis and treatmtent. Muscle Nerve 2009;40:10-8.

138. A bladder neuroprosthesis applies electrical stimulation to intact sacral parasympathetic nerves (S2-S4) to produce effective micturition and improve bowel function. A posterior rhizotomy from S2-S4 is typically also performed at the same time in order to (a) decrease pain and increase patient acceptance of the neuroprosthesis. (b) improve bladder emptying and lower the postvoid residual. (c) improve external urethral sphincter relaxation. (d) decrease autonomic dysreflexia when the bladder is emptying.

138 Answer: (d) Commentary: Micturition by electrical stimulation requires intact parasympathetic neurons to the detrusor muscle. This stimulation is often combined with posterior sacral rhizotomy to increase bladder capacity and decrease reflex incontinence and sphincter contraction. Detrusor sphincter dyssynergia is avoided with rhizotomy, protecting the upper tracts and reducing autonomic dysreflexia. The pudendal nerve controls the external sphincter via the somatic nervous system, which is not affected by rhizotomy. Reference: (1) Sheffler LR, Chae J. Neuromuscular stimulation in neurorehabilitation. Muscle Nerve. 2007;35:571-3. (2) Consortium for Spinal Cord Medicine.Clinical practice guidelines. Bladder management for adults with spinal cord injury. J Spinal Cord Med. 2006;29:541, 550-1, 560-1.

139. A 2-month-old infant presents to you for evaluation of delay ed d evelopment. He was the product of a normal term pregnancy, labor, and delivery. Birth weight was 3500 grams. He has had difficulty feeding since birth. Family history is negative for developmental problems. On physical examination, he is awake, but not alert. Weight is 3600 grams. Respiration is unlabored. He has poor head control and decreased tone throughout. Deep tendon reflexes are absent. What is the most likely diagnosis? (a) Kugelberg Welander syndrome (b) Duchenne muscular dystrophy (c) Infantile botulism (d) Tetraplegic cerebral palsy

139. (d) This patient illustrates the diagnostic dilemma of the floppy infant. Causes of this problem include central nervous system lesions (both brain and spinal cord), myopathies, neuropathies, and neuromuscular junction problems. This infant has had abnormalities since birth, which argues against infantile botulism. Kugelberg Welander syndrome (also known as spinal muscular atrophy type 3) has onset during childhood, as does Duchenne muscular dystrophy. Tetraplegic cerebral palsy often presents in infancy with floppiness and hyporeflexia, which later change to spasticity and hyperreflexia.

14. A 13-year-old boy presents with waddling gait and difficulty in climbing s tairs. O n examin ation , h e demon stra tes s ig nific an t wea kness i n h is pr oximal lo wer ex tr emity muscles, especially the quadriceps, and some calf hypertrophy. What is the genetic inheritance of this disorder? (a) Autosomal dominant (b) X-linked recessive (c) Autosomal recessive (d) No genetic linkage

14 (b) The abnormal gene for Duchenne and Becker muscular dystrophy (DMD, BMD, respectively) is on the short arm of the X chromosome at position Xp21Reference . Both DMD and BM D are inherited Xlinked recessive diseases affecting primarily skel et al and myocardi al musc les. Dystr oph in is a large cytoskele tal pr ot ein in the s ub sarcole mmal lat ti ce, t he pro tei n t hat stab il izes t he pla sma me mbran e d ur ing musc le contracti ons. Mutations in the dystrophin gene that result in a complete loss of dystrophin lead to the DMD phenotype. Mutations that cause a reduced, truncated, or dysfunctional form of dystrophin to be produced lead to the BMD phenotype. Both DMD and BMD are progressive myopathies, although DMD is much more severe and is universally fatal. BMD shows a similar pattern of muscle weakness to DMD but with later onset and much slower rate of progression.

13 A 27-year-old manual laborer presents with a 6month history of right shoulder pain. He has a past medical history of a right shoulder dislocation after a wate r-s kiing accident 3 years ago. On physical exam, he has normal strength and sensation with symmetric reflexes. The shoulder apprehension test is positive. Impingement tests and the O'Brien active compression test are negative. Which diagnosis is most consistent with this presentation? a) Rotator cuff tendinitis a. b) Anterior-inferior labrum tear b. c) Glenohumeral osteoarthritis c. d) Superior labral anterior to posterior (SLAP) lesion

14 Option b is correct. Commentary: This patient presents with an anterior-inferior labrum tear related to chronic anterior shoulder instability following a prior traumatic even t (di slo cation). Unidirectional instability refers to instability in only 1 direction, anterior direction being the most common. This type of instability is common after a traumatic event. Multidirectional instability refers to laxity in more than 1 direction and is associated with congenital laxity or chronic repetitive microtrauma. Anatomically, there is disruption in the anterior-inferior glenohumeral joint capsule and anterior-inferior labrum. Superior labral anterior to posterior (SLAP) lesions may occur, but that possibility is less likely in this patient, because of his history and a negative O'Brien compression test. Reference: Finnoff JT. Musculoskeletal problems of the upper limb. In: Braddom RL, editor. Physical medicine and rehabilitation. 3rd ed. Philadelphia: El sevie r; 2007. p 832-3, 835.

14 Temporomandibular joints (TMJ) are involved in 50% of children with juvenile idiopathic arthritis (JIA) and this involvement can affect development of the m andible, leading to micrognathi Shou lder involvement in JIA is in frequ ent, as is involvement of the small joints of the han Which factor is a contraindication for phonophoresis? a. Contractures b. Neuromas c. Tendonitis d. Skeletal immaturity

14 Reference(s) Weber DC, Hoppe KM. Physical agent modalities. In: Braddom RL, editor. Physical medicine and rehabilitation. 4th e Philadelphia: Elsevier; 2011 . p 449-463. Option d is correct.In phonophoresis ultrasound is used to facilitate transdermal migration of topically applied medications. Corticosteroids are the most f requently used agents and the anti-i nfla mmatory effec ts of the me dicat ion and ultrasound may be synergisti Indications for phonophoresis are osteoarthritis, bursitis, capsulitis, tendonitis, strains, fasciitis, epicondylitis, tenosynovitis, contracture, scar tissue, neuromas, and adhesions. Contraindications for use of phonophoresis (or ultrasound) include general heat precautions, skeletal immaturity, and locations near the eyes, brain, reproductive organs, a pacemaker, a menstruating or gravid uterus, laminectomy sites, or malignancy.

14. A 57-year-old woman comp lain s of the onset of tingling in her feet about 2 weeks ago. She now has muscle weakness of both legs and weakness in her handgrip. She is complaining of pain in the back of both thighs. On examination, she is noted to have mild weakness of the orbicularis oculi bilaterally, intact extraocular muscle movements, intact peripheral sensation, decreased grip strength and lower limb strength measured at 3-/5. Biceps reflexes are present at 1+ and other reflexes are absent. Immediate management measures would include(a) oral multivitamins and f olat e.(b) twice daily vital capaci ty t estin g.(c) intrathecal steroid inje ctio n.(d) serial erythrocyte sedim enta tion rat es.

14. (b) This patient is most likely to have acute inflammatory demyelinating polyneuropathy of GuillainBarré syndrome. This syndrome presents with ascending symmetrical weakness, generally has mild sensory involvement although pain complaints are prominent, and commonly spares extraocular movements despite involvement of other cranial nerves. Management would include careful monitoring of vital capacity, since respiratory muscle weakness could result in the need for ventilation. Steroids by any route of administration have not been shown to be effective.

14. Myositis is defined as (a) muscle aching. (b) muscle aching with weakness. (c) muscle symptoms with creatine kinase eleva tio n. (d) muscle symptoms with creatine kinase and c rea ti nine elevations.

14. (c) Myopathy refers to a disease or an abnormal condition of striated muscle, whereas myalgia is defined as muscle aching or weakness without serum creatine kinase (CK) elevations. Myositis implies muscle symptoms accompanied by CK elevations. Rhabdomyolysis signifies muscle complaints with CK elevations 10 times the upper limits of normal (ULN) with creatinine elevation. Clinically important myopathy with CK elevations greater than 10 times ULN is estimated to occur in approximately 0.1% of patients who receive statin monotherapy. Clinically important myopathy and rhabdomyolysis have been reported with all statins with an overall death rate of .15 per 1 million prescriptions. Ref: Strommen JA, Johns JS, Kim C-T, Williams FH, Weiss LD, Weiss JM, Rashbaum IG. Neuromuscular rehabilitation and electrodiagnosis. 3. Diseases of muscles and neuromuscular junction. Arch Phys Med Rehabil 2005;86(3 Suppl 1):S18-27.

"

14. Myositis is defined as (a) muscle aching. (b) muscle aching with weakness. (c) muscle symptoms with creatine kinase elevation. (d) muscle symptoms with creatine kinase and creatinine elevation s." 14 (c) Myopathy refers to a disease or abnormal condition of striated muscle; whereas, myalgia is defined as muscle aching or weakness without serum creatine kinase (CK) elevations. Myositis implies muscle symptoms accompanied by CK elevations. Rhabdomyolysis signifies muscle complaints with CK elevations 10 times the upper limits of normal (ULN) with creatinine elevation. Clinically important myopathy with CK elevations greater than 10 times ULN is estimated to occur in approximately 0.1% of patients who receive statin monotherapy. Clinically important myopathy and rhabdomyolysis have been reported with all statins with an overall death rate of .15 per 1 million prescriptions.

140. What is the primary motion used to open a voluntary-opening terminal device in a b od y-p owe red tran srad ial pros the sis ?:(a) Glenohumeral flexion:(b) Glenohumeral extension:(c) Biscapular elevation:(d) Biscapular adduction

140 (a) The control cable for opening the terminal device of a standard transradial prosthesis using a voluntary-opening terminal device passes posterior to the el bow joint , a long the posteri or arm, an d a tta ches to the posterior s upport strap of a figur e-o f-8 harn ess . This ar ran ge men t all ows gl en ohumoral fl exion t o produc e ten sion in th e c able and op en the t erm ina l d evice. Glenohumoral extension and biscapular adduction decrease tension on the control cable. Biscapular elevation would not produce cable tension required for terminal device operation.

142. A 60-year-old woman wit h dy sarth ria and r ight arm weakness is admitted to your stroke unit. Her lower extremity strength is intact. You would expect the infarct to be located in the(a) posterior frontal lobe.(b) ventromedial thalamus.(c) lateral medulla.(d) anterior limb of the int erna l cap sul e.

142. (d) This is a classic description of the dysarthria-clumsy hand syndrome. The lesion is most commonly located in the anterior limb of the internal capsule but may also be seen with certain pontine lesions.

140. A patient presents to your clinic with a 1-month history of mild hand numbness and clumsiness without weakness. Electrodiagnosis confirms a primarily sensory median neuropathy at the wrist without axon loss. Symptoms are not interfering with work. What is the most appropriate treatment recommendation to provide short-term relief for this patient? (a) Immediate referral to surgery for carpal tunnel release (b) Neutral wrist splints to be worn at night (c) Thumb spica splint (d) Injection of platelet rich plasma into carpal tunnel

140 Answer (b) Commentary: Wrist splints are shown to effectively decrease symptoms of carpal tunnel syndrome in the short-term. Splints should be worn at night and during the day if possible. Brace should place wrist in neutral (up to 5o of extension): note that many off-the-shelf carpal tunnel braces place the wrist in excessive extensions. A thumb spica splint is not effective in treating carpal tunnel syndrome. Conservative treatment is essential in mild to moderate cases of carpal tunnel syndrome. Surgical referral should be considered for patients with weakness or worsening symptoms not improved with conservative treatment. Platelet rich plasma injections are not an effective treatment for carpal tunnel syndrome. Reference: a) Kelly BM, Patel AT, Dodge CV. Upper limb orthotic devices. In: Braddom RL, editor. Physical Medicine and Rehabilitation. 4th ed. Philadelphia: Elsevier Saunders; 2011:325326. b) Huistedde BM, Hoogvliet P, Randsdorp MS, Glerum S, Middelkoop M, Koes BW. Carpal tunnel syndrome. Part I. Effectiveness of nonsurgical treatments-a systematic review. Arch Phys Med Rehabil. 2010;91(7):981-1004.

140 Which phenomenon is an effect of functional electrical stimulation (FES) as it pertains to gait? (a) Decrease in muscle spasticity (b) Increase in physiologic cost of gait (c) Decrease in voluntary muscle strength (d) Decrease in stride length

140 Answer: (a) Commentary: In addition to a decrease in muscle spasticity, FES decreases the physiologic cost of gait, increases voluntary muscle strength, and increases stride length. Ref: Bogey R, Sisto S. Gait restoration and gait aids. In: DeLisa JA, Gans, BM, editors. Physical medicine and rehabilitation: principles and practice. 4th ed. Philadelphia: Lippincott; 2005. p 1397.

140 At mid stance, where is the ground reaction force vector located? (a) Anterior to ankle, posterior to knee (b) Anterior to ankle, anterior to knee (c) Anterior to knee, anterior to hip (d) Posterior to knee, posterior to hip

140 Answer: B Commentary: In mid stance, the ground reaction vector lies anterior to the ankle, anterior to or through the knee axis, and posterior to the hip center. The passive torques created by this vector alignment are ankle dorsiflexion, knee extension and hip extension. Ref: Rab GT. Muscle. In: Rose J, Gamble JG. editors. Human walking. 2nd Ed.Baltimore :Williams & Wilkins; 1994. p 114.

141. In order to obtain a semi-electric hospital bed for a patient w ho requir es freque nt ch ang es in body position, which Medicare guideline must be met? (a) The patient requires a bed with side rails for positioning the b ody in wa ys not fe asibl e w ith an ordinary bed. (b) The patient requires a heavy-duty bed due to morbid obesity, wit h body we ight over 500 lbs . (c) The caregiver requires a bed to be raised up to 48 inches to fac ilitate w ound care . (d) The patient requires traction equipment that can be attached onl y to a ho spital be d.

140. (d) By Medicare guidelines, to obtain a semi-electric hospital bed for a patient the physician must complete a certificate of medical necessity. Although a full electric bed used in most hospitals may be beneficial for many patients, Medicare guidelines consider the electric powered, variable height feature a convenience and, therefore, Medicare will not cover a full electric bed. A semi-electric hospital bed will be covered if the patient requires frequent changes in body position or has an immediate need for change in body position along with at least 1 of the following conditions: (1) The patient requires positioning of the body in ways not feasible with an ordinary bed. (2) For pain relief, the patient requires positioning of the body in ways not feasible in an ordinary bed. (3) Because of congestive heart failure, pulmonary disease, or aspiration, the patient requires the head of the bed to be elevated more than 30° most of the time. (4) The patient requires traction equipment that can only be attached to a hospital bed. The standard hospital bed has many disadvantages. The beds have a 10-inch excursion from a low of 24 inches to a high of 34 inches. Additionally, the standard hospital bed frame has a weight limitation of around 500 pounds.

141. A 39-year-old mail sorter complains of severe right elbow pain for 6 months. H e ha s tri ed ant iinfl amma tor ies w ith out relief. He d enie s n um bness, but does report weakness in his grip. On examination, the patient has severe pain with palpation just inferior to the lateral epicondyle. Which of the following findings will most likely be found on further examination?:(a) Pain will be increased with ulnar deviation of the wrist with resisted flexion.:(b) The patient's brachioradialis reflex will be significantly diminished or absent .:(c) Resisted wrist extension with a straightened elbow will reproduce the patient's pai n.:(d) Atrophy of the extensor digitorum communis and extensor indicis proprius will b e pr es ent .

141 (c) This patient has lateral epicondylitis or "tennis elbow," a condition brought on by repetitive flexionextension or pronation-supination of the forearm. The pai n wil l b e inc reased by r es isted wr ist ex tens ion wit h the elbow at 180°. The refl exes w ill not be aff ected, no r w il l a troph y be n ot ed. This is not a neurolog ic co nditi on , b ut a my ofas cia l o ne.

141. Which of the following is c orrec tly assoc iated with its kinetic chain?(a) The hand during a biceps cur l rep res ents a n open kinetic chain.(b) The foot during heel str ike repre sen ts an open kinetic chain.(c) The hand waving is an ex ampl e of a c losed kinetic chain.(d) The foot during a squat repr esent s a n exam ple of an open kinetic chain.

141. (a) An open kinetic chain occurs when the terminal segment is free to move. A closed kinetic chain occurs when the terminal segment is fixed. The hand during a biceps curl is free to move and represents an open kinetic chain.

143. Extraskeletal manifestations of ankylosing spondylitis include:(a) Achilles enthesopathy.:(b) tricuspid regurgitation.:(c) cataracts.:(d) nail pitting.

143 (a) Extraskeletal manifestations of ankylosing spondylitis include uveitis/iritis, aortic regurgitation, and enthesopathy.

143. A 25-year-old man with localized low back pain and knee pain that are wor se with rest and impr ove w ith a ctiv ities i s not ed to h ave r ec ent hi st ory of sa lmo nel la gastroenteritis. The most likely diagnosis is (a) enteropathic arthritis. (b) ankylosing spondylitis. (c) reactive arthritis. (d) septic arthritis.

143 (c) Reactive arthritis is a form of peripheral arthritis that ha s 1 or more extra-articular manifestations and appears 2 to 4 weeks after gastrointestinal or genitou rinary infections including salmonella, shigella, and campylobacter in you ng males . It is a type o f sp ondylo art hropath y, which also in clu des e nte rop athi c ar thropat hy, ankl os ing s po ndyl iti s, and psor ia tic ar th riti s.

143. A 12-year-old girl comp lain s of loc alized mid back pain for 3 months, with no history of trauma. She has a non-focal examination with a prominent thoracic kyphosis and compensatory cervical and lumbar lordosis. She also has compensatory hamstring tightness. The most likely diagnosis is(a) idiopathic juvenile scol iosi s.(b) juvenile ankylosing spon dyli tis.(c) Pagets disease.(d) Scheuermann's disease.

143. (d) These are all characteristics of Scheuermann's disease.

144. A 29-year-old man with slowly progressive neuromuscular disease is eager to maint ain over all f unc ti on an d pr eserv e mobil it y. Wh at ty pe of ex er cise p rog ram wo ul d you prescribe? (a) Moderate resistance, low repetition (b) High resistance, low repetition (c) Isokinetic resistance (d) Isometric resistance

144 (a) There have been several well-controlled studies looking at t he effect of exercise as a means to gain strength in neuromuscular diseases (NMDs). In slow ly progres sive NM Ds a 12-week, moderate resistance (30% of maximum isometric force) ex ercise program resulte d in stren gth gains ranging from 4% to 0.2 w ith out any n ot able de lete riou s eff ec ts . Ho wev er, in the sa me p op ul atio n, a 1 2-wee k, hi gh resis tance (train ing at the maximum weight a subject could lift 12 times) exercise program showed no further added beneficial effect compared to the moderate resistance program and there was evidence of overwork weakness in some of the subjects.

144. Which approach is an initial management strategy for sialorrhea that does not interfere with swallowing? (a) Antihistamine medication (b) Botulinum toxin injection (c) Irradiation of salivary glands (d) Salivary duct ligation

144 Answer: A Commentary: Antihistamine and anticholinergics have both been used with varying success and are often first line management options. Botulinum toxin has been used in cases of axillary hyperhidrosis and case reports identify use in sialorrhea. This approach should be considered after less invasive measures fail. For very thick secretions, hydration will help make the discharge thinner and easier to manage. In some cases, irradiation or surgery may be needed to allow safe swallowing. Ref: Bach, JA. Swallowing and gastrointestinal concerns In: Bach, JA. Management of patients with neuromuscular disease. Philadelphia: Hanley & Belfus; 2004. p 68.

144. A 52-year-old African A meri can w oma n pres ents with a 3-month history of fatigue, weight loss, proximal weakness, and muscle pain. Laboratory studies reveal an elevated creatine kinase level. Which treatment would NOT be considered appropriate?(a) Glucocorticoids(b) Methotrexate(c) Plasmapheresis(d) Intravenous gamma globul in

144. (c) In the treatment of inflammatory myopathies such as polymyositis and dermatomyositis, glucocorticosteroids are considered the first drug of choice. Patients refractory to steroids or unable to tolerate high doses because of complications require an immunosuppressive agent. Immunoglobulins are effective and are also used in patients with recurrent relapses. Plasmapheresis and leukapheresis are ineffective in these patients.

145. What is one reason for placing a suprapubic cathete r in a pe rson with a complete cervical spinal cord injury who currently uses intermittent catheterization? (a) Decreased rate of bladder/kidney infections (b) Decreased high bladder pressures (c) Decreased rate of bladder/kidney stone formation (d) Reduced risk of developing autonomic dysreflexia

145 (b) The rates of infections and stones are higher with suprapubic catheters. An indwelling catheter results in a slight increased risk of bladder cancer. High internal bladder pressures may occur as a result of detrusor sphincter-dyssynergia and avoiding reflux by allowing continuous drainage can be safer than intermittent catheterization for some individuals.

145. Autonomic dysreflexia is (a) best treated by placing the patient supine. (b) a common occurrence in patients with T8 spinal cord injuries. (c) predominantly characterized by parasympathetic activity. (d) rarely occurs earlier than 1 month after injury.

145 Answer: D Commentary: Autonomic dysreflexia is most commonly found in patients with spinal cord injury at T6 and above. It is associated with a release of sympathetic activity, which results in regional vasoconstriction. It is usually present by 6 months to 1 year after injury. Initial treatment involves prompt removal of the noxious stimulus and sitting the patient up. Ref: (a) Kirshblum S. Rehabilitation of spinal cord injury. In: DeLisa JA, Gans BM, editors. Physical medicine and rehabilitation: principles and practice. Philadelphia: Lippincott Williams & Wilkins; 2005. p 1724-5. (b) Campagnolo DI, Merli GJ. Autonomic and cardiovascular complications of spinal cord injury. In: Kirshblum S, Campagnola DI, DeLisa JA, editors. Spinal cord medicine. Philadelphia: Lippincott Williams & Wilkins; 2002. p 123-34.

146. A patient with stable angina develops chest pain and EKG changes during s tage V o f the Br uce p rot oc ol. Y ou i nform h im th at his risk of su ffe ri ng a m yoc ard ial i nfarction during moderate exertion is (a) none. (b) low. (c) moderate. (d) high.

146 (b) Patients who are able to complete Stage IV of the Bruce prot ocol without anginal symptoms have a low risk of myocardial infarction.

146. In a patient being evaluated for pulmonary rehabilitation, which sign suggests res tr ict ive lu ng di seas e?:(a) Plentiful sputum production:(b) Pursed lip breathing:(c) Paradoxical breathing:(d) Hyperresonant lung sounds

146 (c) Patients with restrictive lung disease, contingent on the underlying etiology, will rely on purely diaphragmatic or paradoxical breathing. Sputum produc tio n, a uxill ary musc le use, and h yperreso nan t l ung soun ds are characte ristic of pat ient s with pr edominan t o xygenatio n i mp air ment. Pursed lip breathing is used to prevent end expiratory airway collapse.

146. The most common symptom ass ociat ed with c ancer and its treatment is(a) pain.(b) weakness.(c) anorexia.(d) fatigue.

146 (d) Fatigue is the most common complaint of cancer patients, affecting up to 78% of patients, with 70% noting that fatigue affected their daily routine.

146. Which finding supports a diagnosis of complex regional pain syn drome typ e I? (a) Sensory deficits in a dermatomal distribution (b) Cutaneous allodynia (c) Focal motor weakness (d) Side-to-side symmetry of dermal temperature

146. (b) The Task Force on Taxonomy convened by the International Association of Pain developed the descriptor "chronic regional pain syndrome, type I" (CRPS I) to refer to sympathetically maintained pain that is not associated with a nerve injury. The presence of motor or sensory deficits suggests the presence of complex regional pain syndrome, type II which requires the presence of nerve compromise. The four criteria for CRPS I are: (1) the presence of an intiating noxious event, or a cause of immobilization; (2) continuing pain, allodynia, or hyperalgesia with which the pain is disproportionate to any inciting event; (3) evidence at some time of edema, changes in skin blood flow, or abnormal sudomotor activity in the region of the pain; and 4) CRPS I is excluded by the existence of conditions that would otherwise account for the degree of pain and dysfunction.

147. In order to appropriately follow Medicare regulatio ns for te achi ng physicians, when caring for a patient with a resident physician, the attending physician must (a) review the chart and personally document his/her lev el of inv olve ment in patient care, separate from documentation performed by the resident. (b) examine the patient with the resident and co-sign th e residen t no te. (c) examine the patient and review the resident's medica l record docu mentation. (d) examine the patient, review the resident's documenta tion, and per sonally document involvement in the history, exam, and medical decision-making.

147 (d) In order to appropriately follow Medicare regulations for teaching physicians, when caring for a patient with a resident physician, the attending physician must see the patient, review the medical record documentation of the resident, and personally document involvement in key aspects of the history, exam, and medical decision-making. Documentation from the resident alone does not confirm the level of attending physician involvement. The attending physician documentation combined with the resident documentation can be used to determine the level of care provided and the appropriate level of billing.

147 You recommend work hardening for a worker recovering from a shoulder injury. You explain to the worker to expect a therapy program that (a) builds aerobic conditioning and will be performed 2 hours daily. (b) simulates work duties and will be performed approximately 4 hours daily. (c) simulates a heavy manual labor job and will be performed 6 hours daily. (d) improves aerobic conditioning while simulating a light duty job and will be performed 8 hours daily.

147 Answer: B Commentary: Work hardening is a rehabilitation program designed to simulate the individual worker's job. It can be performed at a center or at the worker's jobsite. These programs are often recommended to be done 5 days a week. The worker performs an individualized program based on his/her specific job requirements. Physician follow-up is needed to determine if goals have been achieved. Work conditioning is a program used to enhance aerobic conditioning but does not attempt to replicate the tasks of a specific job. Ref: (a) Nadler SF, Stitik TP, Malanga GA. Optimizing outcome in the injured worker with low back pain. Crit Rev Phys Med Rehabil 1999;11:139-69.(b) Wohlberg K, Sontag M, Cole AJ, Wilder RP, Stratton SA. Return to work functional optimization programs. In: Cole AJ, Herring SA, editors. The low back pain handbook: a guide for the practicing clinician. 2nd ed. Philadelphia: Hanley & Belfus; 2003. p 179-99.

147. A 49-year-old, right-handed man who works at an automobile plan t paintin g cars pr esent s w ith a 4week history of right shoulder pain. His pain increases with overhead activities and with donning his coat. He is unable to sleep due to pain at night. With regards to work, you recommend physical therapy, ibuprofen and (a) modifying work activities to reduce overhead activities. (b) staying off work for 4 weeks. (c) no work restrictions or modifications. (d) staying off work until pain free with strengthening activities.

147. (a) Modifying the activity that is causing the repetitive overload is necessary if treatment is to succeed. Treatment success is measured by reduced pain and return to function, in this case to an occupation that requires repetitive overhead activities. The healthcare provider must communicate with the employer to best understand the specific musculoskeletal job requirements and modify these activities during the recovery phase of rehabilitation. Taking the worker off work entirely engages disability behavior and may not serve a specific recovery function. Not making a work modification is also a mistake, because the overload to the upper quadrant will continue and that will impair rehabilitation.

148. A 73-year-old hospitalized patient complains of pain in the ing uinal are a 6 hours afte r a cardiac angiogram. Examination reveals hip flexion and knee extension weakness. There is decreased sensation over the medial ankle. Which test would you order first? (a) Electrodiagnostic testing (b) Arteriogram of the legs (c) Venogram of the legs (d) Computerized tomography of the pelvis

148. (d) The nerve most likely affected in this type of situation is the femoral nerve, due to a retroperitoneal hematoma. The best way to assess for this acutely is by radiologic studies. Electrodiagnostic testing would be inappropriate in an acute setting, since needle examination findings usually take a few days to weeks to evolve, and the findings would not necessarily help in the acute management of this patient. Vascular studies may help identify the source of bleeding if the symptoms persist.

149. In traumatic brain injury in children, outcome is primarily related to (a) severity of original injury. (b) location of injury. (c) age at time of injury. (d) associated injuries.

149 (a) Although there is considerable variability from case to case, outcome is primarily related to the severity of the injury.

149. The most common musculoskeletal abnormalities seen in a chil d wi th L5 myelodysplasia with sparing of the L5 segment and above are (a) cavus foot, early hip dislocation, hip and knee flex ion contr actu res. (b) calcaneus foot, late hip dislocation, hip and knee f lexion co ntra ctures. (c) cavus foot, late hip dislocation, hip adduction cont ractures. (d) calcaneus foot, early hip dislocation, hip adduction contract ures .

149 (b) The child with L5 myelodysplasia typically has late hip dislocation, calcaneus foot, hip flexion contractures, and may have either knee extension or flexion contractures, depending on whether quadriceps (L2-4) or hamstrings (L4-S1) are stronger. Gluteus medius (hip abductor, L4-S1) and hip adductors (L1-3) are innervated higher than L5 and are typically balanced in L5 myelodysplasia. Late hip dislocation is due to either unbalanced hip musculature or spinal deformities.

149. The Education for All Handicapped Children Act (EHA, PL 94-142) , passed in 1976, and t he Individuals with Disabilities Education Act (IDEA, PL 105-17), passed in 1997, guarantee that children with disabilities have (a) education in special schools. (b) medical care at school. (c) education in the least restrictive environment. (d) education in regular classrooms.

149. (c) The Education for All Handicapped Children Act and the Individuals with Disabilities Education Act guarantee children with disabilities education in the least restrictive environment. They also guarantee necessary health care be provided in the school environment (eg, intermittent catheterization) but do not require medical care be provided.

149. Which finding would ind icat e a p oor long- term outcome in a 9-year-old child with a severe traumatic brain injury?(a) Bladder and bowel incont inen ce(b) Agitation(c) Dysphagia(d) Hypertension and hyperpy rexi a

149. (d) Most children with severe traumatic brain injury have dysphagia, incontinence, and agitation at some time during the recovery period. Central autonomic dysfunction (hypertension, hyperpyrexia, sweating, tachypnea, and rigidity) is associated with worse cognitive and motor outcomes a year or more after injury.

149. The most common congenital limb deficienc y i s (a) right transtibial limb deletion. (b) rig ht tr ansradial limb deletion. (c) left transtibial limb deletion. (d) left transradial limb deletion.

149. (d) The most common congenital limb deficiency is the left midlength transradial deficiency. Ref: Gaebler-Spira D, Uellendahl J. Pediatric limb deficiencies. In: Molnar GE, Alexander MA, editors. Pediatric rehabilitation. 3rd ed. Philadelphia: Hanley & Belfus; 1999. p 334.

15. Compared to persons with traumatic spinal cord injury, persons with non-tr aumatic spinal c ord i nju ry are more like ly to b e (a) under the age of 35 years. (b) female. (c) tetraplegic. (d) single.

15 (b) Persons with nontraumatic spinal cord injury (SCI) are older , more likely married, female, retired, and have significantly more paraplegia and incomple te injury than pe rsons with SCI of traumatic etiology, with neoplasm (53%) and cervi ca l spon dylosis ( 25%) as the leadi ng causes of nontraumatic inj ury .

15. For cystic fibrosis patients, the effects of exercise and an active lifest yle include (a ) improve d pulmona ry fu ncti on. (b) increased work capacity. (c) increased ventilatory muscle fatigue. (d) increased exercise-induced airway mucus. Ref: (a) Orenstein DM, Winnie GB, Altman H. Cystic fibrosis: a 2002 update. J Pediatr 2002 ;140:156- 64 (b) Orenstein DM, Hovell MF, Mulvihill M, et al. Strength vs aerobic trainin g in children with cyst ic fibros is: a ran domized controlled trial. Chest 2004;126:1204-14. Educational Activity 2.7

15 (b) Several studies of exercise programs for patients with cystic fibrosis have sho wn increased wor k c apacity, im proved cardiorespiratory fitness, improved ventilator y mu scl e endura nce, an d enh anced im mun e function. Increased exercise may not improve pulmonary function but may slow the progressive pulmonary decline. Acute effects include the mobilization of airway mucus.

152. As its mechanism of action, botulinum tox in (a) inactivates the calcium pump at the sarcop las mi c reticulum. (b) inhibits the troponin-tropomysin complex. (c) inhibits the production of acethylcholine. (d) inhibits the release of acethylcholine.

152. (d) Botulinum toxins act on the neuromuscular junction where they inhibit the release but not the production of acethylcholine (ACh). Botulinum toxin does not affect the sarcoplasmic reticulum, nor does it work at the troponin-tropomysin complex. Ref: Elovic E, Bogey R. Spasticity and movement disorder. In: DeLisa JA, Gans BM, Walsh NE, editors. Physical medicine and rehabilitation: principles and practice. 4th ed. Philadelphia: Lippincott Williams & Wilkins; 2005. p 1435.

15. What function is expected in an individual with a C7 ASIA class A spinal cord i njur y?:(a) Need assistance to perform level transfers:(b) Pressure reliefs primarily by side-to-side weight shift:(c) Independence in bowel and bladder management:(d) Independent dressing and bathing with adaptive equipment

15 (d) For persons with motor level C5, activities of daily living include drinking from a cup and feeding with static spoons and set-up, some oral/facial hygi ene , wr iting an d typ ing with eq ui pment, a nd pos sibl y so me upper-body d ressing. At t he C 6 inju ry level, i ndi viduals a re ab le to fe ed and p erform uppe r bod y dressing with set- up an d c an p erfo rm lev el su rfa ce tra nsfers with assistance. Persons with motor level C7 ASIA class A should be able to independently feed, dress, and bathe themselves, using adaptive equipment and built up utensils. They should be independent with bed mobility, and level surface transfers and should be able to propel a wheelchair outdoors. Independence in bowel and bladder function is generally seen with injury at level T1 and lower.

15. An individual with T4 ASIA C paraplegia must have (a) normal sensory function below T4. (b) sensation in the sacral segments S4-S5. (c) a muscle grade of 3 or greater in at least half of the key muscles below T4. (d) voluntary sphincter contraction.

15 Answer: B Commentary: All ASIA levels except ASIA A must include sensation through the sacral segments S4-S5. The ASIA C classification can include voluntary sphincter contraction but it is not required. An injury classed as T4 ASIA C would include sensation below T4 but the sensation may be normal or impaired. A muscle grade of less than 3 in more than half of the key muscles below the neurologic level would be expected with ASIA C. Ref: American Spinal Cord Injury Association, International Medical Society of Paraplegia. International standards for neurological and functional classification of spinal cord injury. Chicago: American Spinal Cord Injury Association; 2006.

15 Which orthosis best corrects a swan neck deformity in rh eumatoid arthrit is? a. Ring splint b. Spica splint c. Sugar tong splint d. Ulnar deviation splint

15 Option a is correct. Ring splints can be used to help correct swan neck defor mities in rheuma toid art hri ti s. Th ey all ow flex ion bu t b lo ck h ype rext en si on at the prox imal int erpha lan gea l ( PIP ) jo int. Thumb spica splints are used for conditions such as DeQuervain tenosynovitis and osteoarthritis. Sugar tong splints are used to immobilize a joint such as the elbow or wrist in patients with fractures. Ulnar deviation splints may be used to correct ulnar deviation in rheumatoid arthritis.

15 What is the benefit of using ankle-foot orthoses (AFOs) for a patient with Duchenne muscular dystrophy? a) Improves the patient's ambulation a. b) Assists the patient with rising from the floor b. c) Prevents contractures when used at rest c. d) Improves wheelchair positioning

15 Option c is correct. Commentary: Resting AFOs can help prevent ankle plantar flexion contractures, but are not required for proper wheelchair positioning. Duchenne muscular d ystro phy is an x-linked disease with progressive muscle weakness/degeneration that is usually diagnosed in early childhood. Loss of independent ambulation generally occurs in early adolescence, necessitating the transition to a wheelchair. The progressive weakness leads to compensatory strategies for ambulation such that AFOs may further impede ambulation or transferring. Reference: (a) Busby K et al. Diagnosis and management of Duchenne muscular dystrophy, part 2: implementation of multidisciplinary care. Lancet Neurol 20 10;9: 177 -89. (b) McDonald C, Han J, Carter G. Myopathic disorders. In: Braddom RL, editor. Physical medicine and rehabilitation. 4th ed. Philadelphia: Elsevier-Saunders; 2011. p1126.

15. Based on the revised edition of the Americ an Sp inal Injury Association (ASIA) Impairment Scale, published in 2000, which condition would be sufficient to categorize a spinal cord injury as motor incomplete? (a) Some motor function more than 1 level belo w t he motor level (b) Voluntary anal sphincter contraction (c) A well-defined zone of partial preservatio n (d) An anterior spinal artery syndrome

15. (b) For an individual to receive an ASIA classification of motor incomplete (ASIA C or D), he/she must have either voluntary anal sphincter contraction or sensory sacral sparing with sparing of motor function more than 3 levels below the motor level. The zone of partial preservation is used only in complete injuries. Individuals with anterior spinal artery syndrome are often motor complete. Ref: American Spinal Injury Association/International Medical Society of Paraplegia. International standards for neurological and functional classification of spinal cord injury patients. Chicago:ASIA/IMSP; 2002.

150. A 75-year-old woman falls and fractures her distal humerus. In the process, the ulnar nerve is damaged. She presents 6 months later with weakness in the fourth and fifth digits, claw deformity, and loss of grip power. Which of the following orthotic components could benefit this patient and, therefore, should be included in the orthotic prescription? (a) Dorsal outrigger (b) C bar (c) Lumbrical bar (d) Opponens bar

150 (c) A dorsal metacarpophalangeal extension stop (also called a lumbrical bar) to the fourth and fifth digits is usually quite effective at preventing hyperextension of the fourth and fifth metacarpophalangeal joints. This permits the proper wrapping of the fingers around an object and thus allows a stronger grip.

151. The DeLorme axiom states that:(a) low-weight high-repetition exercises build strength, and high-weight low-repeti tion e xer cis es build end ura nce.:(b) high-weight low-repetition exercises build strength, and low-weight high-repeti tion e xer cis es build end ura nce.:(c) low-weight low-repetition exercises build endurance, and high-weight high-repet itio n exe rci ses buil d st ren gth.:(d) high-weight high-repetition exercises build endurance, and low-weight low-repet itio n exe rci ses buil d st ren gth.

150 (c) Trauma is the most common cause of upper extremity amputation in adults. Malignancy is a more common cause of amputation in the pediatric population. Up per ext remit y a mputa tions occur m ost comm onl y i n th e 20 to 40-year-ol d age group, as a resul t o f trauma an d work re lat ed ac ciden ts. Th e dominant ex tremi ty is affe cted more co mmo nly and amp uta tio ns at th e t ran sradial level are the most common level of upper extremity amputation. Upper extremity amputations occur much less frequently than lower extremity amputations. Upper extremity amputations account for approximately 20% of all major limb amputations.

152. A 45-year-old man with left hemiparesis following a stroke comp lains of left shou lder pai n with ambulation. Which of the following is the most probable cause? (a) Cervical radiculopathy (b) Impingement syndrome (c) Adhesive capsulitis (d) Shoulder subluxation

152. (d) Inferior subluxation of the glenohumeral joint occurs frequently following stroke. Pain in the shoulder is often felt in the upright position, since gravity further aggravates the subluxation.

150. A balanced forearm orth osis is i ndi cated for patients with weakness in which muscle-group combination?(a) Deltoid and elbow flexor s(b) Deltoid and triceps(c) Pectoralis group and pro nato rs(d) Pectoralis group and tri ceps

150. (a) A balanced forearm orthosis can be attached to a wheelchair. It consists of a forearm trough, which is attached by a hinge joint to a ball-bearing swivel mechanism and a mount. It supports the weight of the forearm and arm against gravity. With only minimal muscle force requirement at the shoulder girdle and trunk, the patient can move the arm horizontally and flex the elbow to bring the hand to the mouth. This orthosis is primarily used for patients with severe upper limb weakness (especially the deltoid and elbow flexors), as in high quadriplegia or other severe neuromuscular conditions. The patient must also have sufficient range of motion of the shoulder and elbow, as well as adequate trunk stability (provided or innate) while sitting.

150. In adults, the prevalence of phantom limb pa in , phantom sensation or residual limb pain after amputation is (a) approximately 70% at 6 months postamputati on. (b) dependent on age at the time of amputation . (c) directly related to surgical technique. (d) primarily dependent upon the level of ampu tat io n.

150. (a) Phantom sensation, phantom pain, and residual limb pain have all been reported about equally in over 70% of amputees 6 months or more after lower limb amputation. This is typically not dependent upon the person's age at the time of amputation, the level of amputation, or surgical technique. Ref: Ehde DN, Czerniecki JM, Smith DG. Chronic phantom sensation, phantom pain, and residual limb pain and other regional pain after lower limb amputations. Arch Phys Med Rehabil 2000;81:1039-44.

150. One disadvantage to adding camber to a wheelchair is that it (a) decreases side to side stability. (b) exposes the hands to injury. (c) results in poor ergonomic positioning of push rims. (d) makes maneuvering in narrow spaces difficult.

150. (d) Camber has several advantages. The footprint of the chair is widened creating greater side to side stability; camber allows quicker turning; camber helps to protect the hands by having the bottom of the wheels scruff edges; and camber positions push rims more ergonomically for propulsion. A disadvantage is that the increased width of the wheelchair may make it difficult to maneuver in an environment made for walking (ie, narrow spaces).

151. A 37-year-old woman presents with complaints of paresthesias in the fourt h and fi fth digi ts of he r domin ant hand fo r the l ast 3 year s. Sh e h as vague co mpl ain ts of aching in and around her shoulder and neck. She reports great difficulty with overhead activities. She reports intermittent swelling and discoloration of her hand with these activities. She has no wasting of her muscles on exam, and Tinel's maneuver at the elbow is negative. You suspect (a) thoracic outlet syndrome (b) myofascial pain syndrome (c) rotator cuff tendonopathy (d) Raynaud's syndrome

151 (a) For a provocative test to be positive for thoracic outlet sy ndrome (TOS), the patient must volunteer that the maneuver exactly reproduces the symptoms. Adson's t est is performed by palpating the radial pulse while turning the patient's h ead to ward the affecte d si de, ex ten ding hi s/her neck, and holding the affected arm at the side. This will result in lo ss of the radial pulse. The costoclavicular maneuver is performed by having the patient mov e the shou lders b ack and down, resulting in a loss of the radial pulse. The elevated a rm str ess test (a/k/a Roos ' test ) i s perfo rmed by having t pat ie nt hol d hi s/ he r arms over head w hil e op enin g a nd clo sing th e ha nd s. A p ers on with TOS w ill repo rt pain and will fatigue within 30 seconds. A person without TOS will not report pain and will be able to perform the test for up to 3 minutes. Compression of the ulnar nerve is not a maneuver for the diagnosis of TOS. For a provocative test to be positive for thoracic outlet sy ndrome (TOS), the patient must volunteer that the maneuver exactly reproduces the symptoms. Adson's t est is performed by palpating the radial pulse while turning the patient's h ead to ward the affecte d si de, ex ten ding hi s/her neck, and

151. Which spinal level has the greatest depth of poster ior epidu ral space? (a) C3-4 (b) C4-5 (c) C5-6 (d) C6-7

151 (d) The C6-7 and C7-T1 epidural levels have the greatest amount of space. Interlaminar epidural injections should be performed with caution in the spaces that have a smaller diameter, such as those at stenotic levels or high cervical levels. Practitioners should also be aware that the ligamentum flavum may have defects in a high percentage of individuals.

151. The medical literature supports the use of lumbar epidural ster oid injec tions for whic h c ondition? (a) Sclerotomal pain from facet-mediated low back pain (b) Referred pain from a quadratus lumborum trigger point (c) Neurogenic claudication due to lumbar spinal stenosis (d) Radiculopathy caused by foraminal disc herniation

151. (d) Epidural steroid injections (ESIs) are indicated for radicular rather than referred pain. While anectodotal descriptions of ESIs used for neurogenic claudication may present them as helpful, the efficacy of ESIs have yet to be established for this condition.

152. A 67-year-old patient with coronary artery disease suffered a stroke 1 we ek ago. His rest ing v ita l signs are : blo od pres su re 14 0/86, p uls e 8 7 beats per mi nut e, respiration 18 breaths per minute and oxygen saturation 97%. Which change in vital signs would warrant stopping a therapy session? (a) Oxygen saturation 92% (b) Diastolic blood pressure 110mmHg (c) Systolic blood pressure 130mmHg (d) Heart rate 105 beats per minute

152 (b) An increase in the diastolic blood pressure is indicative of an unstable cardiac condition. The other choices are normal responses to exercise.

153. What is the World Health Organization definition of osteoporosis?:(a) Bone mass 2.5 standard deviations below normal:(b) Bone mass 2.0 standard deviations below normal:(c) Bone mass 1.5 standard deviations below normal:(d) Bone mass 1.0 standard deviation below normal with history of fracture

152 (d) Clinical presentation is not nonspecific. Patients often complain of severe headaches ("worst of their lives") and present with loss of consciousness. A tri oven ous m alf ormat ion present w ith hemo rrh age s ea rlie r i n life, in t he second or thir d deca de. Aneurys ms are most com mo nly foun d in t he anterior r egion o f the ci rcle of Wi ll is, pa rtic ular ly nea r bra nch es of the ant erior communicating, internal carotid, and middle cerebral arteries.

157. Which financial barrier impedes return to work?:(a) Partial salary and potential monetary gain through litigation:(b) Lack of a retirement plan at work:(c) Low salary prior to the injury:(d) lack of employer-provided health insurance

157 (a) A worker may have 2 financial disincentives to return to work. First, employees usually receive a portion of their salary while unable to work, if the i nju ry o ccurr ed while the worker w as perfo rmi ng his/ her job . Second, th e worker may not want t o r eturn to wo rk if mon eta ry ga in is being s ought throu gh pe nd ing liti gatio n.

153. A 60-year-old man with left total knee arthroplasty 5 days prior continues to have difficulty with ambulation during rehabilitation. On exam, he has 70oof active knee flexion, a 20oextensor lag, and a distal lower limb normal to palpation. You then notice that he has trouble clearing his toes during swing phase. You suspect the major cause of his difficulty walking is due to (a) weak quadriceps strength. (b) inadequate knee flexion range. (c) commonperoneal nerve palsy. (d) tibialis anterior tendon tear.

153 Answer: C Commentary: The patient has a common peroneal nerve palsy which can occur after total knee arthroplasty. Weak quadriceps strength and inadequate knee flexion may cause difficulty with ambulation, but not the loss of ankle dorsiflexion. Tibialis anterior tendon tear will cause difficulty with ankle dorsiflexion, but is not a common complication after knee arthroplasty. Also, acute tendon tears present with sudden pain and palpatory defect. Ref: Guzman J. Rehabilitation of patients with rheumatic diseases. In: Braddom RL, editor. Physical medicine and rehabilitation. 3rd ed. Philadelphia: Saunders; 2007. p 791.

153. What articular abnormality is associated wit h systemic lupus erythematosus and its treatment? (a) Avascular necrosis (b) Erosions with subchondral sclerosis (c) Erosions with overhanging edges (d) Syndesmophytes

153. (a) Avascular necrosis (or ostenecrosis) is the second form of joint disease that occurs in persons with SLE; it is associated with use of corticosteroids. Erosions are not seen with systemic lupus erythematosus. Erosions with subchondral sclerosis are seen in rheumatoid arthritis and overhanging edge in gout. Syndesmophytes occur in spondyloarthropathies at the anterior and posterior longitudinal ligaments of the spine. Ref: Klippel JH, editor. Primer on rheumatic disease. 12th ed. Atlanta (GA): Arthritis Foundation; 2001. p 337-8.

154. Which muscle fiber is characterized by fast-twitch oxidative met abolic properties? (a) Type 1 (b) Type 2a (c) Type 2b (d) Type 3

154 (b) Humans have 2 primary types of muscle fiber. They are divided according to many different characteristics, including speed of contraction and sources of fuel. Type 1 muscle fibers are slowtwitch with oxidative metabolic pathways. Type 2 muscle fibers are fast-twitch fibers. The type 2 fibers can then be further divided into fast-twitch oxidative (type 2a) and fast-twitch glyclolytic (type 2b). There are no muscle fibers designated as type 3.

154. A 28-year-old man returns to clinic after failing conservative management for clinical medial epicondylitis. In order to determine the appropriateness of a surgical referral, what is the most cost effective diagnostic test to localize the site of pathology? (a) Plain radiographs of the elbow and forearm (b) Magnetic resonance imaging of upper extremity (c) Real time ultrasound (d) Electrodiagnostic studies

154 Answer: C Commentary: Real time ultrasound is less costly than magnetic resonance imaging (MRI) and has similar sensitivity and specificity in diagnosing medial epicondylitis. Plain radiographs and electrodiagnostic studies will not help localize or confirm your diagnosis of medial epicondylitis, but may help with diagnosing a fracture or nerve injury, respectively. Ref: Park G, Lee S, Lee M. Diagnostic value of ultrasonography for clinical medial epicondylitis. Arch Phys Med Rehabil 2008:89:738-42.

155. Which antispasticity medication is relatively contraindicated in individu als who have mot or in com pl ete s pina l cor d injur y? (a) Baclofen (Lioresal) (b) Dantrolene (Dantrium) (c) Diazepam (Valium) (d) Tizanidine (Zanaflex)

155 (b) Dantrolene sodium depresses the release of calcium from the sarcoplasmic reticulum and is unique in having a direct effect on skeletal muscles. It acts on all sk eletal muscles and may weaken partially innervated muscles, causing muscle s which are funct ional t o be coming no nfuncti onal.

155. Which statement is TRUE concerning traumatic spinal cord injury (SCI)? (a) More than 80% of individuals identified as having motor incomplete SCI at 72 hours after their injury will walk. (b) There is a plateau of functional recovery after incomplete SCI that occurs after the first 3 months. (c) More than 80% of individuals with complete tetraplegia will regain 2 motor levels below their initial injury level. (d) Approximately one-third of individuals with SCI have complete injuries and two-thirds have incomplete injuries.

155 Answer: A Commentary: The majority of patients with complete tetraplegia regain 1 level below their original injury. Up to 87% of motor incomplete subjects (ASIA C) identified at 72 hours postinjury were ambulating at 1 year. The ratio of complete to incomplete SCI is close to 50:50. Recovery after incomplete SCI is often most rapid up to 6 months postinjury but can still occur at a slower rate after 2 years. Ref: Ditunno JF, Flanders AE, et al. Predicting outcome in traumatic spinal cord injury. In: Kirshblum S, Campagnola DI, DeLisa JA, editors. Spinal cord medicine. Philadelphia: Lippincott Williams & Wilkins; 2002. p 108-22.

156. A patient with Kugelberg-Welander disease is hospitalized with acute respirato ry f ai lur e. Wor k-up reve als the pat ien t to hav e p ea k co ugh f lows less than 270L/min. After successfully weaning the patient from ventilatory support, in order to minimize future episodes of acute respiratory failure, you instruct the patient in:(a) manually assisted cough.:(b) air shifting.:(c) pursed lip breathing.:(d) inspiratory resistive loading.

156 (a) During otherwise benign upper respiratory tract infections, patients with predominantly ventilatory impairment generally develop acute respiratory failu re due to in eff ectiv e coughing. W hen peak co ugh flo ws a re less than 30 0L/min patien ts s hould be taught m axi mal insuf fla ti on techn iques.

156. Which fitness program has been shown to improve both balance an d eccentr ic quadri ceps str ength? (a) Pilates (b) Tai Chi (c) Hatha Yoga (d) Jazzercise

156 (b) Tai Chi, an ancient Chinese form of the martial arts increases eccentric knee extensor muscle strength, with a consequent improvement in postural sway. However, studies have not found significant changes in concentric knee flexion strength among practitioners of Tai Chi.

156. Which type of cryotherapy uses conduction for energy transfer? (a) Cold packs (b) Fluidotherapy (c) Vapocoolant spray (d) Whirlpool baths

156 Answer: A Commentary: Conduction is a process of transferring thermal energy between 2 entities placed in direct contact with each other, for example cold packs on skin. Convection is a process of using a medium to transfer energy: Examples of convection include the use of husks with fluidotherapy, and the use of water with whirlpool therapy. Vapocoolant sprays are an example of evaporation, not conduction. Ref: Weber D, Hoppe K. Physical agent modalities. In: Braddom R, et al. Physical medicine and rehabilitation 3rd ed. Philadelphia: WB Saunders; 2007. p 468.

157. Disability as defined by the Americans with Disabilities Act (ADA) is (a) a physical or mental impairment that substantially limits 1 or more major life activities. (b) abnormality of the physiologic or anatomic structure or function. (c) the barriers society places on the individual interacting in his/her community. (d) a rating based on an independent medical examination.

157 Answer: A Commentary: The Americans with Disabilities Act defines disability as a physical or mental impairment that substantially limits 1 or more of a person's major life activities. The person has a record of such impairment, or is regarded as having such impairment. Impairment is the actual physiologic, anatomic, or psychologic abnormality. Handicap refers to the barriers society places on an individual to perform function in the community. A permanent disability rating is used to determine financial compensation for an injury. Ref: US Equal Employment Opportunity Commission. Americans with Disabilities Act. Washington, DC: US Dept of Justice, Civil Rights Division; Updated August 2002.

157. You see the significant other of a close fri en d in your office for knee pain. As part of her past medical history you note that she has a congenital heart defect. She says she has not yet told your friend that she has this condition. You decide to tell your friend about her congenital heart defect even though the patient did not give you permission to do so. What penalty do you face for knowingly disclosing individually identifiable health information, which is in violation of HIPAA rules? (a) $50,000 and up to 1year of imprisonment (b) No penalty (c) $250,000 and up to 10 years imprisonment (d) $100,000 and up to 5 years imprisonment

157. (a) A person who knowingly discloses individually identifiable health information in violation of HIPAA faces a fine of $50,000 and up to a 1-year imprisonment. The criminal penalties increase to $100,000 and up to 5 years imprisonment if the wrongful conduct involves false pretenses, and to $250,000 and up to 10 years imprisonment if the wrongful conduct involves the intent to sell, transfer, or use individually identifiable health information for commercial advantage, personal gain, or malicious harm. Ref: US Department of Health and Human Services. Summary of the HIPAA Privacy Rule, May 2003. US Dept Health and Human Service Web site. Available at: http://www.hhs.gov/ocr/privacysummary.pdf. Accessed July 4, 2007.

158. A 35-year-old woman is sent to you for electrodiagnostic evaluation. Her symptoms include numb nes s in th e ri ght 4 th and 5t h dig its a nd in th e medial ha nd dis ta l to the wrist. Your primary diagnosis is MOST likely to be (a) ulnar neuropathy. (b) C8/T1 radiculopathy. (c) lower trunk plexopathy. (d) medial cord plexopathy.

158 (a) The distribution described is most consistent with involveme nt of the ulnar nerve proximal to the dorsal cutaneous branch. Involvement of the C8/T1 roo ts, lower trunk o f the brachial plexus, or medial cord of the brachial plexus would us ually result in some a ddit ional sen sory de ficits over the med ial f ore arm pro xi ma l to th e wr ist.

158. Studying F-wave responses would be most helpful in making an early diagnosis o f:(a) cervical radiculopathy.:(b) myotonic dystrophy.:(c) multiple sclerosis.:(d) Guillain-Barré syndrome.

158 (d) Guillain-Barré syndrome (acute idiopathic demyelinating polyradiculoneuropathy) commonly affects the most proximal portion of the nerve while sparing th e m ain nerve tr unk i n the early stages. While motor conduction velocities obtained with ordinary techniques may be normal, Fwave studies help detect proximal abnormalities.

158. Palliative care is characterized by (a) a holistic approach to comprehensive symptom management. (b) symptom only management in persons with terminal illnesses. (c) disease modifying therapies. (d) care provided in the home setting only.

158 Answer: A Commentary: Palliative care involves a holistic approach to comprehensive symptom management. This care has a potential role in the management of all disease states that feature an intense and adverse symptom complex. Palliative care typically does not include disease modifying therapies, although it can be provided in conjunction with these treatments. Palliative care can be provided in a variety of health care settings and is not limited to persons with terminal illnesses. Ref: Cheville AL. Palliative care. In: DeLisa JA, Gans BM, Walsh NE, editors. Physical medicine and rehabilitation: principles and practice. 4th ed. Philadelphia: Lippincott-Raven; 2005. p 531.

159. What condition casues the typical "myopathic gait" seen in a young boy with Du chen ne mu scu lar dyst roph y w ith a cce ntu ated lum bar l ordo sis a nd toe walking?:(a) Hip and knee extensor weakness:(b) Hip flexion and ankle plantar flexion contractures:(c) Hip extensor weakness and plantar flexion contracture:(d) Hip flexion contracture and knee extensor weakness

159 (a) The typical "myopathic gait" seen in early Duchenne muscular dystrophy is caused by weakness of the gluteus maximus and quadriceps muscles. In order to mai ntai n upr igh t pos ture the ch il d assume s t he hype rlor dot ic stance. C ontractures o f th e gast roc soleus a nd iliopsoas mu sc les occu r late r in the diso rder.

159. In terms of continuous quality improvement, a sentinel event is defined as (a) a benchmark event that sets the standard for patient care. (b) an occurrence that requires dismissal of personnel. (c) a single occurrence that is highly problematic or socially unacceptable. (d) an event that results in the opening of a new hospital program.

159 Answer: C Commentary: In terms of continuous quality improvement, a sentinel event is defined as a single occurrence that is highly problematic or socially unacceptable. Sentinel events will typically trigger an in-depth root cause analysis to determine the cause of the event as well as potential solutions. The focus of these investigations is to evaluate the processes and systems that are in place rather than to focus blame on individual practitioners. Ref: Johnston MV, Maney M, Wilkerson DL. Systematically assuring and improving the quality and outcomes of medical rehabilitation programs. In: DeLisa JA, Gans BM, editors. Rehabilitation medicine: principles and practice. 3rd ed. Philadelphia: Lippincott-Raven; 1998. p 290.

16. During discussion with the parents of a 2-year-old leukemic patient, you inform the m tha t s ign ifica nt b rai n irr adi ati on almos t u ni form ly pr oduces:(a) attention deficits.:(b) focal motor weakness.:(c) ataxia.:(d) anosmia.

16 (a) An adverse sequela of intensive pediatric anticancer therapy is learning difficulty. Impaired learning can exert a deleterious long-term impact. Whole b rai n ir radia tio n for leukemic p ro phylaxis re sul ts i n en lar ged cerebral sulci and ve ntri culome gal y on cra nia l imaging . C li nic al symptomatology roughly correlates with scan findings. Virtually all patients in whom a substantial portion of the brain is radiated complain of memory l oss and atte nti onal deficits.

16. Which factor predicts increased mortality from pulmonary sarcoidosis? (a) African-American descent (b) Supplemental oxygen dependence greater than 2L/min (c) Low mean pulmonary artery pressure (d) Chronic low-dose corticosteroids Ref: Shorr AF, Davies DB, Nathan SD. Predicting mortality in patients with s arcoidosis awa iting lun g transpl antat ion. Chest 2003;124:922-8. Educational Activity 2.8

16 (a) Variables found to predict mortality in pulmonary sarcoidosis are being of Afri canAmerican desc ent , having a mean pu lmonary artery pr essure of 40mmHg, and using a mea n f low of s uppleme nt al oxygen o f 3 L/min. Chronic low-dose corticosteroids were shown to improve pulmonary function.

16 A 72-year-old woman is receiving warfarin (Coumadin) for deep venous thrombosis (DVT) prophylaxis after repair of a hip fracture. She is on several other medications. The medication that will significantly elevate her international normalized ratio (INR) is (a) diphenhydramine (Benadryl). (b) acetaminophen (Tylenol). (c) carbamazepine (Tegretol). (d) ranitidine (Zantac).

16 Answer: (b) Commentary: Warfarin (Coumadin) is used for anticoagulation in several different disease conditions while patients are under the care of a physiatrist. One of the drug's most common applications is for DVT prophylaxis after repair of a hip fracture. Many medications can alter the therapeutic efficacy of warfarin. Sulfonamides, acetaminophen, amiodarone, aspirin, and nonsteroidal anti-inflammatory drugs (NSAIDs) may increase the prothrombin (PT)/INR. Adrenocorticoids, antacids, antihistamines, carbamazepine, haloperidol, and vitamin C can decrease the PT/INR. Ref: Stitik TP, Klecz R, Grenwald B. Pharmacotherapy of disability. In: DeLisa JA, , Gans BM, Walsh, NE, editors. Physical medicine and rehabilitation: principles and practice. 4th ed. Philadelphia: Lippincott Williams & Wilkins; 2005. p 1231.

16. Which scale evaluates sensory perception, moisture, activity, mobility, nutrition, and friction/shear to determine risk of pressure ulcers? (a) Norton (b) Barthel (c) Braden (d) Beck

16 Answer: C Commentary: Both the Braden scale and the Norton scale are used to assess pressure ulcer risk. The Braden scale consists of 6 factors: sensory perception, moisture, activity, mobility, nutrition, and friction/shear. The Norton scale assesses 5 factors: physical condition, mental condition, activity, mobility, and incontinence. The Barthel index measures activities of daily living and mobility and is not related to assessing pressure ulcer risk. The Beck Depression Inventory and Beck Anxiety Inventory are not related to pressure ulcers. Ref: (a) O'Connor K. Pressure ulcers. In: Physical medicine and rehabilitation: principles and practice. 4th ed. DeLisa JA, Gans BM, Walsh NE, editors. Philadelphia: Lippincott Williams and Wilkins; 2005. p 1609. (b)Cheville AL. Palliative care. In: Physical medicine and rehabilitation: principles and practice. 4th ed. DeLisa JA, Gans BM, Walsh NE, editors. Philadelphia: Lippincott Williams and Wilkins; 2005. p 533.(c) Christiansen CH. Functional evaluation and management of self-care and other activities of daily living. In: Physical medicine and rehabilitation: principles and practice. 4th ed. DeLisa JA, Gans BM, Walsh NE, editors. Philadelphia: Lippincott Williams and Wilkins; 2005. p 983.

16 A 13-year-old girl with a T6 ASIA D spinal cord injury s ince age 2 years comp lai ns of r igh t k nee pa in w ith am bul at ion. Yo u fi nd t hat sh e ha s de creas ed right hi p a bdu cti on a nd a shorter right leg. Knee examination is normal except for tenderness to palpation anteriorly. The most likely finding on radiographs would be a. heterotopic ossification. b. avascular necrosis of the femoral head. c. hip subluxation. d. fracture of the patella.

16 Option c is correct. Children with spinal cord injuries that occur prior to a ge 10 years are at hi gh ris k of hi p s ubl uxa tion (93% had h ip s ubl uxat io n compar ed t o 9% with SC I occ urr ing ov er age 10). Classic signs of hip subluxation are knee pain, decreased hip abduction, and apparent shortening of the thigh length. Hip subluxation on radiographs is defined as migration index greater than 20% and medial joint space greater than 7mm.

16 The iliotibial band (ITB) is a fibrous band and increasing its tensile force increases compressive forces against the fat pad underneath the ban Biomechanic al contributions to ITB syndrome inc lude hip flexor i mb alance, dy namic knee valgus, contralateral hip drop. Activating and strengthening gluteus maximus, the most powerful femoral external rotator, can help prevent dynamic knee valgus and prevent ITB syndrome. Using the rectus femoris instead of the iliopsoas to generate hip flexion may lead to compensatory tensor fascia lata (TFL) activity causing hypertonicity of the TFL and hip flexor imbalance. Although strengthening hip abductors is helpful, the abductors to be strengthened should be the gluteus minimus and medius, not the TFL. Strengthening hip adductors can increase dynamic knee valgus tendencies , exacerbating symptoms. Activating and strengthening gluteus medius, quadratus lumborum, and external obliques prevents contralateral pelvic drop. A 35-year-old office worker presents with 1 week of right neck and upper limb pain that radiates down her arm and forearm to her long finger. She does not r emember any inciting trauma or exert ion associated wi th the onset of h er symptoms. Her biceps reflex is preserved and the triceps reflex is diminished in the affected arm. Which nerve root is most likely affected? a) C5 b) C6 c) C7 d) C8

16 Reference(s) (a) Rhee JM, Yoon T, Riew K Cervical radiculopathy. J Am Acad Orthop Surg 2007; 15(8):486-94. (b) Feinberg JH, Moley PJ. The physical examinati on. In: Frontera WR, DeLisa JA, Gans BM, Walsh NE, Ro bi nson LR, e ditor s. DeLisa's physical medicine & rehabilitation: principles and practice. 5th e Philadelphia: Lippincott Williams & Wilkins; 2010. p56. Option c is correct.

16. Lambert-Eaton myasthenic syndrome is most com mo nly associated with cancer in the (a) prostate. (b) breast. (c) lung. (d) brain.

16. (c) Lambert-Eaton myasthenic syndrome is most commonly associated with small-cell lung cancer, but it may also be seen in kidney and rectal cancer, malignant thymoma, basal cell carcinoma, and leukemia. Ref: Tunkel RS, Lachman E, Ho ML. Cancer. In: Grabois M, Garrison SJ, Hart KA, Lehmkuhl LD, editors. Physical medicine and rehabilitation: the complete approach. Malden (MA): Blackwell Sci; 2000. p 1699.

160. Which statement is NOT TRUE about the practice of evidence-base d medicin e? (a) It incorporates the conscientious and explicit use of the curren t best ev idence in maki ng treatment decisions. (b) It includes use of the medical literature to develop strict stan dards of care for clini cal practice. (c) It integrates individual clinical expertise with the best availa ble clini cal evide nce f rom systematic research. (d) It includes life-long, self-directed learning because caring for patients creates the n eed for clinically important information.

160 (b) Evidence-based medicine does not include the development of strict standards of care for clinical practice. Instead, evidence-based medicine emphasizes the conscientious and explicit use of the current best evidence in making decisions about the care of individual patients. Integrating individual clinical expertise with the best available clinical evidence from systematic research and life-long, self-directed learning in which caring for patients creates the need for clinically important information are also part of evidence-based medicine.

160. The validity of a functional outcome measurement tool is defined as the ability (a) of two different raters to obtain the same conclusion. (b) of the tool to measure what it is designed to measure. (c) to minimize random error. (d) to measure several different outcomes simultaneously

160 Answer: B Commentary: The validity of a functional outcome measurement tool is defined as the ability of the tool to measure what it is designed to measure. The ability to measure different outcomes simultaneously does not impact the validity of the instrument, but the validity of the tool would need to be established for each of the outcomes being measured. The ability of two different raters to obtain the same conclusion is referred to as inter-rater reliability. Freedom from random error is also related to the reliability of the instrument. Ref: Granger CV, Black T, Braun SL. Quality and outcome measures for medical rehabilitation. In: Braddom RL, editor. Physical medicine and rehabilitation. 3rd ed. Philadelphia: Elsevier Inc; 2007. p 155.

160. Informed consent requires that risks asso cia te d with participation in a study be described in terms of (a) type, severity, and probability. (b) probability alone. (c) physical, but not psychological, risks. (d) major, but not minor, risks.

160. (a) Informed consent for participation in a research study requires that the risks be described in terms of type, severity, and probability. It is important to describe risks in each of these areas in order to fully inform the patient of the potential risks associated with study participation. Ref: Lo B, Feigal D, Cummins S, and Hulley SB. Addressing ethical issues. In:Hulley SB, Cummings SR, editors. Designing clinical research. Baltimore: Williams & Wilkins; 1988. p 153.

161. An 18-year-old girl presents with a 2 week history of fever, chills, and right kn ee pain. On e xam in ation you note h er ri gh t kne e is ho t a nd sw ollen. Sh e h as a painless, red maculopapular rash on her trunk. She denies trauma, but has an antalgic gait. What is the most likely cause of her symptoms? (a) Staphylococcus epidermidis (b) Neisseria gonorrhoeae (c) Monosodium urate monohydrate crystals (d) Haemophilus influenzae

161 (b) This patient has fever, chills, and rash. She likely has an infectious cause for her monoarthropathy. She denies traumatic etiology of knee swelling. S he may be sexuall y active, and she should be questioned about this. Her symptoms mos t closel y match t he diag nosi s of g ono ccocal arthritis. This con dit io n o ccu rs f ol lo wing in fect ion wi th go no rrhe a, whi ch prese nt s as m on oart hro pat hy in 25 % of patie nts. The Nei sseria gonorrhoeae organism can be recovered in less than 50% of purulent effusions. It affects women as often as men (1:1), and its highest incidence is among sexually active adolescents and young adults. It is the most common cause of septic arthritis in patients younger than 30 years. If the strain of bacteria is not antibiotic-resistant, recovery is expected. Single joint arthritis follows generalized spread (dissemination) of the gonococcal infection. Dissemination is associated with symptoms of fever, chills, asymmetric polyarthralgias (which may be migratory), and rashes (1-mm to 2-cm red macules). The most commonly involved joints are the large joints such as the knee, wrist, and ankle. Tenosynovitis may also be seen with disseminated disease. Gouty arthritis does not present with rash, fever and chills. In neonates, the most common cause of joint infection is Haemophilus influenzae.

161. A 23-year-old postgraduate student presents to your office with bilateral knee pain. She just began training for a half marathon but has been limited by her knee pain. She reports pain in the anterior aspect of the knee and describes it as "beneath the knee cap." The pain is worse when arising after sitting for a prolonged period of time. Which physical examination finding might you expect in this patient? (a) Pes cavus (b) Strong hip abductors (c) Negative Ober test (d) Tight quadriceps muscles

161 Answer: D Commentary: Patellofemoral arthralgia is thought to result from tracking problems of the patella within the trochlear groove. Several biomechanical issues, such as tight and inflexible quadriceps, pes planus, tight iliotibial band, weak and ineffective vastalis medialis, and weak hip abductors, may contribute to incorrect tracking of the patella. The Ober test assesses the tensor fascia lata and iliotibial band for contracture and inflexibility. Ref: Hansen PA, Willick SE. Musculoskeletal disorders of the lower limb. In: Braddom RL, editor. Physical medicine and rehabilitation. 3rd ed. Philadelphia: Elsevier; 2007. p 862-3.

161. A 45-year-old woman with fibromyalgia presents to y ou with c ompl aints of poor sleep. She notes that she has no trouble falling asleep but has a difficult time staying asleep. She has improved her sleep hygiene and eliminated caffeine from her diet. Which medication would you recommend? (a) Diazepam (Valium) (b) Eszopiclone (Lunesta) (c) Chloral hydrate (d) Fluoxetine (Prozac)

161. (b) Patients with trouble initiating sleep may require shorter acting medications, while those with fragmented sleep and frequent awakenings may more ideally benefit from medications with an intermediate to long half-life. A third nonbenzodiazepine hypnotic, eszopiclone, is FDA approved for long-term management of insomnia and retains a greater half life (5h-5.8 hours) with evidence of greater sleep maintenance efficacy as compared to the current relatively shorter half-life Z-drugs.

162. A 60-year-old woman had a stroke 1 week ago. On examination you find loss of pain-andtemperature sensation on the right side of her face as well as on the left side of her body. You also note some nystagmus, with right eye ptosis and miosis. What is the most likely location of the lesion? (a) Lateral pons (b) Frontoparietal lobe (c) Lateral medulla (d) Medial basal midbrain

162 Answer: C Commentary: A lesion in the lateral medulla causes Wallenberg syndrome and is associated with ipsilateral loss of facial pain- and temperature-sensation and contralateral loss of body pain-andtemperature sensation. Ipsilateral Horner syndrome (ptosis, miosis and anhidrosis) is found, as well as nystagmus, dysphagia and dysphonia. Ref: Whyte J, Hart T, Laborde A, Rosenthal M. Rehabilitation issues in traumatic brain injury. In: DeLisa JA, Gans BM, Walsh NE, editors. Physical medicine and rehabilitation: principles and practice. 4th ed. Philadelphia: Lippincott Williams & Wilkins; 2005. p 1660.

162. Which factor is a risk for heterotopic os sif ic ation in traumatic brain injury? (a) Late seizures (b) Prolonged coma (c) Male gender (d) Diabetes insipidus

162. (b) Significant risk factors for heterotopic ossification in traumatic brain injury include prolonged coma (>1 month), increased muscle tone, limited movement in the involved lower extremity, and associated fractures. Late seizures, gender, and diabetes insipidus are not associated with increased risk of heterotopic ossification. Ref: Whyte J, Hart T, Laborde A, Rosenthal M. Rehabilitation issues in traumatic brain injury. In: DeLisa JA, Gans BM, Walsh NE, editors. Physical medicine and rehabilitation: principles and practice. 4th ed. Philadelphia: Lippincott Williams & Wilkins; 2005. p 1691.

162. A patient with a recent stroke and hemiplegia prese nts to yo ur c linic and is noted to have a genu recurvatum gait pattern. An aggressive stretching program has improved ankle range-of-motion, but not her spasticity and gait. The most appropriate treatment is (a) an ankle foot orthosis with 5º of plantarflexion. (b) Achilles tendon lengthening. (c) phenol motor point injection to the hamstrings. (d) botulinum toxin injection to the gastrocsoleus muscl e group.

162. (d) Genu recurvatum is a common atypical gait pattern in patients with upper motor neuron pathology. It may be caused by ankle plantarflexor spasticity, heel cord contracture, quadriceps weakness, or spasticity and a combination of the above impairments. In this case an ankle foot orthosis with 5º of plantarflexion would worsen the gait. A tendon lengthening would be aggressive and more conservative management should be attempted first. A phenol motor point injection to the hamstrings would make knee control more problematic. Botulinim toxin can be very helpful for focal spasticity and can decrease ankle plantarflexor spasticity and decrease the backward force at the knee.

163. Which recommendation promotes correct posture in patients with ankylosing spondyl itis? (a) Daily corner wall stretches (b) Soft mattress (c) Sidelying (d) Full sit ups

163 (a) Spinal extension should be promoted so that patients fuse in the most functional position. This also includes lying prone on a firm mattress.

163. Which pharmacologic agent is a secondary cause of osteoporosis?:(a) Heparin:(b) Diltiazem:(c) Naproxen:(d) Hydrochlorothiazide

163 (a) There are many secondary causes of osteoporosis. Drug induced osteoporosis is seen with alcohol, heparin, steroids, phenytoin and tobacco. Other causes inc lude immo bil izati on, multipl e myeloma, an d e ndoc rine re lated syndro mes such as h yper thyroi dis m, hyper par athyroidi sm, a nd diabe tes me ll itus type I I.

163. According to the motor unit size principle, which statement is TRUE about muscle activation? (a) Recruitment of smaller units is followed by recruitment of larger units. (b) Motor unit size gradually decreases with increased recruitment. (c) Motor unit size is independent of the force of muscle contraction. (d) Larger motor units are found in larger muscles.

163 Answer: A Commentary: The motor unit size principle, which has been supported by many investigators, states that during muscle activation, smaller motor units are activated first and the larger motor units are recruited with more forceful contraction. Ref: De Lateur B. Therapeutic exercise. In: Braddom R, et al. Physical medicine and rehabilitation 2nd ed. Philadelphia: WB Saunders; 2000. p 398.

163. A patient presents to your office with kn ee pa in from a flare of rheumatoid arthritis. She has a mild effusion and warmth at her knee. The therapist wants to use ultrasound to her knee for treatment. You advise that ultrasound (a) accelerates healing. (b) helps with pain control. (c) is contraindicated. (d) will not help with the pain.

163. (c) Ultrasound use is contraindicated in acute rheumatoid arthritis. Ultrasound has been shown to help with pain, but not in the context of acute inflammation. Ref: Basford JR. Therapeutic physical agents. In: DeLisa JA, Gans BM, Walsh NE, editors. Physical medicine and rehabilitation: principles and practice. 4th ed. Philadelphia: Lippincott Williams & Wilkins; 2005. p 258.

164. Parents of children with Duchenne muscular dystrophy who want them to par ticipate in an e xerci se pr ogram sho uld b e advis ed to (a) encourage them to exercise to the point of fatigue. (b) initiate a graduated, submaximal exercise program. (c) have them participate in a competitive sports program. (d) preclude exercise because it could make the disease progress faster.

164 (b) Intervention with submaximal exercise training in neuromuscu lar disease improves physical performance and increases muscle efficiency, and thus helps f ight fatig ue and improve quality of life. However, in children, the exercise program m ust be conducte d in an enj oyable se tting. Supervision is n ece ssa ry to ma ke s ur e the chi ldre n do n ot pl ay to exh aus tio n, wh ic h co ul d prod uce mu scle dam ag e a nd ov erwork weakn ess.

164. Regarding diabetic peripheral neuropathy, which statement is TRUE?:(a) Autonomic fiber involvement is rare.:(b) Motor fibers are affected before sensory fibers.:(c) Prognosis for recovery is good with adequate serum glucose control. (d) Both ax ons an d m yel in are a ffec ted .

164 (d) Diabetes-related peripheral neuropathy involves both motor and sensory fibers and both the axon and myelin are affected. Sensory fibers are the earliest to be affec ted and autonomic i nv olvement is co mmon . Ti ght serum gluco se control li mits progr ess ion of t he neuropath y, bu t d oes n ot hel p recovery.

175. Your adult patient with a spinal cord inj ury n eeds to access his bathroom in his standard-width wheelchair. If no turn is required following entry into the bathroom, the minimal width of the doorway should be (a) 26 inches. (b) 32 inches. (c) 36 inches. (d ) 40 inches.

175. (b) The proper minimum width of a doorway for a wheelchair without a turn is 32 inches. If a turn is involved, then the doorway width should be at least 36 inches. Ref: Hsiao I, Hodne T. Architectural considerations for improving access. In: Lin VW, editor. Spinal cord medicine principles and practice. New York: Demos; 2003. p 975-86.

164. Which muscle fiber type uses only glycolytic metabolism for energy? (a) Type 1 (b) Type 2a (c) Type 2b (d) Type 3

164 Answer: C Commentary: There are 2 primary muscle fiber types in humans. They are categorized according to speed of contraction and sources of fuel. Type 1 muscle fibers are slow-twitch with oxidative metabolic pathways. Type 2 muscle fibers are fast-twitch fibers. The type 2 fibers can then be further divided into fast-twitch with both oxidative and glycolytic metabolism (type 2a) and fasttwitch glycolytic (type 2b). Ref: Wilder R, Jenkins J, Seto C. Therapeutic exercise. In: Braddom R, et al. Physical medicine and rehabilitation 3rd ed. Philadelphia: WB Saunders; 2007. p 424.

164. The maximum safe exposure of the skin to heated water is 45° Ce lsius for 30 minut es. Par affin dipwrap coats the skin with a mixture of mineral oil and paraffin at a temperature of 52° Celsius. Why does the skin not get burned? (a) Protective subcutaneous fat layer enhanced by mineral oil (b) More rapid cutaneous vasodilation with paraffin (c) Conversion of thermal energy to kinetic energy (d) Paraffin's lower thermal conductivity

164. (d) Paraffin has a lower thermal conductivity than water, which allows it to be placed on the skin at a higher temperature without causing injury to the skin. The heat is more slowly transferred from the paraffin to the skin, which provides for heating over a longer period of time.

165. You are caring for a patient with a T3 ASIA class A spinal cord injury who com plai ns of bu rni ng pa in i n h is le gs. Ad ditional re vi ew o f s ys tems includes urinary leakage between catheterizations, sexual dysfunction, and difficulty sleeping. The best pharmacologic intervention at this time would be:(a) amitriptyline (Elavil).:(b) paroxetine (Paxil).:(c) trazodone (Desyrel). (d) fluoxetine (Prozac).

165 (a) Amitriptyline, a tricyclic antidepressant is among the classic first line treatments in neuropathic pain. Most common side effects related to tricyclic ant idep ressa nts are related mai nl y to the an tic holi nerg ic effects and include dry m outh , urin ary retenti on, and seda tio n. Fo r thi s pati en t who has d iffic ul ty sleep ing, as we ll as ur inar y le aka ge betwe en cat het erizati ons, the anticholinergic sideeffects may prove to be of benefit. Trazodone has not been demonstrated to reduce pain in for spinal cord injury. Paroxetine causes insomnia and sexual dysfunction and therefore would not be appropriate in this patient. Venlafaxine, sertraline, and fluoxetine have proven to be of limited benefit for neuropathic pain.

165. For an individual who has C5 tetraplegia, orthotic splinting att empts to maintain the functional position of the hand. This usually includes (a) closing the thumb web space. (b) 30º to 40º of metacarpophalangeal flexion. (c) promoting flattening of the palmar arch. (d) supporting the wrist in 20º to 30º of extension.

165. (d) The functional position of the hand includes supporting the wrist in 20º to 30º of extension, supporting the palmar arch with the 4th and 5th metacarpals slightly anterior to the second and third digits. Metacarpophalangeal flexion of 30° to 40° would be excessive. The thumb web space should be preserved.

166 What is the most beneficial combination of weight and plane of orientation when ordering cervical traction to treat an acute cervical radiculopathy? (a) 15--25 pounds applied with neck in extension (b) 75--100 pounds applied with neck in flexion (c) 55--75 pounds applied with neck in neutral (d) 25--35 pounds applied with neck in flexion

166 Answer: D Commentary: The weight for cervical traction is most beneficial at 25 to 35 pounds of force. Positioning the neck at 20o to30o of flexion provides the maximal effect of distraction between the vertebrae. Ref: Wieting JM, Andary MT, Holmes TG, Rechtien JJ, Zimmerman G, et al. Manipulation, massage, and traction. In: DeLisa JA, Gans BM, Walsh NE, editors. Physical medicine and rehabilitation: principles and practice. 4th ed. Philadelphia: Lippincott; 2005. p 301-5.

167. Under the Health Insurance Portability an d A cc ountability Act (HIPAA) Protected Health Information is data that (a) a physician can withhold from a patient. (b) a patient's job supervisor can obtain from a ph ysician. (c) can be used to identify a patient. (d) can be shared with family without permissi on.

167 (c) Protected Health Information includes individually identifiable health information. This is information, including demographic data, that relates to any of the following particulars: the individual's past, present or future physical or mental health or condition; the provision of health care to the individual; or the past, present, or future payment for the provision of health care to the individual. It can also be information that identifies the individual or for which there is a reasonable basis to believe it can be used to identify the individual. Individually identifiable health information includes many common identifiers (e.g., name, address, birth date, Social Security Number). Ref: US Department of Health and Human Services. Summary of the HIPAA Privacy Rule. US Dept Health and Human Service Web site. Available at: http://www.hhs.gov/ocr/privacysummary.pdf. Accessed July 4, 2007.

167. Abdominal belts worn by workers have been shown to restrict whi ch motion s in the lumba r s pine? (a) Lateral bending and rotation (b) Lateral bending and flexion (c) Rotation and translation (d) Flexion and translation

167. (a) Research models have shown that abdominal support belts limit lateral bending and rotation. A 10cm leather belt increased stiffness at the torso during lateral bending, thereby reducing motion in these planes. No changes in torso stiffness were noted, despite the belt being worn at full lumbar flexion.

167. Which condition is a cumulative trauma disorder tha t has bee n as sociated with intensive computer use? (a) Herniated thoracic disc (b) Shoulder adhesive capsulitis (c) Post-traumatic stress syndrome (d) Cervical myofascial pain

167. (d) The United States Department of Labor has determined that computer work is associated with a significant number of musculoskeletal disorders, many of which are considered cumulative trauma disorders. Examples include cervical and thoracic myofascial pain, rotator cuff tendonitis, medial and lateral epicondylitis, de Quervain tenosynovitis, and carpal tunnel syndrome.

168. In electromyographic testing of the ulnar nerve for entrapment at the elbow, t he p ri mar y r eas on fo r fl exi ng th e e lbo w to 45° or m ore is th at this position:(a) causes the ulnar nerve to be more superficial and thus easier to stimulate.:(b) allows more accurate assessment of the length of the nerve.:(c) allows for the least possible stimulus overflow to the radial nerve.:(d) causes increased compression of the nerve, thus improving the diagnostic yield.

168 (b) For testing of ulnar conduction around the elbow, flexion of 45° or greater is preferred because conduction studies frequently show slowing in the elbow se gmen t whe n t he fu lly extende d position is us ed. The dis crepancy is thought to ar ise from t he underest ima tion of t he ac tua l len gth of t he nerve wh en us in g surfac e dis tance m eas ure ment .

168 Which attribute is a characteristic of an experimental research design? (a) Results primarily provide information on associations between variables. (b) Manipulation of experimental variables is controlled. (c) It is retrospective in nature. (d) It is an observation of the natural history of a disease process.

168 Answer: B Commentary: Experimental research designs enable investigators to determine a cause and effect relationship between 2 variables; whereas, non-experimental research designs can only establish an association between 2 variables. In an experimental research design an intervention or experimental variable is manipulated and its effect on other variables is measured. Experimental designs are prospective in nature. Cohort studies, in which researchers observe the natural history of a disease process, are non-experimental. Ref:(a) Katz RT, Priebe MM, Campagnolo DI. Research in physical medicine and rehabilitation. In: Braddom RL, editor. Physical medicine and rehabilitation. 2nd ed. Philadelphia: WB Saunders; 2000. p 168-9.(b) DeLisa JA, Millis SR, Gans BM. Research in physical medicine and rehabilitation. In: DeLisa JA, Gans BM, Walsh NE, editors. Physical medicine and rehabilitation: principles and practice. 4th ed. Philadelphia: Lippincott-Raven; 2005. p 1110.

168. Federal research regulations stipulate that each institutional research review bo ard ( IRB ) is responsible for approving all research studies at their particular institution. What condition is the IRB NOT specifically required to determine prior to approving a research protocol? (a) That risks related to the research are minimized (b) That selection of subjects is equitable (c) That protections for privacy and confidentiality are in place (d) That research does not involve vulnerable subjects such as priso ners

168. (d) Federal research regulations stipulate that each institutional research review board (IRB) is responsible for approving all research studies at their particular institution. Before approving a research protocol, the IRB must determine that risks are minimized, selection of subjects is equitable, protections for privacy and confidentiality are in place, informed consent is appropriate and will be documented in writing, and that the study has plans for data monitoring where appropriate. Research may involve subjects who are considered vulnerable, such as prisoners, but the research must add extra protections for these subjects.

169. A research study is performed to assess the degree of spasticity experienced by individuals after traumatic brain injury. Spasticity in the gastrocnemius muscle is evaluated in 10 patients using the Modified Ashworth Scale (0-5 scale). Which descriptive statistic is appropriate for summarizing the ordinal data measured with this scale? (a) Ratio (b) Median (c) Correlation (d) Mean

169 Answer: B Commentary: The appropriate descriptive statistical method used to summarize ordinal data such as the values used in the Modified Ashworth Scale is the median. The median value is defined as the value that occurs in the middle of a set of values. The mean would be used to summarize ratio or interval type of data. The other options listed are not descriptive statistical measures. Ref: (a) Katz RT, Priebe MM, Campagnolo DI. Research in physical medicine and rehabilitation. In: Braddom RL, editor. Physical medicine and rehabilitation. 2nd ed. Philadelphia: WB Saunders; 2000. p 171-2. (b) Findley TW, Daum MC. Research in physical medicine and rehabilitation. III. The chart review, or how to use clinical data for exploratory retrospective studies. Am J Phys Med Rehabil 1988;70(Suppl):S25-6.

169. Once an individual becomes board-certified in the specialty of physical medicine and r eha bilitation, he/she must continue to fulfill certain requirements in order to maintain certification status. Which action is NOT a requirement for maintenance of certification? (a) Continuing medical education credits (b) Maintenance of active medical licensure (c) Completion of a recertification examination every 10 years (d) Publication of at least 1 article in a scientific journal every 10 years

169. (d) Once an individual becomes board certified in the specialty of physical medicine and rehabilitation, he/she must continue to fulfill certain requirements in order to maintain their certification status. Publication of 1 article in a peer-reviewed journal every 10 years is not a requirement for maintenance of certification. All of the other options listed are required.

17. Global Initiative for Obstructive Lung Disease criteria are used to stage levels of chro nic obstr uctive pu lmona ry d isease based on (a) clinical signs and symptoms. (b) physical examination findings. (c) pulmonary function tests.(d) current medical illnesses. Ref: (a) Celli BR. Standards for the optimal management of COPD: a summary. Chest 1998;113 (4 Suppl) :283S-7S. (b) Celli BR. ATS standards for the optimal management of chronic obstructiv e pulmonary di sease. Re spirology 1997 ; 2( Suppl 1):S1-4. S74 Clinical Activity 2.9

17 (c) Global Initiative for Obstructive Lung Disease criteria are used to classify ch ronic obstructiv e p ulmonary di sease i nto mild, moderat e, and severe levels based on pul mon ary func tion te st s ( forced e xpi ratory volume in 1 second, forced vital capacity). Clinical symptoms are not considered until the severe stage, when presence of respiratory failure and right heart failure are evident.

17. Why is it important to ask the injured worker if he/she is involved in litigati on a ga ins t t he workp lace ?:(a) It allows the physician to deny seeing the patient to avoid a deposition.:(b) It is not an appropriate question for the physician to ask.:(c) Workers with attorneys involved are less likely to return to work.:(d) The employer will be more cooperative in facilitating return to work.

17 (c) Several studies have found a strong relationship between attorney involvement and not returning to work. In workman's compensation cases involving lawye rs, 0.73 of i nju red w orkers did no t return to wo rk. In c ase s without la wyer involvem ent, 32% d id not retu rn to work. The se fa ctors are i mp ortant for the p hy sician t o kno w ini ti all y t o be st u nde rst and t he psy cho social barriers involved in treating the injured worker.

17 A pediatrician asks your advice about ordering a speech therapy referral for a 2 ye ar o ld chi ld who se p arents fe el he isn 't s pe ak ing as wel l as his old er si bli ng did at the same age. By what age should a child be able to use 2-word phrases? a. 18 months b. 24 months c. 30 months d. 36 months

17 Option b is correct. By 18 months a child should have a 10- to 25-word vocabu lary using singl e wor ds at a ti me. By 24 mo nths this sho ul d ha ve incr ea se d to a t le ast 50 wo rds and the y s hou ld be p utting 2 words together, for example, "go now." By age 3 a child should be able to state his or her full name, count to 3 and recognize basic colors.

17 The C7 root is the most likely affected nerve root in this case, since the biceps reflex is preserved and the triceps reflex is diminished, and the patient' s pain radiates into the long finger . A history of tr au ma or phys ical exertion preceding the onset of symptoms occurs in less than 15% of patients In trying to differentiate psoriatic arthritis (PA) from rheumatoid arthritis (RA), persons with PA have as a distinguishing characteristic a) dactylitis. b) involvement of the metacarpophalangeal joints. c) erosions on radiographs. d) positive family history.

17 Reference(s) Boumpas DT, Illei GG, Tassiulas IO. Psoriatic arthritis. In: Klippel JH, Weyand CM, Crofford LJ, Stone JH, Arthritis Foundation, editors. Primer on the rheumatic diseases. 12th e Atlanta (GA): Arthritis Foundation; 2001. p 233-7.Gladman D Psoriatic arthritis. In: Klippel JH, Stone JH, Crofford LJ, White PH, editors. Primer on rheumatic disease. 13th e Atlanta (GA): Arthritis Foundation; 2008 p 173-4. Option a is correct. Dactylitis (diffusely swollen digits due to tenosynovitis and arthritis) are distinguishing characteristics of PA and not R The other choices occur in both psoriatic arthritis and rheumatoid a rthr itis patients .

170. Which shoe modification would NOT be appropriate for an individual with loss o f pr ot ect ive se nsati on o n t he fo ot sec ondary t o d ia bete s?:(a) Wide toe box:(b) Rocker bottom:(c) Soft inner shoe liner:(d) Solid steel shank

170 (d) All of the shoe modifications listed would be appropriate for an individual with loss of protective sensation except for the use of a solid steel shank bui lt i nto t he sole of the shoe . This mod ifi cat ion woul d d ecrease the accommodative pro pertie s o f the sh oe and may i ncr ea se the p otenti al for skin b reakd ow n.

170. Informed consent documents for participation in research protocols must include language stating (a) that confidentiality cannot be assured. (b) that once the document is signed, participation will be required. (c) the procedures to be used in the study. (d) that the results of the study will be shared with the subject.

170 Answer: C Commentary: Informed consent documents for participation in research protocols must include the procedures to be used in the study. Informed consent documents must also assure confidentiality of the subjects and outcomes of individuals participating in the research. Informed consent documents must also state that subjects have the right to terminate involvement in the study at any time, even after they have signed the consent document. It must also state that the decision to terminate involvement in the study will not affect the individual's ongoing medical care. Informed consent documents do not provide information that research results will be shared with the individual subjects. Ref: Lo B, Feigal D, Cummins S, and Hulley SB. Addressing ethical issues. In: Hulley SB, Cummings SR, editors. Designing clinical research. Baltimore: Williams and Wilkins 1988. p 153-4.

170. Which attribute is a characteristic of an experimental research design? (a) It allows one to determine a cause-and-effect relationship. (b) It permits no manipulation of experimental variables. (c) It is retrospective in nature. (d) It is an observation of the natural history of a disease process .

170. (a) Experimental research designs allow researchers to determine a cause-and-effect relationship between 2 variables, whereas non-experimental research designs are only able to establish an association between 2 variables. In experimental research an intervention or experimental variable can be manipulated, and its effect on other variables can be measured. Experimental designs are prospective in nature. Cohort studies that focus on observation of the natural history of a disease process are non-experimental.

170. Which statement describes an advantage of a single- subject r esea rch design (that is, A-B-A or multiple baseline design)? (a) It can account for variability between subjects. (b) It permits medication trials with no washout period. (c) It can establish cause and effect relationships. (d) It is useful for interventions with prolonged or ext ended eff ects .

170. (c) An advantage of a single-subject research design (A-B-A design) is that this design can establish cause and effect relationships similar to other true experimental designs. Single subject research designs involve systematic, repeated measurement of a dependent variable over time through 1 or more baseline and intervention phases. The primary limitation with a single-subject research design is that it only establishes the cause and effect relationship for the subject involved in the study. Therefore, these results cannot be assumed to occur in others, because of the variability between subjects. Typically, a single subject research design requires a washout period between medication trials to ensure that the effects of the medication are no longer active. Single subject research designs are especially useful for interventions that do not have extended or prolonged effects. If the intervention has only short-term effects, then a difference in the outcome measured can be clearly demonstrated by comparing results when the intervention is in use against results obtained when it has been removed.

171. Muscle energy technique is a manual medicine treatment that is used to:(a) stabilize unstable segments in the spine.:(b) strengthen painful muscles by isotonic contraction.:(c) increase functional range of motion.:(d) improve efficiency of muscular oxygen extraction.

171 (c) Muscle energy technique is used to increase mobility of a hypomobile segment, increase functional range of motion, allow the return of symmetrical motio n t o af fecte d s egmen ts, strengt he n weaken ed mus cles , an d l engthen cont racted or spa stic muscl es. It uses is ometric c ont ra cti ons, not is ot onic ones. It is n ot used for u nstab le se gme nts, and do es not i mpr ove pr oprioce ption or focus on facilitating volitional activity.

176. Performing a leg press exercise is an example of an (a) open kinetic chain exercise. (b) closed kinetic chain exercise. (c) isokinetic exercise. (d) isometric exercise.

176 (b) Open kinetic chain exercise occurs when the most distal segment is not in contact with a surface (eg, leg extensions). Closed kinetic chain exercise occurs when the most distal segment is in contact with a surface (eg, a leg press). In isokinetic exercise a muscle contracts with a constant angular velocity and variable resistance. In isometric exercise a muscle contracts against an immovable object and there is no joint angular movement.

171. Which statement is TRUE regarding complex regional pain synd rome (CRPS)? (a) Pain is characterized by allodynia. (b) Local osteopenia is a common early occurrence. (c) CRPS type 1 is also known as causalgia. (d) Adults with CRPS have a better prognosis than childr en with C RPS.

171. (a) Complex regional pain syndrome (CRPS), previously known as reflex sympathetic dystrophy, as well as by other names, is characterized by a preceding noxious event; allodynia is an exaggerated pain response (ie, hyperesthesia) in response to a non-noxious stimulus or to vascular changes such as those indicated by paleness and coolness or by edema. Sudeck's atrophy is a name previously given to late stage CRPS when osteopenia is present. Osteopenia is a rare and late occurrence with CRPS. CRPS type 2 is also referred to as causalgia and is instigated from an initial nerve injury. Children with CRPS have a better prognosis than adults.

172. Which statement is TRUE regarding central nervous system (CNS) tumors? (a) Meningiomas are the most common form of primary tumor (b) Metastatic disease makes up approximately 50% of CNS tumors (c) Glioblastoma multiforme has a median survival rate of 5 years (d) Brain tumor treatment side effects do not affect outcome

172 (b) Meningiomas are the second most common form of primary tumor s. Gliomas are the most common. Glioblastoma multiforme has a median survival rate of less than 1 yea r. Trea tment side effects do affect outcome. Fifty percent of CNS tumors a re metas tatic.

172. The Western Aphasia Battery provides (a) an aphasia quotient as a measure of the severity of aphasia. (b) a classification of the aphasic features observed in a particular patient. (c) a statistical summary of language impairments and an outcome prediction. (d) an overall rating of functional communication.

172 Answer: A Commentary: The Western Aphasia Battery measures various parameters of language and provides the aphasia quotient as a measure of aphasic severity. The Boston Diagnostic Aphasia examination produces a classification of the features of a particular patient and a score of severity and is similar to the Western Aphasia battery, but not the aphasia quotient. The Porch Index of Communication Ability (PICA) is different and evaluates verbal, gestural and graphic responses. The Functional Communication profile provides an overall rating of functional communication. Ref: Brandstater ME. Stroke rehabilitation. In: DeLisa JA, Gans BM, Walsh NE, editors. Physical medicine and rehabilitation: principles and practice. 4th ed. Philadelphia: Lippincott Williams & Wilkins; 2005. p 1667.

172. Which pharmacologic agent should be avoided because of its cogn itive sid e-effects in i ndi viduals with brain injury? (a) Metoclopramide (Reglan) (b) Omeprazole (Prilosec) (c) Erythromycin (E-Mycin) (d) Sucralfate (Carafate)

172. (a) The use of metoclopramide (Reglan) should be avoided because it is known to cause sedation and significant cognitive difficulties for individuals with brain injuries, especially for those regaining consciousness. It also has potential side effects of extrapyramidal movements and tardive dyskinesia. The other agents do not have significant cognitive effects on brain-injured individuals.

173. Effects of prolonged bed rest include (a) increased maximum oxygen consumption. (b) increase of plasma volume. (c) decreased resting heart rate. (d) decreased cardiac stroke volume.

173 (d) Prolonged bed rest has detrimental effects, which include an increased resting heart rate, loss of plasma volume, decreased cardiac stroke volume, and decreased maximum oxygen consumption.

173. Which massage technique has as it goal the breakup of tissue and muscle adhesions? (a) Acupressure (b) Petrissage (c) Friction Massage (d) Effleurage

173 Answer: B Commentary: Petrissage is a method of compression massage that is used to break up tissue and muscle adhesions. In effleurage massage the practitioner uses a stroking motion that is beneficial for vascular and lymphatic drainage. Friction massage is a method of massage that uses circular motions to small areas of tissue to help with tendonitis and fasciitis. Acupressure is a massage method in which pressure is applied on specific body points to help reduce pain. Ref: Brault J, Kappler R, Grogg. Manipulation, traction, and massage. In: Braddom R, et al. Physical medicine and rehabilitation 3rd ed. Philadelphia: WB Saunders; 2007. p 450-4.

174. A 50-year-old woman presents with a 3-month history of progressive swallowing prob le ms. Sh e h as de velo ped left le g w eakness. Sh e deni es an y sensory problems or bowel or bladder incontinence. On examination, she has extensor plantar responses. What test would be an appropriate next step?:(a) Lumbar puncture:(b) Trial of pyridostigmine:(c) Brain stem auditory responses:(d) Electrodiagnostic studies

174 (d) The patient has symptoms suggestive of upper motor neuron involvement, given extensor plantar responses. This finding would point away from a neuromuscu lar jun ction di sorde r such as m ya sthenia gra vis . Th ese sym ptoms could indicate mult iple scler osi s but th e c linical p ict ur e d escri bed is m ore typical of a my otrophic late ral s cl ero sis . Ne edle ex am of th e m usc les can he lp confirm lower motor neuron findings.

174. In contrast to cryotherapy, heat may (a) be used safely over insensate areas. (b) increase edema in the treated area. (c) be used over areas with decreased vascular supply. (d) decrease pain while cold will not.

174 Answer: B Commentary: Cryotherapy and heating modalities share several therapeutic benefits and therapeutic contraindications. Both heat and cold modalities have some analgesic properties and can be used as adjuvant treatments for pain management. Neither cold nor heat modalities should be used over insensate areas or over areas with decreased vascular supply. Heat modalities may increase edema in the area treated, whereas cold therapy will not lead to increased edema. Ref: Weber D, Hoppe K. Physical agent modalities. In: Braddom R, editor. Physical medicine and rehabilitation 3rd. ed. Philadelphia: WB Saunders; 2007. p 459-70.

174. What adaptations to strength training are se en in elderly persons engaged in a consistent exercise program? (a) Hypertrophy of muscle only. (b) Revascularization of the exercised muscle (c) Strength gains from Neural and learning fa cto rs only (d) Gains from both neural factors and hypertr oph y

174. (d) Significant evidence exists to show that elderly persons benefit from strength training. In the past it was believed that adaptations were due to only neural factors. Recent evidence has shown that strength gains in elderly persons are attributable to both neural factors and muscle hypertrophy. Ref: (a) Hoffman MD, Sheldahl LM, Kraemer WJ. Therapeutic exercise. In: DeLisa JA, Gans BM, Walsh NE, editors. Physical medicine and rehabilitation: principles and practice. 4th ed. Philadelphia: Lippincott Williams & Wilkins; 2005. p 409.(b) De Lateur BJ. Therapeutic exercise. In: Braddom R, editor. Physical medicine and rehabilitation. 2nd ed. Philadelphia: WB Saunders; 2000. p 407.

176. A 35-year-old AIDS patient is referred to your clinic for evaluation of mild c ogni ti ve def ici ts an d lo wer extr emi ty weakness . S he was di ag nosed with AIDS 5 years ago and her most recent CD4 count was 190/ml. On mental status examination you note decreases in attention, concentration, and short-term memory, with relatively preserved insight. Her motor examination is significant for cogwheel rigidity, ataxia, and tremor. What is most likely etiology of these patients clinical findings?:(a) Toxoplasmosis:(b) Cryptococcal meningitis:(c) AIDS dementia complex:(d) Primary central nervous system lymphoma

176 (c) The patient's presentation includes virtually all of the clinical characteristics of AIDS dementia complex (ADC). This is a relentlessly progressive enc eph alop athy who se pr esentation is typical of su bcor tica l d ementia. It is a late com plic ation of AIDS, us ual ly occurr ing w hen the antige ni c marker on hilp ir T cells (the CD4 co unt ) f alls bel ow 250 /ml. Whi le eac h of th e other choices can complicate AIDS, it would be unusual for them to present with the constellation of signs and symptoms seen in this patient.

176. Which of the listed vaccinations should be given as part of appropriate m edical m anagemen t for pa ti ents part icipa ti ng in a comp rehen si ve pul mo nary r eha bil ita ti on program? (a) Legionella and mycoplasma avium intracellulare (b) Influenza and mycoplasma avium intracellulare (c) Pneumococcus and legionella (d) Influenza and pneumococcus

176 (d) Medical management including appropriate use of inhalers, ea rly antibiotic therapy for respiratory infections, pneumococcal vaccine, and annual influen za vaccine .

176. Cervical and lumbar traction applied over a 20 --60 minute time period is defined as (a) continuous. (b) sustained. (c) intermittent. (d) pulsed.

176. (b) Sustained traction is the use of force greater than that applied in continuous traction, but less than that used in intermittent traction, and the application time is 20--60 minutes. This time frame makes sustained traction more practical in clinical use. Continuous traction is the use of low force of traction over long periods of time, 20--40 hours. Intermittent traction is the use of greater forces, but for shorter periods of time, 10--60 seconds. There is no pulsed traction. Ref: Atchison JW, Stoll ST, Cotter A. Manipulation, traction and massage. In: Braddom R, editor. Physical medicine and rehabilitation. 2nd ed. Philadelphia: WB Saunders; 2000. p 428.

177. A 25-year-old construction worker fell from a ladder this morning. He pre sents wi th knee pain, im me diate swe lling , and i na bilit y to be ar wei gh t on t he inj ure d leg. The best initial recommendations would be (a) hydrocodone, a knee brace, and return to work with light duty restrictions . (b) nonsteroidal anti-inflammatory medication, x-ray, and return to work with a climbing restriction (c) ice, crutches, x-ray, and sedentary work restriction. (d) tramadol, ace wrap and off of work for 1 week

177 (c) This worker has sustained a significant injury and may likel y have torn his anterior cruciate ligament. The most appropriate recommendations in the acu te setting includ e ice, compression, elevation, and x-ray to rule out fracture. Work r estric tions sho uld all ow t he wor ker to rem ain with restric ted we ig ht bea ring u nt il impr ovem ent is no te d or f urt her te sting i s co mp le ted.

178. One factor that is NOT pertinent to determining an individual's decision -making c apaci ty concerning informed consent and participation in treatment decisions is the ability to (a) express a choice, either verbally or nonverbally. (b) understand specific information related to treatment decisions. (c) seek advice from other health care providers. (d) appreciate the significance of information as it applies to one' s conditi on.

178 (c) Decision-making capacity is a requirement for providing informed consent and having a patient participates in treatment decisions. Central to determining an individual's capacity for making decisions is his/her ability to express a choice, ability to understand specific information related to treatment decisions, and ability to appreciate the significance of information as it applies to his/her condition and circumstances. The individual's ability to seek advice from other health care providers is not a central part of the individual's decision-making capacity.

178. According to the American Medical Association Code of Ethics Opinion, which statement is TRUE? (a) Individual gifts of minimal value from pharmaceutical representatives to physicians are permissible so long as the gifts are related to the physician's work. (b) It is acceptable for physicians to request free pharmaceuticals for personal use or use by family members. (c) Subsidies to underwrite the costs of continuing medical education are permissible when provided directly from the pharmaceutical company to the physician. (d) Faculty presenting at conferences cannot accept honoraria and reimbursement for travel, lodging, and meal expenses.

178 Answer: A Commentary: According to the AMA Code of Ethics Opinion, individual gifts of minimal value from pharmaceutical representatives to physicians are permissible, so long as the gifts are related to the physician's work. It is not acceptable for physicians to request free pharmaceuticals for personal use or use by family members. Subsidies to underwrite the costs of continuing medical education are permissible when they are accepted by the conference's sponsor and are not provided directly to the physician. It is acceptable for faculty at conferences to accept reasonable honoraria and reimbursement for travel, lodging, and meal expenses. Ref: What you should know about gifts to physicians from industry. American Medical Association web-based educational materials. Available at: http://www.amaassn.org/go/ethicalgifts. Accessed July 22, 2004.

179. A clinical trial can best be defined as a (a) retrospective study examining the natural history of a disease process. (b) prospective study that is randomized and double-blinded. (c) retrospective study with subjects selected on the basis of presence or absence of an illness. (d) prospective study comparing the effect of an intervention with a control.

179 Answer: D Commentary: A clinical trial can best be defined as a prospective study comparing the effect and value of an intervention with a control. A study measuring the natural history of a disease process is more observational in nature and can be either prospective or retrospective. Clinical trials are not necessarily randomized or double-blinded. Clinical trials are prospective and not retrospective in nature. Ref: Tate DG, Findley T Jr, Dijkers M, Nobunaga AI, Karunas RB: Randomized clinical trials in medical rehabilitation research. Am J Phys Med Rehabil 1999;78:486-99.

191. Which biomechanical factor is NOT implicated in persons at risk for recur rent str ess frac tures ? (a) Excessive supination of the foot (b) Forefoot valgus (c) Leg length discrepancy (d) Hip joint hypermobility

191 (d) Reduced (not excessive) hip rotation, along with excessive s upination or pronation of the foot, forefoot varus or valgus, and leg length inequality hav e all been implic ated in those at risk for recurrent stress fractures.

179. Preventable medical errors are (a) rarely associated with significant morbidity. (b) most commonly a result of individual human error. (c) associated with no impact on patient satisfaction. (d) associated with patients' loss of trust in the healt h care sy stem .

179. (d) Preventable medical errors can result in lower levels of patient satisfaction and loss of trust in the health care system. Preventable medical errors often result in significant morbidity and even mortality. Estimates are that 44,000 to 98,000 people die each year as a result of medical errors that could have been prevented. These errors are frequently the result of system type errors rather than individual human error.

179. The process of developing and adopting qu ali ty standards for clinical practice (a) is dependent on providers being mandated f or re porting data. (b) is ineffective in changing physician pract ice p atterns. (c) allows health insurance providers to deny rei mb ursement for care. (d) helps to form expectations for safety amon g b ot h providers and consumers.

179. (d) The process of developing and adopting quality standards for clinical practice helps to form expectations for safety among both providers and consumers. Ref: National Institutes of medicine report. To err is human: building a safer health system. November 1999

18. Which myopathy is most likely to demonstrate a "normal" EMG?:(a) Myotonic dystrophy:(b) Polymyositis:(c) Steroid myopathy:(d) Duchenne muscular dystrophy

18 (c) In steroid myopathy the only abnormalities are atrophy of the type II fibers. Since these fibers are recruited last, when the screen is full of motor un its , it is u sua lly d ifficult to a ppreciat e s ubt le a mpli tud e changes. T he other myop athi es not ed typicall y m ay exhibi t p os iti ve wa ves an d fibrillatio ns wi th motor u nit c hange s in the typ e I fib ers .

18 A 57-year-old woman with a history of hypertension compl ains of vertigo and p oor ba la nc e w hen tr yin g to sit u p. He r vo ice is ho ar se, an d sh e ha s a n aso gastr ic tub e i n p lace . On examination she is noted to have a constricted left pupil, left eye ptosis, weakness and poor motor control of the left arm and leg. Sensory exam reveals impaired pinprick sensation on the left face and right arm and leg. These signs and symptoms indicate she likely has a brain stem stroke with a a. lateral medullary infarction. b. medial medullary infarction. c. midbrain infarction. d. medial pontine infarction.

18 Option a is correct. These signs and symptoms described a lateral medullary i nfarction, also known as th e la ter al med ull ary syndro me (W alle nbe rg s yn dr ome). It i s ca used by an in far cti on in the lateral wedge of the medulla and occurs with occlusion of the vertebral artery or the posterior inferior cerebellar artery.

18 A 59-year-old woman with metastatic breast cancer presents with painful swelling of her right arm over the last year. She underwent a radical mastectomy two y ear s ago followed by radiation therapy. She describes an aching discomfort along with an illdefined sensation of numbness and tingling. Effective decongestive therapy for the treatment of secondary lymphedema requires a) long-stretch bandaging of the affected limb. b) truncal clearance to facilitate drainage of the affected limb. b. c) intermittent pneumatic compression devices. c. d) low-level laser therapy.

18 Option b is correct. Commentary: Truncal clearance is necessary to facilitate lymphatic drainage from affected limbs by promoting effective pressure gradients, reducing lymph atic net work resistance, and stimulating lymphatic contractility. Short-stretch bandaging is recommended to create large functional dynamic pressures with low resting pressures helping to prevent circulatory compromise. Adjuvant intermittent pneumatic compression devices have been found to significantly reduce limb volume during both Phase 1 and Phase 2 of complete decongestive therapy. However, the older generation, non-programmable pumps may generate higher pressures than therapeutically necessary and risk the development of truncal or genital edema, as well as produce fibrotic cuffs. Low-level laser therapy is still being investigated for the management of secondary lymphedema, since the wavelength, pulse duration and frequency, dose and dose rate, duration of treatment, and repetition of treatment must be further defined. Reference: (a) A randomized, prospective study of a role for adjunctive intermittent pneumatic compression. Cancer 2002;95(11):2260-7. (b) Mayrovitz HN. The s tan dard of care for lymphedema: current concepts and physiological considerations. Lymphatic Res Biol 2009;7(2):101-108. (c) Oremus M, Dayes I, Walker K, Raina P. Systematic review: Conservative treatments for secondary lymphedema. BMC Cancer 2012;12(6):1-15. (d) Szuba A, Achalu R, Rockson SG. Decongestive lymphatic therapy for patie nts w ith breast carcinoma-associated lymphedema. Cancer 2002;95(11):2607. http://www.ncbi.nlm.nih.gov/pubmed/12436430. Accessed July 20, 2012.

180. Your inpatient rehabilitation unit participates in the Uniform Data System for Functional Independence Measures (UDS-FIM) program by submitting information on the outcomes of all patients treated at your facility. Your unit subsequently receives information back from the UDSFIM database about how outcomes from your center compare to centers in your geographic region and centers across the nation. This process is referred to as (a) root-cause analysis. (b) benchmarking. (c) performance indicator identification. (d) peer performance evaluation.

180 Answer: B Commentary: A benchmark is a target value or standard for comparison for a performance indicator. Functional outcomes and efficiency of functional improvement during inpatient rehabilitation admission are examples of performance indicators. The UDS-FIM database provides a means by which individual rehabilitation units can compare their outcomes to other centers across the nation. This process of comparing outcomes to a standard is referred to as benchmarking. Ref: Johnston MV, Eastwood E, Wilkerson DL, Anderson L, Alves A. Systematically assessing and improving the quality and outcomes of medical rehabilitation programs. In: DeLisa JA, Gans BM, Walsh NE, editors. Physical medicine and rehabilitation: principles and practice. 4th ed. Philadelphia: Lippincott Williams and Wilkins; 2005. p 1166, 1172.

180. Your co-resident presents an article in journal clu b on a ne w me dication and its impact on outcomes following traumatic brain injury. On which point would you NOT need assurance before you decide to use this medication in your clinical practice? (a) That the research study results are clinically signi ficant (b) That bias was eliminated from the study (c) That the research study results are statistically si gnificant (d) That research investigators used valid outcome measu res

180. (b) When critically evaluating the medical literature, it is important to consider if the results of the study are both clinically and statistically significant. It is also important to consider whether the outcome assessment tools have been validated for both accuracy and reliability. While biases that may impact the outcome of the study also must be considered, it is often impossible to completely eliminate bias from the study.

194. Performing a seated leg extension exercise is an example of what type of kinetic chain exercise? (a) Open (b) Closed (c) Static (d) Mixed

194 Answer: A Commentary: In open kinetic chain exercises the most distal segment moves around a fixed proximal segment. In closed kinetic chain exercises the proximal segment moves around a fixed distal segment. Static muscle contractions occur when the muscle is contracted but the proximal and distal segments do not move. There are no mixed kinetic chain exercises. Ref: Micheo W. Concepts in sports medicine. Braddom RL, editor. Physical medicine and rehabilitation. 3rd. ed. Philadelphia: WB Saunders; 2007. p 1023-5.

181. A 31-year-old female office worker complains of a "lump" on the volar asp ect of h er left wrist . T he lump on her w ri st is p ainfu l esp ec ial ly la te in the da y a nd has been present for 2 weeks. There is no discoloration, and the lump is soft and tender to palpation. It is located between the flexor carpi radialis tendon and the radial styloid. Sensory and motor testing are normal, as are reflexes. How would this lump be treated initially? (a) Iontophoresis (b) Needle aspiration (c) Wrist splint (d) Surgical excision

181 (c) This patient likely has a benign volar ganglion cyst. These cysts are often located between the flexor carpi radialis tendon and the radial styloid. Th ey can in unusual cases place pressure on the median nerve and result in sensory and m otor s ymptoms. This pa tien t does no t compl ain of either of th ese . The cy st c an b e painf ul, espe ci all y wi th e xtr eme fl exion o r ex te ns ion. Fo r a cute, pa in ful cyst s placement in a wrist splint can decrease pain and can decrease the size of the cyst. A ganglion cyst will transilluminate with a pen light placed beside the cyst. A plain radiograph should be obtained to rule out bony pathology, such as an intraosseous ganglion, benign or malignant bone tumor, or collapse of the lunate (Kienbock's disease). A magnetic resonance image is not necessary for this patient. Needle aspiration is not necessary. Fluid removed is consistent with synovial fluid in the joint, and does not need to be sent for pathological evaluation. Surgical excision may be indicated for cosmetic purposes, but is not required early.

181. A 50-year-old administrative assistant presents with low back pain. After taking her history, performing a physical examination, and reviewing her imaging studies, you determine that her pain is likely discogenic. She asks if there are any positions which would be better for her back while at work. Which position exerts the most pressure on the lumbar discs? (a) Standing erect (b) Standing erect and flexed forward (c) Seated in a chair (d) Seated in a chair and flexed forward

181 Answer: D Commentary: Nachemson measured the relative pressure changes within the third lumbar disc with changes of position. Standing erect was the reference position and pressures decreased with lying supine and increased in the seated position. Seated and flexed forward further increased disc pressures. Several other positions were evaluated. Ref: Nachemson AL. Disc pressure measurements. Spine 1981;6:93-7.

182. What is the single largest indirect cause of traumatic brain injury? (a) Falls (b) Alcohol (c) Motor vehicle accidents (d) Polypharmacy

182 (b) Alcohol is believed to be involved in 60% of traumatic brain injury (TBI) cases. Polymedication is not a common indirect cause of traumatic brain injur y. Motor v ehicle accidents and falls are direct causes of TBI and not indirect.

182. What is the main principle underlying the Bobath neurofacilitation techniques for rehabilitation? (a) Work from proximal to distal muscle groups. (b) Promote diagonal movement patterns. (c) Focus on multiple joint movements. (d) Establish synergistic patterns.

182 Answer: A Commentary: The Bobath technique of therapy focuses on good posture and works on proximal muscle groups first before proceeding to distal muscle groups. Brunnstrom method uses synergistic patterns and focuses on general movement patterns before moving to more isolated movements. Proprioceptive neuromuscular facilitation (PNF) focuses on multijoint movement patterns in a "diagonal" pattern. The Rood approach focuses on specific muscles selected according to the recovery stage of the stroke. Ref: Ragnarsson KT. Gait restoration and gait aides. In: DeLisa JA, Gans BM, Walsh NE, editors. Physical medicine and rehabilitation: principles and practice. 4th ed. Philadelphia: Lippincott Williams & Wilkins; 2005. p 1396.

182. In traumatic brain injury, magnetic resonance imaging (MRI) is preferred to compu ted t omo graphy (CT) scan in the (a) evaluation of acute brain injury. (b) detection of subarachnoid hemorrhage. (c) detection of epidural hematomas. (d) evaluation of diffuse axonal injury.

182. (d) Magnetic resonance imaging (MRI) is considered better than computed tomography (CT) for evaluating diffuse axonal injury (DAI). A CT scan is superior to MRI for the detection of acute extra-axial hematomas, and in the evaluation of acute brain injury.

183. A recovering alcoholic complains of a painful swollen right knee. She has no hist ory of t rauma bu t tells you she ha s rec en tly l ost 1 0 pou nds o n the Atk ins di et . You aspirate her knee and expect to find a 20,000 white blood cell count with (a) gram-negative rods. (b) positive birefringent rods. (c) 100,000 red blood cells. (d) negative birefringent rods.

183 (d) This patient has gout. These crystals are intracellular, rod shaped, with negative birefringence when parallel in a light microscope. There is no histo ry of trau ma, the refore, excessive red blood cells would not be expected.

183. Lying quietly is equivalent to how many metabolic equivalents (METs)? (a) 0.5 to 1.0 (b) 1.5 to 2.5 (c) 3.0 to 3.5 (d) 4.0 to 4.5

183 Answer: A Commentary: Lying quietly is 1.0 MET. Climbing stairs is equivalent to 3-4 METs, and heavy gardening is equivalent to 4-5 METs. Ref: Whiteson, J. Cardiac rehabilitation. In: Braddom R, et al. Physical medicine and rehabilitation 3rd ed. Philadelphia: WB Saunders; 2007. p 713-4.

184. A 42-year-old man with multiple sclerosis has disturbed sleep because of painful calf spa sms. You r initi al c hoice f or ph ar macol ogica l tre atm en t of t his is (a) baclofen (Lioresal). (b) clonidine (Catapres). (c) hydromorphone (Dilaudid). (d) gabapentin (Neurontin).

184 (a) Baclofen is the agent of choice in this setting. Dantrolene, although effective in reducing muscle tone, causes too much weakness to be an effective ag ent in amy otrophi c lateral sclerosis. Diazepam may produce too much sedation and res pi ratory impairme nt. Gab apen tin, a lth ough ef fective for neur opa thi c pai n, has on ly a mild eff ect on mu sc le spa sti cit y.

196. Acupuncture is described as reducing pain through neuromodulati on. Thro ugh what mecha nis m is neuromodulation theorized to reduce pain? (a) Stimulation of large sensory afferent fibers that suppress pain perceptio n (b) Release of local cytotoxins that induce inflammation (c) Inhibition of sensory afferent fibers that transmit pain percept ions (d) Central nervous system reorganization of the somatosensory corte x

196 (a) Acupuncture is considered to help reduce pain through neuromodulation. The 2 theories of neuromodulation with acupuncture are (1) stimulation of large sensory afferent fibers suppress pain perception through gate control and (2) the needle insertion may act as a noxious stimulus and induce endogenous production of opiatelike substances.

184. A patient has been receiving ultrasound treatments for contractures of the gastrocnemius and soleus muscle. You become concerned about the adequacy of the treatment technique when the patient states that (a) he has the sensation of a transient deep, dull ache in the calf area during treatment. (b) the ultrasound application only lasts for 8 to 10 minutes. (c) the ultrasound application is followed by 10 minutes of rest prior to the stretching. (d) he is standing during and after the treatment.

184 Answer: C Commentary: The physician should be concerned if there is a time gap between the ultrasound application and the stretching activity. In order to be effective, the deep heating that is accomplished with the ultrasound application should be combined with a period of prolonged passive stretching, both during and immediately after the ultrasound application. This can be achieved by having the patient stand during the ultrasound application and after it. An appropriate treatment time with ultrasound is 8 to 12 minutes and the patient may experience a transient deep ache in the treatment application area. Ref: Weber D, Hoppe K. Physical agent modalities. In: Braddom R, editor. Physical medicine and rehabilitation 3rd. ed. Philadelphia: WB Saunders; 2007. p 464-7.

184. High frequency electrical stimulation pre dom in antly stimulates which type of muscle fiber? (a) Type 1 (b) Type 2 (c) Type 3a (d) Type 3b

184. (b) Low frequency electrical stimulation predominantly stimulates type 1 muscle fibers. High frequency electrical stimulation predominantly stimulates Type 2 muscle fibers. There are no type 3 muscle fibers. Ref: Pape K, Chipman M. Electrotherapy in rehabilitation. In: DeLisa J, Gans BM, Walsh NE, editors. Physical medicine and rehabilitation: principles and practice. 4thh ed. Philadelphia: Lippincott Williams & Wilkins; 2005. p 437.

185. Which statement is TRUE regarding the vascular supply to the spinal cord? (a) The paired anterior spinal arteries provide circulation to the anterior tw o-thirds of the spina l c or d. (b) The posterior spinal artery is a single vessel that provides circulation t o the po sterior oneth ird o f the spi nal c or d. (c) The watershed region is supplied only via the radicular arteries. (d) The artery of Adamkiewicz provides circulation to the lumbosacral region.

185 (c) There is a watershed zone from approximately T4 to T6 that i s highly vulnerable to ischemia. The zone has 1 anterior and 2 posterior spinal arteries. T he artery of Adam kiewicz enters between about T9 and T10.

186. In which modality do charged particles mi gra te across biological membranes under an imposed electrical field? (a) Iontophoresis (b) Phonophoresis (c) Ultrasound (d) Laser therapy

186 (a) Iontophoresis is the migration of charged particles across biological membranes under an imposed electrical field. Phonophoresis is the use of ultrasound to facilitate transdermal migration of topically administered medications. Ultrasound is a type of heating that occurs as a result of acoustic vibration. Laser therapy is light amplification by stimulated emission of radiation. It consists of a coherent, collimated beam of photons of identical frequency. Ref: (a) Weber DC, Brown AW. Physical agent modalities.In: Braddom R, editor. Physical medicine and rehabilitation. 2nd ed. Philadelphia: WB Saunders; 2000. p 449-53. (b) Basford JR. Therapeutic physical agents. In: DeLisa JA, Gans BM, Walsh NE, editors. Physical medicine and rehabilitation: principles and practice. 4th ed. Philadelphia: Lippincott Williams & Wilkins; 2005. p 264.

186 Which statement is TRUE when comparing a functional restoration program to active individual therapy for chronic low back pain? (a) Flexibility is increased to a greater extent with active individual therapy program. (b) Pain intensity is reduced to a greater extent with active individual therapy. (c) Functional restoration programs have a greater effect on flexibility and pain than do active individual therapy programs. (d) Functional restoration programs produce greater improvements in endurance than do active individual therapy programs.

186 Answer: D Commentary: Functional restoration programs produce a greater improvement in endurance, but no differences are noted between functional restoration programs and active individual therapy programs. Ref: Roche G, Ponthieux A, et al. Comparison of a functional restoration program with active individual physical therapy for patients with chronic low back pain: a randomized controlled trial. Arch Phys Med Rehabil 2007:88:1229-35.

186. Which method of paraffin bath heats the subcutaneou s area to a g reater degree? (a) Dipping method (b) Continuous method (c) Wrap method (d) Paint on method

186. (b) Paraffin bath use has many methods. The dipping method increases subcutaneous temperatures by 3o Celsius, and the intra-articular temperature by 1o Celsius. The continuous method increases the subcutaneous temperature by 5o Celsius and the intramuscular area by 3º Celsius. The paint on method heats the subcutaneous area less than the dipping method. There is no formal wrap method.

187. What is a barrier for return to work for the injured worker? (a) Receiving a portion of salary while recovering (b) Having a good relationship with supervisor (c) All lost salary being recoverable upon return to work (d) Being married to a spouse who is employed

187 (a) Financial barriers that are disincentives for return to work include that the worker receives a portion of his/her salary while not working and that th e worker h as pote ntial for monetary gain through legal settlement.

188. Which statement accurately characterizes a meta-ana lysis? (a) It summarizes the results of randomized controlled t rials. (b) It summarizes the findings of an expert panel. (c) It groups research on a particular topic area into 3 tiers. (d) It summarizes findings of a single research protocol that is carr ied out a multiple centers.

188 (a) A meta-analysis summarizes the results of randomized controlled trials on a particular topic or research question. A consensus statement summarizes the findings of an expert panel. In developing a consensus statement and reviewing the literature, research studies are typically divided into 3 tiers based on the type of research performed. A multi-center study implements a particular research protocol at multiple centers at different institutions.

188. The benefits of quality outcomes manageme nt in clude (a) providing guidance for alignment of the pr ogr am with the needs of the patient. (b) improved insurance reimbursement from thir d p ar ty payors. (c) improved retention of staff. (d) reduced expenses independent of outcomes.

188 (a) The primary benefit of quality outcomes management is that it provides guidance for aligning the program with the needs of the patient. None of the other options are benefits of quality outcomes management. Ref: Granger CV, Black T, Braun SL. Quality and outcome measures for medical rehabilitation. In: Braddom RL, editor. Physical medicine and rehabilitation. 3rd ed. Philadelphia: Elsevier; 2007. p 1578.

188. As the medical director of an inpatient rehabilitation program, you become concerned because you have recently noticed an increase in the number of urinary tract infections in the patients on your service. Which action would NOT be considered a reasonable initial management strategy in this scenario? (a) Discuss the issue with the Rehabilitation Center Quality Improvement Committee and examine the rate of urinary tract infections over the past year. (b) Perform a literature review examining the incidence and prevalence of urinary tract infections in an inpatient rehabilitation setting. (c) Immediately order that a urine culture be obtained on every patient at the time of admission to the rehabilitation service. (d) Provide an educational inservice to the nursing staff regarding catheter and bladder management.

188 Answer: C Commentary: Continuous quality improvement should be a part of each physician's clinical practice. All of the options listed would be appropriate to consider, with the exception of immediately ordering a urine culture on every patient at the time of admission to the rehabilitation service. This option would not be appropriate without gathering more information and understanding the implications of this intervention strategy. Ref:Johnston MV, Eastwood E, Wilkerson DL, Anderson L, Alves A. Systematically assessing and improving the quality and outcomes of medical rehabilitation programs. In: DeLisa JA, Gans BM, editors. Physical medicine and rehabilitation: principles and practice. 4th ed. Philadelphia: Lippincott Williams and Wilkins; 2005. p 1175-7.

189. You are asked to evaluate a child who was born at 25 weeks gestation and had a gr ade 4 in trave ntr ic ular hemo rrhag e. What t ype o f cer eb ral pa ls y are you mo st li kely to find? (a) Athetoid (b) Hemiplegic (c) Diplegic (d) Hypotonic

189 (c) Grade 4 intraventricular hemorrhages in premature infants ar e most commonly associated with spastic diplegia.

189. You have just finished admitting a 60-year-old diabetic man who has rece ntly unde rgone a right below-knee amputation. The patient's son stops you in the hallway and inquires about his father's health status and prognosis for walking again. You have never met the patient's son before, and before answering the questions, you would first (a) further review the patient's medical record and determine his ca rdiac sta tus. (b) perform a literature review of outcomes research in individuals with belo w-knee am putat ion s. (c) ask the patient for permission to discuss his health status with his son. (d) ask the son if the patient has a living will or a health care po wer-of-at torney.

189 (c) Maintaining confidentiality of patient information is important even when discussing health information with family members. Before discussing the patient's health status with his son, the appropriate first step would be to ask the patient for permission. The other options listed would not be appropriate initial management strategies.

189. What is the primary advantage of a body powered upper limb prosthesis compared to a myoelectric prosthesis? (a) Greater sensory feedback (b) Moderate or no harnessing (c) Less body movement to operate (d) Enhanced cosmesis

189 Answer: A Commentary: The advantages of body powered upper limb prostheses include the following factors: moderate cost, most durability, highest sensory feedback, and a variety of prehensors available for various activities. Their disadvantages are that they require the most body movement to operate, have the most harnessing and require increased energy expenditure to use. Myoelectric and/or switch controlled upper limb prostheses have the following advantages: they require moderate to no harnessing, require fewer body movements to operate, have moderate cosmesis, provide more function in proximal areas and, in some cases, provide a stronger grasp. Battery powered prostheses are the heaviest and most expensive prostheses. They also require the most maintenance, provide limited sensory feedback and require extended therapy time. Ref: Gitter A, Bosker G. Upper and lower extremity prosthetics. In: DeLisa JA, Gans BM, Walsh NE, editors. Physical medicine and rehabilitation. 4th ed. Philadelphia: Lippincott Williams and Wilkins; 2005. p 1342.

19. One of your 4-year-old patients exhibits the following characteristics: distres s ov er mi nor ch anges in env ironm ent , e cholalia , l ac k of aw ar eness of the existence of feelings in others, nonparticipation in simple games. The most likely diagnosis is:(a) autism.:(b) cerebral palsy.:(c) hearing impairment. (d) mental retardation.

19 (a) Autism is characterized by echolalia, inability to play reciprocally, and abnormal relationships with people. While children with mental retardation, ce reb ral palsy , a nd he aring impai rm ent may hav e s ome of t hes e features, they do not h ave all of th em in th e a bsence of au ti sm.

19 What is the mechanism of action of gabapentin for modula ting neuropathic pain ? a. GABA (?-aminobutyric acid) agonist b. NMDA (N-methyl-D-aspartate) receptor blockade c. Presynaptic calcium channel blockade d. Norepinephrine reuptake inhibitor

19 Option c is correct. Gabapentin, although structurally related to ?-aminobuty ric acid (GABA), is a n a lph a2 - del ta lig and . Th e alph a2- de lta rec epto r is a pro tein ass ociat ed with neu ron al vol tage -gated calcium channels. Binding to this channel reduces presynaptic calcium influx into the cell at the dorsal horn, reducing the release of several neurotransmitters (glutamate, substance P, norepinephrine, and calcitonin gene-related peptide [CGRP]).

19. The Gross Motor Functional Measure (GMFM) is designed to measure (a) motor changes over time. (b) quality of motor performance. (c) achievement of motor milestones. (d) only walking, running, and jumping ability.

19. (a) The Gross Motor Functional Measure (GMFM) evaluates motor changes over time in children with cerebral palsy. It includes activities in prone and supine positions, rolling, sitting, crawling, kneeling, standing, walking, running, and jumping. It does not measure the quality of motor performance.

190. Practice-based learning and improvement is consider ed by the Acc reditation Council of Graduate Medical Education (ACGME) to be an aspect of medical practice in which all physicians need to achieve and maintain competency. Which characteristic is NOT a key aspect of practice-based learning and improvement? (a) The ability to locate, appraise, and assimilate evid ence from sci entific studies related to their clinical practice (b) The ability to access and use information technology to suppo rt t heir own education (c) The ability to apply knowledge of study designs and statistic al m ethods to the appraisal of medical literature (d) The ability to advocate for quality patient care and assist p atie nts in dealing with system complexities

190 (d) All of the options listed are key aspects of practice-based learning and improvement, with the

191. 20-year-old college basketball player was seen in the training room after practice. He reports "twisting" his ankle while attempting to rebound a missed shot. On further questioning, he describes an inversion-type injury. He has swelling along the lateral aspect of the ankle. He is tender to palpation over the anterior talofibular ligament and calcaneofibular ligament as well as the 5th metatarsal base. He has no pain over the lateral or medial malleolus or proximally over the fibular head. You obtain plain radiographs, which show a nondisplaced avulsion fracture of the 5th metatarsal base. What is the next step in treating this individual? (a) Provide clearance for return to playing basketball without immobilization. (b) Obtain a surgical consult for possible screw or pin fixation. (c) Recommend immobilization with a postoperative shoe for 1-2 weeks. (d) Prescribe non-weight bearing with crutches for 6-8 weeks or until radiographically verified healing occurs.

191 Answer: C Commentary: Nondisplaced or minimally displaced avulsion fractures of the 5th metatarsal base can occur with inversion ankle sprains. These generally are treated nonsurgically with a short course of immobilization (1-2 weeks) with a postoperative shoe or a short walking boot. Displaced fractures may require screw or pin fixation. It is important to differentiate an avulsion fracture of the base from a fracture of the metaphyseal-diaphyseal junction (Jones fracture), since treatment is different. Ref: Hansen PA, Willick SE. Musculoskeletal disorders of the lower limb. In: Braddom RL, editor. Physical medicine and rehabilitation. 3rd ed. Philadelphia: Elsevier; 2007. p 873.

192. A 46-year-old woman was involved in a rear-end type motor vehicle acciden t in whi ch she h it he r h ea d aga inst the st eerin g wheel and br ief ly lo st con sci ous nes s. Her initial cognitive complaints have improved. She experiences 3 throbbing, unilateral headaches a week associated with nausea. These are graded 8/10 and last for 4 to 8 hours. Which medication would be most appropriate to prescribe? (a) Intranasal butorphanol (Stadol) 1 spray 6 times daily (b) Ibuprofen (Motrin) 600mg 4 times daily (c) Sertraline (Zoloft) 100mg daily (d) Valproic acid (Depakote) ER 500mg twice daily

192 (d) Divalproex sodium reduces the number and severity of migrain e headaches. With 3 migraine-like headaches a week, prophylactic treatment for migraine is indicated. There is concern for developing dependence and abuse potential for butorp ha nol. D aily use of ibup rofe n is m ore likely to result in re bou nd he ada che s wh en d isconti nued . Th er e i s no evi den ce tha t the s elec ti ve ser oto nin rele ase i nhi bitor s are effect ive in the treatment of headache.

192. You perform an extraocular muscle exam of a patient with multiple sclerosis. You note that when looking to the right the left eye will not cross midline and she complains of diplopia when looking to the right but not straight ahead or to the left. You classify her findings as (a) Parinaud syndrome . (b) Horner syndrome. (c) Internuclear ophthalmoplegia. (d) Millard-Gubler syndrome.

192 Answer: C Commentary: Internuclear ophthalmoplegia (INO) is caused by a lesion in the medial longitudinal fasciculus (MLF) in the paramedian brainstem. It is characterized by impaired adduction of the contralateral eye with gaze toward the side of the lesion. In practice, an isolated case of INO is rare since the 2 sides of the MLF are very near the midline of the brain stem. The two most common causes of INO are multiple sclerosis and paramedian brain stem infarct. The Horner syndrome is due to a superior cervical sympathetic ganglion lesion and causes miosis, ptosis and anhydrosis. Parinaud syndrome causes impaired upward gaze with dilated and nonreactive pupils and is the result of a lesion in the midbrain, usually a pineal tumor. Millard-Gubler syndrome is due to an ipsilateral pons lesion causing ipsilateral palsy of cranial nerve (CN) 6 and CN 7 and a contralateral hemiparesis. Ref: (a) Sadiq SA. Multiple sclerosis. In: Rowland, LP, editor. Merritt's neurology. 11th ed. Philadelphia: Lippincott Williams & Wilkins; 2005. p 949.(b) Misulis KE. Neurologic localization and diagnosis. Boston: Butterworth-Heinemann; 1996. p 77-86.

192. The association of apolipoprotein-E (APOE-4) allele and history of traum atic brai n inj ury increases the risk of developing (a) hydrocephalus. (b) alzheimer's disease. (c) post-traumatic epilepsy. (d) cerebral neoplasms.

192. (b) Individuals with a history of traumatic brain injury (TBI) and apolipoprotein-E (APOE-4) allele have 10 times the risk of developing Alzheimer's disease compared with 2 times the risk for APOE-4 allele alone and no increased risk with TBI alone.

193. What is the most common reason for revision of a total hip arthroplasty? (a) Recurrent dislocation (b) Peri-prosthetic fracture (c) Aseptic loosening from polyethylene wear (d) Infection

193 (c) Aseptic loosening caused by polyethylene wear is is the most common reason for hip arthroplasty revision surgery.

193. When compared to conventional stroke rehabilitation methods, mirror therapy has been shown to (a) improve Modified Ashworth Scale scores for spasticity. (b) improve self-care Functional Independence Measure (FIM) score. (c) not show any benefit for spasticity or self-care on FIM scores. (d) improve motor FIM score only.

193 Answer: B Commentary: When comparing a conventional stroke rehabilitation program with mirror therapy for stroke patients, researchers found that mirror therapy resulted in improvement only in the selfcare FIM score; it did not improve scores on the Modified Ashworth Scale for spasticity. Ref: Yavuzer G, Selles R, et al. Mirror therapy improves hand function in subacute stroke: a randomized controlled trial. Arch Phys Med Rehabil 2008:89:393-8.

194. What is the most reproducible method for muscle strength testing in patie nts with neuromu scula r d is order s? (a) Manual muscle testing (b) Hand-held dynamometer (c) Isokinetic measurements (d) Single-repetition maximum

194 (b) Manual muscle testing is not reliable in this setting. The h and-held myometer is much more reproducible. Quantitative strength testing methods are high ly accurat e but r equire sophisticated equipment.

194. An exclusion criterion for resistance training in p atients w ith stable cardiac disease is (a) peak exercise capacity at 7 metabolic equivalents (M ETs). (b) prior history of a stroke. (c) controlled hypertension. (d) severe valvular disease.

194 (d) Exclusion criteria for resistance training in stable cardiac patients include congestive heart failure, severe valvular disease, poor left ventricular function, uncontrolled dysrhythmias, and peak exercise capacity under 5 METs.

196. During aerobic exercise, patients with cystic fibrosis should (a) avoid use of supplemental oxygen. (b) receive balanced fluid supplementation. (c) receive hypotonic fluid supplementation. (d) utilize supplementation oxygen for oxygen saturation <94%.

196 (b) Patients with cystic fibrosis are at risk for excessive sodi um and chloride loss during exercise. They should therefore receive balanced fluid suppleme ntation. O xygen i s indicated if patients' oxygen saturation drops below 88%.

197. The Commission on Accreditation of Rehabilitation F acilities (CA RF) (a) requires mandatory surveys of all inpatient rehabili tation fa cili ties. (b) provides accreditation status that confers a preferr ed status wit h payors. (c) provides accreditation status that signifies the reh abilitati on f acility holds itself to the highest standards in the field. (d) provides accreditation for comprehensive inpatient r ehabilita tion programs, but not specialty programs in areas such as spinal cord injury.

197. (c) The Commission on Accreditation of Rehabilitation Facilities (CARF) provides accreditation status that signifies the rehabilitation facility holds itself to the highest standards in the field. CARF accreditation is voluntary and not all inpatient rehabilitation facilities participate. Accreditation by CARF does not confer any preferred status with payors, and CARF provides accreditation in general comprehensive inpatient rehabilitation as well as specialty programs such as spinal cord injury and traumatic brain injury. exception of the ability to advocate for quality patient care and assist patients in dealing with system complexities. This statement is a key aspect of systems-based practice as defined by the Accreditation Council of Graduate Medical Education.

199. You are asked to evaluate a child with arthrogryposis and equinovarus. Wh ich trea tment st rateg y w ou ld be the best t o emp lo y? (a) Short leg braces attached to orthopedic shoes (b) Botulinum toxin injections, stretching, and plastic ankle-foot orthoses (c) Oral baclofen (Lioresal), range of motion, and extra depth shoes (d) Casting followed by surgical releases

199 (d) Arthrogryposis multiplex congenita (AMC) may be caused by a variety of conditions, but is usually not accompanied by spasticity. Thus botulinum toxin a nd baclofe n are n ot indicated. Most club feet (equinovarus) in patients with AMC are r esista nt to con servati ve m ethods an d requi re surgical rele ase s.

199. Investigators must address ethical considerations w hen desig ning and implementing research studies. One such consideration requires investigators to design protocols that will provide generalizable knowledge and ensure that the benefits of the research are proportionate to the risks assumed by the subjects. This ethical consideration is referred to as (a) respect. (b) beneficence. (c) justice. (d) autonomy.

199. (b) Beneficence requires investigators to design protocols that will provide generalizable knowledge and ensure that the benefits of the research are proportionate to the risks assumed by the subjects.

2. A 24-year-old man was in a motor vehicle collision 36 hours ago. His initial Gla sgow C oma Sc ale scor e wa s 1 3 and hi s i nitial h ead c ompu ted t omography scan showed a small frontal contusion. Initial blood alcohol level was .15g/dL. He is currently disoriented, combative, and tachycardic. He reports visual hallucinations. Management of this case should include:(a) benzodiazepines for alcohol withdrawal.:(b) anticonvulsants for agitation.:(c) neuroleptics to treat hallucinations.

2 (a) Premorbid alcohol abuse is commonly seen in people sustaining brain injury and alcohol withdrawal causes agitation and hallucinations.

2 Which finding is a relative contraindication to cryotherapy? a) Acute inflammation a. b) Pain b. c) Acute hematoma c. d) Impaired sensation

2 Option d is correct. Commentary: Cryotherapy, that is, the therapeutic use of cold by means such as ice, cold packs, or cold water immersion is commonly used to decrease pain , mus cle soreness, fatigue and acute inflammation. Relative contraindications include cold intolerance, cryoglobulinemia, impaired sensation or cognitive defects. Cold intolerance can lead to decreased compliance and increased muscle guarding. Cryoglobulinemia results in immune complex precipitation at lower temperatures. Impaired sensation or cognitive defects may lead to tissue injury. Cryotherapy can be effective in decreasing the swelling or bleeding that commonly accompany tissue injuries. Reference: Lane E, Latham T. Managing pain using heat and cold therapy. Paediatr Nurs 2009;21:14-18.

22 A 23-year-old woman with a traumatic brain injury from a motor vehicle crash is seen in clinic 1 year after her injury. She is in a minimally conscious state and still requires total assistance with all her activities of daily living. The family wants to pursue treatment with hyperbaric oxygen therapy (HBOT). You advise them, that HBOT can (a) reduce the size of the injury to the brain. (b) cause short-term visual disturbances. (c) increase the incidence of mortality. (d) improve the functional outcome.

22 Answer: (b) Commentary: Hyperbaric oxygen therapy (HBOT) delivers 100% oxygen under pressure, which increases the amount of oxygen dissolved in the blood, thereby increasing the oxygen delivered to the body tissues. HBOT may also enhance the formation of new blood vessels, decrease inflammation, and increase the volume of blood flow. Treatment sessions occur inside a sealed, pressurized space known as a hyperbaric chamber. The oxygen is delivered either by mask or directly into the chamber. The pressures used are expressed in units of atmospheric pressure and commonly range from 1.5 to 3 atmospheres. The sessions last from 30 to 90 minutes and many practitioners recommend 100 sessions (range, 80-150 sessions). The cost ranges from $200 to $400 per session. HBOT is not FDA approved for treatment of traumatic brain injury. A number of more minor complications may occur due to HBOT. Visual disturbance, usually a reduction in visual acuity secondary to conformational changes in the lens, is common. While the great majority of patients recover spontaneously over a period of days to weeks, a small proportion of patients continue to require correction to restore sight to pretreatment levels. The second most common adverse effect associated with HBOT is aural barotrauma. Barotrauma can affect any air-filled cavity in the body (including the middle ear, lungs and respiratory sinuses) and occurs as a direct result of compression. There is limited evidence that HBOT reduces the chance of dying following a traumatic brain injury. There is little evidence that more survivors have a good outcome. Thus, the routine adjunctive use of HBOT in these patients cannot be justified. Because evidence of lesion resolution or change in size of persistent defect obtained by magnetic resonance imaging (MRI) or computed tomography (CT) has not been studied, there is no evidence to suggest this occurs. Ref: (a) Bennett MH, Trytko B, Jonker B.Hyperbaric oxygen therapy for the adjunctive treatment of traumatic brain injury. Cochrane Database Syst Rev. 2004;( 4):CD004609. doi:10.1002/14651858.CD004609.pub2. (b) McElligott J, Davis AM, Hecht JS, Kothari S, Muenz Jr JA, Wang GG. Complementary and alternative medicine. In: Zasler ND, Katz DI, Zafonte RD, editors. Brain injury medicine: principles and practice. New York: Demos Medical Publishing; 2007. p 1076-8.

20. Which of the following orthoses or shoe modifications is used in the conservative management of plantar fasciitis? (a) Heel lift (b) Posterior night splint (c) Lateral heel wedge (d) Metatarsal bar

20 (b) A heel lift plantarflexes the foot and is used for Achilles tendinitis. A metatarsal bar is used for metatarsalgia. A lateral heel wedge can be used for the conservative management of osteoarthritis of the knee. A posterior night splint dorsiflexed to 5/ is the correct answer.

20. Which statement regarding rehabilitation after limb-sparing surgery for os teosarcoma is TRUE? (a) There is a lo w ri sk of deep venous thrombosis. (b) Healing may be delayed secondary to chemotherapyor radiation therapy. (c) Motor strength and gait are preserved in most cases. (d) Incidence of unintentional nerve injury is about thesame as for total knee arthroplasty. Ref: Plotz W, Rechl H, Burgkart R, et al. Limb salvage with tumor endoprosth eses for malig nant tumo rs of the knee . Cl in Orthop Relat Res 2002;Dec(405):207-15. Clinical Activity 3.2

20 (b) Healing may be delayed because of chemotherapy or radiation therapy and because of the extent o f t he surgery. Risk o f deep venous thr ombosis is high. Intensive ph ysic al therapy is usua ll y r equired for restoration of gait and compensatory muscle strength. The incidence of unintentional nerve injury or planned nerve sacrifice is higher than for total knee arthroplasty.

20. Of the following modalities, which is the most effec tiv e in treating phantom limb pain? (a) Iontophoresis (b) Transcutaneous electrical nerve stimulation (c) Short wave diathermy (d) Paraffin baths

20 Answer: (b) Commentary: Of the options listed, transcutaneous electr ica l ner ve stimulation (TENS) is the modality that may be useful in treating phantom limb pain. Iontophoresis is generally used for dispersion of medications. Short wave diathermy is a method of deep heat. Paraffin bath is a superficial heat modality. Reference: (a) Esquenazi A. Upper limb amputee rehabilit ati on an d prosthetic restoration. In: Braddom RL, editor. Physical medicine and rehabilitation 3rd ed. Philadelphia: Elsevier; 2007. p 271. (b) Kuiken TA, Miller L, Lipschutz R, Huang ME. Rehabil ita ti on of people with lower limb amputation. In: Braddom RL, editor. Physical medicine and rehabilitation. 3rd ed. Philadelphia:Elsevier; 2007. p 292. (c) Weber CD, Hoppe KM. Physical agent modalities. In: Bra dd om RL, editor. Physical medicine and rehabilitation. 3rd ed. Philadelphia: Elsevier; 2007. p 459-74. (d) Gitter A, Bosker G. Upper and lower extremity prosth eti cs . I n: Delisa JA, Gans BM, Walsh NE, editors. Physical medicine and rehabilitation: principles and practice. 4th ed. Philadelphia: Lippincott Williams & Wilkins; 2005. p. 1347.

20 A 60-year-old woman had a left total hip arthroplasty 4 weeks ago. During her gait evaluation, she is noted to have a left lateral trunk lean during left stan ce phase. This gait deviation is most likely a result of weakness in which left lower limb muscle? a) Gluteus medius a. b) Gluteus maximus b. c) Tensor fascia lata c. d) Vastus lateralis

20 Option a is correct. Commentary: Gluteus medius weakness leads to a Trendelenburg gait. This woman's lateral trunk lean is a compensated Trendelenburg gait. Gluteus medius or hip abd uctor weakness is common following total hip arthroplasty. In one study, 36 of 76 (47%) patients with total hip arthroplasty had hip abductor weakness. Of those 36 patients, all 36 had weakness in the gluteus medius, 28 had weakness in the gluteus minimus, and 4 had weakness in the tensor fascia latae. Reference: (a) Hicks JE, Joe GO, Gerber LH. Rehabilitation of the patient with inflammatory arthritis and connective-tissue disease. In: DeLisa JA, Gans BM, W als h NE. Physical medicine and rehabilitation: principles and practice. 4th ed. Philadelphia: Lippincott Williams & Wilkins; 2005. p 736. (b) Bhave A, Mont M, Tennis S, Nickey M, Starr R, Etienne G. Functional problems and treatment solutions after total hip and knee joint arthroplasty. J Bone Joint Surg Am. 2005; 87(Suppl 2):9-21.

20 A patient presents to clinic for evaluation of her left sided poststroke spas tic ity . Pa ssi ve mov eme nt o f the lef t elbo w f lexo r ov er a 1 sec ond time fra me re vea ls a c atc h at 100 degrees, followed by minimal resistance throughout the remainder of the range of motion. Using the Modified Ashworth Scale, what is the correct grading of this patient's elbow flexor tone? a. 1 b. 1+ c. 2 d. 3

20 Option b is correct. Modified Ashworth Scale is scored as follows 0 is no in crease in muscle tone ; 1 is s li ght in cre ase in muscle to ne man ife sted b y a catc h an d re lease at end ran ge of mot ion; 1+ is slight increase in muscle tone manifested by a catch, followed by minimal resistance throughout the remainder (less than half) of the range of motion; 2 is more marked increase in tone, through most of the range of motion, but joint easily moved; 3 is considerable increase in muscle tone, passive movement is difficult; 4 is affected part is rigid in flexion or extension.

20 What is the most common primary source of brain metastases? a) Lung cancer b) Colorectal cancer c) Melanoma d) Genitourinary cancer

20 Reference(s) (a) Cheville AL. Cancer rehabilitation. In: Frontera WR, editor. Physical medicine and rehabilitation: principles and practice. 5th e Philadelp hia: Lippincott Williams & Wilkins; 2010 . p 1378-1379 . Option a is correct. Lung cancer is the most common primary source of brain metastases, with as many as 64% of patients with stage 4 lung cancer developing brain metastases. Bre ast cancer is the second most common sou rce (2% to 25 % of these p atien ts), followed by melanoma (4% to 20%). Brain metastases from colorectal cancers, genitourinary cancers and sarcomas occur with considerably less frequency (1% of these patients).

200. You have been asked to evaluate a 60-year-old man who suffered a left in ternal ca psule is chemic stroke 3 days ago. He is currently hospitalized and he has been deemed by his primary care provider to be medically stable for transfer to an inpatient rehabilitation program. The patient has a right hemiparesis and dysarthria. On your assessment, cognition appears intact. You agree that the patient is an appropriate candidate for admission. You discuss the benefits of inpatient rehabilitation with the patient and his family, but the patient elects to go home with home health services instead of being admitted for inpatient rehabilitation. The ethical principle followed in abiding by the patient's wishes is the principle of (a) beneficence. (b) autonomy. (c) paternalism. (d) social justice.

200 (b) The ethical principle demonstrated in this case in which the physician concedes to the patient's desires and decisions is the principle of patient autonomy. The principle of beneficence refers to a moral obligation to help other people and to refrain from harming them. Inherent in the principle of autonomy is respect for the values and beliefs of other people. There is often tension between these 2 principles when patients refuse to accept information and advice from their health care providers. With a paternalistic approach, the physician or other health care provider is the decision maker and the patient takes on a more passive role of accepting his/her decisions. Social justice refers to the provision, rationing, and distribution of health care resources.

200 The primary stabilizing effect of the flexible lumbosacral orthosis is its ability to (a) restrict spinal extension. (b) prevent atrophy of trunk muscles. (c) elevate intra-abdominal pressure. (d) enhance kinesthetic feedback.

200 Answer: C Commentary: Although they do not effectively restrict motion to a significant degree, flexible lumbosacral orthoses elevate intra-abdominal pressure, thereby unloading the spine and supporting structures. This action also provides inhibitory kinesthetic feedback and warmth. Long-term use of binders and other flexible lumbosacral orthoses may, unfortunately, result in atrophy of trunk muscles. Ref: Pomerantz F, Durand E. Spinal orthotics. In: DeLisa JA, Gans BM, Walsh NE, editors. Wilkins; 2005. p 1362 -3. Rehabilitation medicine: principles and practice. 4th ed. Philadelphia: Lippincott Williams and

200. In instances where a researcher has financial inves tments in a c ompany and is researching the effectiveness of one of the company's products, the researcher is obligated to (a) terminate the investigation if the investigational a gent is f ound to be not effective. (b) disclose this involvement in writing to subjects tha t are bei ng e nrolled in the study. (c) end financial involvement in the company before the results o f th e research are revealed to the public. (d) disclose this involvement to the investigator's medi cal cente r, t o funding organizations, and to journals publishing the results.

200. (d) Conflicts of interest in biomedical research are becoming more apparent as private companies increasingly develop relationships with academic research scientists. Avoidance of real or perceived conflicts of interest in clinical research is necessary if the medical community is to ensure objectivity and maintain individual and institutional integrity. Financial investments should only transpire outside of the time that the investigator is involved in any research activity and the results of the research are known to the public. If conflicts of interest exist, the investigator is obligated to disclose this involvement in writing to the investigator's medical center, organizations funding the research, and anytime that the research is presented or published.

21 One week after mitral valve replacement, which functiona l activity is sa fe to pr esc ri be ? a. Toilet transfer with a sliding board pulling on grab bars b. Wheelchair propulsion using bilateral lower limbs c. Bed mobility with a trapeze bar d. Ambulation using a pickup walker

21 Option b is correct. Sternal precautions are recommended for the first 6 week s following ster notom y f or he ar t v alv e s urg ery. If a pat ie nt p erf orms i na ppropr iate tra nsfer te chniq ues or fo rce ful upper body exercises such as pulling on grab bars, using a trapeze, or pushing down on a pickup walker, sternal dehiscence could result.

21 Continuous quality improvement (CQI) is a method of quality control widely used in the manufacturing industry to analyze and improve production processes. A pplied to health care delivery, what is a general pri nc iple of CQ I? a) Systemization of care is preferred, although it may be perceived as a loss of provider autonomy. b) Most errors are attributable to individuals, rather than to a system being suboptimal. c) A negative medical event must occur in order to trigger the CQI process. d) The increased cost of health care by implementing "best practices" is offset by the continued access to new technologies.

21 Reference(s) Andrews KL, Ohl HS, Kessler MF, Clohan D Quality and outcome measures for medical rehabilitation. In: Braddom RL, editor. Physical medicine and rehabilitation. 4th e Philadelphia: Sau nders Elsevie r; 2011. p 1 59-60 . Option a is correct. Continuous quality improvement (CQI) programs focus on both outcome and process of care. Quality improvement relies on a critical evaluation of current prac tice to develop process improvements , re duce practice v ariation, and o ptimize resource consumption. A work environment with needless variation increases the likelihood of medical errors by the health care personnel involve Physicians often resist standardizing care, fearing a loss of autonomy or loss of their ability to provide individualized care. Frequently, an inherent tension exists between standardization to excellence and physician autonomy. That tension needs to be understood and confronte CQI focuses on process or system improvement, rather than placing blame on individuals. A general principle behind CQI is that 85% of errors occur because of a suboptimal system, and only 15% of errors are attributable to individuals. Experts using CQI in health care believe it is possible to improve quality and save money at the same time. The objectives of quality improvement are to ensure access to new technology, good procedural outcomes, and patient satisfaction, while concurrently identifying opportunities that will reduce expense.

22. Radiation-induced brachial plexopathy is characterized by (a) burning or lancinating pain in the distribution of affected nerves. (b) acute onset during the course of radiation treatment. (c) involuntary, repetitive, grouped firing of single motorunit potentials on electromyograp hy. (d) the presence of an infiltrating lesion on magnetic resonance imaging. Ref: Cross NE, Glantz MJ. Neurologic complications of radiation therapy. Neu rol Clin 2003; 21:249-77 . Clinical Activity 3.4

22 (c) The presence of myokymic discharges on needle electromyography is almost pathog nomonic for a ra dia tion-induce d injur y to the brachial plexus. It is usually a late com pli cation o f radia ti on therapy and is generally painless, although secondary pain associated with musculoskeletal weakness can occur. Even though imaging studies are usually unrevealing in a pure radiation-induced brachial plexopathy, it is important to obtain magnetic resonance imaging to exclude the coexistence of tumor recurrence.

22. Which modifiable risk factor MOST increases the rela tiv e ris k of stroke? (a) Smoking (b) Hypertension (c) Hypercholesterolemia (d) Diabetes mellitus

22 Answer: (b) Commentary: Hypertension, defined as a systolic pressure gr ea ter than 165mmHg, or a diastolic pressure greater than 95mmHg, increases the relative risk of stroke by a factor of 6. The Framingham study has confirmed that smoking is independently associated with stroke. The relative risk for heavy smokers (more than 40 cigarettes a day) is twice that of light smokers (fewer than 10 cigarettes a day). Cessation of smoking reverses the risk to that of nonsmokers within 5 years of quitting. Hypercholesterolemia has not been epidemiologically linked to increased stroke incidence, but its strong influence on atherosclerosis makes it an indirect risk factor. Diabetes mellitus increases the relative risk of stroke by 3 to 6 times the general population. Reference: Harvey RL, Roth EJ, Yu D. Rehabilitation in str ok e s yndromes. In: Braddom RL, editor. Physical medicine and rehabilitation. 3rd ed. Philadelphia: Elsevier; 2007. p 1184-6.

22 A 33-year-old woman who is 6 months pregnant complains o f right-sided, s tabbi ng, lo w ba ck pai n t hat is worse wit h move men t. W hi ch ortho sis is m ost a ppr opria te for he r? a. Sacroiliac belt b. Cruciform anterior spinal hyperextension orthosis c. Silesian belt d. Minerva brace

22 Option a is correct. Pregnant women frequently develop low back pain, and a s acroiliac belt c an be he lpf ul . A c ruc ifo rm ante rior s pin al hyp ere xten si on (CASH ) br ace is ge ner ally use d f or ost eopo rotic compression fractures. A Silesian belt is a type of suspension for transfemoral prostheses. The Minerva brace is a cervicothoracic orthosis.

22 You see a patient in clinic with what appears to be a non-infected diabetic foot ulcer over the first metatarsal head and order an ankle-brachial index ( ABI) stu dy. The patient's ABI is 1.4. What is your next step in treatment? a) Proceed with off loading the ulcer, since blood flow is normal. a. b) Order additional testing, such as an arterial duplex. b. c) Refer for to vascular surgery for urgent revascularization. c. d) Refer for consideration of a transmetatarsal amputation.

22 Option b is correct. Commentary: Evaluation of vascular status is critical in any patient presenting with diabetic ulcer. The ABI is considered a useful screening tool to loo k for pe ripheral arterial disease. Values under 0.91 are considered consistent with peripheral arterial disease. However, calcified vessels can lead to higher values and possibly false neg ative te st results. If ABI is >1.3, this most likely due to calcified, non-compressible vessels; therefore, other means of testing vascular status should be used. Reference: Salameh MJ, Ratchford EV. Update of peripheral arterial disease and claudication rehabilitation. Phys MedRehabil Clin N Am 2009;20:632.

23. After aspirating 10cc of cloudy fluid from a patients knee, you find a white bl ood ce ll cou nt of 20 ,000 /cc , wit h i ntr acellula r, ne gati vel y birefringent rod-shaped structures under the polarized light microscope. Your diagnosis is:(a) septic arthritis.:(b) gout.:(c) pseudo-gout.:(d) anterior cruciate ligament tear.

23 (b) A septic joint would reveal a white blood cell count greater than 50,000/cc, pseudogout has positive-birefringent crystals, an anterior cruciate ligamen t t ear would re veal a bloody as pi rate. Ur ate cr ysta ls f rom gout are ne gatively bire frin gent, nee dle- or rod -shaped c rys ta ls that can be i ntracellula r.

23. Which muscle fiber types are recruited first in isom etr ic co ntractions? (a) Type 1 (b) Type 1b (c) Type 2 (d) Type 2b

23 Answer: (a) Commentary: Fatigue-resistant type 1 fibers are recruit ed in iti ally followed by type 2b fibers. There are no type 1b fibers. Reference: Mysiw WJ, Jackson RD. Electrical stimulation . In : B raddom RL, editor. Physical medicine and rehabilitation. 3rd ed. Philadelphia: Saunders; 2007. p 479.

23 A 35-year-old woman comes to your clinic to advance her rehabilitation p rogra m. She i s pla nni ng a t rip with a 10 h our air plan e ri de and ask s fo r you r a dvice ab out pr eve ntin g blood clots. Her BMI is 25.2 kg/m2 and her only medication is oral birth control pills. What is the best recommendation for her? a. Sit in an aisle seat and ambulate frequently during t he flight. b. Wear graduated compression stockings that provide 8-1 0mm Hg of pressu re at th e a nk le . c. Take low dose aspirin for 7 days prior to travel thro ugh 2 days after trav el. d. Take prophylactic low-molecular weight heparin 1 day prior to travel throu gh 2 d ay s aft er tra vel .

23 Option a is correct. A flight of 8 hours or more is considered long-distance travel. As a lon g-dis tan ce tr av ele r u sin g e stro gen in he r oral bi rth co nt rol pi lls, she is a t i ncrea sed ri sk for ven ous thromboembolism (VTE). Recommendations from the American College of Chest Physicians (Grade 2C evidence) to reduce the risk of VTE in this population include frequent ambulation, calf muscle exercise, sitting in aisle seat if possible and wearing properly fitted below-knee graduated compression stocking providing 15 to 30mm Hg pressure at the ankles during travel. Other risk factors for VTE during long-distance travel include previous VTE, recent surgery or trauma, active malignancy, pregnancy, advanced age, limited mobility, severe obesity or known thrombophilic disorder. The patient's BMI of 25.2 kg/m2 puts her in the overweight category for body mass; extreme obesity is BMI greater than or equal to 40 kg/m2. Graduated compression stockings would be an appropriate intervention to recommend, however the pressure gradient should be in the 15-30mm Hg category. Aspirin or anticoagulants are specifically not recommended for VTE prophylaxis for long-distance travel.

23 Which finding is associated with a more favorable prognosis in amyotrophic lateral sclerosis (ALS)? a) Bulbar involvement presentation a. b) Predominance of lower motor neuron findings on electrodiagnostic studies b. c) Young age and male sex c. d) Short time period from symptoms to diagnosis

23 Option c is correct. Commentary: Several prognostic predictors exist for determining the severity of a person's ALS course. Presentation with bulbar or pulmonary dysfunction (or b oth ), short time period from symptom onset to diagnosis, electrodiagnostic findings indicating primarily lower motor neuron involvement and advanced age all potentially indicate a poor prognosis. Women present with bulbar symptoms more frequently than men do. Bulbar palsy, which indicates a poor prognosis, appears to progress more rapidly in women. Young males with ALS have the best prognosis and may have a longer life expectancy. Overall, the median 50% survival rate is 2.5 years after diagnosis. In patients who present with bulbar symptoms, the 50% survival rate drops to 1 year. Survival rates vary depending on the patient's decision to use a feeding tube and assisted ventilation. Nonetheless, by 5 years postdiagnosis, the overall survival rate is only 28%. Reference: (a) Czaplinski A, Yen AA, Simpson EP, Appel SH. Predictability of disease progression in amyotrophic lateral sclerosis. Muscle Nerve 2006;34:7 02-8. (b ) Krivickas LS, Carter GT. Motor neuron disease. In: DeLisa JA, Gans BM, Walsh, NE, editors. Physical medicine and rehabilitation: principles and practice. 4th ed. Philadelphia: Lippincott Williams & Wilkins; 2005. p 934-5. (c) Carter GT. Rehabilitation management of neuromuscular disease. eMedicine [serial on line]2006. October [cited April 24, 2007]http://www.emedicine.com/pmr/topic233.htm. Accessed July 19, 2012.

23 After a lateral ankle injury, which ligament is most commonly ruptured? a) Tibiotalar ligament b) Calcaneofibular ligament c) Tibocalcaneal ligament d) Anterior talofibular ligament

23 Reference(s) Hyman GS, Solomon J, Dahm Physical examination of the foot and ankle. In: Malanga G, Nadler S, editors. Musculoskeletal physical examination: an evidence-based approach. 1st e Ph ilad elphia: Elsev ie r; 2006. p 42-3 . Option d is correct.Though not all inversion ankle injuries result in the rupture of the ankle ligaments, when rupture does occur, isolated anterior talofib ular ligament ruptures occur in 65% of t he cases. Com bi ned anteri or ta lofibular ligament and calcaneofibular ligament ruptures occur in approximately 20% of cases. The tibiotalar and tibiocalcaneal ligaments are part of the deltoid ligament complex and are not involved in classic lateral ankle injuries.

23. An otherwise healthy eld erly woma n w ith hi story of osteoporosis presents with the acute onset of focal thoracic spine pain. Your management program should include(a) William's flexion exerci ses.(b) epidural steroid injecti ons.(c) spinal extension brace.(d) steroid iontophoresis.

23. (c) Spinal flexion will increase pain related to vertebral compression fractures. An extension brace will promote a position of comfort during the healing process. These braces may include a Jewett brace, cruciform anterior spinal hyperextension brace, and a chairback or warm and form brace.

24. What is the pathophysiology of Duchenne muscular dystrophy? (a) merosin deficiency (b) abnormally low levels of dysferlin (c) absence of dystrophin (d) mutations of alpha-sarcoglycan

24 (c) The absence of dystrophin is the basis of the pathophysiolog y of Duchenne muscular dystrophy (DMD). Most genes in the affected area of the X chromosome encode fo r compo nents of the dystrophin-glycoprotein complex (DGC), an assembly of tr ansmem brane and membra ne-a ssocia ted protei ns that form a s tru ctu ra l l ink age be tw een the F-a ctin c yto sk el eton an d t he extra ce llul ar m atri x i n m uscle . T he pr otein s that compr ise the DGC are organized into 3 subcomponents, the cytoskeletal proteins, the sarcoglycans and the sarcospan. Many of the different types of muscular dystrophies arise from primary mutations in genes encoding components of this complex. However, the other choices noted above are not involved in DMD or Becker MD. Deficiencies in those proteins are associated with forms of limb girdle muscular dystrophy.

24. Which symptom most frequently impacts quality of lif e i n pat ients with incurable cancers? (a) Fatigue (b) Anorexia (c) Weakness (d) Depression

24 Answer: (a) Commentary: Cancer patients experience a much broader ra nge o f s ymptoms that impact their quality of life and their ability to address existential issues at the end of life than those listed here. A systematic review of symptom prevalence studies in patients with incurable cancer identified fatigue (74%), pain (71%), lack of energy (69%), weakness (60%) and anorexia (53%) being the most prevalent that impact quality of life. The prevalence of nausea is 40% in the last 6 weeks of life. Fatigue is often the primary condition adversely affecting quality of life. Reference: (a) Brooksbank M. Palliative care: Where hav e w e com e from and where are we going? Pain 2009;144 (3):233-5.(b) Cheville AL. Palliative care. In: DeLisa JA, Gans BM, Walsh NE, editors. Physical medicine and rehabilitation: principles and practice. 4th ed. Philadelphia: Lippincott-Raven; 2005. p 536-7.

24 A 20-year-old man is involved in a diving accident resul ting in C6 ASIA A tet rap leg ia w ith tr ace mo tor functi on in C7 and C8. Y ou confi rm t his ASIA sco re 30 da ys aft er his injury and make the following statement regarding his 1-year prognosis a. His chance of regaining motor function (3/5) in the C 7 myotome is 80% or b ett er. b. Most upper extremity recovery occurs in the first 2 m onths. c. The initial strength of a muscle does not predict the rate of strengt h rec ove ry. d. There is up to a 10% chance of his becoming a communi ty ambulator.

24 Option a is correct. Most upper extremity recovery occurs in the first 6 mont hs, and the grea test rat e o f ch ang e o ccu rs in t he fir st 3 mont hs. The i ni tial s tren gth of a mus cle i s a si gni fic ant predictor of achieving antigravity strength and the rate of recovering strength. There is only a 2% to 3% chance of recovery from ASIA class A to ASIA class D in the clinically complete patient and it is unknown what percentage of that small group will become ambulatory. Most individuals with ASIA class A designation and trace function 1 level below will gain motor function (3/5) in the myotome with trace activity.

24 An individual with T3 ASIA A paraplegia complains of burning pain in his legs. Additional review of symptoms includes urinary leakage between catheteriza tions an d difficulty sleeping. The best pharmacologic intervention at this time would be a) fluoxetine (Prozac). a. b) amitriptyline (Elavil). b. c) alprazolam (Xanax). c. d) trazodone (Desyrel).

24 Option b is correct. Commentary: Amitriptyline, a tricyclic antidepressant, can be effective in the treatment of neuropathic pain but has a significant side effect profile th at in clu des an anticholinergic and sedative effect. These side effects would be desirable in this patient with leaking and difficulty sleeping. Prozac may be helpful with pain but may actually cause insomnia and has little anticholinergic effects. Trazodone is a mild sedative with slight anticholinergic properties. Alprazolam is primarily a sedative and is not commonly used for neuropathic pain. Reference: Bockenek WL, Stewart, JB. Pain in patients with spinal cord injury. In: Kirshblum S, Campagnola DI, DeLisa JA, editors. Spinal cord medicine. Phila del phia: Lippincott Williams & Wilkins; 2002. p 389-408.

24. What is a common musculo skel etal com plicat ion of acute inflammatory demyelinating polyneuropathy?(a) Joint capsule contractur es(b) Lumbar scoliosis(c) Achilles tendinitis(d) Two-joint muscle contrac ture s

24. (d) Patients with acute inflammatory demyelinating polyneuropathy (AIDP) commonly develop painful tightness in the two-joint muscles, including the hamstrings, tensor fascia lata, and gastrocnemius. Actual joint capsule contracture is much less common. The vast majority of persons with AIDP significantly improve and are unlikely to develop scoliosis as a result of muscle weakness. Tendon inflammation is not a feature of AIDP.

25. Which activity is outside the purview of psychologists on the cancer rehab ilitation team ? (a) Dev eloping c oping ski lls during treatment (b) Documenting and intervening with respect to cognitive deficits (c) Providing biofeedback training for pain and anxiety (d) Managing medications for depression and anxiety Ref: (a) Gaze MN, Wilson IM. Handbook of community cancer care. San Francisc o: Greenwich M edical Me dia; 2003 . (b) Moo ery S, Greer S. Cognitive behaviour therapy for people with cancer. New York: Oxford Univ Pr; 2002. (c) Schneider CM, Dennehy CA, Carter SD. Exercise and cancer recovery. Champ aign: Human Ki netics; 2 3 Educational Activity 3.7

25 (d) The clinical psychologist should address issues of depression and anxiety and p rovide appropria te psychothera py, but management of an tidepressant and anxiolytics is t he responsi bility of th e treati ng physician.

25. What is the leading cause of traumatic spinal cord injury in the United St ates? (a) Falls (b) Sports related injury (c) Gunshot wound (d) Motor vehicle crash

25 (d) The leading cause of traumatic spinal cord injury in the Uni ted States is motor vehicle crash. The incidence of spinal cord injury from gunshot wounds is decreas ing nat ionally; falls are now the second most common cause nationwide, fol lo wed by sports r elated inju ries.

25 Which artery provides the arterial vascular supply to the ventral grey matter of the spinal cord? (a) Anterior spinal (b) Posterior spinal (c) Vertebral (d) Radicular

25 Answer: (a) Commentary: The single anterior spinal artery and its sulcal branches provide blood supply directly to the anterior two-thirds of the spinal cord after arising from branches off the vertebral arteries. The paired posterior spinal arteries similarly originate from the vertebral arteries, and supply the posterior one-third of the cord. Radicular arteries are segmental branches from the thoracic and abdominal aorta. These arteries provide vascular supply to the thoracic, lumbar, sacral and coccygeal cord. Ref: (a) Goshgarian HG. Anatomy and function of the spinal cord. In: Lin VW, editor. Spinal cord medicine: principles and practice. New York: Demos; 2003. p 15-34. (b) Sapru HN. Spinal cord: anatomy, physiology, and pathophysiology. In: Kirshblum S, Campagnolo DI, DeLisa JA, editors. Spinal cord medicine. Philadelphia: Lippincott Williams & Wilkins; 2002. p 20-1.

25. An 18-year-old female with a history of depression and C2 ASIA A spinal cord injury acquired in a diving accident requires continuous ventilation. She is diagnosed with a major depressive disorder 8 weeks after her injury. Which factor has increased her risk for developing depression after her spinal cord injury? (a) Ventilator use (b) Prior history of depression (c) Level of injury (d) Traumatic nature of injury

25 Answer: (b) Commentary: Prior history of depression is a general risk factor for depression after a spinal cord injury. Etiology, level of injury and ventilator use are not risk factors. Reference: (a) Consortium for Spinal Cord Injury Medicine. Depression following spinal cord injury: a clinical practice guideline for primary care physicians. Washington (DC): Paralyzed Veterans of America; 1998. (b) Kirshblum S. Rehabilitation of spinal cord injury. In: Delisa JA, Gans BM, Walsh NE, editors. Physical medicine and rehabilitation: principles and practice. 4th ed. Philadelphia: Lippincott-Raven; 2005. p 136-7.

25 A 62-year-old woman complains of right knee pain and stiffness. On physical examination, she has a genu varum deformity. A physical therapy prescription shoul d i nclude a) isokinetic hamstring strengthening. a. b) isometric hamstring strengthening. b. c) closed kinetic chain quadriceps strengthening. c. d) open kinetic chain quadriceps strengthening.

25 Option c is correct. Commentary: For knee osteoarthritis, quadriceps strengthening has been well studied and is shown to be beneficial. In closed kinetic chain exercises, the dist al aspect of the limb is fixed against a source of resistance, whereas in open kinetic chain exercises, the distal part of the limb is free in space. Closed chain exercises are preferred because they result in less shear force across the joints and are also more functional. Reference: Stitik TP, Foye PM, Stiskal D, Nadler RR. Osteoarthritis. In: DeLisa JA, Gans BM, Walsh NE. Physical medicine and rehabilitation: principles a nd pr act ice.4th ed. Philadelphia: Lippincott Williams & Wilkins; 2005. p 772.

25 A 17-year-old patient with cerebral palsy presents at re st as shown in t he ph oto . O n ex am, he ha s a rep etitiv e t wi stin g m ovem en t and fu ll p assi ve ra nge of m oti on wit h v aryi ng resistance felt during velocity-dependent testing in the joints of the right upper limb. This presentation is best described as a. clonus. b. hypertonic rigidity. c. contracture. d. spastic dystonia.

25 Option d is correct. Spastic dystonia is muscle overactivity present at rest; it is spontaneo us, h as no pr im ary tr igg eri ng f actor and c ause s d efor ma ti on of join ts a nd bo dy postu res . I t f req uent ly produces twisting or repetitive movements. In contrast to spastic dystonia, hypertonic rigidity is characterized by consistent resistance being felt throughout a passive range of motion, regardless of the speed or direction of movement. Clonus is a rhythmic, often self-sustaining contraction evoked by rapid muscle stretch. Contracture is fixed resistance to passive stretching of muscles due to shortening or wasting (atrophy) of muscle fibers or the development of scar tissue (fibrosis) over joints.

25 What do fibrillation potentials represent? a) Spontaneous depolarization of muscle fiber b) Demyelination of peripheral nerve c) Central nervous system pathology d) Activation of a motor unit

25 Reference(s) Dumitru D, Stegeman DF, Zwarts MJ. Electrical sources and volume conduction. In: Dumitru D, Amato AA, Zwarts MJ, editors. Electrodiagnostic medicine. 2nd e Philadelphia: Hanley & Belfus; 2002. p 47. Option a is correct. Fibrillation potentials are simply spontaneous depolarizations of a single muscle fiber seen in pathologic tissue and are often elicited with needle movemen t while performing electromyography (EMG ). Fibrillati on potential s are seen in neurogenic conditions with acute or chronic axonal loss, but also in myopathic disease. Fibrillations are not characteristic for central nervous system disease states or conduction block. Conduction block is evaluated on nerve conduction studies. However, fibrillations could be seen in cases of conduction block if axonal loss has occurred as well.

25. In the emotional stages of r ecove ry from s pinal cord injury, most individuals(a) have prolonged feelings of g uilt or worthl essness.(b) undergo a true depressiv e ep isode .(c) experience bereavement.(d) feel diminished interest or pleas ure in al most all activities.

25. (c) Although the pattern of emotional reaction is unique to every person, coping with a spinal cord injury normally involves sadness, yearning, and intense feelings of loss. While bereavement might appear similar to depression, it does not ordinarily involve prolonged feelings of guilt, worthlessness, self-reproach or thoughts of death as seen in depressive disorders. Because grieving or bereavement is universal in the context of spinal cord injury, it is important to differentiate bereavement from a depressive disorder.

26. In addition to surgery, treatment for impending pathologic fractures of th e femur includ es all of the foll owing EX- CEPT (a) bisphosphonates. (b) radiation therapy. (c) bedrest. (d) chemotherapy. Clinical Activity 3.8

26 (c) Bed rest will not completely prevent pathologic fractures from occurring, becau se these fractur es are frequen tly a r esult of progress ive, erosive, lytic disease. Pati ent s with b one pai n not relieve d b y medical management should be referred for prophylactic surgery. Those responding to conservative measures should be mobilized as tolerated to prevent complications associated with immobility.

26. A 67-year-old man with chronic obstructive lung disease (COPD) is about to start a pulmonary rehabilitation program. Which option is an appropriate breathing retraining technique for the patient to learn? (a) Diaphragmatic breathing (b) Localized expansion exercises (c) Rapid, shallow breathing (d) Head up and bending backward postures

26 Answer: (b) Commentary: Breathing retraining techniques for COPD include pursed lips breathing, head down and bending forward postures, slow deep breathing, and localized expansion exercises (also known as segmental breathing, wherein the patient is asked to inspire while the clinician applies pressure to the thoracic cage to resist respiratory excursion in a segment of the lung). These techniques maintain positive airway pressure during exhalation and help reduce overinflation. Although diaphragmatic breathing (done by expanding one's belly and thereby allowing the diaphragm to move down creating more room for the lungs to expand) is widely taught, it has been shown to increase the work of breathing and dyspnea compared with the natural pattern of breathing in the patient with COPD. Reference: Alba A, Chan L. Pulmonary rehabilitation. In: Braddom RL, editor. Physical medicine and rehabilitation. 4th ed. New York: WB Saunders; 2011. p 742.

26. Two-thirds of infections that occur 1 to 6 months post-transplant are caused by (a) methicillin-resistant staphylococcus aureus (MRSA). (b) pneumocystis carinii (PCP). (c) clostridium difficile (C. diff). (d) cytomegalovirus (CMV).

26 Answer: D Commentary: Transplant patients are at high risk for cytomegalovirus (CMV) infection and frequently receive prophylaxis with acyclovir or ganciclovir. Ref: Zafonte RD, Pippin B, Munin M, Thai N. Transplant medicine: a rehabilitation perspective. In: Physical medicine and rehabilitation: principles and practice. 4th ed. DeLisa JA, Gans BM, Walsh NE, editors. Philadelphia: Lippincott Williams and Wilkins; 2005. p 1914-5.

33 A 28-year-old man with C4 ASIA B tetraplegia after a bicycle accident one week ago is in the intensive care unit. Four days after his injury he was elective ly intubated due to a progressive de crea se in his vit al capacity. Asid e from his spinal cord injury, he suffered no other injuries in his accident. You are consulted to help facilitate weaning him off the ventilator. In order to facilitate the fastest wean and to minimize atelectasis, you suggest starting with a tidal volume of how many milliliters/kilogram (ml/kg) of ideal body weight? a) 5 b) 10 c) 15 d) 20

33 Reference(s) Consortium for Spinal Cord Injury Medicine. Respiratory management following spinal cord injury. Clinical practice guidelines. Washington (DC): Paralyzed Veterans of America; 2005 . p 15 Option c is correct.

26 Repetitive transcranial magnetic stimulation (rTMS) of the motor cortex appears to have therapeutic value in the treatment of a) fatigue b) hypertension c) stroke d) seizure

26 Option c is correct. Commentary: With demonstrated facilitation of motor recovery after stroke, rTMS has been used in 2 ways: (1) frequency stimulation less than 1 Hz to the unaff ect ed hemisphere, reducing its inhibitory effects on the affected hemisphere and (2) frequency stimulation greater than 1Hz to the affected hemisphere, increasing its excitability. Studies suggest possible greater benefit when the lesion is subcortical rather than cortical. Frequencies greater than 20 Hz, especially when intensity is higher, theoretically may increase the risk of seizure. Benefits of rTMS have been shown in preliminary studies of those with both chronic and acute stroke. Possible adverse or undesirable effects of rTMS include seizure (with treatment frequencies greater than 20 Hz) and increasing blood pressure (with highe r tre atm ent frequencies after chronic stroke.) Transcranial magnetic stimulation can be used to evaluate central fatigue, but has not been used for treatment of fatigue. Reference: Corti M, Patten C, Triggs W; Repetitive transcranial magnetic stimulation of motor cortex after stroke; Am J Phys Med Rehabil 2012 ;91:254-70

26 You are working with a 65-year-old man with dysphagia wh o has decreased oral bol us co nt rol , r igh t-s ided oral and p hary nge al w ea kn ess, a nd d ecre ased air way p rot ect ion . W hich position should he avoid during swallowing? a. Head rotation to the right b. Head tilt to the left c. Neck extension d. Chin tuck

26 Option c is correct. The goal of neck extension in swallowing compensation is to use gravity to in cre ase t he sp eed an d e ffic iency of or al t ran sit; t hi s is c ontr aind icate d i n pat ien ts suc h a s th is gentleman with decreased oral control of bolus for swallowing. Neck extension strategy is also contraindicated when the airway cannot be adequately protected. Head rotation to the weaker side or head tilt to the stronger side directs the food bolus to the stronger and more sensate side of the oral cavity and pharynx allowing for better sensory input and motor control and more efficient propulsion of the bolus. Chin tuck shifts the anterior pharyngeal structures posterior; this is thought to decrease the vallecular space and vallecular residue but can increase residue in some patients and generally needs to be verified by swallow study.

26. A radical neck dissectio n fo r hea d a nd nec k cancer, by definition, involves sacrifice of which nerve?(a) Glossopharyngeal(b) Hypoglossal(c) Spinal accessory(d) Auriculotemporal

26. (c) The external jugular vein, spinal accessory nerve, and sternocleidomastoid muscle are removed during a radical neck dissection. Loss of the spinal accessory nerve leads to shoulder dysfunction with long-term adverse functional sequelae.

27. Which situation is associated with good treatment outcome in the injured worker ?:(a) Poor evaluation by the employer within the past 6 months:(b) The employee works in middle management:(c) The employee received a recent increase in salary:(d) Communication between the worker and supervisor is empathetic

27 (d) Good treatment outcome is associated with a history of personal empathetic communication between the supervisor and injured employee. No data show impro ved out comes in inju red workers w ho have rec ent ly r ecei ved a raise or have worked f or t he emp loy er more tha n 5 years . P oo r o utcom es hav e been found in in ju red work ers w ho ha ve re cei ved a po or eva luati on wit hin the 6 months preceding the injury. The level of position within the workplace is irrelevant in a worker's compensation outcome.

27. A 47-year-old woman with T8 ASIA A spinal cord injury (SCI) applied for a position as a store clerk. She felt that she was being discriminated against because of her SCI. Under the Americans with Disabilities Act (ADA), she may have a right to file a complaint if (a) the employer requested a pre-employment physical to see if she qualified. (b) the employer hired her, but then requested a pre-placement physical to determine the most appropriate position for her. (c) the job description required climbing ladders and working from heights. (d) the employer did not make all accommodations to allow her to work from her wheelchair.

27 Answer: (a) Commentary: The Americans with Disabilities Act (ADA) is a federal law designed to help protect the rights of disabled citizens. Employers must not discriminate against hiring a disabled applicant if that person is able to perform the key components of the job. Pre-employment physicals are not allowed under the ADA, but a pre-placement physical can be used after an individual is hired to help determine the most appropriate job for that person. An employer may decline to hire a disabled individual if that person is unable to perform the essential functions of the job, so long as the employer has attempted to make reasonable accommodations to allow the disabled individual to perform these job functions. This individual would not be able to climb ladders or work from heights because of her SCI, despite any accommodations. Reference: Foley BS, Buschbacher RM. Occupational rehabilitation. In: Braddom RL, editor. Physical medicine and rehabilitation. 3rd ed. Philadelphia: Saunders Elsevier; 2007. p1053.

27. Repeatedly lifting the shoulder past which degree of flexion or abduction is associated with an increased prevalence of shoulder disorders? (a) 10o (b) 30o (c) 45o (d) 60o

27 Answer: D Commentary: Repeatedly lifting the shoulder past 60 degrees of flexion or abduction is associated with an increased prevalence of shoulder disorders. Ref:a) Bjelle A, Hagberg M, Michaelson G. Occupational and individual factors in acute shoulder-neck disorders among industrial workers. Br J Ind Med 1981;38:356-63.(b) Ohlsson K, Attewell R, Paisson B, Karlsson B, Balogh I, Johnsson B, et al. Repetitive industrial work and neck and upper limb disorders in females. Am J Ind Med 1995;27(5):731-47.

27 A pediatrician requests your assessment of a 9-month-old infant. The chi ld's par ent s fe el som eth ing is wrong bec au se h e i s no t ke eping up w ith a cou sin of s imi lar ag e. You are concerned because the child is not yet able to a. cruise along furniture. b. hold objects with his fingertips. c. transfer toys from one hand to another. d. point to his nose on request.

27 Option c is correct. By 18 months a child should have a 10- to 25-word vocabu lary using singl e wor ds at a ti me. By 24 mo nths this sho ul d ha ve incr ea se d to a t le ast 50 wo rds and the y s hou ld be p utting 2 words together, for example, "go now." By age 3 a child should be able to state his or her full name, count to 3 and recognize basic colors.

27 An injured worker with complex regional pain syndrome (CRPS), type 1, asks his physician to prescribe methadone instead of morphine because of ongoing pa in. T he physician orders blood work and an electrocardiogram (EKG) first. What finding would be a strong contraindication to prescribing methadone for this patient? a) Hyperkalemia a. b) Hypermagnesemia b. c) QT interval prolongation c. d) Premature atrial complexes

27 Option c is correct. Commentary: A prolonged QT interval and serious arrhythmia (torsades de pointes) have been reported during treatment with methadone. Patients with cardia c hyp ert rophy, concomitant diuretic use, hypokalemia or hypomagnesemia are at higher risk for development of prolonged QT interval because methadone inhibits cardiac potassium channels. Premature atrial complexes without other cardiac abnormalities that would predispose the patient to QT interval prolongation would not be considered an absolute contraindication. Reference: U.S. Food and Drug Administration website http://www.fda.gov/downloads/Drugs/DrugSafety/PostmarketD rugSafetyInformationforPatientsandProvider s/UCM 142 842.pdf .Accessed 05-09-2012.

27 What advantage does a suspension system that features a gel liner pin have over a sleeve suspension system in a transtibial amputee? a) Better heat dissipation and reduced sweating b) Improved cushioning and reduction in shear forces c) Ease of donning and doffing d) Ease of liner care and cleaning

27 Reference(s) Huang ME, Miller LA, Lipschutz R, Kuiken T Rehabilitation and prosthetic restoration in lower limb amputation. In: Braddom RL, editor. Physical medicine and rehabilitation. 4th e Philadelphia: Elsevier Saunders; 2011. p 279-281. Option b is correct. Because they transmit good control of the prosthesis, offer better cushioning and reduce shear forces on the residual limb, gel liner suspension systems are very popular. They do not provide b ette r heat dissip at ion, and e xcess ive heat retention and sweating are often major disadvantages. The donning can be difficult, since a gel liner must be positioned correctly on the residual limb to ensure that the pin engages correctly. Lastly, the gel liners require daily cleaning to avoid skin rashes and skin infections.

27. A 50-year-old construction worker has rece ive d a 30% whole person impairment rating because of his pericardial heart disease. According to the fifth edition of the American Medical Association Guides to the Evaluation of Permanent Impairment, this worker's status indicates that (a) he is 100% disabled from performing his wo rk ac tivities. (b) his general functioning and ability to per for m activities of daily living is reduced by 30%. (c) he has a 30% reduction in work capability. (d) he should receive 30% of his future wages and b enefits in a disability payment.

27. (b) According to the AMA guides, a 30% whole person impairment rating indicates a 30% reduction in general functioning, excluding work. The whole person impairment rating does not directly correlate to the patient's work abilities and it does not determine the disability compensation. Ref: Cocchiarella L, Andersson G, eds. Guides to the evaluation of permanent impairment. 5th ed. American Medical Association. Chicago (IL): AMA; 2001. p 4-5.

27. Under the prospective payment system for inpatient r ehabilita tion facilities, which item is used in assigning a patient to a case-mix group? (a) Mini Mental Status Examination (b) Disability Rating Scale (c) Previous hospitalization (d) FIM instrument motor score

27. (d) The prospective payment system for inpatient rehabilitation facilities requires that all patients admitted for inpatient rehabilitation be assigned to an impairment group code category. Payment to the rehabilitation facility is further determined by the patient's subclassification into a case-mix group. The FIM instrument motor score is used to help determine the case-mix group designation under the prospective payment system for inpatient rehabilitation facilities. None of the other options listed are used in this process.

28 Which characteristic best describes fasciculation potentials? (a) Semirhythmic in their firing pattern (b) Morphologically the same as motor unit potentials (c) Produced by ephaptic conduction between single muscle fibers (d) Randomly firing single muscle fibers

28 Answer: (b) Commentary: Fasciculation potentials are spontaneously firing motor unit potentials with the same morphologic characteristics as that of a motor unit or polyphasic action potential. They have an irregular firing pattern usually and the site of origin is unclear. Ephaptic conduction between single muscle fibers is thought to be the mechanism for complex repetitive discharges. Single muscle fiber potentials are much smaller and represent units such as a fibrillation potential. Ref: Dumitru D, Zwarts MJ. Needle electromyography. In: Dumitru D, Amato AA, Zwarts MJ, editors. Electrodiagnostic medicine. 2nd ed. Philadelphia: Hanley & Belfus; 2002. p 278-9.

28 Which scenario is typical of patients with spinal stenos is and neurogeni c cla udi cat io n? a. Worsening leg pain with downhill walking b. Pain relief with standing still after a long walk c. Worsening leg pain with lumbar flexion d. Pain relief with increased anterior pelvic tilt

28 Option a is correct. Lumbar extension, created when walking downhill, decreas es spinal canal diame ter an d wo rse ns neu rog enic claud ica ti on s ymp toms . Ne urogen ic c laud icati on is cl ass ica lly im prov ed when a person changes from a standing or walking position to a seated position. Lumbar flexion (forward trunk lean) with uphill walking may improve leg pain because lumbar flexion increases spinal canal diameter. Anterior pelvic tilt is associated with lumbar extension and increased pain.

28 What aspect of gait is improved when a manual locking knee design is chosen for an individual with a unilateral transfemoral dysvascular amputation? a) Overall gait mechanics with decreased energy consumption a. b) Foot clearance in swing phase b. c) Stability in stance phase c. d) Ability to vary the gait cadence

28 Option c is correct.Commentary: The only advantage of a manual locking knee is its inherent stability. It is typically used for patients with significant weak nes s or instability, such as very low level household ambulators or patients using prosthetic limb for transfers. Since the knee does not bend during swing phase it compromises gait mechanics. Toe clearance is more difficult and the prosthetic limb is typically designed to be shorter than the intact limb. Reference: Huang ME, Miller LA, Lipschutz R, Kuiken TA. Rehabilitation and prosthetic restoration in lower limb amputation. In: Braddom RL, editor. Physical medicine and rehabilitation. 4th ed. Philadelphia: E lsevi er Saunders; 2011. p 305.

28 A gymnast is experiencing ulnar-sided wrist pain that is exacerbated by forearm rotation. Physical exam of the painful wrist suggests greater distal radius palmar-dorsal movement relative to t he u lna compared to the other wris t, suggesting distal radioulnar joint laxity. You suspect she has a) a scaphoid fracture. b) De Quervain syndrome. c) a pulley injury. d) a triangular fibrocartilage complex (TFCC) tear.

28 Reference(s) Earp BE, Waters PM. Hand and wrist injuries. In: Frontera WR, Herring SA, Micheli LJ, et al, editors. Clinical sports medicine: medical managem ent and rehabilitation. Philadelphia : El sevier; 2007. p 351-73. D eal D N, Chhabra A Soft tissue injuries of the wrist and han In: O'Connor FG, Casa DJ, Davis BA, et al. ACSM's sports medicine: a comprehensive review. Philadelphia: Lippincott Williams & Wilkins; 2013; p 366-74 Option d is correct. The triangular fibrocartilage complex (TFCC) is a stabilizer of the distal radioulnar joint (DRUJ). Athletes who participate in repetitive loading of the wr ist may be susceptible to degenerati ve c hanges, or a fa ll on an o utstr etched arm can cause acute injuries. Scaphoid fractures typically present with dull and aching pain in the anatomic snuffbox at the radial wrist. De Quervain syndrome refers to tenosynovitis of the abductor pollicis longus and extensor pollicis brevis, and it presents with pain over the radial styloi It often occurs in racquet sport athletes. Pulley injuries most often occur in climbers, and presents with pain over the volar aspect of the phalanx.

28. Electrodiagnostic findings in a person with multiple myeloma monoclonal an tibody and amy loid depo sition mi ght i nclu de all of the following EXCEPT (a) distal symmetric axonal sensorimotor polyneuropathy. (b) mononeuropathy multiplex. (c) anterior horn cell disease. (c) demyelinating polyradiculoneuropathy. Clinical Activity 3.10

28. (c) Patients with multiple myeloma may commonly have distal demyelinating a nd axonal periph era l neuropath ies. Mo noneuropathy mult iplex with conduction blocks at a typ ical sit es is s ug ges tive of amy loid deposition. Amyloid deposition may also cause small fiber neuropathy, with normal nerve conduction study and electromyography results. Radiculopathy may be associated with pathologic spinal fracture or neuroforaminal stenosis. Multiple myeloma is not associated with isolated anterior horn cell disease.

28. A 27-year-old previously hea lthy wom an awo ke with severe right scapular and shoulder pain 4 weeks ago. There is no history of trauma. She has no constitutional symptoms. Three weeks ago, her pain began resolving and scapular winging developed. Electromyography (EMG) reveals 3+ positive waves and fibrillations with markedly decreased recruitment in the right serratus anterior. EMG of the right deltoid, biceps, pronator teres, abductor pollicis brevis, first dorsal interosseous, and cervical paraspinals is normal, as is EMG of the left serratus anterior. The most likely diagnosis is(a) systemic lupus erythemat osus .(b) compression neuropathy o f th e dor sal scapu lar nerve.(c) idiopathic brachial neur opat hy (n eur algic amyotrophy).(d) C5 radiculopathy due to cerv ical dis c hern iation.

28. (c) This is a classic history for neuralgic amyotrophy or idiopathic brachial plexopathy involving the long thoracic nerve. In 30% of patients with neuralgic amyotrophy, EMG abnormalities can be found in the asymptomatic upper extremity; however, the absence of such findings does not obviously exclude the diagnosis. The findings are inconsistent with the other diagnoses.

28. A 40-year-old man sustained an injury to his left ar m, 3 week s ag o, when he lost his balance and crashed into a bookshelf. His complaints include left arm pain, weakness with extension of his wrist and fingers, and decreased hand grip. He denies any numbness but has odd sensations over the dorsum of the left hand. Prior to any testing, which problem would you consider as the most likely? (a) Posterior interosseous neuropathy (b) C7 radiculopathy (c) Posterior cord brachial plexopathy (d) Radial neuropathy

28. (d) Based on the clinical presentation, radial nerve injury is the most likely cause of the patient's symptoms. Considering the location of the trauma the other possibilities seem less likely. In a posterior interosseous nerve injury one would not expect any sensory problems.

29 In a patient with inflammatory arthritis, which type of exercise is LEAST likely to raise the white blood cell count in the synovial fluid of the affecte d joi nt? a) Isotonic a. b) Isometric b. c) Isokinetic c. d) Plyometric

29 Option b is correct. Commentary: Isometric exercise allows for tension to be generated in the muscle without any visible joint movement occurring. An example of an isometric exerc ise would be pushing against a wall. Isometric exercise does not alter synovial fluid composition, in contrast to other forms of exercise that can increase white blood cell count and synovial fluid volume in patients with a history of inflammatory arthritis. Isotonic exercises use constant external resistance with variable speed of movement. An example of this would be performing a biceps curl with a dumbbell . Isokinetic exercises are characterized by constant joint speed and variable external resistance. Special equipment is needed for this type of exercise. I n bot h i sotonic and isokinetic exercise, there is visible joint movement. Plyometric exercise involves fast, powerful movements. An example of a plyometric exercise would be jumping from a squatting position. Reference: (a) Physiology of synovial joints and articular cartilage. In: Gonzalez EG, Myers SJ, Edelstein JE, Lieberman JS, Downey JA, editors. Downey a nd Da rli ng's physiological basis of rehabilitation medicine. 3rd ed. Boston: ButterworthHeinemann; 2001. p 160. (b) Choi H, Sugar R, Fish DE, Shatzer M, Krabak B. Physical medicine and rehabilitation pocketpedia. Philadelphia: Lippincott Williams & Wilkins; 2003. p 58. Hicks JE, Joe GO, Gerber LH. Rehabilitation of the patient with inflammatory arthritis and connective-tissue disease. In: DeLisa JA, Gans BM, Walsh NE. P hysic al medicine and rehabilitation: principles and practice. 4th ed. Philadelphia: Lippincott Williams & Wilkins; 2005. p 737-8.

29 Small muscle motor units a. are recruited after large motor units. b. have fewer muscle fibers. c. have a higher threshold of recruitment. d. have larger cross sectional areas.

29 Option b is correct. The muscle motor unit size principle states that smaller motor units are recr uit ed pr io r t o l arg e m otor units . F ur ther , s mall er m otor u nits hav e few er muscl e f ibe rs, lo wer threshold of recruitment and smaller cross sectional areas.

29 A 45-year-old woman with paraparesis due to relapse-remitting multiple sclerosis diagnosed 5 years ago presents to your outpatient clini She started using a cane for ambulation about 3 months ago. She previous ly did not w ant t o take any medications for her disease, but now asks you whether there are any that can improve her ability to walk. The most appropriate option for her is a) dalfampridine ER (Ampyra). b) glatiramer acetate (Copaxone). c) interferon-?1a (Avonex). d) natalizumab (Tysabri).

29 Reference(s) (a) Goodman AD, Brown TR, Krupp LB, et al. Sustained-release oral fampridine in multiple sclerosis: a randomized double-blind controlled study. Lancet 2009;373(9662):732-8. (b) Kr aft GH, Brown TR, J ohnson S. Multi ple sclerosis. In: Braddom RL, editor. Physical medicine and rehabilitation. 4th e Philadelphia: Elsevier; 2011 p 1241. Option a is correct. Dalfampridine ER (Ampyra), also known as 4aminopyridine, has been shown to improve walking speed in patients with all forms of multiple sclerosis. This agen t is a potassium-channel blocker tha t in creases acety lc holine rel ease at the neuromuscular junction and in the central nervous system. The benefits may be related to improved nerve conduction and the frequency response in demyelinated nerve fibers via prolongation of the repolarization phase of the action potential. The other 3 medications listed are all shown to reduce relapse rate and new lesions on MRI, but not to improve function

29. Whole-brain irradiation for primary and metastatic central nervous system tumors during inpatient rehabili tatio n (a ) results in a higher rate of medical transfers. (b) is associated with lower FIM instrument scores at discharge. (c) should not be performed because it interferes withtherapy schedules. (d) carries a high risk of acute injury and functional statusdecline. Ref: (a) Cross NE, Glantz MJ. Neurologic complications of radiation therapy. Neurol Clin 2 003;21:24 9-77. S75 (b) Cohen SR, Payne DK, Tunkel RS. Lymphedema: strategies for management. Ca ncer 2001;92(4 Suppl):9 80-7. Clinical Activity 3.11

29. (a) Whole-brain irradiation is associated with a higher-than-average rate o f medical transf ers but did no t produ ce a statisticall y significant decrease in FIM ins tru ment sco res. Wi th cu rrent pr oto cols there is little risk of acute injury, but this risk is increased with coadministration of methotrexate or anticonvulsants. Therapy schedules can and should be adjusted to accommodate for radiation treatments.

29. Which statement is true rega rding sp inal c ord injury without obvious radiologic abnormality in children?(a) It most commonly occurs in l umbar ra ther t han cervical injuries.(b) There is a lower inciden ce i n you nge r chil dren.(c) It is associated with la rger head si ze and relatively weak neck muscles.(d) Neurologic impairmen, if it occur s, is usu ally apparent within 2 to 4 hours post-injury.

29. (c) Spinal cord injury without obvious radiologic abnormality (SCIWORA) usually occurs in young children, is thought to be due to the relatively large head size and weak neck muscles, and motor abnormalities may not be apparent for up to several days. SCIWORA most commonly occurs in the cervical region.

3. Which of the following synovial fluid findings is most specific for infection? (a) White blood cell count of 5,000/mm3 (b) Transparent, straw-colored fluid (c) Ninety-eight percent neutrophils on a differential leukocyte count (d) Negatively birefringent crystals under a polarizing microscope

3 (c) All synovial fluid removed for diagnostic purposes should be sent for gram stain and cell count. Noninflammatory synovial fluid typically has white blood cell counts of less than 2000/mm3, is transparent or yellow colored, and has less than 50% neutrophils. Inflammatory fluid usually is translucent or opaque, can have very high cell counts (up to 100,000/mm3), and usually has less than 90% neutrophils. Synovial fluid that has cell counts over 100,000/mm3, is purulent and has more than 95% neutrophils should be considered infected.

3 In the United States, upper extremity amputations are a) most often due to vascular disease. a. b) more common in males than females. b. c) rarely caused by workplace injuries. c. d) increasing due to the relaxing of occupational safety standards.

3 Option b is correct.Commentary: Of all upper extremity amputations, 90% are due to trauma. The majority of these are related to workplace injuries involv ing s aws or blades. Males account for 75% of all upper extremity amputations. Trauma related amputations have decreased over the last 20 years and they are expected to remain flat or decrease, due to ongoing enforcement of safety standards. Reference: Sheehan TP. Rehabilitation and prosthetic restoration in upper limb amputation. In: Braddom RL, editor. Physical medicine and rehabilitation. 4th ed. Philadelphia: Elsevier Saunders; 2011. p 257.

3 Riluzole is a neuroprotective agent that appears to inhibit glutaminergic neurotransmission in the spinal cor It is modestly effective in improving the life expectancy in the patient with ALS. Compared to adults, children with burn injuries have a higher frequency of a) heterotopic ossification. b) contractures requiring surgical releases. c) psychological or behavioral problems. d) long-term physical deficits.

3 Reference(s) Massagli T, Engel JM. Special consideration for pediatric patients with disability due to traum In: Robinson LR, editor. Trauma rehabilitation. Philadelphia: Lippincott Williams & Wil kins; 2006. p 3 4 Option b is correct.

33. On the 4th day after rev isio n of his left total hip arthroplasty with an anterior approach, your patient complains of pain in the left thigh after bridging in bed for the bedpan. You notice that his left leg is externally rotated and appears shorter than his right. The LEAST likely factor contributing to your patient's predicament is(a) a surgically malposition ed i mplan t.(b) aseptic loosening of the imp lant.(c) inadherence to precautio ns.(d) profound soft tissue wea knes s.

33. (b) Aseptic loosening is seen 10 years after implant of prosthesis, the other choices are common etiologies for dislocations during the first few weeks after implant.

34. Clinical features of Friedreich's ataxia include (a) sparing of sensory function. (b) onset in the fourth decade of life. (c) a high incidence of scoliosis. (d) ambulation into late adulthood.

34 (c) Friedreich's ataxia is a spinocerebellar degenerative syndrome with onset in the first 2 decades of life. Weakness, proprioceptive sensory loss, and ataxia dominate the clinical picture. The incidence of scoliosis approaches 100%; it is typically more severe when onset occurs at a young age. Ambulation is lost by early adulthood.

32 Which drug is NOT associated with increased seizure risk in patients with traumatic brain injury? (a) methylphenidate (Ritalin) (b) ciprofloxin (Cipro) (c) amitriptyline (Elavil) (d) bupropion (Wellbutrin)

32 Answer: A Commentary: Methylphenidate and dextroamphetamine do not appear to be associated with increased seizure risk among patients with traumatic brain injury. However, amitriptyline, bupropion and quinolones decrease seizure threshold. Ref: Yablon SA, Dostrow VG. Post-traumatic seizures and epilepsy. In: Zasler ND, Katz DI, Zafonte RD, editors. Brain injury medicine: principles and practice. New York: Demos Medical Publishing; 2007. p 444.

"

3. The initial treatment for osteoarthritis is (a) medication to reverse articular cartilage damage. (b) surgical correction of joint deformities. (c) therapy to relieve joint symptoms. (d) immobilization of the joint to prevent deformity." 3 (c) General treatment principles of osteoarthritis include medications and/or therapy to relieve joint symptoms, along with maintaining or improving function and minimizing drug toxicity. To date, no medications can reverse or repair damaged articular cartilage. Exercises, such as range of motion and strengthening, are part of nonpharmacologic therapy of osteoarthritis. Surgical correction is not an initial treatment strategy.

33 The circle of Willis is comprised which complex of arter ies? a. Anterior communicating, posterior communicating, midd le cerebral, sup erior ce reb el la r b. Anterior communicating, posterior cerebral, anterior cerebral, verteb ral c. Posterior communicating, posterior cerebral, superior cerebellar, bas ilar d. Anterior communicating, posterior communicating, post erior cerebral, anter ior ce re br al

33 Option d is correct. The circle of Willis is made up of the following arterie s the anterior commu nic ati ng a rte ry; th e a nter ior ce reb ra l ar ter ies an d poster ior comm unica tin g art eri es as the y co me off the middle cerebral arteries; and the posterior cerebral arteries after they come off the basilar artery.

30. A 16-year-old male with a right above-knee amputation presents to your clinic t o ge t you r o pin ion o n op era ting a m oto r vehicl e. Yo u ad vis e him that he would be required to:(a) strengthen his hip extensors.:(b) change the position of the car's accelerator and brake.:(c) obtain a special driving prosthesis.:(d) install a handbrake on the vehicle's left floor.

30 (b) The only situation that applies in this scenario is changing the position of the pedals in order to operate the vehicle with the left lower limb. Instal lin g a handb rak e is reserved fo r persons wit h r ight upp er limb amputat ions. Other a ccep table rec ommendat ion s could i ncl ud e a utoma tic tr an smission an d/or ha nd contr ols.

30 A weight-activated stance-control knee unit would be indicated in a transfemoral amputee who (a) has cognitive deficits. (b) has a contralateral weak limb. (c) is an unlimited ambulator. (d) is a new amputee.

30 Answer: D Commentary: Weight-activated stance-controlled knees are often used for individuals with a transfemoral amputation. They are especially useful as a preparatory prosthesis in a new amputee, because their simplicity and safety help new amputees learn to walk with a prosthesis. To flex the knee, the amputee must shift weight onto the opposite leg, which requires the opposite limb to accept increased weight. Additionally, the amputee must have the cognitive ability to learn to weight shift. The requirement to shift weight off of the prosthesis to allow knee flexion presents few problems at slow cadences, but if the amputee attempts to walk at a more normal speed, the gait pattern is disrupted by the premature weight shift. Ref:Michael JW. Prosthetic suspensions and components. In: Smith DG, Michael JW, Bowker JH, editors. Atlas amputations and limb deficiencies. 3rd ed. Rosemont: AAOS ;2004. p 420.

30 What prosthesis is most appropriate for a 6-monthold child who has a congenital transhumeral amputation? a) Curve shaped "banana" arm a. b) Myoelectric hand b. c) Body powered hook c. d) Friction elbow arm

30 Option a is correct. Commentary: The banana arm is a passive prosthesis designed to help in reaching and bimanual midline activities in the very young child. A myoelectric ha nd is mo st appropriate to initiate about age 1 year; a body-powered hook is appropriate for children age 4-5 years; a friction-elbow arm is appropriate about when a child with a transhumeral amputation starts to walk. Reference: Gaebler-Spira D, Lipschutz R. Pediatric Limb Deficiencies. In: Alexander MA, Matthews DJ, editors. Pediatric rehabilitation: principles and practice. 4th ed. New York: Demos Medical; 2010. p 342-344, see fig13.9, p343,"banana arm."

30 When comparing the compound muscle action potential (CMA P) latency with the l ate ncy o f the F res pon se, which fin di ng i s m ost li ke ly to occu r as you mov e the st imu lat ing ele ctrode to a more proximal location in the limb? a. Both CMAP and F-response latency will decrease. b. Both CMAP and F-response latency will increase. c. CMAP latency will increase and F-response latency wil l decrease. d. CMAP latency will decrease and F-response latency wil l increase.

30 Option c is correct. The F response is a late response that becomes evident a fter the CMAP re spons e. The F r esp ons e i s d eriv ed fro m a n anti dro mic st im ulatio n of a p eriph era l ner ve tha t p rop agat es an impulse proximally to the anterior horn cells of the spinal cord and returns an orthodromic response to the distal muscle. The F response is a pure motor response creating a small action potential representing 1% to 5% of the muscle fibers. Moving the stimulator proximally shortens the distance to the spinal cord's anterior horn cells. Consequently, the F response latency decreases as you increase the distance from the distal muscle pick up site and the CMAP amplitude or direct motor potential latency increases.

30 You are consulted on the surgical floor of the hospital to manage an agitated patient with traumatic brain injury. Which antipsychotic medication has the mo st favorable side-effect profile for thi s patient? a) Haloperidol (Haldol) b) Olanzapine (Zyprexa) c) Risperidone (Risperdal) d) Quetiapine (Seroquel)

30 Reference(s) (a) Wagner AK, Arenth PM, Kwasnica C, Rogers EH. Traumatic brain injury. In: Braddom RL, editor. Physical medicine and rehabilitation. 4th e Ph iladelphia: Saunders; 2011. p 1154-1 155 (b) Kim E, B ij lani M. A pilot study of quetiapine treatment of aggression due to traumatic brain injury. J Neuropsychiatry Clin Neurosci 2006;18:547-49. Option d is correct. Quetiapine is a frequently selected agent for postTBI agitation for its favorable side-effect profile and its relatively low action as a D2 receptor antagon ist. A recent pilot study suggests t hat quetiapine is c linically effec tive in reducing agitation symptoms post-TBI, with associated improvements in cognition. Animal studies showed the detrimental effect that the antipsychotic medication, haloperidol (a classic D2 receptor antagonist), has on motor recovery. Experimental studies using many atypical antipsychotic medications, including olanzapine and risperidone, have shown negative effects on cognitive recovery.

31. Which statement regarding cervical traction is correct?:(a) The best angle of pull is between 10° and 20° of cervical extension.:(b) Its use in patients with rheumatoid arthritis is absolutely contraindicated.:(c) At least 25 pounds of force is necessary to counter the effects of gravity on t he h ea d.:(d) With an over-the-door traction unit, the patient should face away from the door to wh ich th e p ulley is att ached .

31 (b) Patients with rheumatoid arthritis have ligamentous instability. This can lead to subluxation of cervical vertebrae, especially at the atlantoaxial join t ( C1-2 ). Be cau se in stability c an lead to sp ina l co rd c omp ression, cer vical tractio n is , ther efo re, abso lut ely contr ain di cat ed in perso ns with rheum atoid a rthritis . The best a ngl e o f pu ll i s b etw een 2 0° and 30 ° of fl exion. The most common reason for cervical traction to fail or to exacerbate symptoms is applying the force in extension rather than in flexion. The home traction unit should always be placed so the patient is facing toward the door to which the pulley is attached. At least 10 pounds of force is needed to counter the effects of gravity on the head. To straighten the cervical lordotic curve requires 25 pounds of force.

31 For a person with an upper extremity amputation, what is the advantage of choosing a body-powered device over a myoelectric device? a) Stronger grip force a. b) Better cosmesis b. c) Lighter weight c. d) Less dependence on motor strength

31 Option c is correct. Commentary: Main advantages of body powered systems are lower initial costs, lighter weight, easier repairs, and better tension feedback to body. Advanta ges o f m yoelectric devices are cosmesis, less need for motor strength/coordination to operate limb, and stronger grip force. Reference: Sheehan TP. Rehabilitation and prosthetic restoration in upper limb amputation. In: Braddom RL, editor. Physical medicine and rehabilitation. 4th ed. Philadelphia: Elsevier Saunders; 2011. p 270.

31 A 34-year-old woman diagnosed with fibromyalgia presents with symptoms c onsis ten t w it h cen tra l p ain sen sitiza tio n and dep ress ed m ood. Y ou a re c onsid eri ng in iti ati ng a t rial of an antidepressant medication, which you hope will also help with her pain. Based on its mechanism of action, which medication would ONLY be expected to help with her mood and NOT also help with her pain, independent of its antidepressant effect? a. Nortriptyline (Pamelor) b. Duloxetine (Cymbalta) c. Milnacipran (Savella) d. Citalopram (Celexa)

31 Option d is correct. When prescribing an antidepressant medication for pain, it is important to co nsi der t he im pac t o f e ach medica tio n on s ero toni n an d nore pine phri ne. S ero tonin up reg ula tio n is typically believed to help with mood, and norepinephrine upregulation is typically believed to help with central pain. Nortriptyline is a tricyclic antidepressant (TCA) that enhances both serotonin and norepinephrine neurotransmission. Duloxetine and milnacipran are serotonin-norepinephrine reuptake inhibitors (SNRIs) and, therefore, are expected to enhance both serotonin and norepinephrine neurotransmission. Citalopram, on the other hand, is a selective serotonin reuptake inhibitor (SSRI) and, therefore, it does not directly enhance norepinephrine neurotransmission.

31 Aside from motor vehicle crashes (MVC), which etiology is the most common cause of spinal cord injury in children younger than 5 years? a) Medical/surgical complications b) Violence c) Sports d) Falls

31 Reference(s) DeVivo MJ, Vogel L Epidemiology of spinal cord injury in children and adolescents. J Spinal Cord Med 2004. 27 (suppl 1): S7-S10. Option a is correct. Motor vehicle crashes are the most common cause of spinal cord injury (SCI) in all pediatric age groups, accounting for 65% of SCI in children younger than 5 years. The second most common caus e in of SCI in th is age group is m edical/surgical complications, accounting for 11.6% of spinal cord injuries, while the rate among older pediatric patients (age 16-21 years) is only 0.6%. More common in the older age groups are violence, sports injuries, and falls. For example, among children younger than 5 years, 8.7% experience SCI from violence, while 21.6% in the 16- to 21-year range fall victim to SCI for that reason. Sports account for 0.2 % of SCI in children below age 5, while the percentage in youngsters between the ages of 16 and 21 is 18.3%. Falls account for 6.5% of SCI in children younger than 5 years; 8.1 % of spinal cord injuries in the 16- to 21year age group are attributed to falls, and 24% of adults over age 22 sustained their spinal cord injury in a fall. Because both physiology and behavior change with age in the pediatric population, the pediatric-onset SCI population exhibits distinct epidemiologic characteristics among different age groups and when compared to the adult-onset SCI population.

31. Lumbar spondylolisthesis is the t erm for s lippage of one vertebral body on the adjacent body below. All of the following statements are true EXCEPT(a) It is graded 0-4, by the per centa ge of sli ppage of the superior body on the inferior one.(b) It is caused by a fractu re o r def ect in th e pars interarticularis.(c) A TLSO brace is the best met hod t o s tabili ze an unstable spondylolisthesis.(d) A spondylolisthesis may caus e neu rol ogic c ompromise of the cauda equina.

31. (c) Spondylolisthesis in the lumbar spine is a common finding, occurring 70% of the time at L5-S1 and 25% at L4-5. It is caused by a defect or fracture in the pars interarticularis and is graded 0-4 on the basis of the amount of slippage of one body on the other. It may indeed lead to spinal stenosis and compromise of the cauda equina. A spinal orthosis will not be effective in stabilizing this defect but can be useful in reducing lumbar lordosis, decreasing pain, and reducing gravitational forces on the slippage.

31. A 35-year-old female tennis player and data entry clerk presents with a 6 week his tory of lateral elbow pain. Which factor is promoting her injury? (a) Her strong scapular stabilizer muscle strength (b) Her leading with the shoulder when performing the backhand tenni s stroke (c) Her keyboard being placed at too high a level at work (d) Her tennis racquet string tension being recently increased from a low to a moderat e str ing tension.

31. (c) This patient has lateral epicondylitis or so-called tennis elbow. Weak scapular stabilizer muscle strength may cause decreased force production through the elbow, thus causing more force and load through the elbow. Leading with the elbow during the backhand stroke can cause an increased load on the wrist extensor muscles. Keyboards placed too high or too low may cause lateral epicondylitis. Racquet string tension should be adjusted to mild to moderate tension (52 to 55 pounds if stringing machine is properly calibrated) in a normal (non-oversized) racquet. A racquet strung to less than 30 pounds will require the tennis player to use more force. This may, in turn, cause increased grip pressures and wrist extensor overuse.

39 What is the best predictor of fractures in a person with osteoporosis? a) Low body weight a. b) Recent falls b. c) Low physical activity c. d) Prior fractures

39 Option d is correct.Commentary: The best predictor of future fractures is prior fractures. Low body weight is also a major risk factor. Recent falls and low p hys ical activity are additional risk factors.

32 An 18-year-old female on your inpatient traumatic brain injury service is inconsistently oriented and does not recall your name on a day-to-day basis. Sh e can fo llow single-step commands. She gets more confused when stressed but can be re-directed and can finish her therapy sessions with encouragement. She is more consistent with goal-directed behavior but needs cueing. Greater participation in activities of daily living is evident and she is developing a better awareness of self and others. On the Rancho Los Amigos scale, what is her level of cognitive function? a) IV b) V c) VI d) VII

32 Option c is correct. Commentary: She is presently displaying characteristics consistent with the sixth stage of recovery in the Rancho Los Amigos scale of cognitive function. This pa tient is not out of posttraumatic amnesia and is still confused. However, she responds appropriately to feedback and is able to participate in therapies. She is improving in goal-directed behavior and is developing greater awareness of self and others. On the Rancho Los Amigo scale, the other options listed are described as follows: Level V - Confused and Inappropriate; Level VI - Confused and Appropriate; Level VII - Automatic and Appropriate. For level VII, you would anticipate that she would no longer need cuing for goal-directed behavior but will still have problems with new activities or with planning and following through with activities. Reference: (a) Cifu DX, Kreutzer JS, Slater DN, Taylor L. Rehabilitation after TBI. In: Braddom RL, editor. Physical medicine and rehabilitation. 3rd ed. Phil ade lphia: Elsiever; 2007. p1138. (b) Sullivan KJ. Therapy intervention for mobility impairments and motor skill acquisition after TBI. In: Zasler ND, Katz DI, Zafonte RD, editors. Brain injur y m edicine: principles and practice. New York: Demos Medical 2007. p 937-938.

32 A 52-year-old man presents with slowly progressive leg weakness. He also complains of clumsiness with his right hand, which creates difficulties with button s or turning keys. Examination revea ls m ild bilateral f oot drop a nd le g weakness. Fasciculations and mild wasting are observed in the calf muscles. There is no spasticity or impaired sensation. His speech is normal, but tongue fasciculations are appreciate Respiration, pulse and temperature are normal. Electrodiagnostic studies show evidence of denervation with reinnervation. The most likely diagnosis is? a) amyotrophic lateral sclerosis b) Charcot-Marie-Tooth disease c) Guillain-Barré syndrome d) myasthenia gravis

32 Reference(s) (a) Krivickas LS, Carter GT. Adult motor neuron disease. In: Frontera WR, DeLisa JA, Gans BM, Walsh NE, Robinson LR, editors. DeLisa's physical medicine and rehabilitation: princi ples and practice . 5th e Phil adelp hia: Lippincott-Raven; 2010. p 719-21. (b) Carter GT, Bednar-Butler LM, Abresch RT, Ugalde VO. Expanding the role of hospice care in amyotrophic lateral sclerosis. Am J Hosp Palliat Care 1999;16: 707-10. Option a is correct. Flaccid paresis involving the lower extremities, foot drop, hand clumsiness, muscle wasting, and especially fasciculation in a middle-aged person are highly suggestive of amyotrophic lateral s cler osis (ALS). T he se symptom s res ult from degeneration of the motor neurons in the spinal cord (lower motor neuron) and lead to denervation of skeletal muscle. His tongue fasciculations result from degeneration of motor neurons of cranial nerve nuclei. Surviving neurons may reinnervate the denervated myofibers by axon sprouting. The finding of denervation/reinnervation on needle EMG examination is confirmatory of the clinical diagnosis. The patient will later develop evidence of corticospinal and corticobulbar (upper motor neuron) degeneration as his disease progresses

32. A 45-year-old woman with mul tiple sc lerosi s reports that her fatigue is interfering with the activities of daily living. You prescribe an energy conservation program and(a) amantadine (Symmetrel).(b) valproic acid (Depakote) .(c) beta-interferon (Avonex) .(d) adrenocorticotropic horm one (ACTH ).

32. (a) Fatigue is a common, limiting symptom in patients with multiple sclerosis. Behavioral techniques such as energy conservation and well-planned rest periods are often required. Amantadine is traditionally the first choice; however, pemoline may provide relief. Beta-interferon and ACTH are more disease-modifying agents used during periods of acute exacerbation.

32. What is the most common primary malignant tumor of the brain in adults? (a) Medulloblastoma (b) Meningioma (c) Glioblastoma multiform (d) Ependymoblastoma

32. (c) More than 90% of the primary malignant tumors of the brain in adults are high-grade astrocytomas and, of these, the most common is glioblastoma multiform. Meningiomas are tumors that occur in the membranes that cover and protect the brain and spinal cord (the meninges). Meningiomas usually grow slowly. Medulloblastomas are almost always found in children or young adults. Ependymoblastomas are rare cancers that usually occur in children.

33. Mr. Jones comes to your office complaining of a hot, painful, swollen left wris t. A dd iti ona l f indin gs o n p hysic al exa mination in cl ude swa n- neck and boutinierre deformities, subluxation of the metacarpophalangeal joints with ulnar deviation of the digits. What will x-ray findings of his wrist reveal?:(a) Chondrocalcinosis of articular cartilage:(b) Severe marginal erosions with juxta-articular osteopenia:(c) Bony erosion with an overhanging edge:(d) Pencil-in-cup deformity

33 (b) This patient has rheumatoid arthritis which shows juxta-articular osteopenia. Gout characteristically reveals "overhanging edge" lytic lesions. Chondroc alc inos is is se en in pseudo-gou t, and pen cil =-in cup defo rmi ty is seen w ith psoriatic art hritis .

33. Which statement is TRUE regarding the rehabilitation of anterior cruciate ligament (ACL) reconstruction/repair? (a) Immediate postoperative weight bearing adversely affects subsequent knee function. (b) A self-directed program is not as effective as regular physical therapy visits. (c) Use of a continuous passive motion machine improves outcome. (d) Postoperative functional bracing does not improve outcome.

33 Answer: (d) Commentary: The use of postoperative functional bracing does not improve outcome. Immediate postoperative weight bearing does not adversely affect subsequent knee function. A self-directed program is as effective as regular physical therapy visits in a motivated patient. The use of a continuous passive motion machine does not improve outcome. Ref: Wright RW, Spindler KP. Anterior cruciate ligament tear. N Engl J Med 2008;359:2135-42.

33 A physiatrist assesses each of the following patients during hospitalization and determines that each one is clinically appropriate for an admission to y our a cut e inpatient rehabilitation unit. Which patient does NOT qualify for the Center for Medicare and Medicaid Services (CMS) 60% Rule (formerly known as the 75% Rule)? a) 28-year-old woman with multiple sclerosis exacerbation, status-post intravenous methylprednisolone (Solu-Medrol) a. b) 60-year-old woman with active polyarticular rheumatoid arthritis, status-post left total shoulder arthroplasty b. c) 63-year-old man with end-stage kidney disease, status-post kidney transplant, complicated by sepsis c. d) 87-year-old man with end-stage osteoarthritis, status-post elective left total knee replacement

33 Option c is correct. Commentary: Inpatient Rehabilitation Facilities (IRFs) must follow the Center for Medicare and Medicaid Services (CMS) criterion regarding admissions in order to receive payment. Previously known as the "75% Rule," CMS changed the percentage to 60% in 2007. Currently, 60% of all admissions to an IRF must have 1 or more of 13 selec ted c ond itions that include stroke, spinal cord injury, congenital deformity, amputation, major multiple trauma, fracture of femur (hip fracture), brain injury, neurological disorders (including multiple sclerosis, motor neuron disease, polyneuropathy, muscular dystrophy, Parkinson disease), and burns. Other conditions such as rheumatologic arthritides or osteoarthritis have additional stipulations. Patients undergoing elective hip and/or knee joint replacements must also meet 1 of 3 additional criteria, including (1) bilateral joint replacement surgeries, (2) body mass index greater than 50, or (3) age 85 or older. The CMS criterion does not prohibit admission to an IRF if a patient does not have 1 of these 13 conditions, though IRFs must continually monitor compliance to ensure reimbursement. While the patient who underwent a kidney transplant may be able to participate in and benefit from 3 hours of interdisciplinary therapy daily and requires ongoing inpatient nursing and physician management, his diagnosis is not included in the 13 medical conditions recognized by the CMS criterion. Reference: Department of Health and Human Services, Center for Medicare and Medicaid Services, Medicare Learning Network fact sheet: Inpatient rehabilita tion fac ility prospective payment system http://www.cms.gov/Outreach-and-Education/Medicare- Learning-Network- MLN/MLNProducts/downloads//InpatRehabPaymtfctsht09- 508.pdf Accessed 05/08/2012.

34. In patients with neuromuscular disease, which measurement is shown to be the most influential in determining the intensity of their aerobic exercise? (a) 70%-85% maximum heart rate (b) 60%-80% of heart rate reserve (c) Borg scale of perceived exertion (d) Delayed-onset muscle soreness

34 Answer: (d) Commentary: Thirty-three studies, to date, report absent or negligible adverse effects of exercise in neuromuscular disease patients. Maximum heart rate and cardiac reserve are the same goals for able bodied and neuromuscular patients. Delayed onset muscle soreness is shown to be the most influential factor for patients to modify their own exercise program. Ref: Cup EH, Pieterse AJ, Ten Broek-Pastoor JM, Munneke M, van Engelen BG, Hendricks HH et al. Exercise therapy and other types of physical therapy for patients with neuromuscular disease: a systemic review. Arch Phys Med Rehabil 2007; 88:1452-64.

34 Which complaint is NOT an early sign of dysphagia in amyotrophic lateral sclerosis? (a) Increasing hoarseness (b) Persistent coughing after swallowing (c) Painful swelling in the oropharynx (d) Inability to manage thin liquids

34 Answer: C Commentary: Dysphagia in amyotrophic lateral sclerosis is directly due to weakness and spasticity of the oropharyngeal musculature and does not involve pain or swelling. The presence of any of the other symptoms indicates dysphagia that can lead to aspiration. A speech therapist should be consulted for clinical swallowing evaluations and recommendations on dietary modification. Ref:(a) Carter GT, Krivickas LS. Adult motor neuron disease. In: Kirshblum S, Campagnolo DI, DeLisa JA, editors. Spinal cord injury medicine. Philadelphia: Lippincott Williams & Wilkins; 2002. p 537-52.(b) Strand EA, Miller RM, Yorkston KM, Hillel AD. Management of oralpharyngeal dysphagia symptoms in amyotrophic lateral sclerosis. Dysphagia 1996;11:129-39.

34 Which of the following opioid prescriptions is the LEAST potent in terms of m orp hin e eq uiv ale nts pe r da y? a. Hydrocodone/acetaminophen (Norco), 5/325 mg 1 tab ora lly, 4 times dai ly b. Oxycodone/acetaminophen (Percocet), 5/325 mg 1 tab or ally, 4 times da ily c. Methadone (Methadose) 5 mg, 1 tab orally, 4 times dai ly d. Hydromorphone (Dilaudid) 5 mg, 1 tab orally, 4 times daily

34 Option a is correct. When using various opioid medications, it is important t o understand the diff ere nt po te nci es in ter ms o f morp hin e equi val ents p er day ( MEDs ). T here are many co nve rsi on tabl es that provide this information. It is particularly important to pay attention to methadone, since its potency does not increase in a simple linear fashion. Using opioid calculators such as the one in the reference cited below, the Norco prescription is 20 MEDs, the Percocet prescription is 30 MEDs, and the hydromorphone and methadone prescriptions are both 80 MEDs.

34 Myotonic discharges produce a distinctive sound on electromyography (EMG). This sound results from a) spontaneous firing of motor unit action potentials (MUAPs). a. b) intermittent blocking of muscle fibers during voluntary activity. b. c) firing of just a few muscle fibers per motor unit during voluntary activity. c. d) spontaneous firing of muscle fibers.

34 Option d is correct. ommentary: The classic myotonic discharge of waxing and waning in frequency and amplitude is often described as either a dive-bomber or revving engine. T he so urc e generator of a myotonic potential is muscle fiber, and thus may take the form of either positive wave or a brief spike potential. Neuromyotonia and myokymia result in spontaneous firing of motor unit action potentials as opposed to muscle fiber action potentials. Intermittent blocking of some muscle fibers within a motor unit result in unstable MUAPs, which are characterized by changes in either amplitude or number of phases from potential to potential. Unstable MUAPs are seen in primary neuromuscular junction (NMJ) disorders or with new and immature NMJs, as can be seen in early reinnervation. Mypopathic MUAPs result from a decreased number of muscle fibers firing within a motor unit. These signals are seen as short-duration, small amplitude, and polyphasic potentials. Reference: (a) Myotonic muscle disorders and periodic paralysis syndromes. In: Preston DC, Shapiro BE, editors. Electromyography and neuromuscular disord ers. 2nd ed. Philadelphia: Butterworth-Heinemann; 2005. p 591. (b) Dumitru D, Zwarts MJ, Amato A, Peripheral nervous system's reaction to injury: electrodiagnostic medicine. 2nd ed. Philadelphia: Hanley & Belfus; 2002. p 277-8.

34 When initially ventilating a patient with tetraplegia and no significant associated lung tissue injury, it is recommended to start at 15ml/kg of ideal body weight. Tidal volumes can be increas ed i n small incre me nts if nee ded t o treat any atelectasis that develops, and peak airway pressure should be kept under 40cm of water. Which lower extremity amputee group has the highest probability of successful mobility? a) Dysvascular bilateral transtibial over age 85 b) Dysvascular unilateral transtibial under age 85 c) Traumatic unilateral transtibial under age 85 d) Traumatic unilateral transtibial over age 85

34 Reference(s) Huang ME, Miller LA, Lipschutz R, Kuiken T Rehabilitation and prosthetic restoration in lower limb amputation. In: Braddom RL, editor. Physical medicine and rehabilitation. 4th e Philadelphia: Elsevier Saunders; 2011. p 277. Option c is correct.The person with a traumatic lower extremity amputation has higher mobility success: 97% of traumatic amputees are ambulating at 3 months . Dysvascular amputees can be succes sful ambulators b ut at a lowe r rat e than traumatic amputees. Mobility success in bilateral amputees is less than unilateral amputees. Two studies report that 70% of bilateral transtibial amputees use their prostheses for ambulation. Age greater than 85 years is associated with very low rate of mobility success (2% success in 1 study).

34. A nonambulatory 15-year- old boy w ith spina l muscular atrophy is requesting a new power wheelchair after a growth spurt. An important feature of the wheelchair prescription will be(a) a solid seat with foam p addi ng.(b) extra room at each side to a llow for growt h.(c) seat back below the scap ular ridg e.(d) back-slanted seat with p omme l.

34. (a) Because of growth and increasing weakness, a common sequela of motor neuron disease is scoliosis. It is important to provide adequate pelvic support with a firm seat to avoid hip asymmetry. If wheelchairs are too big, asymmetric spinal posture is encouraged. Slanted seats with pommels are useful to control extensor spasticity, which should not be an issue here. Seat backs should be high to help control spinal posture.

35 Which score range on the Galveston Orientation and Amnesia Test (GOAT) indicates the end of posttraumatic amnesia (PTA)? 75-85 a. 55-65 b. 35-45 c. 15-25

35 Option a is correct. Commentary: A standard technique for assessing posttraumatic amnesia (PTA) in adults is the Galveston Orientation and Amnesia Test (GOAT), a brief struct ured int erview that quantifies orientation and recall of recent events. The GOAT score can range from 0 to 100, with a score at or above 75 defined as normal. The end of PTA is defined as when the GOAT score is at or above 75 for 2 consecutive days. Reference: (a) Cifu DX, Kreutzer JS, Slater DN, Taylor L. Rehabilitation after TBI. In: Braddom RL, editor. Physical medicine and rehabilitation. 3rd ed. Phil ade lphia: Elsiever; 2007. p1138. (b) Brandstater ME. Stroke rehabilitation. In: DeLisa JA, Gans BM, Walsh NE, editors. Physical medicine and rehabilitation: principles and practice. 4th ed. P hil adelphia: Lippincott Williams & Wilkins; 2005. p 1012.

35 Which sign is associated with central dysautonomia follo wing severe trau matic br ain i nj ury ? a. Flaccidity b. Hyperthermia c. Hypotension d. Bradycardia

35 Option b is correct. Central dysautonomia can occur acutely after severe trau matic brain inju ry. I t h as al so be en cal led die ncepha lic s eizu res , au to no mic or neu rost ormin g o r hyp oth ala mic dy sreg ulation syndrome. Signs include elevated temperature with a normal fever work up, tachycardia, elevated blood pressure, rapid respiratory rate and posturing. Facial flushing and diaphoresis may also be seen.

35 You conducted a successful intrathecal baclofen trial in a patient with a traumatic brain injury, administering a 75-microgram bolus via a lumbar puncture. After pump implantation, the neurosu rgeo n requests yo ur recommend ation for a starting dose of baclofen to be delivered via the implanted pump. In micrograms per day, what is the highest starting dose you would recommend? a) 75 b) 100 c) 150 d) 200

35 Reference(s) (a) Elovic EP, Eisenberg ME, Jasey NN Jr. Spasticity and muscle overactivity as components of the upper motor neuron syndrome. In: Frontera WR, editor. Physical medicine and rehab ilit ation: princi pl es and pra ctice . 5th e Philadelphia: Lippincott Williams & Wilkins; 2010. p 1334-1335. (b) Nance PW, Satkunam L, Ethans K. Spasticity management. In: Braddom RL, editor. Physical medicine and rehabilitati on. 4th e Philadelphia: Saunders; 20 11p 654 Option c is correct. The recommended starting dose of baclofen to be delivered by intrathecal pump is twice the dose of the successful trial. The higher dosage is because the re sponse will be muted by the slow del iver y of medicati on with a pu mp, v ersus the rapid delivery with the bolus injection at the initial baclofen trial. In this case, because the trial was a success at a 75microgram dose, the pump should be set to deliver 150 micrograms of baclofen every 24 hours.

35. You are called to the be dsid e of an indivi dual with a T3 spinal cord injury sustained 7 ½ weeks earlier. The person complains of pounding headache and appears to have piloerection on the upper extremities, neck, and face, as well as flushing. Blood pressure is 150/90. The first thing you do is(a) instill a topical anesth etic into th e rect um in order to decrease sensation for a rectal check.(b) apply 1 inch of topical nitr opast e a bove t he level of injury.(c) irrigate the indwelling urin ary c ath eter w ith a small amount of normal saline.(d) sit the person up and lo osen any clo thing.

35. (d) This individual is experiencing autonomic dysreflexia, seen typically in individuals with spinal cord injury with lesions at or above T6. A treatment algorithm that outlines the timing of treatment recommendations was established by the consortium for spinal cord medicine in 1997. When an individual presents with autonomic dysreflexic symptoms including elevated blood pressure (systolic blood pressure greater than 150mm Hg), the very first thing to do is to sit the patient up with his/her clothing and constrictive devices loosened. If the blood pressure remains elevated and the individual has an indwelling catheter, kinks and twists should be removed. If there is no urine flow, the catheter then needs to be irrigated. If the individual does not have an indwelling catheter, a Foley catheter must be inserted and again if there is no urine flow, it should be irrigated. If there is good urine flow and/or the blood pressure drops down to normal, then the work-up as well as other interventions would cease. If the blood pressure remains elevated after irrigation or initiation of catheter, and the systolic blood pressure remains above 150mm Hg, a short-acting antihypertensive medication such as topical nitropaste is initiated. After this, if the individual continues to be hypertensive, he/she may have to be admitted to a hospital to control blood pressure. If, after the short-acting antihypertensive, the blood pressure drops, evaluation of the rectum for fecal impaction begins, including installation of lidocaine into the rectum and allowing it to sit for approximately 5 minutes to decrease sensation before probing the rectum with a gloved finger and subsequently attempting to disimpact.

36 What secondary condition requires the most medical management in adults with cerebral palsy? a) Constipation a. b) Pain b. c) Cardiovascular disease c. d) Osteoporosis

36 Option b is correct. Commentary: In adults with cerebral palsy (CP) The incidence of pain is high (67%-82%) across all disability groups and appears to increase with age. It is mu lti factorial with musculoskeletal issues being of primary concern. Constipation issues persist into adulthood and adjustments to the bowel program may be needed. Adults with CP are not reported to have a higher risk of cardiovascular disease but are less likely to be screened than the general population. Osteoporosis is most common (up to 50% in some studies) in non-ambulatory persons, (Gross Motor Function Classification System, GMFCS levels 4, 5), and it increases with age. Reference: (a) Tosi, LL, Maher N, Moore DW, Goldstein M, Aisen ML. Adults with cerebral palsy: a workshop to define the challenges of treating and preven ting the secondary musculoskeletal and neuromuscular complications in this rapidly growing population. Develop MedChild Neurol 2009;5:3-5. (b) Turk, MA. Health, mortality, and wellness issues in adults with cerebral palsy. Devel MedChild Neurol 2009;51:26. (c) Turk MA, Logan LR, Kanter D. Aging with pediatric onset disability and diseases. In: Alexander MA, Matthews DJ, editors. Pediatric rehabilitation: principles and practice. 4th ed. New York: Demos Medical; 2010. p428-430, 439.

36 Which option is a negative sign of the upper motor neuro n syndrome? a. Dyssynergic cocontraction patterns b. Clonus c. Fatigability d. Babinski sign

36 Option c is correct. The upper motor neuron syndrome (UMNS) can occur followi ng any lesion af fecti ng the d es cen din g m oto r pa thways / co rtic osp inal t ra ct. Cl inic al f eatur es are t he sim ult ane ous presence and interaction of positive and negative signs. The negative features of the UMNS are characterized by a reduction in motor activity. Fatigability is a negative sign of the UMNS, as well as muscle weakness and loss of dexterity. Positive features of the UMNS are increased muscle stretch reflexes, clonus, positive Babinski, spasticity, extensor or flexor spasms, dyssynergic patterns of cocontraction and other associated reactions, and dyssynergic and stereotypical spastic dystonias.

36 Which medication is most likely to cause osteoporosis? a) Atorvastatin b) Enalapril c) Phenytoin d) Metformin

36 Reference(s) (a) Sinaki M. Osteoporosis. In: Braddom RL, editor. Physical medicine and rehabilitation. 4th e Philadelphia: Elsevier Saunders; 2011. p 914. ( b) Saag KG. Osteoporosis. In: Klippe l JH , Stone JH, C ro fford LJ, White PH, editors. Primer on rheumatic disease. 13th e Atlanta (GA): Arthritis Foundation; 2008. p 590. (c) Lee RH, Lyles KW, Colon-Emeric A review of the effect of anticonvulsant medications on bone mineral density and fracture risk. Am J Geriatr Pharmacother 2010 Feb;8(1):34-46. Option c is correct. Anticonvulsants such as phenytoin cause the liver to increase metabolism of 25-hydroxyvitamin D to inactive metabolites, resulting in metabolic bone disease and osteoporosis.Many medications c an c ontribute to bo ne loss. T hese include corticosteroids, thyroxine, anticonvulsants such as phenytoin and carbamazepine, cholestyramine, barbiturates, heparin, warfarin, cyclosporine, aromatase inhibitors, and gonadotropin-releasing hormone agonists.

36. The generation of speech fol lowin g a trach eoesophageal puncture procedure requires(a) use of an electrolarynx.(b) swallow prior to vocaliz atio n.(c) manual tracheostomy occl usio n.(d) insertion of a one way v alve .

36. (c) The generation of speech following trachealesophageal puncture requires that air flow be directed from the trachea into the esophagus and through the oropharyngeal cavity. This can only be achieved if the patient's tracheostomy site is manually occluded, usually with a digit.

37. What work-place situation is the most frequent cause of low back pain in worker s?:(a) Jobs that result in falls at work:(b) Jobs that require standing for more than 4 hours:(c) Jobs that requires lifting and material handling:(d) Jobs that require sitting more than 2 hours

37 (c) Jobs that require lifting and material handling place the worker at increased risk for low back injury. Lifting frequency, load movement, trunk twisting an d tr unk s agi ttal angle predi ct medium and hi gh-r isk occ upational lo w back pain. No r isk co rre lation h as been foun d f or le ngth of tim e sitting or stand in g. Altho ugh f alls in th e w ork plac e p ut the w ork er at risk fo r low back injury, the repetitive motion involved in lifting and material handling is thought to cause low back pain more frequently.

37. Which statement regarding an independent medical exa min at ion (IME) is TRUE? (a) The traditional physician-patient relationship is no t m ai nta ined, and confidentiality is not guaranteed. (b) The examiner is exempt from potential liability sinc e t he pu rpose of the evaluation is to assess medical-legal issues, not clinical issues. (c) Treating providers may conduct an IME as long as rec ord s fro m other providers are also reviewed. (d) Because of potential conflicts of interest, only pro vid er s n o longer in clinical practice should conduct IMEs.

37 Answer: (a) Commentary: In the IME context, a traditional physician- pat ie nt relationship does not exist, since the evaluation does not include "intent to treat." Confidentiality is not guaranteed, since the examiner is expected to share certain medical information and findings with the referring party. Because a "limited doctor-patient relationship" exists during an IME, the physician is responsible for disclosing in the IME any medical findings that could affect the patient's health, and he or she is potentially liable for any harm, direct or indirect, that may be sustained by the person examined. Only a provider who is uninvolved with an examinee's treatment may conduct an IME, although a treating provider may be an "expert witness." Legal requirements for qualification as an expert witness vary from state to state. There is no restriction regarding a provider's clinical status and eligibility to conduct IMEs. Reference: (a) Ky P, Hameed H, Christo PJ. Independent m edi ca l e xaminations: facts and fallacies. Pain Physician 2009;12:811-8. (b) Baum K. Independent medical examinations: an expandi ng so urc e of physician liability. Ann Intern Med 2005;142:974-.8 (c) Andrew LB. Expert witness testimony: the ethics of b ein g a m edical expert witness. Emerg Med Clin N Am 2006;24:715-31.

37. Compared to a younger individual, an older worker who suffers a musculoskeletal injury is more likely to (a) return to work sooner. (b) have a recurrent injury. (c) have the injury treated nonsurgically. (d) sustain a less serious injury.

37 Answer: (b) Commentary: Compared to a younger individual who suffers a musculoskeletal injury, an older individual is more likely to have a recurrent injury, a decreased likelihood of returning to work after the injury, increased time lost from the job as a result of the injury and a more serious injury. Also, an older individual with a spine injury is more likely to have surgery than is a younger individual. Ref: Zuhosky JP, Irwin RW, Sable AW, Sullivan WJ, Panagos A, Foye PM. Industrial medicine and acute musculoskeletal rehabilitation. 7. Acute industrial musculoskeletal injuries in the aging workforce. Arch Phys Med Rehabil 2007;88(3 Suppl):S35.

37 A patient with a left transfemoral amputation demonstrat es a lateral tru nk le an tow ar ds hi s p ros the tic side. Wha t is t he most l ik ely ca use? a. Prosthesis too long b. Long residual limb c. Prosthesis aligned in adduction d. Hip abduction contracture

37 Option d is correct. Causes of lateral trunk lean towards the prosthetic side include prosth esis too sh or t, hi p a bdu cti on c ontrac tur e, pro sth esis a li gned i n ab duct ion, and shor t r esi dua l l imb.

37 What is the biggest barrier for participation in outdoor activities for children with physical disabilities? a) Lack of an adult assistant a. b) Rejection by physically able children b. c) Need for aids such as braces or wheelchairs c. d) Inaccessible playgrounds

37 Option d is correct. Commentary: The environment is a significant barrier to play for children with physical disabilities. Fencing, sand, and inaccessible playground equipmen t rel ega te children with disabilities to an observational role and indicates they are not valued/welcomed. Younger children may see an adult assistant as a playmate but older children see them as intrusive and a hindrance in social situations. Physically able children are willing to include children with disabilities in their games but readily acknowledge limitations of playgrounds in accommodating them. Children often view their braces, walkers and wheelchairs as extensions of themselves and helpful in play situations. Reference: Kaitz E, Miller M. Adaptive sports and recreation. In: Alexander MA, Matthews DJ, editors. Pediatric rehabilitation: principles and practice. 4th e d. New York: Demos Medical; 2010. p 82.

"

37. The interdisciplinary approach to patient care emp hasizes (a) common patient and team goals. (b) discipline-specific goals. (c) concentration on specific clinical problems. (d) tr eatment b y mu ltiple team members." 37. (a) The interdisciplinary approach to patient care emphasizes common patient and team goals rather than discipline-specific goals. The patient and family members should be included in the goal setting process. All team members must work in a collaborative way to facilitate achievement of goals. Team members must have an appreciation for all the issues that affect the patient rather than focusing on an isolated problem. Team communication is essential at all points in the rehabilitation process, not just when problems occur.

38. A patient having difficulty late in the day getting up fr om a chair, going up or down stairs, and reaching with his arms presents for electrodiagnostic studies. Physical exam demonstrates normal deep tendon reflexes and normal findings on manual muscle testing. Standard sensory and motor nerve conduction studies are normal. Repetitive axillary nerve stimulation (RNS) performed at 2Hz demonstrates 20% decremental response. Immediately after exercise, the RNS decrement is no longer observed. Three minutes following exercise, however, the decrement is greater. Needle electromyography results are normal. Upon further investigation, you would most likely find w hat a ddi tional clinical finding? (a) Asthma (b) Dry mouth (c) Ptosis (d) Skin rash

38 Answer (c) Commentary: The patient presents with myasthenia gravis (M G) , a postsynaptic neuromuscular junction disorder. Ptosis and extraocular weakness often occur in MG. Lambert-Eaton myasthenic syndrome (LEMS), a presynaptic neuromuscular junction disorder, would demonstrate postexercise facilitation (at least 100% increase in first response CMAP immediately following exercise) and likely have low-amplitude baseline CMAP results. Autonomic symptoms such as dry mouth often accompany LEMS. Long-term steroid treatment for asthma may cause myopathy without significant needle EMG results, but RNS would be normal. Although dermatomyositis typically presents with proximal weakness, no abnormalities characteristic of an inflammatory myopathy were seen on needle electromyography. Reference: Preston DC, Shapiro BE. Neuromuscular junctio n d is ord ers. In: Preston DC, Shapiro BE, editors. Electromyography and neuromuscular disorders, 2nd ed. Philadelphia: ButterworthHeinemann; 2005. p 553-61.

38 According to the documentation requirements of the Center for Medicare and Medicaid Services (CMS), which factor is not considered part of a patient's Hi story ? a) Chief complaint a. b) Allergies b. c) Review of systems c. d) Social history

38 Option b is correct. Commentary: Evaluation and Management (E/M) coding is the process by which physician-patient encounters are translated into numeric codes to facilitate b illin g. Different E/M codes exist for different types of encounters, such as inpatient or outpatient encounters and new patient or established patient encounters. Documentation for E/M services is based on 3 key components, including (1) history, (2) physical exam and (3) medical decision-making. While a patient's allergy history is an important part of the evaluation, E/M coding does not credit this documentation as part of a patient's history. The history is comprised of the patient's chief complaint, history of present illness, review of systems, and past medical, family, and social history. Reference: Department of Health and Human Services, Center for Medicare and Medicaid Services, Medicare Learning Network fact sheet: Evaluation and manag ement (E /M) services: Complying with documentation requirements https://www.cms.gov/Outreach-and-Education/Medicare- Learning-Network- MLN/MLNEdWebGuide/Downloads/Evaluation_Management_ Fact_Sheet_ICN905363.pdf Accessed 05-08-2012.

38 A 6-month-old boy exhibits hypotonia and generalized weakness. What is the most likely diagnosis? a) Myelomeningocele b) Duchenne muscular dystrophy c) Spinal muscular atrophy d) Neuroblastoma

38 Reference(s) McDonald CM. Electrodiagnosis in pediatrics. In: Alexander MA, Matthews DJ, editors. Pediatric rehabilitation. 4th e New York:Demos Medical; 20 10. p 139. Option c is correct. Spinal muscular atrophy presents within the first 2 months of life with generalized hypotonia and symmetric weakness. Myelomeningocele usually involves weak ness of the legs, but not generalize d hy potoni Duchen ne muscular dystr ophy usually presents after age 2 years with weakness, but attainment of motor milestones is usually normal to mildly delaye Neuroblastoma may present with movement disorder, but not hypotoni The differential diagnosis of hypotonia and weakness in an infant is brain lesion (e.g., severe encephalopathy), spinal cord disorders (e.g., spinal cord injury or spinal muscular atrophy), and myopathy.

39. A 6-year-old girl with Erb palsy since birth has an internal rotation deformity of her right shoulder. Her shoulder external range of motion, whether passive or active, is zero degrees. Right elbow, forearm, wrist, and hand function is good, latissimus dorsi strength is normal. Shoulder MRI shows glenoid dysplasia, but no shoulder dislocation. She writes with her right hand but is unable to reach the back of her head to fix her hair on the right. What is the most appropriate management by the physiatrist at this time? (a) Aggressive stretching of right shoulder internal rotators and strengthening of external rotators (b) Evaluation for compensatory strategies and assistive devices for independence in activities of daily living (c) Consultation with orthopedic surgeon (d) Observation until adolescence for anticipated further improvement

39 Answer: (c) Commentary: Shoulder surgery, most often internal rotation contracture release, often combined with latissimus dorsi tendon transfer to provide active external rotation, is shown to improve shoulder function in children with birth brachial plexus palsy. The majority (60%--70%) of infants with birth brachial plexus palsy recover spontaneously, in the first months of life. Longterm sequelae including fixed contractures warrant intervention. Physical or occupational therapy will be appropriate for this child postoperatively, when greater progress can likely be made. Ref: Strombeck C, Krumlinde-Sundholm L, Remahl S, Sejersen T. Long-term follow-up of children with obstetric brachial plexus palsy I: functional aspects. Dev Med Child Neurol 2007;49:198-203.

39. A 3-year-old child has a hig h tho rac ic spi nal cord injury. When he reaches the age 10 years, which complication is the child most likely to have?(a) Isolated lumbar lordosis(b) Thoracolumbar scoliosis(c) Deep venous thrombosis(d) Heterotopic ossification

39. (b) Scoliosis requiring surgery is a common complication seen in children who have had an spinal cord injury (SCI) at a young age. Increased lordosis in the absence of scoliosis is rarely seen. Deep venous thrombosis rarely occurs in young children and when it does occur it usually occurs soon after the SCI. Heterotopic ossification tends to occur soon after the SCI.

4. The ability to perform aerobic exercise is most related to (a) left ventric ular ejection fraction. (b) cardiac output. (c) respiratory oxygen exchange. (d) Blood partial pressure of oxygen. Clinical Activity 1.5

4 (b) The ability to perform aerobic exercise is related to cardiac output and periph eral utilization of oxygen. Th e other choices may also affect aerobic exercise capa city bu t not as direct ly as cardiac ou tput.

4 Prolonged coma is a significant risk factor for the development of contractures in the traumatic brain injury population. What is the most common site for a contracture to develop in this population? (a) Shoulder (b) Hip (c) Elbow (d) Ankle

4 Answer: (b) Commentary: The overall 1-year incidence was 84% for contracture development in the population of persons with brain injury. The hip was the most common joint affected (81%), followed by the shoulder (76%), ankle (74%) and elbow (44%). Ref: Fergusson D, Hutton B, Drodge A. Epidemiology of major joint contractures: a systematic review. Clin Orthop 2006; 456:22-29.

4 An infarct in the lower division of the left middle cerebral artery division would be associated with which type of aphasia? a) Global a. b) Broca b. c) Transcortical sensory c. d) Wernicke

4 CORRECT: Commentary: An infarct in the area of the middle cerebral artery lower division is much less common than an upper division stroke of the middle cerebral arter y. It is usually caused by an embolic event. If the stroke is in the dominant hemisphere it will demonstrate a Wernicke aphasia; if it is in the nondominant hemisphere an affective agnosia is seen. A contralateral homonymous hemianopsia is also caused by a stroke in this area. Classically, global aphasia is associated with an infarction of the middle cerebral artery main stem, and Broca's aphasia with infarction of upper division of the middle cerebral artery. Transcortical sensory aphasia typically is associated with a posterior c erebr al artery infarction. Reference: (a) Harvey RL, Roth EJ, Yu D. Rehabilitation in stroke syndromes. In: Braddom RL, editor. Physical medicine and rehabilitation. 3rd ed. Philad elphi a: Elsevier; 2007. p. 1181-1182. (b) Harvey RL, Roth EJ, Yu D. Stroke syndromes. In: Braddom RL, editor. Physical medicine and rehabilitation. 4th ed. Philadelphia: Saunders; 2011. p 1185-118

42. A 43-year-old man with a his tory of insuli n dependent diabetes mellitus, gastroparesis, hypertension, and obesity had a right cortical ischemic infarct 7 days ago. The nurses note that he is having frequent small urinary voids with a weak voiding stream. What bladder mechanism is most characteristic for this presentation?(a) Small volume bladder wit h sp hinct er flacci dity(b) Spastic detrusor activit y wi th no rma l sphi ncter(c) Flaccid detrusor with la rge volum e b ladder(d) Hyperactive detrusor wit h la rge v olu me bla dder

42. (c) Although the most common bladder among patients with stroke is normal or hyperreflexic, bladder hyporeflexia is very common in diabetics (especially in this case with recorded gastroparesis). These patients will have small frequent voids due to overflow from distended bladders with poor detrusor contraction.

40 What is the primary disadvantage of moving the rear axle of a wheelchair forward? (a) Ascending curbs becomes more difficult. (b) It takes more muscle effort to propel the wheelchair. (c) More strokes are required to push the wheelchair. (d) Ascending a ramp becomes more difficult.

40 Answer: D Commentary: Moving a wheelchair's rear axle forward enables the user to propel the chair with less muscle effort and fewer strokes. Because the modification causes more weight to be centered over the rear wheels, it is easier to pop a wheelie, negotiate obstacles and ascend or descend curbs. However, moving the axle forward can also make the wheelchair more "tippy" (likely to tip backwards) and that tendency to tip backwards makes it more difficult to push the chair up a ramp. Ref:Koontz AM, Spaeth DM, Sichmeler MR, Cooper RA. Prescription of wheelchairs and seating systems. In: Braddom RL. Physical medicine and rehabilitation. Philidelphia: Elsevier; 2007. P 381-411.

40 While performing an electrodiagnostic study for suspecte d carpal tunnel syndr ome , y ou n oti ce tha t t he m edian, ul na r, a nd radi al s ensory ner ve c onduc tio n stu die s d emo nst rate prolonged peak latencies with increased sensory nerve action potential (SNAP) amplitudes and durations. What should be evaluated next? a. Additional nerve conduction studies of the lower extr emity b. F responses on the median and ulnar nerves c. Limb temperature d. Needle electromyography (EMG)

40 Option c is correct. Prolonged peak (or distal) latency with increased SNAP ( or CMAP) amplitu des m ay be an i ndi cat or of cool limb tem pe ratu res . Th is s hould not be c onfus ed with a d emy eli nat ing process affecting the peripheral nervous system. The effects of a cool limb include slowed nerve conduction velocity, prolonged distal and peak latencies, increased amplitude and duration of SNAPs and CMAPs, and increased duration, amplitude, and phases of motor unit action potentials (MUAP). Distal limb temperatures should be recorded and ideally maintained between 33 and 34 degrees Celsius. Additional nerve conduction studies of the lower limb, F responses of the upper limb, or needle electromyography (EMG) would not be the appropriate first intervention; however, these studies may be necessary if abnormalities exist even after temperature correction and if true peripheral nerve pathology is suspected.

40 Which patient is most likely to be treated conservatively? a) A young man who fell from roof and is experiencing bladder incontinence, numbness and tingling in the sacral distribution b) A woman with T12 compression fracture following a motor vehicle collision c) A person with chronic low back pain, grade 3 spondylolisthesis and neurological symptoms d) A 40-year-old woman with progressive foot drop and herniated disc L4-L5

40 Reference(s) Barr KP, Harrast M Low back pain. In Braddom RL. Physical medicine and rehabilitation, 4th e Philadelphia: Elsevier; 2011. p 871-912. Option b is correct. Compression fractures are most often treated conservatively. The man who fell from the roof has cauda equina syndrome, an absolute surgical indication. Prog ressive motor loss, such as the woma n wi th the hernia te d disc, is also an absolute indication for surgical intervention. Relative indications for surgery include intractable pain and static motor loss, and surgery is often considered in these cases following failed conservative treatments. A grade 3 spondylolisthesis may be treated conservatively when no neurological symptoms are present. Although most discogenic low back pain can be treated conservatively, progressive neurological deficits are treated surgically.

40. A patient is referred to you r off ice by hi s primary care physician for evaluation of an unusual gait pattern caused by a remote case of polio. You note excessive lateral trunk flexion to the left during stance phase between foot flat and heel off. Swing phase is normal. On exam there is normal hip flexor strength bilaterally. The gluteus medius is 4-/5 on the left and 5/5 on the right. Knee strength is normal. The tibialis anterior is 4-/5 on the right and 5-/5 on the left. Range of motion is normal at all joints. Which gait abnormality is occurring?(a) Waddling gait(b) Steppage gait(c) Trendelenberg gait(d) Circumducted gait

40. (c) A waddling gait occurs when there is bilateral gluteus medius weakness. A steppage gait occurs as an abnormality in swing phase due to severely weak dorsiflexors of the ankle. Foot slap is seen with moderately weak dorsiflexors and occurs on the side of weakness. Trendelenberg gait is excessive lateral flexion due to ipsilateral weakness. Circumduction is the swinging of the limb in a wide lateral arc.

"

40. The most common reason for prescribing a pl as tic leaf-spring ankle-foot orthosis is to (a) overcome ankle spasticity. (b) reduce lower-extremity edema. (c) prevent plantar flexion deformity. (d) support weak ankle dorsiflexors." 40 (d) A plastic leaf-spring orthosis (PLSO) is probably the most commonly prescribed type of ankle-foot orthosis (AFO). It substitutes for weak ankle dorsiflexors and provides some medial lateral stability. Severe spasticity of the ankle may require prescription of a solid AFO. A plastic spiral AFO may be prescribed for concomitant weakness of both the ankle dorsiflexors and plantar flexors when spasticity is absent. Ref: Ragnarsson KT. Low extremity orthotic shoes and gait aids. In: DeLisa JA, Gans BM, Walsh NE, editors. Physical medicine and rehabilitation: principles and practice. 4th ed. Lippincott Williams & Wilkins. 2005. p 1383-5.

41 Which phenomenon occurs when an axon is depolarized? a. The intracellular region becomes more negatively char ged. b. Sodium channels open, allowing sodium to rush out of the axon. c. Large anions and potassium permeate into the axon. d. Sodium movement is driven by electrical and concentra tion gradients.

41 Option d is correct. The resting potential is about -90 millivolts. With depo larization an in flux of sod iu m occ urs , d riv en b y both an e lect ric al a nd c oncent rati on g radie nt. This so diu m i nfl ux r esults in the axon becoming more positive internally. The axon membrane is impermeable to large negatively charged anions and relatively impermeable to sodium ions in the resting state. The concentration of sodium is higher outside the membrane, whereas the potassium concentration is higher inside.

42. Which statement concerning the use of prophylactic antiepileptics in the management of patients with traumatic brain injury is TRUE? (a) They decrease the functional disability of the injury. (b) They reduce the occurrence of late seizures. (c) They reduce the incidence of death. (d) They reduce the occurrence of early seizures.

42 Answer: (d) Commentary: There is no evidence that prophylactic antiepileptic medications, used at any time after head injury, reduce death and disability. Evidence exists that prophylactic antiepileptics reduce early seizures, but there is no clinical evidence that late seizures are reduced, or that treatment has any effect on death or neurological disability. Ref: (a) Schierhout G, Roberts IG. Anti-epileptic drugs for preventing seizures following acute traumatic brain injury. Cochrane Database Syst Rev. 2001;(4):CD000173. doi:10.1002/14651858.CD000173. (b) Yablon SA, Dostrow VG. Post-traumatic seizures and epilepsy. In: Zasler ND, Katz DI, Zafonte RD, editors. Brain injury medicine: principles and practice. New York: Demos Medical Publishing; 2007. p 452-5.

42 Which electrodiagnostic finding would be most helpful in differentiating crit ica l i ll ne ss myo pat hy (CIM ) from cr it ical il lnes s po lyneur opat hy ( CIP)? a. Normal compound muscle action potential (CMAP) amplit udes b. Abnormal spontaneous activity c. Normal sensory nerve action potential (SNAP) amplitud es d. Decreased CMAP amplitudes

42 Option c is correct. Critical illness myopathy (CIM) presents as weakness in a patient in the inte nsi ve ca re un it (IC U). Cli nicall y i t may be diff ic ul t to d isti ngui sh CI M f rom C IP. A CIM pa tter n would have normal SNAP amplitudes, whereas CIP will show abnormalities on sensory nerve conduction studies consistent with an axonal loss pattern. Both CIM and CIP demonstrate decreased CMAPs and both may have abnormal spontaneous activity on needle examination.

42 Which statement is most correct regarding instrumentation for electrodiagnostic studies? a) Signals pass from the filters to the amplifiers. b) Cathode-ray tube subtracts common electrical activity. c) Electrical signals are transmitted from 2 surface electrodes. d) Filters are used to increase the electrical noise.

42 Reference(s) Malhotra G. Instrumentation. In: Visco C, Chimes G, editors. McLean course in electrodiagnostic medicine. New York; Demos Medical; 2011. p 9-11 . Electrical signals are transmitted from skin surface electrodes or through a needle. These electrodes are referred to as E1 and E2 or the active and the ref erence. Differential amplifiers elim inat e signal comm on to the 2 elect rodes. Filters are used to remove excessive electrical noise after signal amplification. Cathode-ray tube permits the visual display of the electrical signal as a waveform.

43. A 40-year-old woman with a history of irritable bowel syndrome and tension head ache s com pla ins of i ncre asi ng fa tig ue and diff use m uscl e s or eness in her neck, shoulders, and low back. She has a nonfocal neurologic examination. Initial recommendations should include:(a) craniosacral manipulation.:(b) closed kinetic chain exercises to strengthen the shoulder girdle.:(c) electrical stimulation to the upper and mid back. (d) walking on a daily basis.

43 (d) Promoting a restorative sleep and 20-30 minutes of aerobic activity daily is recommended to treat fibromyalgia. Passive modalities are not the optimal t her apeu tic i nte rvent ion.

43. A 40-year-old woman reports left-sided facial pain for the past month along with difficulty in moving her jaw. She hears a clicking noise with chewing along with constant tinnitus. Upon examination, she has tenderness to palpation along her muscles of mastication on the left with deviation of the mandible upon jaw opening. She would like to have pain relief. You suggest (a) referral to an oral surgeon. (b) that she perform jaw isometric exercises in a closed position with massage. (c) a 2-week trial of an oral nonsteroidal anti-inflammatory medication. (d) an ultrasound-guided intra-articular injection with steroids.

43 Answer: (c) Commentary: This woman has a temporomandibular joint (TMJ) disorder most likely myofascial in origin, which is the most common etiology. It is usually self-limited, and is managed conservatively with relative rest (eg, avoiding jaw clenching, gum chewing), heat, and nonsteroidal anti-inflammatory agents. Intra-articular steroid injections are not needed with a myofascial origin of pain. She also does not need a referral to an oral surgeon at this time. Ref: Scrivani SJ, Keith DA, Kaban LB. Temporomandibular disorders. N Engl J Med 2008;359:2693-705.

43 A 59-year-old man had a stroke 6 months ago and has resi dual expressive aphas ia and w ea kne ss of the rig ht upp er li mb w ith onl y sl ight v olun tary cont rac tions at th e s hou lder and elbow. He would like to try constraint-induced movement therapy (CIMT). You inform him that you do not think CIMT will help him because a. CIMT is only effective if initiated within the first 3 months after a stro ke. b. his aphasia prohibits him from effectively understand ing the treatmen t pro toc ol. c. he does not have adequate motor control in the right upper limb to pa rtici pat e. d. the medical literature has shown poorer outcomes with CIMT than conve ntion al tre at me nt.

43 Option c is correct. Constraint-induced movement therapy (CIMT) is a relative ly recently deve loped ty pe of t her apy ba sed on the cu rre nt con cep ts o f ne uropla stic ity. Pati ent s mus t h ave ad equ ate proximal limb control and at least partial wrist and finger extension, as well as sufficient balance during limb restraint. There is emerging support for the effectiveness of CIMT in acute, subacute, and chronic phases of stroke recovery, and outcomes have compared favorably to conventional treatment. Expressive aphasia would not be prohibitive of participation in a CIMT protocol. Logistic and cost considerations have limited the availability of CIMT, as has the candidacy of patients with only mild to moderate unilateral limb paresis.

43 Option d is correct. Distal symmetric sensory or sensorimotor axonopathies are the most common neuropathy associated with cryoglobulinemia and may be the first or only clinical finding in patients. Mononeuropathy multiplex is less common and demyelinating neuropathies are the least common neuropathy in these patients.

43 What is the most frequent peripheral neuropathy in cryoglobulinemia? a) Mononeuropathy b) Mononeuropathy multiplex c) Demyelinating neuropathy d) Distal symmetric sensorimotor axonopathy Reference(s) Rosenbaum R. Neuromuscular complications of connective tissue disease. Muscle Nerve 2001;24: 154-69.

43. The most significant risk factor for conti nue d structural destruction of the knee in osteoarthritis is (a) joint malalignment. (b) obesity. (c) prior surgery. (d) occupational bending and lifting.

43. (a) Joint malalignment is the most significant risk factor for further joint deterioration, since it creates uneven focal loading. Ref: Felson DT. Osteoarthritis of the knee. N Engl J Med 2006;354:841-8.

43. What is the most likely diagnosis in a 45-year-old man with a ra sh over t he upper and l owe r eyelids who complains of insidious onset of difficulty getting out of a chair and ascending stairs? (a) Inclusion body myositis (b) Ankylosing spondylitis (c) Dermatomyositis (d) Guillain-Barré syndrome

43. (c) Individuals with dermatomyositis commonly present with slowly progressive proximal weakness. A heliotrope rash affecting the eyelids occurs in 30% to 60 % of patients with dermatomyositis. Inclusion body myositis primarily affects upper extremities distally and occurs in the older population. Ankylosing spondylitis and Guillain-barré syndrome would not present in the fashion described.

44. Regarding the epidemiology of neurogenic thoracic outlet syndrome, (a) it is a commonly occurring syndrome. (b) it occurs equally in men and women. (c) it occurs more frequently in the young and middle-aged. ssociate d with r epeti tiv e motio n. (d) it is highly associated with repetitive motion

44 (c) Neurogenic thoracic outlet syndrome is rare, occurs most fre quently in young to middle-aged women, and involves the lower trunk of the brachial plexus. Pain is t he most common sensory symptom, and is usually in the medial forearm and u ln ar asp ect of th e hand.

44. Which electrodiagnostic feature is more common in type 2 Charcot-Marie-Tooth (CMT) disease than in CMT type1? (a) It is primarly demyelinating. (b) Lower limbs are more affected than upper. (c) Conduction velocity slows. (d) Secondary axonal changes occur over time.

44 Answer: (b) Commentary: The type-2 form of Charcot-Marie-Tooth disease (CMT2) tends to affect the lower extremities more than the upper extremities. In CMT type 1 (CMT1), which is primarily a demyelinating neuropathy, anatomic changes directly affect the myelin sheath, with secondary axonal changes. In areas of focal demyelination, impulse conduction from 1 node of Ranvier to the next is slowed, because current leakage occurs and the time for impulses to reach threshold at successive nodes of Ranvier is prolonged. The prolongation slows conduction velocity along the nerve segment. CMT2 is often a clinically less severe disease than CMT1. Ref: Thomas MA, Felsenthal G, Fast A, Young M. Peripheral neuropathy. In: DeLisa JA, Gans BM, Walsh NE, editors. Physical medicine and rehabilitation: principles and practice. 4th ed. Philadelphia: Lippincott Williams & Wilkins; 2005. p 898.

44 You are evaluating a patient who may require a transtibi al amputation. W hich fac tor w ou ld be ass oci ated with poo r post sur gica l he aling? a. Ankle-brachial index higher than 0.90 b. Monophasic arterial Doppler ultrasound measurement c. Warm limb temperature d. Capillary refill time of 1-2 seconds

44 Option b is correct. Blood flow plays a critical role in healing of the resid ual limb. An ank le-br ach ial i nd ex (AB I) gre ater than 0.9 i s no rma l an d va lues l ess than 0.5 are asso cia ted wi th woun ds that are unlikely to heal. Normal Doppler ultrasound should reveal triphasic waveforms; monophasic recordings indicate impaired blood flow and would predict poor healing. Cold limb temperature (not warm) would be more associated with poor healing. Capillary refill time of 2 seconds or less is considered normal.

41 What is the strongest single predictor of mortality in adults with pediatric onset disabilities? a) Feeding problems a. b) Presence of epilepsy b. c) Inability to walk c. d) Intellectual disability

44 Option d is correct. Commentary: Feeding problems, epilepsy and inability to walk are conditions associated with pediatric mortality, but intellectual disability is the stron gest sin gle predictor of mortality in adults with pediatric onset disabilities. Intellectual disability affects a person's ability to manage health care monitoring, exercise programs, nutrition, housing, and sexuality. These items are likely to b e mor e c losely monitored by parents and pediatricians for a child than by community workers and adult physicians caring for adults with pediatric onset disabilities. Reference: (a) Turk, MA. Health, mortality, and wellness issues in adults with cerebral palsy. Develop Med Child Neurol 2009;51:25.(b) Turk MA, Logan LR, Kant er D. Aging with pediatric onset disability and diseases In: Alexander MA,Matthews DJ, editors. Pediatric rehabilitation: principles and practice. 4th ed. New York: Demos Medical; 2010. p 434, 446-447.

44 A 25 year old patient presents for electrodiagnostic testing after sustaining a right shoulder injury following a motorcycle crash 4 weeks ago. He has no ac tive elbow, wrist or finger extensio n. T he needle exa mi nation of the r adial innervated muscles shows normal insertional activity, no abnormal spontaneous activity and absent motor recruitment. In the Sunderland classification system, this nerve injury is best described as? a) myelin injury with no Wallerian degeneration b) Wallerian degeneration with preservation of endoneurium c) disruption of axon with intact perineurium d) complete discontinuity of the nerve

44 Reference(s) (1)Dumitru D, Zwarts MJ, Amato A Peripheral nervous system's reaction to injury. In: Dumitru D, Amato AA, Zwarts MJ, editors. Electrodiagnostic medicine. 2nd e Philadelphia: Hanley & Belfus; 2002. p 124-5. (2)Dumitru D, Zwarts MJ. Needle electromyography. In: Dumitru D, Amato AA, Zwarts MJ, editors. Electrodiagnostic medicine. 2nd e Philadelphia: Hanley & Belfus; 2002. p 275-6. Option a is correct. The injury described is limited to the myelin with preservation of the axon. This type of injury is classified as neurapraxia by the Seddon classification a nd as type 1 by the Sunderland class ific ation. In axo na l injury, one w ould typically observe positive sharp waves and fibrillation potential within 3 weeks after the injury. Sunderland type 2 and 3 are both forms of axonal injury and differ in their extent of injury to the supporting neural structures. Sunderland type 2 and 3 are classified as axonotmesis by the Seddon classification. Complete discontinuity of the nerve is classified as type 5 Sunderland and as neurotmesis by Seddon.

45. According to the most recent data from the National Spinal Cord Injury Statistical Center and Model Spinal Cord Injury Systems, which source of trauma is the leading cause of traumatic spinal cord injury among individuals between the ages of 46 and 60 years? (a) Motor vehicle accidents (b) Acts of violence (c) Sports-related injuries (d) Falls

45 Answer: (d) Commentary: Falls comprise the leading cause of traumatic spinal cord injury in the 46- to 60year-old age group, while motor vehicle crashes are the most common etiology for traumatic spinal cord injury among people younger than age 46. Incidence rates for acts of violence and sports-related injuries are lower in the 46-60 age group than in younger age groups. Ref: DeViVo MJ. Epidemiology of traumatic spinal cord injury. In: Kirshblum S, Campagnolo DI, DeLisa JA, editors. Spinal cord medicine. Philadelphia: Lippincott Williams & Wilkins; 2002. p 71-3.

45 Which pharmacologic and non-pharmacologic treatment combination is the most appropriate initial program in a patient with fibromyalgia? a) Duloxetine (Cymbalta) plus aerobic exercise b) Amitriptyline (Elavil) plus high intensity strength training c) Diazepam (Valium) plus trigger point injections d) Fentanyl (Duragesic) plus cognitive behavioral therapy

45 Option a is correct. Commentary: Pharmacologic treatments used for fibromyalgia include tricyclic antidepressants (e.g., amitriptyline), serotoninnorepinephrine reuptake inhi bitor s ( SNRIs, e.g., duloxetine, venlafaxine), and some anticonvulsants such as pregabalin. Opiates (e.g., fentanyl) and benzodiazepines (e.g., diazepam) are generally not recommended. Non-pharmacologic therapies include cognitive behavioral therapy, aerobic exercise (low impact), and complementary therapies. To reduce the pain associat ed wi th exercise, it is recommended to "start low, go slow," with gradual progression in exercise intensity. Patients with fibromyalgia would not likely comply with a high intensity strength training program. Reference: (a) Dadabhoy D, Clauw DJ. Musculoskeletal signs and symptoms. E. The fibromyalgia syndrome. In: Klippel JH, Stone JH, Crofford LJ, White PH, e ditor s. Primer on the rheumatic diseases. 13th ed. New York: Springer Science & Business Media; 2008. p 92. (b) Walsh NE, Dumitru D, Schoenfeld LS, Ramamurthy S. Treatment of the patient with chronic pain. In: DeLisa JA, Gans BM, Walsh NE. Physical medicine and rehabilitation: principles and practice. 4th ed. Philadelphia: Lippincott Williams & Wilkins; 2005. p 514-7.

45 A 22-year-old man with a history of epilepsy and a 30-pa ck-year history of to bac co sm ok ing co mes to see you i n c li nic. He is in te rested in quit ting smo king. Wh ich st rat egy is the best approach to maximize this patient's success with smoking cessation? a. Bupropion (Wellbutrin) b. Varenicline (Chantix) and counseling c. Transdermal patch for nicotine replacement d. Transdermal patch with inhaled nicotine replacement

45 Option b is correct. Varenicline (Chantix), bupropion (Wellbutrin), and combi nation nicotine repla cem ent ( eg , t ran sde rma l pa tch wi th ni coti ne inha le r) are a ll f irst line tr eatme nts to he lp with smoking cessation. Counseling for smoking cessation is recommended for all interventions to optimize treatment efficacy, so first-line pharmacotherapy plus counseling would optimize the patient's chances of quitting smoking over any pharmacotherapy option alone. Choice (a), bupropion (Wellbutrin), is not an appropriate choice for this patient, since seizure disorder is a contraindication. Option (b) is the best choice for this patient from the options provided because it includes first-line pharmacotherapy that is medically acceptable with this patient's medical history and it includes counseling. Choice (c) is incorrect, since single formulation nicotine replacement (ie, patch alone) has a much lower success rate for maintaining abstinence at 6 months compared with the other interventions. Option (d) is also a first-line pharmacotherapy option for smoking cessation but choice (b) is a better option because it includes counseling. Because both varenicline (Chantix) and bupropion (Wellbu trin) have an FD A bla ck box w ar nin g f or neu rops ychiat ric e vent s, clin ic al monit orin g is indi cat ed fo r a ll pat ien ts t aking those medications. Bupropion (Wellbutrin), an antidepressant, also has an FDA black box warning regarding an increased risk of suicidal thinking and behavior in children, adolescents and young adults taking antidepressants. Recent reviews (Cahill, 2014) indicate that the risk of serious adverse events is not increased in any of the smoking cessation treatment options listed here. Tobacco use is an important modifiable risk factor in ma ny disease proce sses, in clu di ng th ose of ca rdio vascul ar di seas e a nd s tr ok e. Smo king ces satio n i s imp ort ant , s inc e sm oking has a dose dependent effect the relative risk of stroke for heavy smokers is twice that of light smokers. Further, the additional health risks from smoking can normalize to baseline for some conditions within 5 years of quitting.

45 Impairments resulting from chronic disease have become increasingly significant risk factors of disability. Which medical condition has the highest prevalen ce of activity limitation? a) Diabetes b) Heart disease c) Mental disorders d) Visual impairments

45 Reference(s) Klingbeil H, Sperber K. Employment of persons with disabilities. In: Braddom RL, editor. Physical medicine and rehabilitation. 4th e Philadelphia: Saunders Elsevier; 2011. p 756. Option b is correct. From highest to lowest, the prevalence of activity limitation for the above conditions are heart disease (22.5%), visual impairments (4.4%), mental disorder s (3.9%), and diabetes (2.7%). Orthopedic impairments (16%) and arthritis (12.3%) also carry significant risks for the development of activity limitation and disability. Diabetes, mental disorde rs a nd visual imp ai rments are less likely to cause activity limitation or disability, when one factors in both the prevalence of the condition and the likelihood that the condition will cause decreased function.

45. You are called to the ne urol ogy i nte nsive care unit to evaluate a patient with new spinal cord injury; you determine that the patient has sustained a C7 ASIA A spinal cord injury. Which change in the respiratory system would be expected?(a) Residual volume will dec line to 3 0 of pre dicted value.(b) Pulmonary function will not impro ve after the first 2 weeks postinjury.(c) Expiratory reserve volum e in creas es 40% 6 weeks postinjury.(d) Vital capacity of 60% p redi cted val ue may be obtained within the first 6 months post-injury.

45. (d) Tetraplegic patients usually have a reduction in all measures of pulmonary function with the exception of residual volume. Residual volume is increased due to lack of active expiratory effort. Vital capacity will continue to improve. Tracheostomy is usually not necessary for pulmonary hygiene, especially with adequate hydration and techniques for facilitating cough. Since the diaphragm is supplied by cervical roots C3, C4, and C5, it is common for persons injured above the C4 level to need ventilator support. In acute spinal cord injury, 67% experience significant pulmonary complications, most commonly atelectasis. Ventilatory failure and aspiration occur the earliest (mean, 4.5 days), followed by atelectasis (mean, 17 days) and pneumonia (mean, 24 days). The late decline coincides with the onset of mucus hypersecretion and muscle fatigue. Ventilator weaning has been demonstrated in 80% of C4 spinal cord injury patients and 57% of C3 patients. Considerable patience is required and respiratory muscle fatigue must be closely monitored.

46. A patient with squamous cell carcinoma of the larynx elected to undergo organ p rese rv ati on the rapy with in tensi ve ext ernal be am ra diat ion t herapy to the anterior neck. He now presents with painless bilateral lower extremity weakness. Which initial diagnostic test is most likely to be abnormal?:(a) Magnetic resonance imaging of the brain:(b) Nerve conduction studies of the lower extremities:(c) Thyroid stimulating hormone level test:(d) Computed tomography scan of the abdomen

46 (c) Patients who receive external beam radiation therapy to the anterior neck are at risk for developing hypothyroidism. If their thyroid function tests are no t mo nitor ed, they may initia ll y presen t w ith sig ns a nd symptoms of hypothyroidis m. M yopath y i s a comm on presentin g c om pla int. This p at ient is also at risk for radiation induced cervical myelopathy. However, since radiation hypofractionation techniques have become the standard of care, the i nci denc e of thi s dre aded compli ca tion has si gni fica ntly di minished.

47 Which clinical characteristic is associated with inclusi on body myositis ? a. Female predominance b. Improvement with corticosteroids c. Onset in middle-aged or older population d. Markedly elevated serum creatine kinase (CK) levels

47 Option c is correct. Inclusion body myositis (IBM) is a slowly progressive my opathy that tend s to aff ect m id dle ag ed and old er ind ivi du als. Cl inic al m anifes tati ons can i ncl ude d ist al as wel l as proximal weakness, which can be asymmetrical. Muscle enzyme levels may be mildly elevated. There is a 31 male predominance. There is generally no significant improvement with corticosteroids.

46 Following a crush injury with axonotmesis, the approximate growth regeneration rate at the wrist is 1 centimeter per a) day. a. b) week. b. c) month. c. d) year.

46 Option b is correct. Commentary: Regenerating axons grow approximately 1 millimeter a day, 1 centimeter a week, 1 inch a month, or 1 foot a year. The rate of axon regeneratio n dep end s chiefly on type of injury (crush or laceration) and whether the lesion is proximal or distal. Growth rate following a crush injury with axonotmesis in the upper arm is about 8 millimeters a day; in the upper forearm it is about 6 millimeters a day, at the wrist about 1-2 millimeters a day, and in the hand about 1.0-1.5 millimeters a day. Easier figures to remember, however, are 1mm/day, 1cm/week, or 1 inch/month. Reference: (a) Stewart, JD. Focal peripheral neuropathies. 3rd ed. Montreal (Quebec):McGill Univ; 2000. p 25. (b) Ref: Dumitru D, Zwarts MJ, Amato A, Peripheral nervous system's reaction to injury: electrodiagnostic medicine. 2nd ed. Philadelphia: Hanley & Belfus; 2002. p -120 1

46 A 55-year-old woman with a subarachnoid hemorrhage and aneurysm clipping presents with mental status decline. Her sodium level is 121 m illi equi valen ts per l ite r. Her bl ood urea nitrogen (BUN) is elevated. On examination she appears mildly dehydrated, and she has lost 5 pounds. The best treatment option is to a. institute a fluid restriction program. b. improve hydration by increasing free water intake. c. give intravenous fluid with normal saline (0.9%) and oral saline (NaC l). d. prescribe nasal desmopressin (DDAVP).

46 Option c is correct. Cerebral salt wasting is believed to be caused by renal salt wasting and is a sso cia te d wit h h ypo nat remi a, wei ght l oss, an d si gn s of deh ydra tion . It is best man age d w ith hyd ration and sodium replacement. Intravenous hydration with normal saline (0.9% NaCl), hypertonic saline (3% NaCl), or oral salt may be used alone or in combination, depending on the severity and the patient's ability to tolerate enteral feeding. Fluid restriction is used for syndrome of inappropriate antidiuretic hormone secretion (SIADH). SIADH is not associated with dehydration and serum osmolality is commonly decreased. Answers a, b and d are wrong because they would worsen the hyponatremia. Inappropriate fluid restriction may result in volume depletion and potential cerebral ischemia in persons with brain injury.

46 You see a 50-year-old man in your clinic in follow-up for a stroke 6 months after he was discharged from your inpatient rehabilitation service. He made good functional gains during his initial reh abilitation, bu t recent f uncti onal gains are slower despite intense outpatient rehabilitation effort. Early functional gains are most likely due to a) structural and functional brain reorganization of partially damaged pathways and expansion of the representational brain maps. b) neuroplasticity and recruitment of neurons not normally involved in an activity. c) early rehabilitation efforts emphasizing forced use of the hemiplegic arm and leg. d) the resolution of ischemia, metabolic injury, edema, hemorrhage, and pressure in the ischemic penumbr

46 Reference(s) Stein J, Brandstater ME. Stroke rehabilitation. In: Frontera WR, editor. Physical medicine and rehabilitation: principles and practice. 5th e P hiladelphia: Lippincott Williams & W ilki ns; 2010. p 5 61 . Option d is correct. In the early phase of stroke recovery the resolution of the ischemic insult and sequelae of secondary injury explain the early and rapid recovery. The time frame of recovery in the area of rev ersi bly injured n eu rons is re lativ ely short and accounts for the improvement in the first several weeks.

46. One year has elapsed sin ce a 56-y ear =-old p atient received aggressive treatment for high grade ovarian cancer. She now presents with a 2-week history of progressive unilateral lower extremity swelling and weakness, as well as urinary incontinence. The most appropriate initial diagnostic test would be(a) electromyogram.(b) urodynamic studies.(c) pelvic CAT scan.(d) venogram.

46. (c) Female gynecologic malignancies tend to recur locally within the pelvis. This patient likely has a lumbosacral plexopathy due to compression by the tumor. An electromyogram could potentially identify the portions of the plexus involved, however spontaneous activity would not yet have developed. Computed tomography of the abdomen and pelvis would determine whether and where recurrent tumor was present. This would inform surgical or radiation oncologic treatment options.

47 Which treatment is shown consistently to improve pain in patients with acute low back pain? (a) Superficial heat (b) Traction (c) Transcutaneous electrical nerve stimulation (TENS) (d) Ultrasonography

47 Answer: (a) Commentary: Superficial heat is the only modality listed that has consistently decreased pain in acute low back pain, which is pain that has been present for less than 4 weeks. Ref: Chou R, and Huffman LH. Nonpharmacologic therapies for acute and chronic low back pain: a review of the evidence for an American Pain Society/American College of Physicians Clinical Practice Guideline. Ann Intern Med 2007;147:492-504.

47. Which abbreviation or symbol is acceptable to use when writing prescriptions, according to The Joint Commission (JC)? (a) QD for once daily (b) U for units (c) ml for milliliters (d) cc for milliliters

47 Answer: C Commentary: Of the options given the Joint Commission has only approved the use of the abbreviation ml for milliliters. Using the abbreviation QD can be dangerous since it may be mistaken for QID, which stands for four times per day. Ref:Official "Do Not Use" abbreviations list. Joint Commission Resources Website. http://www.jointcommission.org/NR/rdonlyres/2329F8F5-6EC5-4E21-B93254B2B7D53F00/0/06_dnu_list.pdf. Accessed May 25, 2008.

47 The definitive diagnostic test for inclusion body myositis is a) muscle enzyme serum levels. a. b) cerebrospinal fluid. b. c) muscle biopsy. c. d) electromyography.

47 Option c is correct. Commentary: Inclusion body myositis (IBM) is a slowly progressive myopathy that tends to affect middle-aged and older individuals. Clinical manifestations can include distal as well as prox imal wea kness, which can be asymmetrical. Muscle enzyme levels may be slightly elevated or normal. Myopathic motor units can be seen, although this finding is nonspecific, since it exists in other inflammatory myopathies, as well. Acquisition of cerebrospinal fluid is not part of the work-up for myopathy. Muscle biopsy is diagnostic with rimmed cytoplasmic vacuoles and cytoplasmic and nuclear inclusions. Reference: Kegan L. Inclusion body myositis. In: Kagen L, editor. The inflammatory myopathies. New York: Humana Press;2009. p 279-294.

47 Overuse syndromes resulting in tendinopathies are ideally treated differently in the progressive phases of recovery. Treatments for tendinopathies may appro priately be initiated in the followi ng o rder: a) relative rest, stretching, muscle activation, strengthening. b) massage, relative rest, strengthening, muscle activation. c) aggressive stretching, relative rest, muscle activation, strengthening. d) ice, strengthening, muscle activation, stretching.

47 Reference(s) Corrado G, d'Hemecourt P Thigh injuries. In: Frontera WR, Herring SA, Micheli LJ, et al, editors. Clinical sports medicine: medical management and rehabilitation. Philadelphia: El sevi er; 2007. p 4 16 Option a is correct. The first goal is to reduce inflammation and irritation. Rest, nonsteroidal anti-inflammatory agents, and ice each help accomplish that first goal. Aggressi ve stretching of sore muscles does n ot a ccomplish the f irst goal of re ducing inflammation and irritation. Strengthening should not begin until pain is controlled and stretching is tolerated without pain. Muscle activation refers to techniques designed to elicit specific muscles within a functional muscular complex that may not be efficiently contracting in response to neural excitation.

47. A 29-year-old painter pr esen ts to yo u with 2 days of knee pain and swelling after falling off a ladder at work. The swelling began immediately after the fall. His neurologic examination is normal, and peripheral pulses are normal at the knee and ankle. He is unable to fully extend or flex the knee because of pain and swelling. He is ambulating with an antalgic gait limp. Your recommendations include ice and(a) knee immobilizer, crutch es, x-ray s, return to sedentary work, recheck in 5 days.(b) crutches, magnetic reson ance imag ing , refe rral to an orthopaedic surgeon.(c) narcotics, physical ther apy, rech eck in 3 weeks.(d) nonsteroidal anti-inflam mato ry dr ugs , x-ra ys, return to work.

47. (a) Appropriate management for acute knee injuries include, ice, elevation, non-steroidal antiinflammatory drugs, protection, weight bearing as tolerated and activity modification. X-rays initially rule out bony injury. magnetic resonance imaging should be reserved for cases where the diagnosis is in question or a surgical procedure is planned. A careful examination to fully exclude ligament or cartilage injury cannot be completed until the effusion has resolved enough to allow for an appropriate examination. Therefore, in the case of an acute knee injury with effusion, the patient should be reexamined within a 1- to 2-week interval in order to narrow the diagnosis and progress treatment.

48. Which type of study best differentiates a severe pol yra di cul opathy from amyotrophic lateral sclerosis (ALS)? (a) Motor nerve conduction studies of upper and lower ex tre mi tie s (b) Needle electromyography of thoracic paraspinals or b ulb ar mu scles (c) Sensory nerve conduction studies of upper and lower ext re mit ies (d) Needle electromyography of multiple extremities

48 Answer (b) Commentary: Sensory nerve conduction studies are normal in b oth radiculopathy and motor neuron disease. Motor nerve conduction studies are also often normal in both diseases. Both diseases may demonstrate abnormal needle examination in multiple extremities. Thoracic paraspinals and bulbar muscle examinations are most helpful in differentiating severe polyradiculopathy from amyotrophic lateral sclerosis (ALS), since one would expect these studies to be normal in radiculopathy but may be abnormal in ALS. Reference: Preston DC, Shapiro BE. Amyotrophic lateral s cle ro sis and its variants. In: Preston DC, Shapiro BE, editors. Electromyography and neuromuscular disorders, 2nd ed. Philadelphia: Butterworth-Heinemann; 2005. p 428-32.

48. A 22-year-old woman with complaints of fluctuating weakness and abnormal fatigability that improves with rest is sent for electrophysiologic testing. Which electrophysiologic finding on routine testing would be most consistent with this clinical presentation? (a) Small sensory nerve action potentials (b) Slow motor nerve conduction velocities (c) Variability in motor unit action potential amplitude (d) Small compound muscle action potentials

48 Answer: C Commentary: This patient's presentation is most consistent with myasthenia gravis. The incidence of this condition is bimodal and affects women more than men in the younger age group. When one suspects myasthenia gravis the test of choice is repetitive stimulation. However, it is still important to assess for other possible problems and routine nerve conduction and needle electromyographic examination should be performed. The sensory component of the peripheral nervous system lacks a neuromuscular junction and hence the sensory responses should be normal. Motor amplitudes can be small, but this is usually only in severe cases. Motor conduction velocities are normal, since this study assesses the conduction along the motor fibers. Motor unit action potential amplitude variability is a characteristic abnormality observed during routine electromyography in patients with neuromuscular junction disorders. This finding is due to the variability in the total number of single muscle fibers being activated at any single time. Ref: Dumitru D, Amato AA. Neuromuscular junction disorders. In: Dumitru D, Amato AA, Zwarts MJ, editors. Electrodiagnostic medicine. 2nd ed. Philadelphia: Hanley & Belfus; 2002.p 1149-50, 1153-4.

48 After completing inpatient rehabilitation, an 18year-old male with complete tetraplegia is able to feed himself with adaptive equipment and requires some assi sta nce with upper body dressing and grooming. He is able to assist with bed mobility, but is dependent for transfers. He is also able to use a manual wheelchair with rim projections indoors on flat surfaces, but when outdoors he prefers to use a power wheelchair with a joystick. His physical therapist reports that he has achieved his maximal expected outcome. What is his level of injury? a) C4 a. b) C5 b. c) C6 c. d) C7

48 Option b is correct. Commentary: Although each person is different, individuals with C5 tetraplegia are in general able to feed themselves with adaptive equipment after set-u p and ar e able to assist with some upper body dressing. Some are able to independently use a manual wheelchair, but most require some assistance, especially on carpets, non-level surfaces and outdoors. Many prefer to use a power wheelchair. People with complete C4 levels of injury are not able to feed themselves, assist with activities of daily living (ADLs), or propel a manual wheelchair, especially if they have no zone of partial preservation. People with C6 and C7 levels of injury are often capable of transferring (independently or with assistance) and of attaining more independence with ADLs. Reference: (a) Consortium for Spinal Cord Injury Medicine. Outcomes following traumatic spinal cord injury: Clinical practice guidelines. Washington (DC) : Par aly zed Veterans of America; 1999. (b) Bryce TN, Ragnarsson KT, Stein AB, Biering-Sorensen F. Spinal cordiInjury. In: Braddom, editor. Physical medicine and rehabilitation.P hilad elp hia: Saunders; 2011. p 1310-1.

48 A ballet dancer is seen in clinic complaining of knee pain. On physical examination, her knees hyperextend more than 10 degrees. What additional physical ex am finding would help support your d iagn osis of joint h ypermobili ty sy ndrome? a) Extend the 5th digit to 90 degrees. b) Abduct the thumb to 90 degrees. c) Touch fingertips to the floor when standing. d) Appose the posterior heels 180 degrees.

48 Reference(s) Ross J, Grahame R. Joint hypermobility syndrome. BMJ 2011 Jan 20;342:c7167. doi: 10.1136/bmj.c7167. Accessed July 29, 2013.Hoppmann R Musculoskeletal problems in instrumental musicians. In: Sataloff RT, Brandfonbrener AG, Lederman RJ. Performing arts medicine. 2nd e San Die go: Singular Publishing Group; 1998. p 2 22 Option a is correct. A 9-point Beighton score, as described below, can be used to assess for joint hypermobility syndrome. Passive dorsiflexion of the fifth metacarpophalangeal joint to 90o or more (1 point per si de). Opposition of t he thumb t o the volar aspect of the ipsilateral forearm (1 point per side).Hyperextension of the elbow to 10o or more (1 point per side).Hyperextension of the knee to 10o or more (1 point per side).Placing of hands flat on the floor without bending the knees (1 point). A Beighton score of 4 or greater is a major criterion in the Brighton criteria for joint hypermobility syndrome.

49. Which measure is the first sign of respiratory muscle dysfunction in boys with Duch en ne mus cul ar dy stro phy ?:(a) Vital capacity:(b) Oxygen saturation:(c) Maximal expiratory force:(d) Negative inspiratory force

49 (c) Recent studies by McDonald and by Bach showed that reduction of maximal expiratory force (MEF) to 40%-60% of normal in the 7- to 14-year-old age group w as the first si gn of respirator y muscle d ysf unc tion in boy s with Duche nne muscular dyst rophy (DM D). The ear lier and mor e sev ere d ecreas es of MEF tha t are g reater t han t he de cr eas es in m axim al ins pirat ory fo rce , corre spond to the clinically observed weakness of abdominal muscles, which like coughing are important in forced expiration. Vital capacity was not found to decrease until an average of 15-16 years. Low oxygen saturation is a late manifestation in DMD, developing after hypercapnia.

49. Pathological drooling in children with spastic quadr ipa re tic cerebral palsy is (a) due to excessive saliva production. (b) unsightly, but has no medical significance. (c) associated with inefficient, uncoordinated swallowin g. (d) associated with increased dental caries.

49 Answer: (c) Commentary: Pathological drooling is the unintentional l oss o f s aliva either anteriorly over the lips or posteriorly over the back of the tongue. It is associated with an inefficient, uncoordinated swallow. Anterior drooling is normal in infants up to 18 months of age. Recent studies have shown that salivary production is similar to that of typical children without cerebral palsy. Medical complications of pathological drooling include chronic aspiration, pulmonary infections and skin irritation. Saliva is protective of dentition. Reference: a) Blasco PA, Allaire JH. Drooling in the dev elo pm ent ally disabled: management practices and recommendations. Consortium on Drooling. Dev Med Child Neurol 1992;34:849862. (b) Jongerius PH, van Limbeek J, Rotteveel JJ. Assessmen t o f sal ivary flow rate: biological variation and measure error. Laryngoscope 2004;114:1801-4. (c) Mirrett PL, Riski JE, Glascott J, Johnson V. Videofl uor os cop ic assessment of dysphagia in children with severe spastic cerebral palsy. Dysphagia 1994;9:174-9.

54. Patients are NOT candidates for bariatric surgery if they are (a) twice their ideal body weight. (b) age 50 or younger. (c) without a psychiatric contraindication. (d) experiencing skin breakdown.

49 Answer: D Commentary: Surgical candidates include persons who are twice their ideal weight, demonstrate recurrent failure to lose weight through dieting, have no cardiopulmonary or psychiatric contraindications, and are usually50 years of age or younger in most cases, with minor exceptions. Some patients may ask about this procedure when it is recommended they lose weight in order to mitigate musculoskeletal pain. Ref:Koffman BM, Greenfield JL, Imran AI, Noor AP. Neurologic complications after surgery for obesity. Muscle Nerve 2006;33:166-76.

49 Where is the lesion site in ataxic dysarthria, as found in Friedreich's ataxia? a) Extrapyramidal system a. b) Bilateral upper motor neuron b. c) Cerebellum c. d) Multiple sites

49 Option c is correct. Commentary: Types of dysarthria include spastic, most commonly found in cerebral palsy; hypokinetic, as in Parkinson's disease; hyperkinetic, as in dystonia; ataxic, as in Friedrich's ataxia; flaccid, as in bulbar palsy; and mixed, as in amyotrophic lateral sclerosis. The site of the causative lesion in spastic dysarthria i s bil ate ral upper motor neuron; hypo- and hyperkinetic is extrapyramidal system; ataxic is cerebellum; flaccid is lower motor neuron; mixed is multiple sites. Reference: Driver L, Ayyangar R, Van Tubbergen M. Language development in disorders of communication and oral motor function. In: Alexander MA, Matthews DJ, e dit ors. Pediatric rehabilitation: principles and practice. 4th ed. New York: Demos Medical; 2010. p 58. spastic, bilateral UMN; CP hypokinetic, extrapyramidal system; Parkinson's hyperkinetic,extrapyramidal system; dystonia; ataxic, cerebellum; Friedrich 's at axi a flaccid,lower motor neuron; bulbar palsy mixed, multiple sites ALS

5 A 50-year-old man with metastatic renal cell carcinoma status post nephrectomy 1 year ago was found to have a T10 lesion on recent post-operative imaging done as part of a work-up for right-sided mid-back pain. The patient's pain is not relieved with recumbency and is not affected by thoracic rotation. He has a normal thoracic kyphosis and is neurologically intact on physical examination. An MRI scan of the thoracic spine shows a T10 lytic lesion, normal alignment, no discernable vertebral body collapse, and unilateral involvement of the T10 posterior elements. You recommend a) neurosurgical consultation for decompression and segmental stabilization. a. b) radiation oncology consultation for palliative radiotherapy treatments. b. c) T10 kyphoplasty. c. d) custom molded thoracic lumbosacral orthosis (TLSO).

5 Option b is correct. CORRECT: Commentary: Palliative radiotherapy treatments directed at the T10 vertebral body will provide symptomatic pain relief from metastatic tumor involvement. The pat ient has a Spinal Instability Neoplastic Score (SINS) of 6, out of 18. A T10 lesion in the semirigid portion of the thoracic spine scores 1, non-mechanical pain scores 1, lytic bone lesion scores 2, normal alignment scores 0, no collapse with > 50% vertebral body involvement scores 1, and unilateral involvement of the posterior spinal elements scores 1. Neurosurgical decompression and segmental stabilization is not required for a stable T10 lesion in a neurologically intact patient. Similarly, a T10 kyphoplasty is not indicated in the absence of significant vertebral body collapse. A custom-molded TLSO is unlikely to benefit this patient who has no mechanical back pain symptoms. Reference: (a) Fourney D, et al. Spinal instability neoplastic score: an analysis of reliability and validity from the spine oncology study group. J Clin Onco l 2 011;29(22):3072-077. (b) Fisher CG, DiPaola CP, Ryken TC, Bilsky MH, Shaffrey CI, Berven S, et al. A novel classification system for spinal instability in neoplastic disease: an evidence-based approach and expert consensus from the Spi ne On col ogy Study Group. Spine 2010;35(22):E1221-9. (c) Stubblefield MD, Pearce CK. Rehabilitation of the cancer patient: identification, evaluation, and rehabilitation of patients with complications of cancer and its treatment from impending fracture to hematologic abnormalities. Paper presented at: American Academy of Physical Medicine and Rehabilitation Annual Assembly; 2011.

49 Maximal aerobic power decreases less rapidly in the geriatric athlete compared to the sedentary geriatric person. This difference is because the geriatric a thlete has a slower rate of a) maximal heart rate decline. b) muscle atrophy. c) cardiac output decline. d) lactate threshold decline.

49 Reference(s) Unwin BK. The geriatric athlete. In: O'Connor FG, Casa DJ, Davis BA, et al. ACSM's sports medicine: a comprehensive review. Philadelphia: Lippi ncott Williams & Wilkins; 2013. p 76 673 Shepard RJ. S pe cial consi derat ions in the older athlete. In: Frontera WR, Herring SA, Micheli LJ, et al, editors. Clinical sports medicine: medical management and rehabilitation. Philadelphia: Elsevier; 2007. p 103-15. Option c is correct. Aerobic power, VO2, is the rate of oxygen consumed during physical activity. Maximal aerobic power, VO2max, decreases less rapidly in physically active geri atric adults in part because they ha ve l ess rapid dec re ase in car diac output. The peak aerobic power required for independent living, which is around 15 ml/(kg?min), is reached at age 80 to 85 in the sedentary adult and 10 to 20 years later in athletes. The key physiological changes with aging, despite regular physical activity, are a decreased maximal heart rate and a decreased maximal aerobic capacity (VO2max). Exercise does not affect the declining maximal heart rate that occurs with age. Lactate threshold can be maintained in older adults with appropriate regular physical activity, but this is not directly associated with aerobic power. Active and sedentary adults have similar rates of muscular atrophy, or sarcopenia, with age.

49. Acquired subluxation or disl ocati on of the hips in spastic cerebral palsy is usually due to muscular imbalance and pull of the(a) hip flexors and tensor f asci a lat a.(b) hip flexors and hip addu ctor s.(c) rectus femoris and hip a bduc tors.(d) tensor fascia lata and h ip e xtens ors .

49. (b) Strong hip flexor and adductor muscles can overpower weak extensors and abductors. Acquired hip dislocation can be prevented in some cases by release of spastic hip flexors and adductors.

49. What percentage of American children with myelomeningocele requi res a shu nt to man age h ydr ocephalus? (a) 10-20 (b) 25-50 (c) 60-70 (d) 80-90

49. (d) Seventy-five percent of lesions in spina bifida cystica (myelomeningocele) affect the lumbosacral spine. Ninety percent of children with spina bifida have hydrocephalus that requires a shunt for management.

5. What bathroom modification should be made for nonambulatory individuals who have the a bil ity to tran sfer th emsel ves in and out of a whe elc ha ir?:(a) Toilet height of 15 inches:(b) Cabinet heights of 36 inches from the floor:(c) Three feet of clear turning space in the bathroom:(d) Separate hot and cold water handle controls

5 (b) When modifying a bathroom for a person who has the ability to transfer him/herself in and out of a wheelchair, it is important to design bathroom space for the pers on' s eff iciency, to a llow him or he r to be as independent as possible. This inclu des providi ng a minimum of 5 fe et of clear t urning spac e and t oilets a t lea st 20 i nch es high (it is re asona ble to us e a sta ndard height toilet with a raised toilet seat). Cabinets should be positioned for easy access. In most cases, the bottom of the cabinet should be 36 inches from the floor. Mirrors also should be positioned so that their bottom edge is 36 inches from the floor. Single-lever type handles should be used for water control. The use of a single handle control with a lever or blade shape that mixes the water to control and adjusts the flow is recommended. Temperatures should be set between 110 and a 120 degrees Fahrenheit at the point of supply so that the delivery temperature will be approximately 105 degrees Fahrenheit. All hot water feed and drainage pipes should be insulated to prevent scalding.

5. According to the Consortium for Spinal Cord Medicine's Clinical Practice Gu idelines for the Prev ent io n of Thro mboem bo lism in Spin al Co rd In jur y, indiv idu als wi th motor incomplete (ASIA class C or D) spinal cord injury should receive (a) warfarin (Coumadin), international normalized ratio target: 2-3. (b) low molecular weight heparin. (c) inferior vena cava filter. (d) unfractionated heparin, 5000 units every 12 hours.

5 (d) According to the guidelines for the prevention of thromboem bolism in spinal cord injury, patients with low risk motor incomplete injuries require only compressi on hose and compression boots; those with intermediate risk require unfrac ti onated heparin, 5000 u nits every 12 hours. Patients with a mo tor c omp let e in ju ry should rec eive e ith er u nfra cti ona ted hepa ri n to a h igh nor mal acti vat ed pa rtial thromboplas tin time (aPTT) or low molecular weight heparin twice daily. Persons with a motor complete injury with other risk factors including lower limb fracture, risk of thrombosis, cancer, heart failure, or other compromising factors may require an inferior vena cava filter in addition to the prescribed drugs.

5. A 22-year-old male with C6 ASIA B tetraplegia secondary to a motor vehicle accident 2 months ago is undergoing inpatient rehabilitation. His bladder is managed with a suprapubic catheter and he is on a daily bowel program using digital stimulation and a bisacodyl (Dulcolax) suppository. While resting supine in bed one evening, he suddenly develops a pounding headache. His blood pressure is found to be 180/100 and his heart rate is 56. His face is flushed. What is the first step in the initial management of this patient? (a) Flush his suprapubic catheter. (b) Using a well lubricated finger, check his lower rectum for fecal impaction. (c) Sit him up and loosen any restrictive clothing. (d) Apply ½ inch of nitropaste to his anterior chest wall.

5 Answer: (c) Commentary: This patient is presenting with autonomic dysreflexia (AD). Once diagnosed, the first step in the management of AD is to sit the patient up, if supine, and loosen any restrictive clothing. If the blood pressure remains elevated, the urinary system should be evaluated. In this case, therefore, the second step would be to flush the suprapubic catheter. If the blood pressure continues to be elevated after bladder distention has been ruled out, the lower bowels should be evaluated for fecal impaction, but only after the systolic blood pressure is reduced to less than 150 mmHg, using medications if necessary. Medications, such as nitroglycerin paste (nitropaste), should be used only after these first 3 steps are taken. In the acute setting the need is unlikely, but to avoid life threatening hypotension in chronic SCI and AD avoid using nitrates with sildenafil (Viagra) and other phosphodiesterase type 5 inhibitors. Reference: Consortium for Spinal Cord Injury Medicine. Acute management of autonomic dysfunction: adults with spinal cord injury presenting to health-care facilities. J Spinal Cord Med. 2002;25:S67-S88.

5 What is the most common initial presenting symptom of br ain metastasis? a. Seizures b. Focal weakness c. Speech difficulties d. Headache

5 Option d is correct. Presenting symptoms at the time of diagnosis with brain metastasis, in o rder of dec re as ing fr equ enc y, a re as fol lo ws hea dach e, 4 9%; me ntal dis turba nce , 32% ; f oca l w eak ness , 30%; gait ataxia, 21%; seizures,18%; speech difficulty, 12%; visual disturbance, 6%. On presentation for medical evaluation a patient can have more than 1 of the symptoms. The most common presenting complaint is a progressive headache, often worse when recumbent. Hemiparesis, seizures, and mental status changes occur frequently. Evaluation includes a complete neurological and clinical examination to exclude other etiologies such as stroke and central nervous system (CNS) infection. Gadolinium-enhanced magnetic resonance imaging (MRI) is the criterion standard.

5 Nonaccidental trauma accounts for an estimated 10% to 20% of pediatric burns and is more common in children younger than 2 years. Children with nonaccidenta l trauma are less likely to attend o utpa tient therapy a nd follow- up cl inics. The anterior interosseous nerve innervates which muscle? a) Brachioradialis b) Extensor indicis proprius c) Extensor hallucis longus d) Pronator quadratus

5 Reference(s) Dumitru D, Zwarts MJ. Focal peripheral neuropathies. In: Dumitru D, Amato AA, Zwarts MJ, editors. Electrodiagnostic medicine. 2nd e Philadelphi a: Hanley & Belfus; 2002. p 1049. Option d is correct.

5. Which change is included in t he re vis ed edi tion of the American Spinal Injury Association (ASIA) Impairment Scale, published in the year 2000? (a) The zone of partial pres erva tion (ZP P) is defined as the most rostral segment with sensory function. (b) The Functional Independe nce Measu re (FIM) has been added to the standards. (c) The definition of a moto r in compl ete injur y requires some motor function more than 3 levels below the motor level. (d) The sensory exam now inc lude s a 5 po int sc ale to include sharp and dull sensations, proprioception, and vibration.

5. (c) The 2000 revisions have clarified a few issues from the previous standards. For a person to receive a classification of motor incomplete spinal cord injury (ASIA C or ASIA D) they must have either 1) voluntary anal sphincter contraction or 2) sacral sensory sparing with sparing of motor function more than 3 levels below the motor level. Previously, the person needed only to have sparing more than 2 levels below the motor level. The FIM was eliminated from the standards. The ZPP is to be documented as the most caudal segment with some sensory and/or motor function. There has been no change in the 3-point (0-2) scale for the sensory exam.

50. In which circumstance is supracondylar suspension on a transtibial prosthetic s ocke t mos t i ndi cated ?:(a) A 4-centimeter residual limb length below the tibial tubercule:(b) A residual limb with mildly adherent distal scar tissue:(c) A cylindrical-shaped residual limb:(d) A residual limb with poor definition above the femoral condyles

50 (a) Supracondylar suspension would be most indicated for an individual with a short transtibial residual limb to provide additional mediolateral support and to inc rease th e wei ght-bearing s urface a rea fo r mo re e ven pressure di stribution. A cyl indric al- shaped t ran stibial l imb i s i deal for al lo wing total conta ct between the resid ua l l imb and the so cke t and is no t a n indic ation for supracondylar suspension. Supracondylar suspension is also not specifically indicated for mildly adherent scar tissue. Supracondylar suspension would be difficult to utilize in a residual limb that has poor definition above the femoral condyles.

50 A 60-year-old woman with right medial knee pain has a genu varum deformity that is observed while she is standing and walking. What shoe modification can help her pain? (a) Medial wedge (b) Lateral wedge (c) Rocker bottom (d) Arch support

50 Answer: (b) Commentary: Medial compartment osteoarthritis causes a genu varum deformity. Lateral heel wedges can be used for conservative treatment of medial compartment osteoarthritis. A medial wedge would exacerbate the genu varum. An arch support would help with pes planus (flatfoot) which may be helpful for genu valgum deformity. Rocker bottoms may be used to offload pressure from the metatarsal heads. Ref: (a) Hennessey WJ. Lower limb orthoses. In: Braddom RL, editor. Physical medicine and rehabilitation. 3rd ed. Philadelphia: Elsevier; 2007 p 344-5, 347. (b) Goldman R, Popescu A, Hess CT, Salcido R. Prevention and management of chronic wounds. In: Braddom RL, editor. Physical medicine and rehabilitation. 3rd ed. Philadelphia: Elsevier; 2007 p 700.

50. What does the acronym SACH stand for? (a) Simple amputation, cadence heel (b) Single axis, carbon heel (c) Standard adult, control heel (d) Solid ankle, cushion heel

50. (d) SACH is an acronym for solid ankle, cushion heel. The SACH foot has a cushioned heel that compresses during heel strike, stimulating plantar flexion, and has a rigid anterior keel to roll over during late stance. It is light, durable, and inexpensive, and is the orthosis most often prescribed for juvenile and geriatric amputees.

53. Use of continuous passive motion after total knee replacement surgery (a) is associated with increased knee flexion at 6 months. (b) decreases the risk of joint infection. (c) facilitates knee flexion within the hospital stay. (d) obviates deep vein thrombosis prophylaxis.

53 (c) Patients treated with continuous passive motion obtain greater early knee flexion (and thus experience fewer hospital days and manipulations). These devices might exacerbate flexion contractures and extension lags. Their use has no effect on rates of infection or deep vein thrombosis.

51 The manager of a call center solicits your advice about ergonomics for computer work stations. Which ergonomic recommendation is correct to reduce cervical strain? a) The distance of the monitor should be no more than two-thirds the length of the employee's arm. b) The bottom third of the computer screen should be situated at eye level. c) Headsets reduce sustained contralateral and forward flexion of the cervical spine. d) Individuals who use bifocals should position materials to reduce sustained cervical extension.

51 Reference(s) Lipetz JS, Lipetz DI. Disorders of the cervical spine. In: Frontera WR, DeLisa JA, Gans BM, Walsh NE, Robinson LR, editors. DeLisa's physical m edicine and rehabilitation: principl es a nd practice. 5t h e Philad elphi a: Lippincott Williams & Wilkins; 2010. p 830. Option d is correct. Instruction in proper positioning at the work station represents an important component in the rehabilitation of patients with cervical disorders. Sustained spinal posturing out of the neutral zon e can lead to f urther agg ravat ion of the affected tissues and a perpetuation of symptoms. Individuals who require bifocal eyeglasses should be instructed to position reading materials in such a fashion as to avoid sustained cervical extension postures. Proper distance from the computer monitor is an arm's length measurement, which is unique to each individual. The upper third of the computer screen should be situated at eye level. Headsets can reduce ipsilateral, lateral flexion of the cervical spine.

51. A 20-year-old female long-distance runner presents with a l-week history of worsen ing l ate ral hip pain. Magnetic resonance imaging reveals bone edema at the lateral aspect of the femoral neck, without evidence of a fracture line or cortical break. What is the next step in management? (a) Relative rest for 4 weeks (b) Refer to orthopedic surgeon for surgical evaluation (c) Order a triple-phase bone scan to determine acuity of the injury (d) Physical therapy with closed kinetic chain exercise emphasis

51. (b) Femoral neck stress fractures are serious injuries. Stress fractures may occur without a cortical break. Stress fractures of the medial side of the femoral neck, the "compression" side, can be treated with a weight-bearing restriction for 6 weeks or longer including no running or high impact activities. Stress fractures on the lateral side of the femoral neck, the "tension" side, warrants strong surgical consideration because of the poor healing capability at that location.

51. A 21-year-old US Army re crui t rep ort s to b oot camp. After 5 days of marching, he reports to the base physiatrist with complaints of severe pain in his left shin. He states his pain began after a 10mile run in full gear this morning. The pain has gotten significantly worse over the last 2 hours. He is now unable to bear weight on his left leg. On examination, his left shin is shiny and edematous. He has severe pain with palpation and the muscles seem tight. The most appropriate treatment plan for this patient would be to(a) obtain an x-ray and a tr iple phas e b one sc an.(b) measure the pressure in his tibia lis anter ior muscle immediately.(c) wrap the foreleg with an Ace band age , appl ying pressure from distal to proximal.(d) apply ice and have the p atie nt el eva te his leg when he gets back to his barracks.

51. (b) Suspicion of a compartment syndrome should lead the physician to get pressure measurements immediately, since delays may result in permanent muscle or nerve damage. Usual pressures are less than 30mmHg. Pressures from 30 to 50mmHg are equivocal, but pressures greater than 50mmHg constitute a surgical emergency. The leg should NOT be elevated, because this will lower arterial perfusion pressure and will further compromise vascular supply. An external circumferential force will increase pressure. An x-ray and bone scan are not indicated in this patient.

52 A 65-year-old man with multiple myeloma (MM) complains of new-onset lower back pain. A skeletal survey performed 6 months earlier showed no evidence of lyti c lesions or vertebral body collapse . Fu rther diagnos ti c work-up shoul d include a) technetium-99m bone scan. b) dual-energy x-ray absorptiometry scan. c) MRI scan of the lumbar spine and pelvis. d) PET/CT scan.

52 Reference(s) (a) Dimopoulos M, Terpos E, Comenzo RL. et al. International myeloma working group consensus statement and guidelines regarding the current rol e of imaging techniques in the diagn osis and monitori ng of multip le my elom Leukemia 2009;23(9):1545. (b) Tryciecky EW, Gottschalk A, Ludema K Oncologic imaging: interactions of nuclear medicine with CT and MRI using the bone scan as a model.Semin Nucl Med 1997;27(2):142. Option c is correct. Magnetic resonance imaging can detect diffuse and focal bone marrow lesions in patients with MM without osteopenia or focal osteolytic lesions seen on stand ard metastatic bone surveys. Convers ely, bone surveys c an detect lesio ns not found on MRI of the axial skeleton. A baseline MRI scan of the pelvis and spine may also be useful for risk stratification in patients with smoldering (asymptomatic) MM. Technetium-99m bone scanning (which primarily detects osteoblastic activity) is inferior to conventional roentgenography for the detection of lytic lesions and should not be use Dual-energy x-ray absorptiometry scanning is not recommende A PET/CT scan using fluorine-18 (labeled FDG or 18FDG) appears to correlate with areas of active lytic bone disease but is not recommended for routine use in the management of myeloma patients.

52. Findings commonly seen a fter a ri ght hemis pheric stroke include(a) right hemiplegia.(b) aphasia.(c) visual-perceptual defici ts.(d) agraphia.

52. (c) Strokes on the nondominant hemisphere present with contralateral hemiplegia and hemianesthesia, aprosody, visual-spatial deficit, and neglect syndrome.

53. Which of the following is NOT associated with a Charcot joint?:(a) Diabetes:(b) Syringomyelia:(c) Tabes dorsalis:(d) Rheumatoid arthritis

53 (d) Destruction of a joint due to loss of nociceptive input describes a Charcot joint.

53. A 25-year-old man with a history of plantar fasciitis complains of low back and buttock pain. The pain is worse at rest and better with activity. Schober test (signifying restricted lumbar flexion) is positive. The laboratory or radiology result that would help confirm your most likely diagnosis is a positive (a) antinuclear antibody (ANA). (b) human leukocyte antigen (HLA) B27. (c) discogram. (d) myelogram.

53 Answer: (b) Commentary: The patient may have ankylosing spondylitis (AS). Enthesitis, such as plantar fasciitis, is common in patients with AS. HLA B27 is usually positive in this condition, which is a seronegative spondyloarthropathy. Schober test is performed by marking a point 5 cm below the iliac crest line and 10 cm above. On forward flexion, the line should increase by more than 5 cm. An ANA test, discogram and myelogram would not help to diagnosis AS. References: (a) Nucatola TR, Freeman E, Brown DP. Rheumatology. In: Cuccurullo SJ, editor. Physical medicine and rehabilitation board review. New York: Demos; 2004. p 106. (b) Van der Heijde D. Ankylosing spondylitis. In: Klippel JH, Stone JH, Crofford LJ, White PH, editors. Primer on rheumatic disease. 13th ed. Atlanta (GA): Arthritis Foundation; 2008. p 193-4.

53. Mr. Smith comes to your offi ce co mpl aining of a hot, painful, swollen left foot. He denies any history of trauma and states that he forgot to take his allopurinol the past several days. X-rays of his foot may reveal(a) chondrocalcinosis of art icul ar ca rti lage.(b) bony erosion with an ove rhan ging edg e.(c) severe juxta-articular o steo penia .(d) pencil in cup deformity.

53. (b) This patient has gout with characteristic "overhanging edge" lytic lesions. Chondrocalcinosis is seen in pseudogout, juxta-articular osteopenia is seen in RA, and pencil in cup deformity is seen with psoriatic arthritis.

53. A 45-year-old man complains of wrist, thumb, and elbow pain and swelling after bei ng on a highprotein diet for 12 weeks. Aspiration of the elbow reveals negatively birefringent intracellular crystals on microscopic examination. What would you expect to see on radiographs of his hands and wrists? (a) Distal interphalangeal joint osteophytes (b) Pencil-in-cup deformity (c) Juxtarticular osteopenia (d) Joint erosion with sclerotic borders

53. (d) This patient has gout, and radiographs of the hands and wrists would likely demonstrate joint erosions with sclerotic borders. The finding in option (b) describes psoriatic arthritis. Juxtarticular osteopenia may be a finding on radiographic studies in patients with rheumatoid arthritis. Distal interphalangeal joint osteophytes are characteristic of osteoarthritis.

54 A 35-year-old woman with T12 ASIA A paraplegia due to a motor-vehicle collision is on your inpatient rehabilitation unit. She has no other risk factors for the development of peptic ulceration . In order to pre ve nt a stres s ulc er, you suggest that she take a proton-pump inhibitor (PPI) for a total of how many weeks after her injury? a) 2 b) 4 c) 8 d) 12

54 Reference(s) Consortium for Spinal Cord Injury Medicine. Early acute management in adults with spinal cord injury. Clinical practice guidelines. Washington (DC): Paralyzed Veterans of America; 200 8. p 42-3. Option b is correct. According the Consortium for Spinal Cord Injury Clinical Practice Guidelines, it is recommended to initiate stress ulcer prophylaxis after acute traumatic s pinal cord injury. Most stress ulcer s ha ppen within t he first 4 w eeks, and prolonged used of PPIs has been associated with increasing rate of Clostridium difficile infection. Therefore, 4 weeks of stress ulcer prophylaxis is indicated in most uncomplicated situations. If other risk factors for peptic ulceration are present one can consider a longer duration of treatment.

54. What is the primary underlying cause of the restrictive lung dis orders th at are co mmon in many neuromuscular disorders? (a) Recurrent pneumonia (b) Prolonged wheelchair use (c) Respiratory muscle weakness (d) Intrinsic lung damage

54. (c) Respiratory impairment in neuromuscular disease is due to weakness of the diaphragm, chest, and abdominal musculature. The other listed factors have not been shown to play a significant role.

54. For patients with amyotr ophi c lat era l scle rosis, exercise should be prescribed for muscles with(a) weakness and less than 2 /5 s treng th.(b) muscles with visible fas cicu latio ns.(c) weakness and better than 43164 stre ngt h.(d) unaffected muscles.

54. (d) Because amyotrophic lateral sclerosis is a relentlessly progressive disease, only muscles with unaffected strength should be exercised, to prevent disuse atrophy. In the postpolio patient, it is acceptable to strengthen weak muscles with greater than fair (or 3/5) strength.

"

54. All hereditary sensory motor neuropathie s a re characterized by (a) decreased Intelligence Quotient scores. (b) absent spinal deformities. (c) muscle weakness. (d) joint contractures." 54. (c) All types of hereditary sensory motor neuropathies (HSMN) are characterized by weakness. The residual muscle force in the later stages of disease is 20%--40% less than normal. Intelligence Quotient reduction, significant joint contractures, pulmonary/cardiac abnormalities and spinal deformities are not typical of these diseases. Ref: Thomas MA, Felsenthal G, Fast A, Young M. Peripheral neuropathy. In: DeLisa JA, Gans BM, Walsh, NE, editors. Physical medicine and rehabilitation: principles and practice. 4th ed. Philadelphia: Lippincott Williams & Wilkins; 2005. p 898.

55. The occurrence of renal calculi during the first 3 months after spinal cor d injury is rela ted t o (a) level of injury. (b) immobilization hypercalciuria. (c) method of bladder management. (d) number of urinary tract infections.

55 (b) Renal calculi occur in approximately 8% of patients with spi nal cord injury. Approximately 98% of renal calculi in persons with spinal cord injury are composed e ither o f calcium phosphate or magnesium ammonium phosphate. These stones a re typic ally asso ciated with urina ry tract i nfections (UTIs) . E arl y sto ne form at io n is li kely sec on dar y to imm obi liz ati on hy po calc em ia , wh ere as later st on e f ormat ion is secon dary to repeated UTIs and long term use of an indwelling catheter.

55. In response to a request for information regarding ejaculation, you advise a 22-year-old man with T4 ASIA A paraplegia who is 1 year postinjury to (a) avoid ejaculation, because of the risk of autonomic dysreflexia. (b) use sildenafil (Viagra) 60 minutes before intercourse. (c) use vibratory stimulation. (d) see a urologist for direct sperm harvest.

55 Answer: C Commentary: In men with spinal cord injury who have an ejaculation reflex (upper motor neuron lesion), there is a 30% to 96% ejaculation rate, depending on the amplitude and frequency of vibratory stimulation. Sildenafil is an option for erectile dysfunction, rather than for ejaculationrelated problems. Autonomic dysreflexia can occur with ejaculation but is more commonly a transient phenomenon and does not lead to complications. Ref: Linsenmeyer TA. Sexual function and fertility following spinal cord injury. In: Kirshblum S, Campagnola DI, DeLisa JA, editors. Spinal cord medicine. Philadelphia: Lippincott Williams & Wilkins; 2002. p 322-30.

55 A 40-year-old man with a history of severe asthma and alcohol abuse complains of progressive right groin pain and a decline in mobility for the past 5 years . Acetaminophen and nonsteroidal ant iinf lammatory dru gs are not h elpfu l. He has also tried physical therapy and the use of a cane without improvement. On physical examination, he has an antalgic gait and limited right hip range of motion, especially with internal rotation. Which procedure would best help the patient's pain and function? a) Total hip replacement b) Lumbar epidural steroid injection c) Radiofrequency rhizotomy d) Hip injection of platelet rich plasma

55 Reference(s) Seyler TM, Marker D, Mont M Osteonecrosis. In: Klippel JH, Stone JH, Crofford LJ, White PH, editors. Primer on rheumatic disease. 13th e Atlanta (GA): Arthritis Foundation; 2008. p 565 571 Option a is correct. This patient has osteonecrosis or avascular necrosis of the femoral hea Risk factors include glucocorticoid, alcohol, and tobacco use. When pain limits mobi lity and conservative management has fai led, total hi p replacemen t is indicate Platelet rich plasma is still experimental for its primary indications of lateral epicondylitis, patellar tendinopathy, and Achilles tendinopathy.

55. Factors associated with poor prog nos is in multiple sclerosis include(a) female gender.(b) age at onset less than 2 0 ye ars.(c) cerebellar involvement a t on set.(d) relapsing remitting cour se a t ons et.

55. (c) Factors associated with a poor prognosis in multiple sclerosis include: 1) progressive course at onset. 2) Male sex. 3) Age at onset greater than 40 years. 4) Cerebellar involvement at onset. 5) Multiple system involvement at onset.

56. A patient with far advanced prostate cancer metastatic to liver and bone achiev es p ai n c ont rol thro ugh the use of a s ubcutane ous h ydro mor ph one infusion. The therapy is complicated by significant sedation. You therefore initiate therapy with:(a) Clonidine (Catapres).:(b) Pemoline (Cylert).:(c) Methylphenidate (Ritalin). (d) Naloxone (Narcan).

56 (c) Opioid-based pharmacotherapy is the current standard of care for severe cancer-related pain. Side effect management is an integral dimension of competen t p ain manag eme nt. O pioid-induc ed sedatio n c an be m anag ed through the use of psycho stim ulants su ch as me thy lphenidat e ( Ri tal in). Epidur al and intrat hecal d rug deli very can a ls o b e u sed to m ini miz e neu rop syc hol ogical toxicity. Pemoline (Cylert), a psychostimulant, is not widely used because of concern over hepatotoxocity, particularly in patients with liver metastases.

56. Which of the following medications exerts its analgesic effect by increasi ng the i nfluence of t he de scend ing spina l tract s on no cicep ti ve pro ce ssing in the do rs al horn? (a) Valproic acid (Depakote) (b) Mexiletine (Mexitil) (c) Amitriptyline (Elavil) (d) Baclofen (Lioresal)

56 (c) Tricyclic antidepressants are believed to exert their analge sic effects through modulation of the descending inhibitory pathways the arise in various b rain stem centers and synapse within the dorsal horn of the spinal cord. Neurons wit hi n thes e tracts are pri ncip ally s ero tonergi c and noradrener gic . B y inf lue ncin g th e reupt ake of t he se mo no amin es, am itr iptiy li ne a nd o ther tr icy clic ant id epr essan ts, enhance the inhibitory influence exerted by the descending tracts.

56 Which physiological change occurs in the cardiovascular system with aging? (a) Increased resting heart rate (b) Increased resting cardiac output (c) Decreased ejection fraction (d) Decreased orthostatic hypotension

56 Answer: (c) Commentary: As a person ages, decreased inotropic responsiveness to adrenergic stimuli leads to decreased myocardial contractility and, hence, to a decrease in ejection fraction. Resting heart rate does not change with aging, but maximal heart rate with exercise does decrease progressively. Cardiac output at rest and with modest exercise is maintained by early involvement of the Frank-Starling mechanism. There is an increased incidence of orthostatic hypotension in the elderly due to decreased baroreceptor sensitivity and diminished reflex tachycardia. Ref: Clark GS, Siebens HC. Geriatric rehabilitation. In: DeLisa JA, Gans BM, Walsh, NE, editors. Physical medicine and rehabilitation: principles and practice. 4th ed. Philadelphia: Lippincott Williams & Wilkins; 2005. p 1537.

56 In a body-powered upper extremity prosthesis, which feature is an advantage of a voluntary opening device over a voluntary closing device? a) It provides larger prehensile forces. b) Variable prehensile force is transmitted through the control cable. c) There is no need for constant pull on the control cable during grasp. d) It provides indirect sensory feedback.

56 Reference(s) Walsh NE, Bosker G, Santa Maria, Upper and lower extremity prosthetics. In: Frontera WR, Gans BM, Walsh NE, Robinson LR, editors. Physical medicine and rehabilitation: principles and practi ce. 5th e Philadelphia: Lippincott-R aven ; 2010. p 203 -8 9 Option c is correct. Closing forces in the voluntary opening terminal device rely on springs or rubber bands to provide prehensile force. Typical closing forces range from 5 lbs to 10 lbs. Voluntary closing device s ar e capable of pr oviding pr ehens ile forces up to 20-25 lbs and provide indirect sensory feedback through the force exerted on the control cable. A disadvantage of the voluntary closing device is the need for a constant pull on the control cable during prolonged grasping.

57. A functional capacity evaluation is required when:(a) the injured worker is ready to return to his/her job.:(b) the worker's ability to perform work-related activities must be assessed.:(c) an injured worker's case remains open more than 7 weeks.:(d) the company physician must determine whether the worker is injured.

57 (b) A functional capacity evaluation (FCE) is an assessment of a worker's ability to perform workrelated activities. A functional capacity examination can b e u sed to de ter mine if a worker m ight ben efi t f rom work ha rdening or w ork condition ing, to de ter mine whe the r a worke r c an re turn to his /h er job, to deter mi ne if wo rk re stric ti ons ar e re comm end ed or if jo b m odi ficatio ns are needed, and to document the worker's activity capability. No absolute time line exists. An FCE can be used in the subacute, or maintenance phase of treatment. An FCE does not determine validity of injury but can reveal the effort a person expends to perform a task.

57. Comparing the functional outcomes at 1-year post tre atm en t o f 2 groups of patients with nonspecific low back pain greater than 12-months' duration and no prior history of lumbar fusion, which finding regarding structured rehabilitation with cognitive behavioral therapy (CBT) versus lumbar fusion is TRUE? (a) Better functional outcomes in the surgical group ver sus t he CBT group (b) Improvements in both groups with similar functional out co mes (c) Better functional outcomes in the CBT group versus t he su rgi cal group (d) Poor functional outcomes in the CBT group, but no co nsi st ent outcome in the surgical group

57 Answer: (b) Commentary: Randomized trials for surgery are difficult to co ndu ct, particularly those that compare surgical to nonsurgical treatment. While available studies do not allow a general statement regarding the efficacy of fusion over nonsurgical care for discogenic back pain, 4 trials suggest any advantage of surgery over nonsurgical care is modest, on average near or below the minimally important change in the disability score. Both groups demonstrated improvement compared to baseline. Highly structured rehabilitation with a cognitive-behavioral component seems nearly equivalent to surgery in efficacy at 1 year, with fewer complications. Reference: Mirza SK, Deyo RA. Systematic review of rando miz ed tr ials comparing lumbar fusion surgery to nonoperative care for treatment of chronic back pain. Spine 2007 32(7):816-23.

57 A physical therapist tells you that he just took a refresher course on therapeutic ultrasound, and you would like to refer a patient to him. The patient mos t likely to benefit from this therap y is a a) 56-year-old woman with an acute lateral epicondylitis. b) morbidly obese woman with a snapping hip, or tight iliopsoas. c) elderly man whose chronic shoulder pain is associated with calcific tendonitis. d) young man who wants to hasten recovery from a quadriceps contusion.

57 Reference(s) Press JM, Burgled D Physical modalities. In: Frontera WR, Herring SA, Micheli LJ, et al, editors. Clinic al sports medicine: medical manageme nt a nd rehabilita ti on. Philad elphi a: Elsevier; 2007. p 211-3. Option c is correct.

57. A case manager comes to your office accompanying the injured work er you are managing. The front desk person asks if you will see the case manager with the patient. You respond that (a) case managers inhibit patient care and you don't wis h to spea k wi th them. (b) as requested by the patient you will see the case ma nager fol lowi ng the interview and examination. (c) you will speak with the case manager after the patie nt signs a re lease of information. (d) the case manager should always be present at the tim e of the pati ent's interview and examination despite the patient's request to avoid the case manager.

57. (b) Case managers are shown to be beneficial liaisons between the physician and workers compensation carrier and their presence facilitates patient care. To be treated as a workers compensation case, the patient must give the carrier full access to his/her medical record. The employee treated under workers compensation cannot restrict the access of the case manager to the physician; however, discussions with the case manger should be done in the environment that the patient requests.

58. In a patient with a neuromuscular junction disorder, wh ic h e lectrodiagnostic results for compound muscle action potential (CMAP), motor unit action potential (MUAP) or nerve action potential (SNAP) may be misleading if the limb is cold? (a) Diminished CMAP decrement on repetitive nerve stimul ati on (b) Diminished polyphasia of the MUAP (c) Shortened distal latency of the CMAP (d) Decreased amplitude of the SNAP

58 Answer (a) Commentary: In neuromuscular junction (NMJ) disorders, c omp ou nd muscle action potential (CMAP) decrement may be diminished if the limb is cold, likely due to decreased functioning of acetylcholinesterase. Cool temperatures may alter results by slowing nerve conduction velocity, prolonging distal latency, increasing amplitude and duration of sensory nerve action potential (SNAP) and CMAP and motor unit action potential (MUAP), increasing phases of MUAP. Reference: Preston DC, Shapiro BE. In: Preston DC, Shap iro B E, editors. Electromyography and neuromuscular disorders, 2nd ed. Philadelphia: Butterworth-Heinemann; 2005. p 71, 87-9.

58. Blink reflex studies can be useful in diagnosing whi ch condit ion? (a) Neuromuscular junction disorder (b) Axonal neuropathy (c) Motor neuron disease (d) Midpontine lesion

58. (d) Blink reflex studies can help assess facial and trigeminal nerve lesions, as well as central lesions in the brain stem. Neuromuscular junction disorders are better assessed by repetitive studies. Axonal neuropathies rarely affect the blink reflex, but demyelinating peripheral neuropathy can affect all potentials of the blink reflex study. Motor neuron disorders such as amyotrophic lateral sclerosis do not typically affect the blink reflex.

59. A 10-year-old child with L4-5 myelodysplasia and shunted hydrocephalus develops spa st ici ty in her l egs. Th e mos t l ike ly cause of t his spa st icity is:(a) shunt malfunction.:(b) symptomatic Chiari malformation.:(c) growth.:(d) tethered cord.

59 (d) Tethered cord is the most common cause of new onset spasticity in patients with myelodysplasia. Linear growth does not cause new spasticity. Symptoms of Ch iari malf orm ation include cr an ial nerv e d iso rder s an d r espiratory p roblems. Shun t ma lfunct ion may be ass ociated w ith h ead aches , vomi ti ng, eye mus cle a bn ormaliti es, a nd so me tim es abdo mina l s ymp toms.

59. When should one be concerned when observing a child with an asymmetric tonic neck reflex (ATNR,"fencer" position) with neck rotation, relative extension of the limbs on the chin side and flexion of the limbs on the occiput side? (a) In a 3-month-old infant who is able to move out of the "fencer" position (b) In a 5-month-old infant who is unable to move out of the "fencer" position (c) In a 6-month-old infant whose leg response to the stimulus is greater than the arm's (d) In an infant of any age, since the ATNR is a primitive pathological reflex.

59 Answer: (b) Commentary: The ATNR is typically present at birth and integrates between 4 and 6 months of age. An obligatory "fencer" position is abnormal at any age. A persistent or obligatory ATNR may be an early clue that a child has a disorder of motor control, most often cerebral palsy. Ref: Carpenter DL, Batley RJ, Johnson EW. Developmental evaluation of infants and children. Phys Med Rehabil Clin N Am 1996;7 561-82.

59 As compared to children with severe traumatic brain injuries, children with severe anoxic encephalopathy are more likely to have (a) rigidity and decreased rate of regaining consciousness. (b) rigidity and increased rate of regaining consciousness. (c) hypotonia and decreased rate of regaining consciousness. (d) hypotonia and increased rate of regaining consciousness.

59 Answer: A Commentary: Compared to children with severe traumatic brain injury, children with severe anoxic encephalopathy are less likely to regain consciousness; they also have shorter survival time, and often have profound rigidity. Ref: Krach LE, Kriel RL. Traumatic brain injury. In: Molnar GE, Alexander MA, editors. Pediatric rehabilitation. 3rd ed. Philadelphia: Hanley & Belfus; 1999. p 262.

59 A 38-year-old woman with C8 ASIA B tetraplegia resulting from a gunshot wound presents to your outpatient clini Upon discharge from acute rehabilitation, sh e managed her bladder with intermitt ent catheterizati on . Due to w orsen ing obesity and hip adductor spasticity, however, she is now having difficulty performing her intermittent catheterization. She wants to continue to be independent with her bladder management and is willing to consider surgery. Which surgical procedure is she most likely to benefit from? a) Bladder augmentation b) Continent urinary diversion c) Cutaneous ileovesicostomy d) Transurethral sphincterotomy

59 Reference(s) (a) Consortium for Spinal Cord Injury Medicine. Bladder management for adults with spinal cord injury. Clinical practice guidelines. Washington (DC): Paralyzed Veterans of America ; 20 06. (b) Carde na s DD, Chio do A, Samson G. Management of bladder dysfunction. In: Braddom RL, editor. Physical medicine and rehabilitation. 4th e Philadelphia: Elsevier; 2011. p 611-3. Option b is correct. Continent urinary diversion is the best surgical option for this patient because she would be able to remain independent performing intermittent catheteriza tions. Cutaneous ileovesicostomy, a spec ific variant of incontine nt ur inary diversion, would maintain her independence, but she would have to adapt to using an external collection device. Bladder augmentation would not resolve the problem associated with performing catheterizations via her urethr Transurethral sphincterotomy is not a good option for females because few good external collection devices for women exist

59. Your 15-year-old patient wit h Duc hen ne mus cular dystrophy complains of new onset morning headaches. What is the most likely cause?(a) Neck extensor tightness(b) Hypercarbia(c) Migraines(d) Vision changes

59. (b) Migraines do not typically occur only in the morning. Neck extensor tightness usually occurs before the loss of ambulation in boys with Duchenne muscular dystrophy, which usually occurs before the age of 15 years. Vision changes usually do not cause morning headaches. Hypercarbia results from hypoventilation during sleep and is an early sign of impending respiratory failure.

6. Which statement regarding ventricular arrhythmias and ischemic heart diseas e is TRUE? (a) Patients prone to arrhythmias are recommended toexercise in the supine pos ition to decre ase myoca rdial oxy gen d eman d. (b) Approximately 50% of patients with a history of ventricular arrhythmias wi ll have a vent ricular a rrhythmia duri ng c ardiac rehabilitation. (c) An exercise stress test is used to determine target heartrate, which is se t at a level b elow whic h arrhyth mias are noted. (d) Patients with good exercise tolerance are less likely toexperience ventric ular arrhythmi as during cardiac rehab ilit ation than patients with poor exercise tolerance. Ref: Lindsay GM, Hanlon WP, Smith LN. Belcher PR. Experience of cardiac reha bilitation aft er corona ry artery surg ery: effects on health and risk factors. Int J Cardiol 2003;87: 67-73. Clinical Activity 1.7

6 (c) Exercise stress tests are used to screen for ventricular arrhythmias and to det ermine the targe t h eart rate, which i s set below the l evel at which arrhythmias are not ed. Upright exerci se pr oduces l ess myocardial oxygen demand than supine exercise, and therefore patients prone to arrhythmias are recommended to exercise in the upright position. Approximately 80% of patients with a history of ventricular arrhythmia will have a ventricular arrhythmia during inpatient cardiac rehabilitation. Patients with good exercise tolerance are more likely to experience ventricular arrhythmias during cardiac rehabilitation than patients with poor exercise tolerance.

6. A head and neck cancer patient completed external beam radiation therapy 1 week ago. Y ou inf orm the pati ent that pr eve ntion of ce rv ical so ft tissue contractures requires daily cervical range-of-motion exercises for at least:(a) 1 month.:(b) 6 months.:(c) 1 year.:(d) 5 years.

6 (d) The late effects of external beam radiation therapy can continue for at least 5 years. Fibrosis of fascia, skin, and muscles following radiation therapy is med iated by radi ation-induc ed microva scu lar inj ury. Da ily cervical range-of -mo tion exerc ise s should be performe d f or at leas t 5 ye ar s after com pleti on of trea tment to i ns ure th at p rogr ess ive fibr osi s d oes not oc cur.

6. Which finding is a functional physiological change seen in the elderly? (a) Increased drug-binding for highly-protein bound drugs (b) Doubling of D-dimer levels (c) Decreased erythrocyte sedimentation rate (d) Macrocytic anemia

6 Answer: (b) Commentary: D-dimer levels are shown to double with aging, especially among African Americans and functionally impaired individuals. Increased erythrocyte sedimentation rate and C-reactive protein have also been seen in the elderly. Although anemia occurs with increasing prevalence with aging, there is convicncing evidence that it is not a normal consequence of aging. Decreased drug-binding for highly protein-bound drugs in the elderly may lead to higher unbound or free drug concentrations. Ref: Clark GS, Siebens HC. Geriatric rehabilitation. In: DeLisa JA, Gans BM, Walsh, NE, editors. Physical medicine and rehabilitation: principles and practice. 4th ed. Philadelphia: Lippincott Williams & Wilkins; 2005. p 1534-5.

6. Which clinical tool BEST measures and predicts the safety of ambulation in older adults? (a) Berg Balance Scale (BBS) (b) Braden Scale (c) Timed Up and Go (TUG) test (d) Katz Index

6 Answer: (c) Commentary: The Berg Balance Scale (BBS) is a 56-point scale to evaluate performance during 14 common activities, such as standing, turning and reaching for an object on the floor. It does not rate walking. The Braden Scale is for predicting pressure sore risk, and is used to help determine the risk of skin breakdown or decubitus ulcer. In the Timed Up and Go (TUG) test, a patient is asked to rise from an armchair, walk 3 meters (10 feet), turn around, walk back to the chair, and sit down again (the score is the time in seconds it takes to complete these tasks). This test has high interrater and content reliability, and predicts whether a patient can safely walk outside alone. The Katz Index is widely used to measure independence in activities of daily living (ADLs), but does not include measures of mobility, such as walking or stair climbing. Reference: Stewart DG, Phillips EM, Bodenheimer CF, Cifu DX. Geriatric rehabilitation. 2. Physiatric approach to the older adult. Arch Phys Med Rehabil 2004;85 (Suppl 3):S7.

6. Which cardiac response is increased as a result of ae rob ic tr aining? (a) Oxygen consumption (VO2) (b) Maximal heart rate (c) Anginal threshold (d) Stroke volume at rest

6 Answer:(d) Commentary: After an aerobic training program, the angi nal t hre shold is unchanged. Oxygen consumption (VO2) at rest, and during any given submaximal load remains unchanged, while VO2 max is increased. The maximal heart rate also does not change, but the heart rate is lower both at rest and during any submaximal load (bradycardia of training). The stroke volume at rest is increased, reciprocal to the decrease in heart rate. Although angina threshold is unchanged, myocardial oxygen demand decreases relative to oxygen consumption, which allows more intense activity before the ischemic threshold is reached. Reference: (a) Moldover JR, Stein J, Krug PG. Cardiopulm ona ry ph ysiology. In: Gonzalez EG, Myers SJ, Edelstein JE, Lieberman JS, Downey JA, editors. Downey and Darling's Physiological basis of rehabilitation medicine, 3rd ed. Boston: Butterworth Heinemann; 200.; p 176-7. (b) Whiteson JH. Cardiac rehabilitation. In: Braddom RL, editor. Physical medicine and rehabilitation. 3rd ed. Philadelphia: WB Saunders; 2007. p 716-28.

6 A 45-year-old man with T4 paraplegia secondary to transverse myelitis is in acute inpatient rehabilitation. Your physical therapist reports to you that t he pa tie nt is asking whether he will ever be able to have sexual intercourse with his wife again. The next day you decide to address sexuality with your patient on morning rounds. What is the best way to approach this patient? a) Explain that there are important medical needs that should be addressed first. a. b) Offer to answer any questions that he has about his injury and sexual function. b. c) Provide him with specific examples of how to treat erectile dysfunction. c. d) Refer him to a therapist for intensive counseling on sexual techniques

6 Option b is correct. CORRECT: Commentary: The PLISSIT model is a framework for educational interventions related to sexuality. It is an acronym for 4 levels of intervention: Permissio n, Li mit ed Information, Specific Suggestions, and Intensive Therapy. "Permission" is the first level of intervention and refers to creating an atmosphere in which it is clear th at di scu ssion about sex will be well received. In this case, answer (b) is most consistent with this level of intervention. Reference: Consortium for Spinal Cord Injury Medicine. Sexuality and reproductive health in adults with spinal cord injury: A clinical practice guideline for hea lth-care professionals. Washington (DC): Paralyzed Veterans of America; 2010.

6 Which factor is a known risk for the development of depr ession following ampu tat ion ? a. Higher income levels b. Dysvascular etiology c. Phantom limb pain d. Paucity of comorbid conditions

6 Option c is correct. Depression is noted in 21% to 35% of persons with limb l oss regardless o f eti olo gy. P os ttr aum ati c s tres s diso rde r (PTS D) is m or e associ ated wit h tra uma tic e tio log y. Pha ntom limb pain and back pain are risk factors for developing clinical depression in persons with limb loss. Other risk factors include low income levels and more comorbid conditions.

6 The pronator quadratus, flexor pollicis longus, and the lateral half of flexor digitorum profundus are innervated by the anterior interosseous nerve. The br achioradialis is innervated by the r adia l nerve, the ex tensor ind icis proprius by the posterior interosseous nerve, and the extensor hallucis longus by the deep peroneal nerve. A 48-year-old man is undergoing rehabilitation after a heart transplant. His resting heart rate is consistently between 90 and 110 beats/minute. What is the most likely explanation for this? a) Prolonged bed rest post-operatively b) Low ejection fraction c) Denervation of the donor heart d) Anemia of chronic disease

6 Reference(s) Zafonte D, Pippin B, Munin M, Thai N. Transplantation medicine: a rehabilitation perspective. In: DeLisa JA, Gans BM, Walsh NE, editors. Physic al medicine and rehabilitation: principles and practice. 5th e Philadelphia: Lippincott Williams & Wilkins; 2010. p 1217-19. Option c is correct. Cardiac transplantation involves removing the diseased heart and leaving an atrial cuff which results in complete denervation of the donor heart, with loss of both afferent and efferent nerve conn ections. The do nor heart will not respond to vagolytic muscle relaxants, anticholinergics, anticholinesterases, digoxin, nifedipine or nitroprusside. The resting heart rate of a denervated heart varies between 90 and 110 beats/minute due to loss of vagal tone, leading to a small resting stoke volume.

6. A patient with multiple myeloma presents with a new pathologic fr acture of the supe rior pub ic ramus. Computed tomography scans and plain radiographs fail to reveal additional myelomatous involvement of the pelvic ring. When consulted regarding weight-bearing recommendations you advise (a) bed rest. (b) non-weight bearing on ipsilateral lower extremity. (c) weight bearing as tolerated. (d) toe touch weight bearing until surgical stabilization.

6. (c) Pathologic fractures of the pelvis that do not involve the acetabulum are generally treated nonsurgically. Patients may bear weight as tolerated. Aggressive analgesic management may be required. Mechanical insufficiency of the acetabulum can only be managed surgically.

60. A 65-year-old woman has right-sided hip pain secondary to osteoarthritis. She denies upper limb arthritis symptoms. Which prescription is the most appropriate? (a) Straight cane used in the left hand (b) Four point cane used in the right hand (c) Platform crutch used in the left arm (d) Lofstrand crutch used in the right arm

60 Answer: (a) Commentary: A straight cane should be used on the unaffected side to lessen the force exerted on the hip with pathology. A Lofstrand crutch is also known as a Lofstrand forearm orthosis. It includes a cuff placed along the lateral aspect of the forearm. Lofstrand crutches are often used bilaterally. Because it does not require the use of the hand or wrist, and does not apply pressure through them, a platform crutch is helpful for patients who need an assistive device and have wrist/ hand pain or weakness. Ref: Hennessey WJ. Lower limb orthoses. In: Braddom RL, editor. Physical medicine and rehabilitation. 3rd ed. Philadelphia: Elsevier; 2007 p 362-3

60. What shoe modification can be used to treat medial c omp ar tme nt knee osteoarthritis? (a) Rocker bottom sole (b) Solid ankle cushioned heel (c) Medial wedge (d) Lateral wedge

60 Answer: (d) Commentary: Medial compartment osteoarthritis results i n g en u v arum. A lateral wedge can help relieve pain by placing a valgus force at the knee. A medial wedge would exacerbate the problem. Solid ankle cushioned heel is a type of prosthetic foot. A rocker bottom sole is helpful for other conditions such as forefoot fractures, hallux rigidus, foot arthritis, and insensitive feet. Reference: (a) Hennessey WJ. Lower limb orthoses. In: B rad do m R L, editor. Physical medicine and rehabilitation. 3rd ed. Philadelphia: Elsevier; 2007. p 347. (b) Frey C. Shoes. In: Goldberg B, Hsu JD, editors. Atl as of or thoses and assistive devices. 3rd ed. St. Louis: Mosby; 1997. p 239.

60 A 49-year-old man with paraplegia secondary to transverse myelitis is having significant spasticity that is interfering with his transfers. He is taking 20m g of baclofen (Lioresal) 4 times dai ly. Your attendin g physician sugge sts starting another oral spasticity agent that mediates its activity though a GABA receptor mechanism. Which medication would be the most appropriate? a) Dantrolene (Dantrium) b) Diazepam (Valium) c) Metaxalone (Skelaxin) d) Tizanidine (Zanaflex)

60 Reference(s) Elovic E, Eisenberg ME, Jasey NN.. Spasticity and muscle overactivity as components of the upper motor neuron syndrome. In: Frontera WR, DeLisa , editors. Physical medicine and reh abil itation: prin ci ples and p racti ce. 5th e Philadelphia: Lippincott Williams and Wilkons; 2012. p. 1326-7. Option b is correct. Both baclofen and diazepam mediate their activity through the GABA system. Baclofen causes presynaptic inhibition of interneurons via GABAB receptors, where it causes hyperpolarization of the memb rane that pre ve nts the in flux of calcium and resultant release of neurotransmitter. Diazepam facilitates postsynaptic effects of GABAA, opening chloride channels and leading to hyperpolarization, which ultimately leads to an increase in presynaptic inhibition. Other spasticity agents work though different mechanisms. Dantrolene interferes with calcium release from the sarcoplasmic reticulum, and tizanidine is an alpha-2 agonist. The mechanism of action of metaxalone is not established, but is thought to be a general nervous system depressant.

60. What is the leading caus e of ampu tat ion in the upper extremity for men between the ages of 15 and 45 years?(a) Diabetes mellitus and/or per ipher al vascul ar disease(b) Other disease states (ex clud ing d iab etes, peripheral vascular disease)(c) Trauma(d) Tumor

60. (c) Trauma is the leading cause (approximately 75%) of acquired amputation in the upper extremity, occurring primarily in men between the ages of 15 and 45 years. Disease and tumors are responsible for about equal numbers of the remaining acquired upper-extremity amputations. In the lower extremity, disease states account for approximately 75% of all acquired amputations, with complications of diabetes and peripheral vascular disease accounting for the great majority of these, especially in persons age 60 years and over. Trauma is the next most common cause for lower extremity amputation (20%), followed by tumors (5%). Among persons between the ages of 10 and 20 years, however, tumor is the most frequent cause of all amputations in both the upper and lower extremities.

61. A patient presents with wrist and hand pain. The radiograph below demonstrates what finding? (a) Osteoarthritis of the carpal bones (b) Impaction syndrome (c) Capitate fracture (d) Scapholunate instability

61 Answer: (d) Commentary: This radiograph demonstrates a gap between the scaphoid and lunate seen on a anterior-posterior x-ray of the wrist. This finding is consistent with scapholunate dissociation. A measured distance of 3mm between the scaphoid and lunate is diagnostic for dissociation and instability. Ref: Finnoff J. Musculoskeletal problems of the upper limb. In: Braddom R. Physical medicine and rehabilitation. Philadelphia: Elsevier; 2007. p 844.

61. Imaged with musculoskeletal ultrasound, normal tendon structure looks a) hypoechoic, with hyperechoic septa. b) hypoechoic , with fascicular pattern. c) hyperechoic, with fibrillar echotexture. d) hyperechoic, with posterior acoustic shadowing.

61 Answer:(c) Commentary: Musculoskeletal ultrasound is an imaging modality that is able to identify and characterize various soft tissue structures. Normal tendons appear as hyperechoic(bright echo) structures with fibrillar or fiber-like pattern. Normal muscle appears as a hypoechoic(low echo) structure with hyperechoic septa. Bone appears as a very hyperechoic structure with posterior acoustic shadowing. Posterior acoustic shadowing is an artifact that refers to the anechoic region(no echo) deep to the bone surface. Reference: Jacobson J. Fundamentals of musculoskeletal ultrasound. Philadelphia: SaundersElsevier; 2007. p 4.

61 Which factor signals an increased risk for underlying malignancy in dermatomyositis? a) Male gender b) Age younger than 40 years c) Elevated C-reactive protein d) Erythroderma (exfoliating dermatitis)

61 Reference(s) Baer A, Wortman R. The risk of malignancy in patients with dermatomyositis and polymyositis. Pu lmonary manifestations of inflammato ry m yopathies. In : Kagen L, e ditor . The inflammatory myopathies. New York: Humana Press;2009. p 307-18. Option c is correct. Dermatomyositis is associated with underlying malignancy in approximately 25% of patients. The erythrocyte sedimentation rate and C-reactive protein are gen erally elevated in persons with unde rlyi ng malignancy . The risk o f mal ignancy is higher in older patients and gender does not appear to be a risk factor. Erythroderma may be a presenting symptom of dermatomyositis but does not appear to be a risk factor for malignancies. Several other features of the rash, such as cutaneous necrosis, ulceration or vasculitis, may be associated with underlying malignancy.

61. In which case is a corti cost eroid in jectio n contraindicated?(a) A police officer with pl anta r fas cii tis wh o will return to his usual street duties 2 days after treatment(b) A professional tennis pl ayer with ac ute el bow pain associated with her backhand 1 week before a tournament(c) A drywall hanger with ch roni c sho uld er pai n who has had 1 prior injection 6 months ago with good results(d) A diabetic weightlifter with suba cut e medi al knee pain that is warm to touch compared to the other leg

61. (d) Patients with diabetes mellitus are at risk for serious infection and for systemic effects of absorbed corticosteroids. Injection into an infected joint, tendon or bursa is contraindicated. Multiple injections should not be performed unless clear improvement has been demonstrated. More than 3 steroid injections are rarely indicated. There is a risk of tendon rupture in patients who return to usual activity too rapidly. Plantar fasciitis and lateral epicondylitis respond well to steroid injection.

62. Six months after a moderate traumatic brain injury, a 32-year-old woman complai ns o f day tim e s omnol ence . H er me dic al work-up is ne gati ve. S he has normal sleep patterns. The medication you would most likely consider in this case is:(a) donepezil.:(b) buspirone. (c) tolcapone. (d) modafinil.

62 (d) From the information given, it is clear that this patient is functioning well overall. She has some difficulty staying awake. Of the answers given, moda fin il i s the me dicat ion most ap pr opriate to hel p wi th a ler tness during the day. Don epez il is an acetylch oli nesterase in hi bit or us ed mos t often to im prove m emory. B uspir one i s use d t o de crea se anx iety. To lca pon e is a newer dopaminergic agent that has not been studied in populations with brain injury.

62. A neurologist refers a patient to you with Parkinson disease and poor gait. What treatment strategy is recommended to prevent frequent falls? a) Methylphenidate medication trial to increase attention and concentration b) Physical therapy with balance training and cueing strategies c) Referral to a neurosurgeon for implantation of a deep brain stimulator d) Maximized levodopa medication to improve balance control

62 Answer: (b) Commentary: Physical therapy with cueing strategies, such as rhythmic auditory stimulation with a metronome and balance and strength training are shown to be useful in improving gait and decreasing falls. Treadmill training is still in its infancy and its role in improving gait is unclear, although early studies are positive. The use of methylphenidate in initial trials was positive but a recent randomized, double blinded study using methylphenidate showed no improvement in gait. The use of deep brain stimulation is very inconsistent in its effect on balance and gait and further study is needed to optimize type of stimulation and to define new targets for stimulation. Levodopa can improve gait, but can also cause a worsening of gait and balance, possibly due to drug-induced dyskinesias. Reference: (a) Boonstra TA, van der Kooij H, Munneke M, Bloem BR. Gait disorders and balance disturbances in Parkinson's disease: clinical update and pathophysiology. Curr Opin Neurol . 2008;21:461-471.(b) Mehrholz J, Friis R, Kugler J, Twork S, Storch A, Pohl M. Treadmill training for patients with Parkinson's disease. Cochrane Database Syst Rev. 2010;(1): CD007830. DOI:10.1002/14651858.CD007830.pub2.(c) Espay AJ, Dwivdei AK, Payne M, Gaines L, Vaughan JE, Maddux BN, et al. Methylphenidate for gait impairment in Parkinson disease: a randomized clinical trial. Neurology 2011;76:1256-1262.

62. An 18-year-old man incur s a mild tra umatic brain injury in a bar dispute. He presents 2 months later complaining of persistent headaches originating in the back of the neck and radiating circumferentially forward. After a thorough history and physical examination, you feel his principal problem is myofascial in origin; specifically, two offending trigger points are found. Your first approach to treatment would be(a) amitriptyline 100mg at b ed t ime.(b) injection with botulinum tox in.(c) establishing an outpatie nt c ognit ive behav ioral program through psychology to address a subacute pain syndrome.(d) injection with 1% lidoca ine.

62. (d) Post traumatic headaches are a common symptom after cervicocranial trauma. The differential diagnosis includes cervical disease, occipital neuralgia and migraines, and myofascial pain. In this scenario, points that reproduce the headaches should be treated either with or without local anesthesia. This helps to reduce pain, inhibit the muscle contracture band, and enhance local muscle blood flow.

63. A 40-year-old woman with irritable bowel syndrome and tension headaches co mplains of incre asing fa ti gue a nd d iffus e muscl e soren ess i n her ne ck , shou lde rs, an d low back. She has a nonfocal neurologic examination. She does not feel rested in the morning. The most appropriate initial recommendation is (a) lorazapam (Ativan) at bedtime. (b) amitriptyline (Elavil) at bedtime. (c) zolpidem (Ambien) at bedtime. (d) acetaminophen with codeine (Tylenol #3 ) at bedtime.

63 (b) This patient has fibromyalgia. Neuropathic medications such as trycyclic antidepressant medications are recommended.

63. Which of the following may be associated with a subacromial corticosteroid inje ctio n?:(a) Dermal keratinification:(b) Localized osteopenia:(c) Tendon rupture:(d) Dermal hyperpigmentation

63 (c) Intra-articular corticosteroid injections have enough systemic absorption that suppression of the adrenal hypopituitary axis may be seen with repeated i nje ctio ns. M ore loca lized delet er ious eff ect s i nclu de s kin depigmentat ion, soft tis sue atroph y, steroid art hropathy, po st inj ectio n flar e, and tendon rupt ur e.

63. A 40-year-old woman with a h istor y o f irri table bowel syndrome and tension headaches complains of increasing fatigue and diffuse muscle soreness in her neck, shoulders, and low back. She has a nonfocal neurologic examination. Initial treatment should include(a) 30mg twice daily of sust aine d rel eas e morp hine.(b) 60mg prednisone with rap id t aper.(c) 1mg lorazepam twice dail y.(d) 25mg nortriptyline each nigh t.

63. (d) Medications that promote sleep, such as low dose tricyclic antidepressant medications, have been shown to be the most helpful pharmacologic approach to patients with fibromyalgia. (This question has been eliminated from the exam, therefore, it was not scored.)

75. A 37-year-old man presents to your office with a Grade 2 sacral pressure sore which appears clean, with no necrotic tissue and only a slight amount of serosanguinous drainage. In order to optimize wound healing, you suggest (a) allowing a protective eschar to form. (b) wet to dry gauze dressings. (c) vacuum-assisted closure. (d) an occlusive dressing.

75 Answer: D Commentary: An occlusive dressing will help to maintain a moist environment, which is ideal for wound healing. Allowing eschar to form will inhibit healing. Wet to dry dressing changes are used only when debridement is required. Vacuum-assisted closure is usually used on grade 3 and grade 4 wounds. Ref: Priebe MM, Martin M, Wuermser LA, Castillo T, McFarlin J. The medical management of pressure ulcers. In: Lin VW, editor. Spinal cord medicine: principles and practice. New York: Demos; 2003. p 567-89.

64. Which symptom is the most predictive of cardiac dise ase i n D uchenne muscular dystrophy? (a) Palpitations (b) Syncope (c) Dyspnea (d) Cachexia

64 Answer: (c) Commentary: The most frequent predictive symptom is dysp nea . Ab sence of exertion dyspnea from lack of physical activity allows myocardial impairments to remain clinically silent and difficult to detect. A high index of suspicion is required. Electrocdardiogram abnormalities in both Duchenne and Becker muscular dystrophy patients are attributed to progressive fibrosis of the cardiac conduction system and impairment in the cardiac autonomous nervous system. Palpitations and syncope will be related to the conduction abnormalities and occurs late in the disease process. Cachexia is a late finding that occurs when feeding becomes difficult as a result of heart failure and dyspnea. Reference:(a)Manzur AY, Kinali M, Muntoni F. Update on the m ana gement of Duchenne muscular dystrophy. Arch Dis Child 2008;93:986-90.(b) Kilmer DD. Myopathy. In: DeLisa JA, Gans BM, Walsh NE, editors. Physical medicine and rehabilitation: principles and practice. 4th ed. Philadelphia: Lippincott-Raven; 2005. p 916-21.(c) McDonald CM. Neuromuscular diseases. In: Alexander MA, Matthews DJ, editors. Pediatric rehabilitation: principles and practice. 4th ed. New York: Demos; 2010. p 277-333.

64 Which condition is a progressive neuromuscular disease that destroys upper and lower motor neurons? (a) Transverse myelitis (b) Amyotrophic lateral sclerosis (c) Kugelberg-Welander disease (d) Multiple sclerosis

64 Answer: B Commentary: Amyotrophic lateral sclerosis (ALS) is perhaps the most severe of all the major neuromuscular diseases. It is a rapidly progressive disease that destroys both upper and lower motor neurons. This destruction results in diffuse muscular weakness and atrophy. Unlike most primary nerve disorders, ALS also produces spasticity because of the loss of upper motor neurons. This loss creates unique clinical management issues. Kugelberg-Welander disease has a very slow progressive course with no upper motor neuron findings of spasticity. Multiple sclerosis and transverse myelitis are demyelinating diseases of the central nervous system with symptoms that mimic lower motor neuron findings, such as weakness. Ref:(a)Krivickas LS, Carter GT. Motor neuron disease. In: DeLisa JA, Gans BM, Walsh, NE, editors. Physical medicine and rehabilitation: principle and practice. 4th edition. Philadelphia: Lippincott Williams & Wilkins; 2005. p 932.(b)Kraft G. Rehabilitation of persons with multiple sclerosis. In: Braddom RL, editor. Physical medicine and rehabilitation. 3rd ed. Philadelphia: Elsevier; 2007. p 1224.(c)Multiple sclerosis and allied demyelinative diseases. In: Adams RA, Victor M, editors. Principles of neurology. 6th ed. New York: McGraw-Hill;1997. p 90225.(d)Carter GT. Rehabilitation management of neuromuscular disease. eMedicine [serial on line] 2006. October [cited April 24, 2007] http://www.emedicine.com/pmr/topic233.htm. Accessed April 24, 2007.

64 A 65-year-old right handed man has a pure Wernicke aphasia without hemiplegia after a stroke. The location of his stroke is in which branch of the left midd le cerebral artery? a) Posterior branch of the lower division b) Anterior branch of the lower division c) Posterior branch of the upper division d) Anterior branch of the upper division

64 Reference(s) (a) Mohr JP, Young WL. Anatomy and physiology of the vascular supply to the brain. In: Gonzalez EG, Meyers SJ, editors. Downey and Darl ing's physiol og ical basis of r ehabilitation medicine. 3rd e Boston: Butterworth-Heinemann; 2001. p 17-28. (b) Harvey RL, Roth EJ, Yu DT, Celnik P. Stroke syndromes. In: Braddom RL, editor. Physical medicine and rehabilitation. 4th e Philadelphia: Saunders; 2011. p 1184-1188. Option a is correct. Pure Wernicke aphasia without hemiplegia is seen with occlusion of the posterior branch of the lower division of the middle cerebral artery supplying the he misphere dominant for speech. Broca apha sia is seen w it h occlusio n of the anterior branch of the upper division. A literal paraphasia, featuring speech errors of mispronounced words, is limited to the posterior branch of the upper division of the middle cerebral artery.

7. Cyclobenzaprine is a medication that is use d t o treat acute musculoskeletal pain. While the exact mechanism of action is unknown, its structure and side effects are similar to what class of drug? (a) Central alpha2-adrenergic agonist (b) Tricyclic antidepressant agent (c) Antihistamine (d) ?-aminobutyric acid agonist

7. (b) Cyclobenzaprine is structurally similar to tricyclic antidepressants and was first studied as an antidepressant. While its exact mechanism of action is unknown, it is presumed to work at the level of the brainstem or higher with a generalized sedative effect. Tizanidine is a central alpha2-adrenergic agonist. Orphenadrine is an antihistamine. Benzodiazepines, such as diazepam, and baclofen are ?aminobutyric acid agonists. Ref: Sullivan WJ, Panagos A, Foye PM, Sable AW, Irwin RW, and Zuhosky [A1] JP. Industrial medicine and acute musculoskeletal rehabilitation. 2. Medications for the treatment of acute musculoskeletal pain. Arch Phys Med Rehabil 2007;88(3 Suppl):S10.

65 Positive sharp waves and fibrillation potentials are most likely to be seen in which condition? a) Anterior horn cell disease b) Steroid myopathy c) Small fiber neuropathy d) Cerebellar ischemia

65 Reference(s) Dumitru D, Zwarts MJ. Needle electromyography. In: Dumitru D, Amato AA, Zwarts MJ, editors. Electrodiagnostic med icine. 2nd e Philadelphia: Hanley & Belf us; 2002. p 2 74 -276 Option a is correct. Positive sharp waves (PSWs) and fibrillation potentials are seen in certain disorders of the muscles, neurogenic disease, and neuromuscular disorders. These may include but are not limited to infl ammatory myos it is, inclus ion b ody myositis, rhabdomyolysis, anterior horn cell disorders, radiculopathies, plexopathies, peripheral neuropathies, myasthenia gravis, and botulism. Steroid myopathy does not result in muscle necrosis and thus abnormal spontaneous activity is not observe Fibrillation potentials and PSWs are not characteristic for central nervous system pathology. Small fiber neuropathies will be undetected on needle EMG. Option b is correct.Interstitial lung disease occurs in up to 65% of patients with myositis and is a significant contributor to morbidity and mortality. Pat ients with myositis have further pul mona ry complicati on s with res pect to muscle weakness and vascular disease resulting in secondary complications of aspiration pneumonia, infection, respiratory muscle failure, pulmonary edema and pulmonary arterial hypertension. Spontaneous pn.

66 Upper extremity exercise (eg, crutch walking) leads to a greater increase in heart rate and blood pressure compared with lower extremity activity (eg, normal walking) due to the (a) smaller upper extremity muscles, which contract at a higher maximal percentage. (b) proximity of the upper extremities to the heart and major blood vessels. (c) upper extremities having to overcome the effect of gravity. (d) greater range of motion of the upper extremities compared to the lower ones.

66 Answer: (a) Commentary: Upper extremity work leads to greater increases in heart rate and blood pressure. When a muscle contracts with a given percentage of its maximum force, its effect on blood pressure is about the same as during the same percentage of contraction of any other muscle. The smaller muscles in the upper extremity contract more, and stimulate the cardiovascular system more relative to the larger lower extremity muscles. Ref: Gonzalez EG, Edelstein JE. Energy expenditure during ambulation. In: Gonzalez EG, Myers SJ, Edelstein JE, Lieberman JS, Downey JA, editors. Downey and Darling's physiological basis of rehabilitation medicine. 3rd ed.Boston: Butterworth Heinemann; 2001. p 420.

66. The surgical transplant service requests physical therapy recomm endations for a pa tient be ginning to reject a single lung transplant. In addition to pursed lip and diaphragmatic breathing, recommendations should include (a) a focus on passive stretching and isometrics. (b) telemetry monitoring while out of bed. (c) an aerobic exercise program. (d) minimizing of supplemental oxygen.

66. (c) During rejection, lung transplant patients should perform basic breathing exercises. They can continue to perform aerobic exercise as tolerated with oxygen supplementation.

66. Which pulmonary parameter is most commonly followed in a pati ent with amyotrophic lateral sclerosis (ALS)? (a) Arterial blood gas (ABG) (b) Oxygen saturation (O2 sat) (c) Forced expiratory volume in 1 second (FEV1) (d) Vital capacity (VC)

66. (d) Vital capacity should be monitored in patients with neuromuscular disease such as ALS. The forced vital capacity is convenient to follow disease progression, and it correlates with disability. FEV1 is normal. Blood gases remain normal until the patient is in near respiratory arrest. Hypercapnia precedes hypoxia, so monitoring oxygen saturation is not helpful.

67. How does work conditioning differ from work hardening? Work conditioning is:(a) a maintenance exercise program.:(b) activities simulating the worker's tasks.:(c) training the worker for a specific job.:(d) aerobic training that may not be job specific.

67 (d) Work conditioning is the physical conditioning portion of work hardening. It has been referred to as an aerobic training program for patients with less com plex and mor e chr onic condit io ns. Ofte n t hes e pa tien ts are not bein g retrained t o re turn t o a specifi c j ob. In a wor k har denin g prog ra m the injur ed wo rk er is gr adual ly st re ngt hen ed a nd r eco ndi tione d t o t he functio nal capacity level required to perform a given job. Work hardening has also been referred to as work simulation, work readiness training, and work rehabilitation.

67. The purpose of the Health Insurance Portability and Accountab ilit y Act (HIPAA) is to (a) ensure that a patient's medical record is available to health car e providers as directed by the patient. (b) allow qualified physicians access to the patient's m edical re cord . (c) allow a lawyer access to a medical record only if li tigation is p ending. (d) prohibit the release of confidential health informat ion to in sura nce carriers.

67. (a) The purpose of the Health Insurance Portability and Accountability Act (HIPAA) is to ensure that a patient's medical record remains private, but is available to health care providers as directed by the patient. A non-treating physician, lawyer, or insurance company may have access to the record with written authorization by the patient or guardian. There are no stipulations about a physician's qualifications with regards to medical information access.

67. A 50-year-old insulin-de pend ent d iab etic t ruck driver with a long-standing history of smoking is sent to you after experiencing left hand weakness following a long haul across the country. Signs and symptoms include weakness in hand intrinsics and wrist, with numbness in the ring and little fingers. Exam demonstrates normal upper extremity reflexes. Motor strength is normal for deltoid, biceps, triceps, pronator teres, and opponens pollicus. There is moderate weakness of the abductor digiti minimi. Your diagnosis is(a) C7 radiculopathy.(b) carpal tunnel syndrome.(c) ulnar compression at Guy on's cana l.(d) ulnar neuropathy at the elbo w.

67. (d) The patient's symptoms and the physical exam findings are consistent with an ulnar neuropathy at the elbow.

68. What is the earliest electrophysiologic abnormality seen in generalized myasthe nia gr avi s?:(a) Increased jitter on single fiber electromyography of the extensor digitorum com muni s.:(b) Blocking on single fiber electromyography of the extensor digitorum communis.:(c) A 10% decrement of compound motor action potential (CMAP) amplitude with 2-3Hz repe ti tiv e s tim ulati on, rec ordin g f rom the abd uct or dig iti m inimi.:(d) A 10% decrement of CMAP amplitude with 2-3Hz repetitive stimulation, recording from t he fro nta lis.

68 (a) Increased jitter on single-fiber electromyography is the earliest abnormality seen in myasthenia gravis. The other abnormalities noted are seen later in th e di sease pr ocess .

69. A 9-year-old girl with an L1 ASIA class A spinal cord injury that occurred at a ge 5 y ear s p res ents in y our offi ce wit h a 1-da y h is tory of a swollen left leg. History is that she woke up with the swollen leg the day before. There is no history of trauma, fever, or shortness of breath. On examination, you find a prepubertal girl in no distress with normal vital signs. Upper extremities are normal. Lower extremities have moderate spasticity and no voluntary movement. Skin is normal. The left leg is warm and swollen from the ankle to the knee. There is no sensation in the legs. Which test is most likely to yield the correct diagnosis?:(a) Bone scan:(b) Plain radiograph:(c) Venous Doppler study:(d) White blood cell count with differential

69 (b) Deep venous thromboses (DVTs) which can be diagnosed by Doppler study usually occur in the first 3 months after spinal cord injury (SCI) and are rare in pr epub ertal ch ildre n. In lower l eg DVTs the fo ot a nd l eg are usually swollen. Hete roto pic os sif ication (HO ), which can b e d etect ed by bo ne scan, oc curs in about 3 % of child re n w ith SCI and ha s o nset an ave rag e of 14 months after injury. Heterotopic ossification most commonly involves the hip. Cellulitis is usually associated with skin lesions and usually involves a discrete area. A fracture is the most likely cause of swelling in this case and can be diagnosed by plain radiographs.

69. Which of the following is part of neurodevelopmental therapy (NDT)? (a) Promotion of primitive reflexes (b) Use of taping and icing (c) Strengthening exercises (d) Facilitating automatic reactions

69 (d) Neurodevelopmental therapy, developed by Bobath, emphasizes inhibition of reflex patterns, normalizing tone, and facilitating automatic reactions. The therapy do es not include strengthening exercises.

69 A 1-year-old boy presents with marked weakness. Parents report a weak cry and cough since birth, and the child cannot sit independently. Exam findings include a bell-shaped thorax, hypotonia, some movement of the hands and feet but minimal movement at the hips and shoulders, and there are tongue fasciculations. The diagnosis is best confirmed by (a) genetic analysis. (b) muscle biopsy. (c) electromyography. (d) repetitive nerve stimulation.

69 Answer: (a) Commentary: This child's presentation is typical for spinal muscular atrophy (SMA) type I, also known as Werdnig-Hoffmann disease. SMA is the second most common neuromuscular disease of childhood, occurring with an incidence of 1:6,000. Deletion of the survival motor neuron gene leads to degeneration of anterior horn cells and can be detected in over 90% of children with SMA. Prior to availability of genetic diagnosis, EMG and muscle biopsies were utilized. Repetitive nerve stimulation has been used in the investigation of botulism and congenital myasthenia gravis but is not helpful in SMA. Ref: Han JJ, McDonald CM. Diagnosis and clinical management of spinal muscular atrophy. Phys Med Rehabil Clin N Am 2008;19:661-80.

69. Which insult is the most lik ely c aus e of s pastic diplegic cerebral palsy?(a) Intrauterine stroke(b) Hyperbilirubinemia in th e ne onata l p eriod(c) Postnatal intraventricul ar h emorr hag e(d) Perinatal asphyxia

69. (c) Spastic diplegic cerebral palsy occurs most commonly in premature infants who have had an intraventricular hemorrhage during the neonatal period. Intrauterine stroke causes hemiplegia. Neonatal hyperbilirubinemia most commonly causes athetosis. Birth asphyxia is more commonly associated with spastic quadriplegic cerebral palsy.

"

69. In children with spastic cerebral palsy, wh ic h approach strengthens weak muscles? (a) Ankle-foot orthotics (b) Tendon transfer surgery (c) Intrathecal baclofen (d) Functional training" 69 (d) Children with cerebral palsy often have weakness as part of their disorder. Treatments such as bracing, tendon lengthening or transfers, and medications such as botulinum toxin or intrathecal baclofen add to this weakness. Strengthening programs or functional training programs can help to strengthen weak muscles. Ref: Damiano DL, Dodd K, Taylor FT. Should we be testing and training muscle strength in cerebral palsy? Dev Med Ch Neurol 2002;44:68-72.

7. A 45-year-old auto mechanic with a history of low back pain and a herniated disc pr esents t o you wi th 2-da y hi story o f leg p ain, numbn es s i n t he lower ex tre mit y, stumbling, and 2 episodes of urinary incontinence. Your recommendation includes (a) Ibuprofen, ice, relative rest, pelvic tilts, and repeat magnetic resonance imaging this week. imaging this we ek. (b) Lumbar epidural steroid injection and physical therapy. (c) Immediate magnetic resonance imaging and referral to a spine surgeon. (d) Acetaminophen, ultrasound, light duty for 5 days, and physical therapy.

7 (c) In this patient with known herniated disc now with urinary i ncontinence the primary concern should be to evaluate for cauda equina syndrome. The MRI is necessary to det ermine if an extruded disc is causing the current symptoms. Surgery i n the acute set ting is ess ential to achiev e neurologic rec ove ry.

7. Which physiologic changes in cardiac function occur after heart transplanta tion? (a) Peak heart rates are typically 25% higher than those ofage-matched control s. (b) Resting bradycardia near 50bpm occurs because ofsympathetic denervation. (c) Increased stroke volume occurs because of improvedsystolic function. (d) Diastolic dysfunction occurs because of increasedmyocardial stiffness. Ref: (a) Daida H, Squires RW, Allison TG, Johnson BD, Gau GT. Sequential ass essment of exe rcise tol erance in hear t tr ansplantation compared with coronary artery bypass surgery after phase II caridan rehabilitation. Am J Cardiol 1996;7:696-700. (b) Bartels MN. Cardiopulmonary assessment. In: Grabois M, Garrison SJ, Hart KA, Lehmkuhl LD, edito rs. Physi cal m edic ine and rehabilitation: the complete approach. Malden: Blackwell Science; 2000. p 351-72. Clinical Activity 1.9

7 (d) After cardiac transplantation, stroke volume may be reduced because of diastoli c dysfunction fr om increased m yocardi al stiffness in t he new heart. There is a rest ing tac hycardia near 1 0 bpm caused by vagal inhibition, and peak heart rate is 20% to 25% lower than that of age-matched controls.

82. Risk factors for heterot opic ossi fic ation in a 27-year-old man with severe traumatic brain injury include(a) total parenteral nutriti on.(b) prolonged coma.(c) seizure disorder.(d) hypotonicity.

82. (b) Risk factors for the development of heterotopic ossification include prolonged coma, immobilization, and limb spasticity.

7. A 67-year-old man who had a stroke is being discharged from the hospital. His 32-year-old nephew plans to care for him at home, but is unfamiliar with the Family Medical Leave Act (FMLA). How does FMLA apply to the nephew? (a) He does not qualify since he is not the spouse or an immediate family member. (b) FMLA only applies to the patient, not to the caregiver. (c) He will be paid 66% of his usual salary while he is taking FMLA. (d) If he takes FMLA, he may lose his employer-sponsored health insurance.

7 Answer: (a) Commentary: The Family Medical Leave Act (FMLA) entitles eligible employees of covered employers to take unpaid, job-protected leave for specified family and medical reasons with continuation of group health insurance coverage. Eligible employees are entitled to 12 work weeks of leave in a 12-month period for any of the following reasons: 1. the birth and care of the newborn child of an employee 2. the placement with the employee of a child for adoption or foster care 3. to care for an immediate family member (spouse, child, or parent) with a serious health condition 4. to take medical leave when the employee is unable to work because of a serious health condition Employees are eligible for leave if they have worked for their employer at least 12 months, have worked at least 1250 hours over the past 12 months, and work at a location where the company employs 50 or more employees within 75 miles. Reference: United States Department of Labor, Wage and Hour Division web site. http://www.dol.gov/dol/topic/benefits-leave/fmla.htm. Accessed June 18, 2011.

7. When considering risk of cumulative trauma in an older individual, it is important to know the typical decreases in strength that occur with aging. Between ages 70 and 80 people typically lose what percentage of their strength? (a) 5 (b) 15 (c) 30 (d) 50

7 Answer: (c) Commentary: Between the ages of 70 and 80 people typically lose 30 percent of their strength. Muscular weakness occurs after age 30 in association with generalized muscle fiber atrophy, decreased muscle density and increased intramuscular fat. Between the ages of 50 and 70 people typically lose 15 percent of their strength. Ref: Zuhosky JP, Irwin RW, Sable AW, Sullivan WJ, Panagos A, Foye PM. Industrial medicine and acute musculoskeletal rehabilitation. 7. Acute industrial musculoskeletal injuries in the aging workforce. Arch Phys Med Rehabil 2007;88(3 Suppl):S35.

7. Injured workers with acute low back pain treated with hi gh do sage opioids compared to low-dose or nonopioid medications demonstrated which outcome? (a) Lower overall medical costs (b) Same duration of disability (c) Higher risk for surgery (d) Shorter duration of opioid use

7 Answer: (c) Commentary: Injured workers with acute low back pain who re ce ive d higher dosages of opioids in early treatment had adverse outcomes compared to patients given no or low-dose opioids. In the high-dose opioid group, adverse outcomes included higher medical costs, prolonged disability, higher risk for surgery, and continued use of opioids. The high-dose opioid group was disabled 69 days longer than the non-opioid group, had a 3 times greater risk for surgery, and a 6times-greater risk of receiving long-term opioids. The severity of the low back injury was a strong predictor of all outcomes. Reference: Webster BS, Verma SK, Gatchel RJ. Relationshi p b et wee n early opioid prescribing for acute occupational low back pain and disability duration, medical costs, subsequent surgery and late opioid use. Spine 2007;32(19):2127-32.

7 A 47-year-old woman injures her back on the job. Her supervisor inquires about the injury and creates a document with the employee's name, outlining how the injury occurred and where the employee is experiencing pain. The information in the document is protected by the (a) Health Insurance Portability and Accountability Act (HIPAA). (b) The Joint Commission (JC). (c) Americans with Disabilities Act (ADA). (d) United States Supreme Court.

7 Answer: A Commentary: Once the document was created by the supervisor and it contained individually identifiable health information it became information that is protected by HIPAA. The term 'individually identifiable health information' means any information, including demographic information collected from an individual, that: (A) is created or received by a health care provider, health plan, employer, or health care clearinghouse; and (B) relates to the past, present, or future physical or mental health or condition of an individual, the provision of health care to an individual, or the past, present, or future payment for the provision of health care to an individual, and identifies the individual. Ref: Centers for Medicare and Medicaid Services Website. http://www.cms.hhs.gov/HIPAAGenInfo/Downloads/HIPAALaw.pdf. Accessed May 26, 2008.

7 Which ergonomic recommendation for computer stations is NOT appropriate for an injured worker with upper extremity cumulative trauma disorder? a) Neutral position of the wrists a. b) Forearm position horizontal to the floor b. c) Elevated positioning of the mouse c. d) Mid-line positioning of the keyboard

7 Option c is correct. Commentary: Ergonomic evaluation of the computer workstation is important for both the prevention and the treatment of cumulative trauma disorders. All t he li ste d ergonomic recommendations are appropriate, with the exception of elevated placement of the mouse. More midline and level mouse placement is recommended, especially in cases of de Quervain tenosynovitis. Reference: Foye PM, Cianca JC, Prather H. Industrial medicine and acute musculoskeletal rehabilitation. 3. Cumulative trauma disorders of the upper limb in co mpu ter users. Arch Phys Med Rehabil 2002;83(3 Suppl):S14-5.

7 A journalist is interested in writing an article about work options for the chronically disabled population. When discussing vocational rehabilitation, you inform him that a) governmental rehabilitation agencies are mostly funded by state resources. b) the emphasis is to provide services for patients with mild disabilities. c) tax incentives cannot be offered to employers who offer job training. d) after completion of training and placement in a job for 60 days, a file is usually close

7 Reference(s) Klingbeil H, Sperber K. Employment of persons with disabilities. In: Braddom RL, editor. Physical medicine and rehabilitation. 4th e Philadelphia: Saunders Elsevier; 2011. p 759-61. Option d is correct. The Rehabilitation Act of 1973 authorized federal funding for state rehabilitation agencies to provide a variety of services to qualified persons with disab ilities. The federal government supp lies 80% of the f un ding for s tate vocational rehabilitation agencies, whereas the states must provide the remaining 20%. State agencies administer the programs under the Rehabilitation Services Administration in the Department of Education. The intent of the Rehabilitation Act was to provide services to persons with disabilities, with emphasis placed on serving those with more severe disabilities. Tax incentives for potential employers can help convince industry to offer training. Once an individual has completed training and has been placed for 60 days, the state vocational rehabilitation agency considers the case a "success" and closes its file. No follow-up evaluation is typically provide

70. When applying a static resting hand splint to a person who had a stroke wi th upper limb pa ralys is, w hat i s th e app ro priat e joint posi ti on to ma intain th e h and i n a functional position? (a) 20° of wrist flexion (b) 90° of metacarpophalangeal flexion (c) Neutral position or slight flexion of the distal interphalangeal joints (d) 30° of thumb opposition across the palm

70 (c) A resting hand splint is designed to maintain a position of function in a hand that is weak or paralyzed. It is applied on the volar surface and extend s from the finger tips to the proximal third of the forearm. The wrist is typically p la ced in slight e xtensio n. T he met aca rpophal angeal joints ar e p lac ed in sl ight f le xion an d th e in te rph al an geal jo int s a re pl ac ed i n a neut ral po sitio n o r in sligh t flexion. T he thumb is supported in a position between palmar and radial abduction.

70. What is the best-established benefit of a microprocessor-controlled knee unit, comp ar ed to a c onven tion al pneum ati c k nee unit ?:(a) It provides both swing and stance phase control.:(b) It allows greater knee flexion to perform bending and lifting activities.:(c) It allows running at faster speeds.:(d) It provides a more natural and symmetric gait pattern.

70 (d) Microprocessor controlled knee units use a computerized system to automatically adjust the knee unit's resistance over a wide range of gait speeds. With th is a utoma tic cont rol, there is greater co nsi sten cy a nd reliability in the knee m ovem ent du rin g the ga it cycle. Th ese b ene fits give t he amputee gr eater c onfidenc e and impr ov e s win g ph ase res pon siven ess an d g ait sym metry, as well. Negative aspects of the microprocessor controlled knee units are heavier weight and greater expense. These units have not been shown to improve running speed.

71. A 45-year-old man presents to his physician with complaints of posterior heel p ain fo r s eve ral week s. T he pain is wor se when he fi rst beg in s to ambulate after resting. He has a noticeable limp. He denies any recent trauma. On examination, he is exquisitely tender along the medial and lateral aspects of the calcaneus and along the Achilles tendon at its most distal portion. The area is mildly swollen and warm to touch. What is the most likely diagnosis?:(a) Achilles tendon rupture:(b) Plantar fasciitis:(c) Posterior calcaneal nerve entrapment:(d) Retrocalcaneal bursitis

71 (d) This scenario is most likely retrocalcaneal bursitis. This condition is seen in middle aged to elderly persons, and may occur with overuse. A limp is co mmo n an d pai n i s usu ally worse wi th first ac tiv ity afte r r est. Swellin g and local t ende rness may be seen , a nd a "pum p b um p" may b e pres en t. This bum p is a prominen ce cr eated b y i nfl amma tion as soc iated wi th sho e wear. The local swelling is due to inflammation of the retrocalcaneal bursa. An acute Achilles tendon rupture is associated with a positive Thompson test, which consists of compressing the calf and causing the foot to plantarflex. If the tendon is ruptured, the foot will not plantarflex. The pain is not located in the bottom of the foot as would be seen with plantar fasciitis. Posterior calcaneal nerve entrapment would not cause swelling, nor would it be warm to touch.

72. Which electroencephalogram pattern is associated with a better prognosis after traumatic brain injury? (a) Low amplitude delta activity (b) Burst suppression (c) Isoelectric activity (d) Spindle pattern

72 Answer: D Commentary: Favorable electroencephalogram (EEG) patterns after a traumatic brain injury are normal activity, rhythmic theta activity, frontal rhythmic delta activity, and spindle pattern. Poor prognosis is associated with epileptiform activity, nonreactive, low amplitude delta activity and burst suppression patterns with interruption of isoelectricity. Complete isoelectric EEG activity had the highest mortality. Ref: Lew HL, Lee EH, Pan SSL, Chiang JYP. Electrophysiologic assessment techniques: Evoked potentials and electroencephalography. In: Zasler ND, Katz DI, Zafonte RD, editors. brain injury medicine: principles and practice. New York: Demos Medical Publishing; 2007. p 159.

73. A 23-year-old man complains of localized low back pain that is worse in the mor ning a nd imp rov es as the da y pro gre sse s with a cti vi ties . H e has no radicular symptoms and a nonfocal neurologic examination. Which finding unequivocally supports your diagnosis:(a) A positive human leukocyte antigen B27:(b) Bilateral sacroiliitis on a radiograph of the pelvis:(c) An elevated erythrocyte sedimentation rate:(d) Atlantoaxial subluxation on a radiograph of the cervical spine

73 (b) Bilateral spondylitis is diagnostic of ankylosing spondylitis. HLA-B27 is not helpful because a percentage of the normal population is positive for HLA- B27 and not all pati ents with a nk ylosing spo ndy liti s ar e p ositive. Ery throcyte sedi ment ation rat e is a g ene ral marke r o f inf lamma tion. En thesopathie s may o ccur in a wid e var ie ty of infl amma tor y d isord ers , i ncl uding s pondyloarthropathies.

73. A 35-year-old gentleman with a history of Lyme disease that was treated adequately with antibiotics 1 year ago complains of continued muscle aches, joint pain, fatigue, and difficulty concentrating. His repeat Lyme serologies have been negative, as have all other laboratory tests. He has had a full medical work-up from his internist that has been unremarkable. You recommend (a) intravenousceftriaxone for 28 days. (b) sulfasalazine for his muscle and joint pains. (c) intra-articular cortisone injections for joint pain. (d) emotional support and symptom management.

73 Answer: D Commentary: The patient has postLyme disease syndrome, which occurs in a minority of patients who have had Lyme disease. There is no specific treatment. Physicians should provide support and management of patient complaints. Antibiotic treatment is contraindicated. Sulfasalazine is not a treatment for Lyme disease. Ref: Feder HM, Johnson BJ, O'Connell S, Shapiro ED, Steere AC, Wormser GP. A critical appraisal of chronic Lyme disease. N Engl J Med 2007; 257:1422-30.

73. A 55-year-old woman complains of bilateral thumb pain with activ ities. E xaminatio n rev eal s pain with axial grinding of the first carpometacarpal joints as well as Heberden and Bouchard nodes in the hands. What is an appropriate initial treatment? (a) Resting wrist splints in neutral (b) Thumb spica splints (c) Compression gloves (d) Buddy taping the digits

73. (b) The carpometacarpal joint is a common location for osteoarthritis in the upper extremity. While cumbersome, a thumb spica splint will often alleviate the pain. The other options listed would not be considered appropriate initial treatment options.

73. A 20-year-old man compla ins of 4- mon th his tory of low back pain that radiates into the buttocks. The pain began insidiously, is worse in the morning, and eases up after a hot shower. The neurologic exam is normal. Which finding will help support your likeliest diagnosis?(a) A negative HLA-B27(b) Symmetric wrist swelling(c) A negative Schober test(d) Acute iritis

73. (d) Acute iritis is an extra-skeletal manifestation of ankylosing spondylitis (AS). HLA-B27 is not necessary to make the diagnosis of AS. Involvement of peripheral joints is infrequent and, when present, is asymmetric. The Schober test is typically positive.

74. For a patient with myopathy, which type of exercise is most harmful?:(a) High intensity concentric:(b) High velocity isokinetic:(c) High intensity eccentric:(d) High intensity isotonic

74 (c) There is evidence of more muscle fiber damage with eccentric exercise. This appears to be true for persons with myopathic disease as well as in control sub ject s. Th ere is s imilar evid en ce in an ima l s tudi es. The type of exe rcise and its pot ential be nefits v ers us detrim ent al ef fects conti nu es to be a contr ov ersial s ubjec t. Ho we ver , m ore and mor e e viden ce sug ges ts that exercise in persons with myopathy may not be as detrimental as once thought.

74. Which one of the following characteristics is typica lly assoc iate d with Charcot Marie Tooth (CMT) disease type 2? (a) minimal level of disability. (b) minimal decrease in nerve conduction velocity. (c) autosomal recessive inheritance. (d) absence of sensory deficits.

74. (b) Charcot Marie Tooth (CMT) disease type 2 has greater variability and produces more disability than type 1. The disability can range from very mild to severe in CMT type 2. In addition to the weakness typical of the hereditary sensory motor neuropathy diseases, paresis of diaphragm, vocal cord, and intercostal muscle has been reported. CMT type 2 disease is characterized by less hypertrophic change in myelin, with more neuronal or axonal involvement. Sensory deficits are common to both forms. Both have autosomal dominant inheritance.

74. Which neuromuscular diso rder is N OT usuall y associated with thyroid disease?(a) Peripheral neuropathy(b) Neuromuscular junction d isor der(c) Myopathy(d) Radiculopathy

74. (d) Thyroid disease is associated with several different aspects of the neuromuscular system. In both hypothyroidism and hyperthyroidism, there can be neuromuscular junction disorders (increased incidence of myasthenia gravis), and myopathy. Hypothyroidism is associated with sensorimotor peripheral neuropathy and entrapment neuropathy, especially carpal tunnel syndrome. Thyroid disease is not associated with radiculopathy.

75 Bone loss following spinal cord injury is characterized by (a) greater loss of cortical rather than trabecular bone. (b) low bone mineral density in the spine. (c) predilection for regions below the level of injury. (d) new bone homeostasis that ensues by 6 months after injury.

75 Answer: (c) Commentary: Bone loss occurs inevitably following spinal cord injury, and is uniquely characterized by a predilection for trabecular more than cortical bone in regions below the level of injury. This is associated with relative sparing of spine bone mineral density, possibly due to continued functional loading of the spine. A new homeostasis in bone resorption and formation is achieved by about 16 months. Ref: (a) Little JW, Burns SP. Neuromusculoskeletal complications of spinal cord injury. In: Kirshblum S, Campagnolo DI, DeLisa JA, editors. Spinal cord medicine. Philadelphia: Lippincott Williams & Wilkins; 2002. p 241. (b) Kiratli, BJ. Bone loss and osteoporosis following spinal cord injury. In: Lin VW, editor. Spinal cord medicine: principles and practice. New York: Demos; 2003. p 539-41.

75. A 60-year-old woman is seen in consultation by your rehabilit atio n team after elective surgery. She has a new finding of 1/5 strength in her lower extremities, but retained propioception and vibratory sense. You make the diagnosis of (a) posterior spinal cord syndrome. (b) central cord syndrome. (c) anterior spinal cord syndrome. (d) conversion disorder.

75. (c) In anterior spinal cord syndrome there is usually paralysis below the level of the lesion, along with bilateral loss of pain and temperature sensation. Proprioception and vibratory sense are partially preserved. This syndrome often occurs after significant intraoperative hypotensive events. Central cord syndrome refers to weakness that is greater in the upper extremities than the lower extremities. Posterior cord syndrome shows loss of proprioception and is the least common of the incomplete spinal cord injury syndromes.

76. A 39-year-old male factory worker suffers from a low vo lt age induced electrical injury. The most serious acute medical complication that can occur is a) cardiac arrhythmia. b) peripheral neuropathy. c) distal extremity amputation. d) myelopathy.

76 Answer: (a) Commentary: Electrical injuries are usually caused by al ter na tin g current of 60Hz. They are classified as high voltage injuries when the person comes in contact with 1000V or more, or low voltage when the voltage is below 1000V. A large number of electrical injuries are work related. Hussman found cardiac arrhythmias to be the most serious medical problem in patients admitted with low voltage injuries (41% of patients). Other complications are soft tissue burns (especially tissues with high water content, such as nerve, muscle and blood vessels), amputations (especially of the fingers and toes), and neurological injuries (to the central or peripheral nervous system). Peripheral neuropathy is reported in up to 34% of high voltage injuries and a lower incidence is found in low voltage injuries. Reference: (a) Esselman PC, Moore ML. Issues in burn reh abi li tat ion. In: Braddom RL, editor. Physical medicine and rehabilitation, 3rd ed. New York: WB Sa und ers; 2007. p 1409. (b) Hussman J, Kucan JO, Russell RC, et al. Electrical injuries - morbidity, outcome, and treatment rationale. Burns 1995;7:530-5.

77. Which factor is associated with increased risk for occupational injury in an older individual? (a) White collar occupation (b) Female gender (c) Impaired hearing (d) Self employment

77 Answer: C Commentary: Predictors of increased injury risk in an older worker include male gender, less education, obesity, alcohol abuse, disability, self report of impaired hearing or sight, and several specific job requirements. Service workers, mechanics, machine operators, and laborers are at increased risk for occupational injury compared to people in white collar occupations. Individuals who are self-employed have a lower risk of injury. Ref: Zwerling C, Sprince NL, Wallace RB, Davis CS, Whitten PS, and Heeringa SG. Risk factors for occupational injuries among older workers: an analysis of the Health and Retirement Study. Am J Public Health 1996;86:1306-9.

77. Which statement about workers' compensation cases is TRUE? (a) The most recent publication of the American Medical Association guideline s for dis abili ty are used uniformly in every state. (b) Parameters that guide entry to evaluation and treatment are uniq ue to eac h state. (c) States require subspecialty board certification for physicians w ho practi ce indust rial med icine. (d) Most states allow injured workers to choose the physician who ma nages the ir care.

77. (b) Guidelines for worker entry and treatment under workers' compensation law are determined by each state. The published American Medical Association guidelines are useful tool in determining disability. Because each state determines which edition of the publication is used, its application is not uniform nationwide. Not all states require subspecialty certification required of a physician wishing to practice industrial medicine. In some states the employer chooses the physician to treat the injured worker.

78. Which compound muscle action potential (CMAP) finding is the mos t compati ble with myast hen ia gravis? (a) A 15% decrement in the amplitude between the 1st and 4th respons es with r epetitive stim ula tion at 3 hertz (b) A low amplitude response with supramaximal stimulation of the ul nar nerve and pick up o ver the hypothenar muscles (c) A 100% increase in the amplitude with stimulation of the spinal accessory nerve an d pic k-u p over the trapezius muscle immediately post exercise (d) A 50% increment in the amplitude between the 1st and 4th respons es with r epetitive stim ula tion at 30 hertz

78. (a) In myasthenia gravis there is a decrement in the amplitude and area of the compound muscle action potential (CMAP) with stimulation at low frequencies (2-3Hz). Stimulation at high rates of 20- 50Hz can result in pseudofacilitation with correction of the amplitude decrement in normal subjects as well as patients with neuromuscular junction disorders. A 100% or greater increase in the amplitude of the post exercise CMAP is usually associated with a presynaptic neuromuscular junction disorder such as Lambert-Eaton myasthenic syndrome (LEMS). Small CMAPs on routine nerve conduction studies are more characteristic of LEMS than of myasthenia gravis.

78. While recording an antidromic sensory nerv e a ct ion potential, you increase the distance between the active and reference electrode from 1cm to 4cm by moving the reference electrode. What is the effect on the onset latency and peak latency? Onset Latency Peak Latency (a) No change Increase (b) Increase No change (c) Decrease Decrease (d) Increase Increase

78. (a) Increasing the interelectrode distance from 1 cm to 4 cm does not alter the onset latency, but increases the peak latency and amplitude of the sensory response. The onset latency does not change because the active electrode position is not changed. The sensory nerve action potential amplitude increases because less of the information is eliminated by differential amplification. Similarly, the peak latency also is prolonged as less of the signal is eliminated. Ref: Dumitru D, Zwarts MJ. Instrumentation. In: Dumitru D, Amato AA, Zwarts MJ, editors. Electrodiagnostic medicine. 2nd ed. Philadelphia: Hanley & Belfus; 2002. p 73.

78. A 55-year-old man presents with a 2-month history of progress ive weakness. On examination he has mild proximal weakness in the upper and lower limbs. His muscle tone and bulk are normal and he has no facial weakness. Sensation is normal and deep tendon reflexes are 1+ and symmetrical. Which finding on electrodiagnostic testing is most consistent with this patient's presentation? (a) Prolonged or absent F waves (b) Decreased recruitment ratio (c) Motor unit potentials with amplitudes of 10 millivol ts (d) Normal number of phases of the motor unit potentials

78. (b) The clinical presentation is most consistent with a myopathic picture. In myopathies the recruitment ratio is usually lower (<5). An increased recruitment ratio and large motor unit amplitude potentials are associated with a neurogenic process. The number of phases of the motor units is usually increased in myopathy and in different forms and stages of neuropathies. F waves are typically prolonged or absent in neuropathies, but not in myopathies.

79. Which maternal factor is associated with an increased risk of spina bifida? (a) Anticonvulsant medications during pregnancy (b) Upper socioeconomic class (c) Alcohol ingestion during pregnancy (d) Folic acid 4mg/day prior to and during pregnancy

79 Answer: A Commentary: The etiology of spina bifida is multifactorial. Both polygenic inheritance and environmental influences contribute. Several studies have shown that there the incidence of spina bifida is reduced if food is fortified with folic acid or if mothers take folic acid prior to conception and during pregnancy. Recommended doses of folic acid are 0.4 mg/day in women who are not at high risk and 4 mg /day in women at high risk (eg, those with a family history of spina bifida). Some studies have also implicated lower socioeconomic class and in utero exposure to anticonvulsant medications as being risk factors. Maternal alcohol ingestion is not related to an increased risk of spina bifida in a baby. Ref: Molnar GE, Murphy KP. Spina bifida. In: Molnar GE, Alexander MA, editors. Pediatric rehabilitation. 3rd ed. Philadelphia: Hanley & Belfus; 1999. p 219.

79. The best predictor of co mmun ity a mbu lation beyond childhood in patients with myelomeningocele is(a) body mass index.(b) quadriceps strength.(c) early surgical closure o f th e men ing ocele.(d) bowel and bladder contin ence .

79. (b) There are many studies about longterm outcomes of ambulation in children and adults with myelomeningocele. While many factors influence outcome, including intelligence, medical problems, and obesity, the best predictor of ambulation into adulthood is strong quadriceps function. Bowel and bladder continence has no relationship to ambulation.

8. Which statement regarding postoperative neurologic and cognitive changes af ter coronary a rtery byp ass surge ry (C ABG) is TRUE? (a) Each additional hour on bypass doubles the probability of postoperative en cephalopathy. (b) Lesions seen on diffusion weight magnetic resonanceimaging correlate highl y with impairm ents. (c) Cognitive status at 3 weeks predicts cognitive functionat 5 years. (d) Cognitive deficits are most prominent at 1 week afterCABG. Ref: (a) Mullges W, Berg D, Schmidtke A, Weinacker MA, Toyka KV. Early natur al course of t ransient encephalo pathy aft er coronary artery bypass grafting. Crit Care Med 2000;28: 1808-11. (b) Stygall J, Newman SP, Fitzgerald G, et al. Cognitive change 5 years afte r coronary art ery bypas s surgery . Hea lth Psychol 2003;22:579-86. Clinical Activity 1.12

8 (a) Each additional hour on cardiac bypass doubles the probability of postoperative encephalopathy. Le sions shown on dif fusion weight mag netic resonance imaging corre late po orly wit h impai rm ent . Impair men ts may be transient, with recovery at 8 weeks predictive of cognitive function at 5 years. Cognitive deficits are most prominent at 3 days after coronary artery bypass graft.

8 A patient with poststroke spasticity is noted on exam to have a hemipleg ic ga it pat te rn wi th typ ica l sy nergy pat te rns of tone . Wh at is the most comm on syner gy pat ter n y ou e xpect to see in the upper limb and lower limb? a. Flexion synergy in the upper limb, flexion synergy in the lower. b. Flexion synergy in the upper limb, extension synergy in the lower. c. Extension synergy in the upper limb, flexion synergy in the lower. d. Extension synergy in the upper limb, extension synerg y in the lower.

8 Option b is correct. The typical spastic posture in the poststroke patient is a flexion syner gy in th e u pp er li mb wit h a fle xed, a ddu ct ed, int erna ll y rotate d ar m wi th pr ona ted f ore arm an d f lexe d wrist and fingers, and an extension synergy in the lower limb with leg extended, internally rotated and adducted, and the ankle plantarflexed and inverted.

8 Which non-surgical treatment for carpal tunnel syndrome is shown to provide significant short-term benefit? a) Magnet therapy a. b) Laser therapy b. c) Therapeutic exercise c. d) Therapeutic ultrasound

8 Option d is correct. Commentary: Patients suffering from carpal tunnel syndrome are often offered nonsurgical treatments. Current evidence shows significant benefit from ther apeut ic ultrasound treatments, splinting, yoga, and carpal bone mobilization. However, trials involving the use of magnet therapy, laser therapy, therapeutic exercise, and chiropractics have not produced significant benefits compared to placebo or control treatments. Reference: O'Connor D, Marshall SC, Massy-Westropp N, Pitt V. Non-surgical treatment (other than steroid injection) for carpal tunnel syndrome. Cochrane Datab ase Syst Rev 2003;(1):14.

8 A patient's dual-energy x-ray absorptiometry (DXA) scan shows a T score of -1.5 at the lumbar spine, 2.0 at the femur, and -1.2 at the radius. This patient has a. normal bone density. b. osteopeni c. osteomalaci d. osteoporosis.

8 Reference(s) (a) Sinaki M. Osteoporosis. In: Braddom RL, editor. Physical medicine and rehabilitation. 4th e Philadelphia: Elsevier Saunders; 2011. p 923. ( b) Saag KG. Osteoporosis. In: Klippe l JH , Stone JH, C ro fford LJ, White PH, editors. Primer on rheumatic disease. 13th e Atlanta (GA): Arthritis Foundation; 2008. p 577. Option b is correct.

8. Which hypothesis does NOT exp lain a n ormal electromyograph (EMG) in a patient who has a lumbar radiculopathy? (a) Involvement of only the sens ory r oot (b) Limited sampling of musc les (c) Oxycodone taken prior to the stud y (d) Timing of the study

8. (c) Pain medication has no effect on EMG findings. All the other choices can be an explanation for a normal EMG in a patient who has a lumbar radiculopathy.

8. Which statement is TRUE about volume conduction as it relates to electroph ysiologic stud ies ? (a) Volume conduction is the transmission of an electrical potential through intracell ular tis sue. (b) Volume conducted potentials produce a biphasic waveform as an ad vancing a ction pot entia l a pproaches and then passes beneath a recording electrode. (c) The negative peak of a recorded waveform represents the time tha t the act ion poten tial is beneath the active electrode. (d) Volume conducted near field potential amplitudes does not charac teristica lly depen d on the distance between the recording electrode and the source.

8. (c) The characteristics of volume conducted near field responses are dependent on the distance from the recording electrode and the electrical source. The responses represent intracellular events transmitted through extracellular fluid and tissue. They usually produce a triphasic waveform and the negative phase is that time during which the advancing wave is directly underneath the recording electrode.

80. A person with diabetes presents with an area of nonblanching erythema on t he plant ar surfa ce of th e foot at t he fi rs t met at arsal head . Rec omm en dation s f or foo tw ear would include (a) custom-molded shoe insert. (b) narrow toe box shoe. (c) heel lift on the affected side. (d) calcaneal bar added to the sole of the shoe.

80 (a) Footwear for the person with diabetes and grade 1 skin chang es on the plantar aspect of the foot should be designed to relieve pressure over the affect ed site wh ile eve nly distributing pressure over the remaining foot surface to preven t other skin brea kdown. A ty pical pre scripti on would include an ex tr a-d ept h sh oe w ith a w ide toeb ox an d a tota l-c ont act , cus to m-mo ld ed ins ert wi th pr ess ur e r elief at the area of skin irritation. Further shoe modifications with a metatarsal bar and rocker bottom sole could also be considered, especially if the patient had grade 2 skin changes or more severe foot deformities. A calcaneal bar or heel lift would not be appropriate considerations in this case.

80. Regarding energy expenditure during prosthetic ambulation,:(a) at self selected walking speeds, someone ambulating with a transtibial prosthes is h as a hig her rate of met aboli c e ner gy expen dit ur e [m lO2 /( kg•meter)] compared with normal human locomotion.:(b) whether a person has a transtibial or transfemoral prosthesis, the metabolic co st [ ml O2/ (kg •me ter)] of amb ulati on is the same .:(c) at self selected walking speeds, cardiac work load during transtibial prostheti c am bu lat ion is comp arab le to th at in normal h uma n loco mot io n.:(d) gait velocity [meters/min] during transtibial prosthetic ambulation is comparab le t o tha t i n t ransf emor al prost het ic ambulati on.

80 (c) The rate of metabolic energy expenditure-mlO2/(kg•minute)-represents energy expenditure per unit of time in comparison to metabolic cost-mlO2/(kg•minute )-w hich is a me asure of energy ex penditur e p er unit dis tan ce, a meter. The metaboli c co st of pro sthetic amb ulation i s g re ate r tha n that u sed in norm al hu ma n locomo tion and t he me tab olic cos t o f t ransf emo ral pr ostheti c ambulation is greater than that in transtibial prosthetic ambulation. Because individuals who ambulate witha prosthesis slow their gait velocity, their rate of metabolic energy expenditure is relatively unchanged compared to normal ambulation at self-selected walking speeds. Because the rate of metabolic energy expenditure is maintained at a level comparable to ambulation without a prosthesis, cardiac work load is also unchanged during ambulation with either a transtibial or transfemoral prosthesis, if subjects are allowed to walk at their self-selected walking speed.

80. Which factor promotes knee stability during the gait cycle of a person with transfemoral amputation? (a) Knee component placed anterior to the socket (b) Hard heel in the prosthetic foot (c) Polycentric 4-bar linkage prosthetic knee (d) Anterior position of the shank on the prosthetic foot

80 Answer: C Commentary: Flexion moment at the hip, a rigid heel in the solid ankle, cushion heel foot and the anterior position of the shank all shift the ground reaction force behind the knee joint to produce a knee flexion moment. The 4-bar linkage with instantaneous center of rotation and the posterior location of instant center in extension creates knee stability, especially at heel strike. Ref:(a) Michael JW. Prosthetic suspensions and components. In: Smith DG, Michael JW, Bowker JH, editors. Atlas of amputations and limb deficiencies. 3rd ed. Rosemont. AAOS; 2004. p 421.(b) Schuch CM, Pritham CH. Transfemoral amputation: prosthetic management. In: Smith GD, Michael JW, Bowker JH, editors. Atlas of amputations and limb deficiencies. 3rd ed Rosemont: AAOS; 2004. p 541-2.

80. The best example of a dy nami c ort hos is is(a) dorsal wrist hand orthos is w ith e xte nsion force in radial nerve injury.(b) thermoplastic ankle foot ort hosis in sever e foot drop.(c) wrist hand orthosis in 1 5° o f ext ens ion fo r carpal tunnel syndrome.(d) a C-bar in median nerve inju ry.

80. (a) Static means that the orthosis is rigid and gives support without allowing movement. These devices are commonly used to rest a part after trauma or surgery, and for acutely inflamed joints and tendons. Dynamic orthoses allow a certain degree of movement. They usually provide some element of assisted motion to the joint, such as the elastic assist to wrist extension in the orthosis for radial nerve palsy.

80. What is a possible cause for circumduction during mi d swing i n th e transfemoral amputee? (a) Insufficient knee friction (b) Prosthesis too short (c) Excessive medial brim pressures (d) Inadequate hip extension

80. (c) Possible causes for circumduction in the gait of a transfemoral amputee include excessive mechanical resistance to knee flexion, prosthesis aligned with too much stability, prosthesis too long, increased medial brim pressures, inadequate suspension, patient lacks confidence or has inadequate hip flexion.

81. Which statement regarding Waddell signs is TRUE?:(a) They are a collection of organic signs that identify malingerers.:(b) They include simulation, distraction, overreaction, and regional disturbances.:(c) A finding of 2 or more positive signs is clinically significant.:(d) These signs are seen much more commonly in chronic pain patients.

81 (b) The Waddell signs are nonorganic physical signs that are used for patients with low back pain. They are a simple screen that can be used to help identif y p atie nts w ho may r equire a mo re detaile d p syc holo gica l a ssessment. T hree or more posi tive s ign s are a cli nically s ign if ica nt fi nding. T he signs in clude s imulatio n, di strac ti on, ov erre acti on, re giona l d ist urb ances a nd either superficial or nonanatomic tenderness.

81. Which statement is TRUE rega rding my ofasci al trigger points?(a) They exhibit electrodiag nost ic ab nor maliti es on needle examination.(b) Their symptoms can be re prod uced wit h palp ation over the trigger point.(c) Their identification req uire s at lea st 8 k ilograms of pressure.(d) They resolve with isokin etic exer cis e of t he affected muscle.

81. (b) Moderate, sustained pressure for approximately 10 seconds on an irritable trigger point causes symptoms in the reference zone for that muscle. No specific amount of pressure is required for this diagnosis. Trigger points should not be confused with fibromyalgia "tender points" which require approximately 4 kg of pressure for diagnosis. Myofascial trigger points are electrically silent and show no resting muscle activity on electromyogram. No elevation in muscle creatine phosphokinase is seen with this condition. Isokinetic exercise is not indicated as a treatment for this condition. Local injection with anesthetic and/or spray with a vaporcoolant spray and stretch of the muscle are the treatments of choice.

81. To allow pronation of the foot, which 2 joints must have thei r ax is of rotation in parallel? (a) Lisfranc and talonavicular (b) Subtalar and calcanocuboid (c) Talocrural and subtalar (d) Talonavicular and calcaneocuboid

81. (d) The transverse tarsal joint, namely the talonavicular and calcaneocuboid joints, must have their joint axes in parallel to allow for a flexible midfoot and pronation. If the axes intersect, the midfoot becomes rigid, which enables proper supination.

82. A 26-year-old man was in a motor vehicle crash last night. Oxygen saturation wa s co ns ist ent ly aroun d 93 %. Intra cra nia l pressu re wa s 15 mmH g, with a mean arterial blood pressure of 110mmHg. Pupils were equally reactive. This patient's prognosis is:(a) poor because oxygen saturation was consistently below 95%.:(b) poor because of the high intracranial pressure.:(c) not affected by pupillary response in the first 24 hours.:(d) good because his cerebral perfusion pressure was greater than 70mmHg.

82 (d) The guideline set by the American Association of Neurological Surgeons included avoidance of oxygen saturations of less than 90%. Cerebral perfusion pre ssu re i s a m ore impo rtant predi ct or of ou tco me than ICP si nce CPP is m ore directly rela ted to me tabolic del ivery and is ch emi a. CP P is m ea n arterial press ur e minus ICP a nd in t his pa tien t is 95 . P upill ary re spo nse is a very important early predictor of eventual outcome.

82 Which statement best describes the effects of repetitive task training after stroke? (a) Lower limb functional recovery is greater than upper limb functional recovery. (b) Improvement in activities of daily living is a major benefit of the training. (c) Training effects are more significant in early stroke therapy. (d) Improvement in functional benefit is sustained for more than a year.

82 Answer: (a) Commentary: This review of 14 studies with 659 participants looked at whether repeated practice of tasks similar to those commonly performed in daily life could improve functional abilities. In comparison with usual care or placebo groups, people who practiced functional tasks showed modest improvements in walking speed, walking distance and the ability to stand from sitting, but improvements in leg function were not maintained 6 months later. Repetitive task practice had no effect on arm or hand function. There was a small amount of improvement in ability to manage activities of daily living. Training effects were no different for people whether the training was given early or late after stroke Ref: (a) French B, Thomas LH, Leathley MJ, Sutton CJ, McAdam J, Forster A, et al. Repetitive task training for improving functional ability after stroke. Cochrane Database Syst Rev. 2007;(4):CD006073. doi: 10.1002/14651858.CD006073.pub2. (b) Basmajian JV. Biofeedback in physical medicine and rehabilitation. In: DeLisa JA, Gans BM, Walsh NE, editors. Physical medicine and rehabilitation: principles and practice. 4th ed. Philadelphia: Lippincott Williams & Wilkins: 2005. p 276-7.

82. In a patient with traumatic brain injury who has impaired speed of processing, inattention and decreased arousal, which medication is regarded as first-line therapy? (a) modafinil (Provigil) (b) methylphenidate (Ritalin) (c) bromocriptine (Parodel) (d) carbidopa/levodopa (Sinemet)

82 Answer: B Commentary: The present evidence suggests that methylphenidate should be regarded as first-line therapy when an agent from this medication class is used. If methylphenidate proves ineffective or produces intolerable side effects, dextroamphetamine, amantadine, or bromocriptine may be useful alternative stimulant medications. Amantadine's side effect profile is worse than methylphenidate and there is some evidence of a lowering of the seizure threshold, but this is controversial. There is no support at this time in the literature for the use of modafinil over methylphenidate. Bromocriptine and carbidopa/levodopa both have worse side effects and are not as well studied as methylphenidate or amantadine. Ref: Arciniegas DB, Siver JM. Pharmacotherapy of cognitive impairment. In: Zasler ND, Katz DI, Zafonte RD, editors. Brain injury medicine: principles and practice. New York: Demos Medical Publishing; 2007. p 1006-17.

83. A 70-year-old obese gentleman presents to your offic e f or fo llow-up with a several month history of increasing left hip and groin pain that occurs with walking. His history is significant for prior alcoholism, hypothyroidism, gout, and right knee osteoarthritis. He completed a physical therapy course that did not help much. He now has difficulty even standing or walking for a few minutes and complains of pain with moving his leg. Radiographs taken today demonstrate sclerosis and slight collapse of the femoral head. What is his main risk factor for developing the condition found on his radiographs? (a) Obesity (b) Alcoholism (c) Hypothyroidism (d) Older age

83 Answer: (b) Commentary: The radiographic findings are typical for a vas cu lar necrosis which can be due to trauma, high doses and/or prolonged use of steroids, heavy alcohol use, and certain systemic diseases (diabetes, systemic lupus erythematosus). Obesity and older age are risk factors for developing osteoarthritis. Typical radiographic findings of osteoarthritis include joint space narrowing, osteophytes, subchondral cysts and sclerosis; collapse of bone is not seen. Hypothyroidism is not a risk factor for avascular necrosis. Reference: Hansen PA, Willick SE. In: Braddom RL, editor . P hy sic al medicine and rehabilitation. 3rd ed. Philadelphia: Saunders; 2007. p 867.

83 An 87-year-old man on your inpatient rehabilitation unit was found on the therapy mat in much pain after hearing a loud "cracking" noise when he transferred himself. What position of his right lower limb would suggest hip fracture? (a) Internal rotation and lengthened (b) Internal rotation and shortened (c) External rotation and lengthened (d) External rotation and shortened

83 Answer: (d) Commentary: In most cases, the lower limb of the fractured hip would be held in external rotation (rotated outward) and would appear shortened relative to the unaffected lower limb. Ref: Sinaki M. Prevention and treatment of osteoporosis. In: Braddom RL, editor. Physical medicine and rehabilitation. 3rd ed. Philadelphia: WB Saunders; 2007. p 934.

83. A 75-year-old manwith a recent calcaneal stress fracture after starting a walking program presents to your clinic. Initially, you should (a) order a bone mineral density test. (b) prescribe a lower extremity strengthening program. (c) obtain a nuclear bone scan. (d) prescribe a swimming program

83 Answer: A Commentary: The initial assessment should include checking his bone density to establish a diagnosis of osteopenia/osteoporosis and then identifying secondary risk factors (such as hypogonadism, corticosteroid use, excessive alcohol use). Once a diagnosis is established, prescribing weight-bearing and strengthening exercises are important. Obtaining a nuclear bone scan is not as helpful. Swimming is a non-weight bearing exercise. Ref: Ebeling PR. Osteoporosis in men. N Engl J Med 2008;358:1474-82.

84. A 47-year-old left handed assembly line worker presents with a 1-week history o f ne ck pa in and left arm we aknes s. Exa mination re ve als dim in ished left triceps reflex. Muscle strength is 4/5 left elbow extensor, 4-/5 wrist extensor, and otherwise 5/5. Sensation is decreased to pinprick in the left index and middle fingers. What is the most appropriate initial intervention?:(a) Refer to a neurosurgeon:(b) Start on a tapering steroid:(c) Start cervical extension exercises:(d) Apply therapeutic ultrasound

84 (b) The clinical presentation is that of a person with an acute cervical radiculopathy. In this setting the use of steroids is often indicated for a potent ant i-in flamm ato ry ef fect. The t re atment i s t ypi call y in iti ated when an inflammatory eti ology is suspecte d a nd there are n o c ontra indica ti ons such as an i nf ection o r his tory of pe pti c ul cer dis eas e.

85. A 28-year-old with T11 paraplegia for 6 months comes to your office to discuss trea tm ent op tio ns fo r er ect ile d ysf unc tion. A tri al of sil de nafil (Viagra) was unsuccessful. He asks about constriction rings. You tell him:(a) they should not be used in men with sickle cell disease.:(b) they can be kept in place for up to 2 hours.:(c) they cannot be used without a vacuum pump.:(d) they are not effective for erectile dysfunction in spinal cord injury.

85 (a) Constriction rings (cock rings) to occlude venous outflow can be used if a person is having a poorly sustained erection. A vacuum pump and constriction rin gs c an be us ed if a person i s having n o e rec tion s. T o p revent skin breakdown fro m pr olonge d v enous co nge stion wit hin t he penis , the ri ng should n ot be k ept in p lace for m or e t han 30 minu tes . T hey s hou ld not be use d in men with sickle cell disease. Anticoagulants are a relative contraindication to their use.

85. The risk of an exacerbation of multiple sclerosis is re du ced a) during pregnancy. b) in the first 3 months after delivery of a baby. c) in the postoperative period after a surgery. d) during a respiratory infection.

85 Answer: (a) Commentary: During pregnancy multiple sclerosis (MS) exa cer ba tio ns decrease to about half of what they would be otherwise. During the first 3 months postpartum, the relapse rate is higher than normal. The net effect of pregnancy on the course of MS is neutral, and women need not make decisions about pregnancy based on fear that it will worsen their disease. There is no documented effect on MS due to surgery or with a respiratory infection. Reference: Kraft GH, Brown T. Comprehensive management o f m ul tip le sclerosis. In: Braddom RL, editor. Physical medicine and rehabilitation. 3rd ed. Philadelphia: Elsevier; 2007. p 1228.

85 A 24 year-old man sustains an acute, traumatic C5 American Spinal Injury Association Impairment Scale (AIS) A tetraplegia and a proximal left femur fracture following a motor vehicle crash. His hemoglobin has remained stable. Based on the Consortium for Spinal Cord Medicine's Clinical Practice Guidelines, venous thromboembolic prophylaxis should include sequential compression devices for a minimum of 2 weeks and (a) coumadin for 4 weeks. (b) low molecular weight heparin for 8 weeks. (c) low molecular weight heparin for 12 weeks. (d) prophylactic inferior vena cava placement.

85 Answer: (c) Commentary: According to the Clinical Practice Guidelines, venous thromboembolic prophylaxis for uncomplicated motor-complete tetraplegia and AIS C injuries should be comprised of low molecular weight heparin or adjusted dose unfractionated heparin for 8 weeks. However, in the presence of complicating factors (eg, lower limb fractures, advanced age, obesity, heart failure, cancer) prophylaxis with low molecular weight or unfractionated heparin should continue for a total of 12 weeks or until discharge from Rehabilitation. Individuals with AIS D paraplegia without other complications require chemoprophylaxis with unfractionated heparin only until the rehabilitation discharge. Prophylactic intravenous chemotherapy filter placement is recommended only if there are contraindications or high risk associated with anticoagulation, and prophylaxis should be initiated as soon as hemostasis is achieved or contraindications resolved. Ref: Consortium for Spinal Cord Medicine Clinical Practice Guidelines. Prevention of thromboembolism in spinal cord injury, 2nd ed. DC: Paralyzed Veterans of America; 1999.

85. Which of the following is a benefit of a phrenic pacemaker in an individual with tetraplegia (a) elimination of ventilator support (b) improved speech (c) improved hearing acuity (d) longer life expectancy

85 Answer: B Commentary: Benefits of p hrenic pacemaking include improved speech, improved smell, ease of transfers and out of home mobility, reduced incidence of respiratory tract infections, and reduced volume of repiratory secretions. Ref: Sapru HN. Spinal cord: anatomy, physiology, and pathophysiology. In: Kirshblum S, Campagnola DI, DeLisa JA, editors. Spinal cord medicine. Philadelphia: Lippincott Williams & Wilkins; 2002. p 5-26.

85. Autonomic dysreflexia is most commonly precipitated by (a) bladder distension (b) bowel impaction (c) heterotopic ossification (d) atelectasis

85. (a) Autonomic dysreflexia occurs in individuals with spinal cord injuries at the level of T6 and above. It occurs because of sympathetic discharge resulting from a stimulus below the injury level. The most common cause is bladder distension, which can result from a clogged or kinked indwelling urinary catheter or from delayed intermittent catheterization. Bowel impaction is the second most common cause of autonomic dysreflexia.

86. A thrombocytopenic cancer patient has severe pain related to osseous metas tases. R ecognizi ng th at pa in fr om o steol yt ic me ta stase s is pr ost agl an din me dia ted , y ou choose to initiate therapy with (a) Naproxen (Naprosyn, Aleve). (b) Celecoxib (Celebrex). (c) Valproic acid (Depakote). (d) Acetaminophen (Tylenol).

86 (b) Nonsteroidal anti-inflammatory drugs reversibly inhibit cycl ooxygenase, the enzyme responsible for the conversion of arachadonic acid to prostaglandins . Given th e impor tance of prostaglandins in mediating metastatic bone pain, use of a n NSAID represent s appro pria te fir st- line th erapy for this p ati ent . For th romb oc yt openic pati ents , use o f a cy clo oxy gen ase-2 s peci fi c inhi bit or such as ce lec oxib or refocoxib will place the patient at a much lower risk of hemorrhage.

86. Hydrocolloid dressings facilitate debridement through which mechanism? (a) Enzymatic (b) Autolytic (c) Sharp (d) Mechanical

86 Answer: B Commentary: Hydrocolloid dressings maintain a moist wound environment. Subsequently, proteases and collagenase digest eschar that is in contact with the wound fluid. This process is called autolysis. In enzymatic debridement, chemical agents such as papain-urea break down necrotic tissue. Sharp debridement is performed using an instrument such as a scalpel. An example of mechanical debridement would be wet-to-dry dressing or whirlpool treatment. Ref: Priebe MM. Pressure ulcers. In: O'Young BJ, Young MA, Stiens SA, editors. Physical medicine and rehabilitation secrets. 2nd ed. Philadelphia: Hanley & Belfus; 2002. p 462. (b) Goldman R, Popescu A, Hess CT, Salcido R. Prevention and management of chronic wounds. In: Braddom RL, editor. Physical medicine and rehabilitation. 3rd ed. Philadelphia: Elsevier; 2007 p 690-1.

86. An inpatient consultatio n is requ est ed on a 56-year-old patient 3 days after uncomplicated coronary artery bypass grafting. The cardiothoracic surgical resident maintains that the patient must remain either in his bed or chair during exercise. In formulating a rehabilitation program you(a) agree with the resident, add ing t hat only isometric exercise be performed.(b) agree with the resident, add ing t hat no up per extremity strengthening be performed.(c) maintain that progressiv e am bulat ion can b e safely initiated after postoperative day 2.(d) maintain that upper extr emit y erg ome try sh ould be used for aerobic conditioning.

86. (c) Patients who have just undergone coronary artery bypass grafting have recently been revascularized and are therefore excellent rehabilitation candidates. Typically, patients begin progressive ambulation training on postoperative day 2, with independent ambulation usually being performed on day 3.

9. A 2-year-old patient with spinal muscular atrophy type 2 (intermediate form ) presen ts with a 25° , C s haped sco liosi s. What i s the best t rea tme nt optio n a t t his t ime? (a) Muscle strengthening (b) Electrical stimulation (c) Spinal fusion (d) Spinal orthosis

9 (d) Muscle strengthening will not reduce the curve or prevent it from progressing and is not easily accomplished in 2-year-old children. Posterior or anter ior spinal fusion is not indicated with a curve of this size and is to be avoided in a young child if at all pos sible. Sp inal or thotics are used in yo un g c hil dren w it h spina l mu scul ar at ro ph y to im pro ve sitti ng bal an ce and to at tempt to h alt curv e progressio n.

86. An 80-year-old man with peripheral neuropathy and mu ltiple me dica l conditions fell at home and was found several hours later. He was admitted to the h ospital f or a sacral insufficiency fracture and failure to thrive. During your initial consultation, you notice a skin ulcer in which the entire thickness of the skin is involved without involvement of the underlying fascia. According to the National Pressure Ulcer Advisory Panel, the patient's ulcer is classified as stage (a) 1 (b) 2 (c) 3 (d) 4

86. (c) Stage 1: Nonblanchable erythema of intact skin not resolved within 30 minutes; epidermis intact. Stage 2: Partial-thickness skin loss involving the epidermis, possibly into dermis. Stage 3: Fullthickness skin loss involving damage or necrosis of subcutaneous tissue that may extend down to, but not through, underlying fascia. Stage 4: Full-thickness skin loss with extensive destruction, tissue necrosis, or damage to muscle, bone, or supporting structures (eg, tendon or joint capsule).

86. A 24-year-old man with T4 paraplegia has a sa cr al pressure ulcer measuring 2 cm by 2 without depth. The ulcer base has pink granulation tissue. Which dressing is LEAST appropriate in this case? (a) Tegaderm (transparent adhesive dressing) (b) Duoderm (hydrocolloid wafer dressing) (c) Curasol (gel dressing) (d) Accuzyme (enzymatic debridement)

86. (d) This man has a stage II pressure ulcer. Debridement with an agent such as Accuzyme is indicated in wounds with necrotic tissue. Since no necrotic tissue is present in this patient's wound, Accuzyme is not appropriate. A transparent adhesive dressing such as Tegaderm, a hydrocolloid wafer dressing such as Duoderm, and a gel dressing such as Curasol are all appropriate for clean wounds such as the ulcer described. Ref: Salcido R, Goldman R. Prevention and management of pressure ulcers and other chronic wounds. In: Braddom RL, editor. Physical medicine and rehabilitation. 2nd ed. Philadelphia: WB Saunders; 2000. p 651-2.

87. A 32-year-old female cashier presents to you with a 2-week history of low back pai n. She d enies sp ec ific trau ma or a ctivi ty with onse t. Sh e h as no lo wer ex tre mi ty pain, numbness, or tingling. She denies night time pain, recent illness, or previous history of low back pain. Which radiological diagnostic test would you recommend? (a) Computed tomography scan (b) Radiographic imaging (c) No imaging (d) Magnetic resonance imaging

87 (c) No imaging is necessary in this patient with no known medica l risk factors. The low back pain can be managed initially without the expense of CT or MRI . Recommen dations for obtaining x-rays for the patient with a first episode of low b ac k pain for less than 7 wee ks inc lud e: over age 65, history of os te opo ros is, hi st ory of urin ary tr act d ys func tio n, per siste nt sen so ry los s, pro gress ive p ain desp ite treatmen t, night or rest pain, fever, chills, unexplained weight loss, history of trauma or repetitive overuse, recurrent pain with no x-rays in 2 years, previous surgery, or fracture.

87. What is the rationale for bracing for low back pain in the injured worker?:(a) It has been shown to prevent further low back pain injury.:(b) It prevents further injury and should be used in all injuries occurring at work .:(c) It has been shown to improve lifting capabilities.:(d) It can provide proprioceptive feedback to reinforce proper mechanics.

87 (d) Lumbar supports do not prophylactically prevent low back injuries or prevent recurrence of low back pain. One study showed a decrease in lost time at wo rk when lumb ar suppo rts were ut il ized wit h a ba ck i njur y e ducational p rogram. No im prov ement in lifting cap acity has be en fo und w ith th e use of brac ing. Wo rkers wh o use lumb ar su ppo rts use the m a s a p rop rio cep tive re minder to use proper mechanics with lifting and other work-related activities.

87 Which approach is shown to be efficacious in treating carpal tunnel syndrome? (a) Oral corticosteroids (b) Exercise therapy (c) Vitamin B6 (d) Botulinum toxin injection

87 Answer: (a) Commentary: Of the choices listed, only oral steroids have been shown to be efficacious in the treatment of carpal tunnel syndrome. In addition to oral steroids, local injection of corticosteroids and wrist splint are shown to be effective. Exercise therapy and botulinum toxin are ineffective in the treatment of carpal tunnel syndrome. Ref: Piazzini DB, Aprile I, Ferrara PE, Bertolini C, Tonali P, Maggi L, et al. A systematic review of conservative treatment of carpal tunnel syndrome. Clin Rehabil 2007;21;299.

87. Which factor is most likely to be associat ed wi th the development of a work-related, repetitivestrain injury? (a) Normal body weight (b) Warm work environment (c) Younger age (d) Rheumatoid arthritis

87. (d) Risk factors associated with a repetitive strain injury include obesity, cold temperature, older age, diabetes, smoking, pregnancy, rheumatoid arthritis, and psychologic stress. Ref: Panagos A, Sable AW, Zuhosky JP, Irwin RW, Sullivan WJ, Foye PM. Industrial medicine and acute musculoskeletal rehabilitation. 1. Diagnostic testing in industrial and acute musculoskeletal injuries. Arch Phys Med Rehabil 2007;88(3 Suppl):S5.

87. A 55-year-old secretary pres ents to you wi th neck, shoulder, and arm pain. Her pain is worse on Friday than Monday. She denies any trauma or history of previous similar complaints. Her work station changed a week before the onset of her symptoms. You advise her that(a) her job has nothing to d o wi th th e p ain sh e is experiencing.(b) taking a week off will r educ e sym pto ms and allow her to return to work without problems.(c) nonsteroidal anti-inflam mato ry me dic ations and ice will resolve her problem.(d) her work station should be e valua ted .

87. Postural changes can produce muscle imbalances that can cause pain syndromes. Changing posture and re-education of muscles through appropriate strengthening programs is appropriate management. The ergonomics at her work site have a lot to do with her pain complaints. Time away from work may temporarily reduce symptoms but will likely have no long-term effect. The use of pain management interventions such as ice and anti-inflammatory medications is appropriate but should not be expected to correct the long term problem.

93. A 40-year-old woman who runs 30 miles per week wants to prevent symptoms o f hip an d knee osteoarthritis. Proven methods include (a) avoidance of estrogen replacement. (b) maintaining optimal flexibility. (c) use of medial wedge orthotics. (d) nonsteroidal anti-inflammatory medications.

93 (b) Major risks for knee and hip osteoarthritis include major in jury, obesity, and occupations that require repetitive motion of the joint. Proper training technique s for b uilding strength and flexibility can reduce risk of major injury an d thus p revent os teoarth riti s. Est rog en may have a protectiv e e ffe ct .

88. Which potentials have manifestations that CANNOT be observed on physical examin atio n?:(a) Fasciculation potentials:(b) Myokymic discharges:(c) Complex repetitive discharges:(d) Cramp potentials

88 (c) Complex repetitive discharges can only be detected with electromyography. The remainder of the discharges have manifestations that can be observed on ph ysi cal exami nat ion.

88. Which filter setting is usually considered to be appropriate for routine n eedle el ectromyo graph y? Low frequency High frequency (a) 2-10Hz 10,000Hz (b) 2-10Hz 2,000Hz (c) 20-30Hz 10,000Hz (d) 20-30Hz 2,000Hz

88 (c) There is no universally accepted guideline for filter settin gs. However, based on clinical experience certain ranges have been determined and are recom mended. Ea ch proc edure has particular filter settings, which are based on optimum fr eq uency content o f mean wave forms rou tinely observed. The re com men de d f ilt er s et ti ng for rout ine ne edl e el ectr omy ogr aph y is 20 -30 he rt z fo r t he low f ilt er an d 10k ilo hertz fo r the high filter.

88 Which electrodiagnostic criterion is included in the diagnosis of peripheral nerve demyelination? (a) Conduction velocity reduced in at least 4 nerves (b) Compound muscle action potential conduction block in at least 3 nerves (c) Prolonged distal motor latencies in at least 4 nerves (d) Prolonged F-wave latency or absent F wave

88 Answer: D Commentary: The criteria require conduction velocity to be reduced in 2 or more nerves, compound muscle action potential conduction block or abnormal temporal dispersion in 1 or more nerves, prolonged distal motor latencies in 2 or more nerves, and prolonged F wave or absent F wave. Three of these four criteria must be present. Ref: Amato AA, Dumitru D. Acquired neuropathies. In: Dumitru D, Amato AA, Zwarts MJ, editors. Electrodiagnostic medicine. 2nd ed. Philadelphia: Hanley & Belfus; 2002. p 940.

88. Regarding the electrodia gnos tic t est ing of a patient with definite myasthenia gravis, which statement is TRUE?(a) An increment on repetiti ve s timul ati on at 1Hz of up to 25% is expected.(b) A stimulation rate of 2- 3Hz is mo st useful in demonstrating a decrement.(c) An initial low amplitude com pound mo tor ac tion potential after a supramaximal stimulus is expected.(d) Motor unit variability i s re flect ed by dec reased jitter during single fiber EMG.

88. (b) A 2 to 3Hz stimulation is optimal for demonstrating a decremental response. At this rate there is no build up of Ca++ concentration within the nerve terminal and the amount of acetylcholine in the readily available stores diminishes, making failure of some of the neuromuscular junctions possible in those patients with an already small safety factor. A decrement of up to 10% on 2 to 3Hz repetitive stimulation is considered normal. Small CMAPs initially are more suggestive of myasthenic (Lambert-Eaton) syndrome than of myasthenia gravis. Single fiber EMG reveals increased jitter and may reveal blocking.

"

88. Which filter setting is usually considered appropriate for rou tine sens ory nerve cond uct ion studies? LOW-FREQUENCY FILTER HIGH-FREQUENCY FILTER (a) 2-10Hz 10,000Hz (b) 2-10Hz 2,000Hz (c) 20-30Hz 10,000Hz (d) 20-30Hz 2,000Hz" 88 (b) There is no universally accepted guideline for filter settings. However, based on clinical experience, certain ranges have been determined and are recommended. Each procedure has particular filter settings based on the optimum frequency content of the mean waveforms that are routinely observed. The recommended filter setting for routine sensory nerve conduction studies is 2-10Hz for the low-frequency filter and 2,000Hz for the high-frequency filter.

9. Which statement regarding coronary artery disease in spinal cord injury (SC I) is TRUE? (a) Cardiovascular disease is the second-leading cause ofdeath for patients wi th SCI. (b) Aerobic capacity achieved with arm ergometry isequal to that achieved with leg exercise. (c) High-density lipoproteins are reduced in people withcomplete, high-level, and longer-dur ation SCI . (d) Dietary advice includes a goal of body mass index lessthan 15kg/m2. Ref: (a) Raymond J, Davis GM, Climstein M, Sutton JR. Cardiorespiratory resp onses to arm c ranking a nd electr ical stim ulation leg cycling in people with paraplegia. Med Sci Sports Exerc 1999;31:822-8. (b) Demirel S, Demirel G, Tukek T, Erk O, Yilmaz H. Risk factors for coronar y heart diseas e in pati ents with spin al c ord injury in Turkey. Spinal Cord 2001;39:134-8. Educational Activity 1.13

9 (c) High-density lipoproteins are reduced further in patients who have complete, hi gher-level, and lon ger-duratio n spina l cord injuries ( SCIs). Cardiovascular disease is the leading cause of de ath for pat ients with SCI. Aerobic capacity achieved with arm ergometry may be up to 40% lower than aerobic capacity with leg exercise. Dietary goals include a body mass index of less than 25kg/m2.

9. Botulinum toxin injections into the hip adductors, hamstrings and gastrocsoleus in children with spastic cerebral palsy are shown to (a) have greater effect in children older than 10 years. (b) be better than serial casting in management of spastic equinus. (c) be ineffective in management of spastic equinus gait. (d) delay progression of hip displacement.

9 Answer: (d) Commentary: Younger children with fewer physical limitations have more potential for improvement than older children with more physical limitations. Serial casting and botulinum toxin injections appear to have similar benefit. Spastic equinus gait is effectively improved with botulinum toxin injections into gastrocsoleus and hamstrings. Hip displacement can be delayed with botulinum toxin injections into hip adductors and hamstrings but does not affect long-term outcome. Reference: Love SC, Novak I, Kentish M, Desloovere K, Heinen F, Molenaers G. Botulinum toxin assessment, intervention and after-care for lower limb spasticity in children with cerebral palsy: international consensus statement. European J Neurol 2010; 17 (Suppl 2): 9-37.

9 What is the best possible expected outcome for an indivi dual with C7 ASI A cla ss A t et ra ple gia ? a. bladder, dependent; bed mobility, independent; some a ssist with all t ransf ers . b. bladder, dependent; bed mobility, independent; modifi ed independent w ith l eve l t ra ns fer s. c. bladder, independent; bed mobility, some assist; inde pendent with som e tra nsf ers . d. bladder, independent; bed mobility, independent; modi fied independent with le vel t ra nsf ers .

9 Option d is correct. Expected functional outcomes after traumatic spinal cord injury have bee n del ine ate d in th e C lin ica l Pr actice Gu id elin es for he al th car e pr ofes siona ls. A mo tiv ate d i ndi vidu al with C7 spinal cord injury may use a tenodesis grasp to manage bladder catheterization. Using tricep strength to stabilize the upper limb, turning in the bed and level transfers are also possible.

9 The T score is the number of standard deviations above or below the mean bone mineral density of normal young adults. Normal bone density is less than 1 sta ndard deviation below the mean. Osteopenia is defined by the World Health Organization as -1 to -2.5 standard deviations below the mean. Osteoporosis is -2.5 or more standard deviations be low the mean. Osteomalacia is a soft enin g of bone fro m decreased miner alization, frequently from vitamin D deficiency. What is the predominant cause of long-term failure of total hip arthroplasty 10 years after surgery? a) Aseptic loosening b) Infection in the hip joint c) Heterotopic ossification d) Low bone density

9 Reference(s) Sledge CB, Ranawat CS. The hip: principles of reconstructive surgery. In: Ruddy S, Harris ED Jr, Sledge CB, editors. Kelley's textbook of rheum atology. 6th e Philadelphia: WB Saun ders ; 2001. p 175 -2 6. Skerker RS, Mulford GJ. Total hip replacement. In: Frontera WR, Silver JK, Rizzo TD, editors. Essentials of physical medicine and rehabilitation. Musculoskeletal disorders, pain, and rehabilitation. 2nd e Philadelphia : Saunders Elsevier; 2008. p 301. Option a is correct.

9. Which reflex is typically NOT seen in a nor mal 4-month-old infant? (a) Extremities extend on th e fa ce si de as the head is turned to the side. (b) Fingers flex when the pa lm i s tou che d. (c) Extremities extend to th e di recti on of dis placement when center of gravity is displaced. (d) Shoulder abduction, and shou lder, el bow, a nd finger extension occur when the neck is suddenly extended.

9. (c) These options all describe reflexes. (a) asymmetric tonic neck reflex, (b) palmar grasp, and (d) Moroare seen until a baby is about 6 months old. Protective extension or parachute reaction (c) does not appear until after 6 months.

9. You are asked to evaluate a 1-year-old pati ent w ho is not yet walking but is developmentally appropriate. Which reflex would you expect to find? (a) Asymmetric tonic neck (b) Symmetric tonic neck (c) Palmar grasp (d) Plantar grasp

9. (d) The asymmetric tonic neck reflex (ATNR) and symmetric tonic neck reflex (STNR) are usually integrated by 6 to 7 months. Palmar grasp disappears by 5 to 6 months. Plantar grasp is integrated when walking is achieved. The normal age of walking varies, but may be as late as 18 months. Ref: Molnar GE, Sobus KM. Growth and development. In: Molnar GE, Alexander MA, editors. Pediatric rehabilitation. 3rd ed. Philadelphia: Hanley & Belfus; 1999. p 20.

90. Which circumstance is most likely to result in left knee hyperextension at init ial co nta ct (he el st rike ) i n a p ati ent with sp ast ic lef t h em iparesis?:(a) Solid ankle, double-metal, upright ankle-foot orthosis set in 5° of plantar fle xion:(b) Addition of a solid one-quarter inch heel to the left shoe:(c) Addition of ankle dorsiflexion spring assistance to a dual-channel, double-meta l, a nk le- foo t o rthos is:(d) Addition of a T-bar strap to a double-metal upright ankle-foot orthosis to redu ce f oo t i nve rsi on

90 (a) Placing a solid ankle, double-metal, upright ankle-foot orthosis in 5° of plantarflexion will increase the extension moment (forces) at the knee and thi s c an r esult in knee hyperexten si on, espe cia lly at the tim e of initial contact (hee l st rike). Ad dition o f a T-bar st rap s hou ld no t effe ct the knee a nd th e addition of d orsif le xio n s prin g as sis tan ce or a one qu arter i nch heel will reduce the tendency to have knee hyperextension.

90. Which of the following i s tr ue re gar ding s keletal design of a lower extremity prosthesis?(a) Endoskeletal tends to we igh less.(b) Exoskeletal is less hard y.(c) Endoskeletal tends to de mand less ma intena nce.(d) Exoskeletal is easily ad just ed af ter fabri cation.

90. (a) Exoskeletal designs tend to weigh more, are more rugged, demand less maintenance, and cannot be adjusted after fabrication. Generally, the opposite is true of endoskeletal designs.

90. The gluteus maximus is primarily active du rin g which part of the gait cycle? (a) Pre swing (b) Loading response (c) Midstance (d) Terminal stance

90. (b) The gluteus maximus is primarily active from terminal swing through initial contact and loading response. During midstance, terminal stance, and pre swing the gluteus maximus is actually silent. Ref: Perry J. Gait analysis, normal and pathologic function. Soack;1992. p 111-8.

"173. What is the measure of the rate of oxygen ut il ization for the production of energy?

? (a) VO2min? (b) Peak V? O2 (c) V?O2max (d) VO2" 173. (d) Oxygen uptake (VO2) is the measure used to describe the rate at which oxygen is used in the production of energy. Maximal oxygen uptake (VO2max) is the maximal rate at which an individual can use oxygen. Peak VO2 is the measure of oxygen uptake stated when the highest attainable VO2 may not have been reached due to external factors. There is no VO2min measure. Ref: Hoffman MD, Sheldahl LM, Kraemer WJ. Therapeutic exercise. In: DeLisa JA, Gans BM, Walsh NE, editors. Physical medicine and rehabilitation: principles and practice. 4th ed. Philadelphia: Lippincott Williams & Wilkins; 2005. p 391

91. 45-year-old concert violinist presents to your clinic for evaluation of left elbow pain. She has been diagnosed with "lateral epicondylitis" and has had pain and impaired function for 8 months. She has been treating her symptoms with relative rest, occupational therapy and alternative therapies, such as acupuncture and massage, without improvement in her symptoms. What other diagnoses should you consider in this patient? (a) Intersection syndrome (b) Musculocutaneous neuropathy (c) Posterior interosseous neuropathy (d) Rotator cuff tendinopathy

91 Answer: C Commentary: Patients whose symptoms are consistent with lateral epicondylitis or "tennis elbow" but who do not respond to conservative treatments should be considered to have a posterior interosseous neuropathy. Mild neural compression of the posterior interosseous nerve may present with proximal and dorsal forearm pain without obvious muscle weakness, wasting, or sensory deficits. Ref: Dumitru D, Zwarts MJ. Focal peripheral neuropathies. In: Dumitru D, editor. Electrodiagnostic medicine. 2nd ed. Philadelphia: Hanley and Belfus; 2002. p 1093.

91. A 22-year-old skier repo rts to yo ur mounta in ski clinic the day after falling over her ski pole. She complains of severe pain in her left thumb along the medial aspect of the metacarpophalangeal joint. She has minimal swelling and no ecchymosis, but is exquisitely tender along the ulnar collateral ligament. No mediolateral laxity is found on exam. Optimal treatment for this patient should include(a) immediate referral to a hand surg eon for a grade III ulnar collateral ligament tear.(b) immobilization in a thum b sp ica s pli nt for a period of 2 to 4 weeks.(c) a corticosteroid injecti on a t the si te of pain to relieve the inflammation.(d) no intervention indicate d; s he sh oul d be a llowed to return to ski.

91. (b) This patient most likely has suffered a grade I or II ulnar collateral ligament injury. This injury is called a "gamekeeper's or skier's thumb." A complete (grade III) tear is diagnosed by a difference of 15° or more of lateral laxity compared to the uninjured side or an absolute laxity of 35°. This is not a grade III injury, and will likely heal with nonsurgical treatment consisting of immobilization in a thumb spica cast or splint for 2 to 4 weeks. This patient may be returned to ski with a thumb spica or cast in place. A steroid injection is not indicated. Ice may help with the pain and swelling, but immobilization is required for this patient. Nonsteroidal anti-inflammatory drugs may be given for pain control.

92. Early post-traumatic seizures are defined as seizures within the first (a) day. (b) month. (c) week. (d) 3 months.

92 (c) Early post-traumatic seizures occur from the first day to 1 week postinjury. Immediate seizures occur within the first 24 hours. Late seizures occur af ter the fi rst 7 d ays.

92. You are consulted to see a 19-year-old woman with a traumatic brain injury after a motor vehicle crash 2 days ago. She is unconscious even though the computed tomography scan of her brain is normal. The most likely cause is (a) diffuse axonal injury. (b) cerebral contusion. (c) arterial vasospasm. (d) epidural hemorrhage.

92 Answer: (a) Commentary: The initial computed tomography and magnetic resonance imaging scans taken soon after injury are often normal. Only 10% of patients with diffuse axonal injury (DAI) demonstrate the classic CT findings of DAI. These are hemorrhagic punctate lesions of (1) the corpus callosum, (2) the gray-white matter junction of the cerebrum, and (3) the pontinemesencephalic junction. Ref: Meythaler JM, Peduzzi JD, Eleftheriou E, Novack TA. Current concepts: diffuse axonal injury-associated traumatic brain injury. Arch Phys Med Rehabil 2001;82:1461-71.

92. The physical therapist calls you concerning the patient with traumatic brain injury you admitted last week. She tells you that his bladder incontinence is disrupting therapy. You have checked his urinalysis and there is no evidence of a urinary tract infection. A postvoid residual bladder ultrasound shows that his bladder is emptying well. Your next step is to initiate (a) an anticholinergic medication. (b) in/out catheterization. (c) a condom catheter with a leg bag. (d) a behavioral modification program and timed voiding.

92 Answer: D Commentary: This patient is exhibiting normal bladder emptying with no evidence of a bladder infection. An anticholinergic in a patient with a traumatic brain injury may exacerbate his confusion. A condom catheter in this population will probably not stay in place. It may increase agitation and will not help the patient. Intermittent catheterization and a Foley catheter will increase the patient's infection risk. The best course at this time is frequent bladder emptying and retraining, with the entire rehabilitation team encouraging the new behavioral modification. Ref: Mysiw WJ, Fugat LP, Clinchot DM. Assessment, early rehabilitation intervention and tertiary prevention. In: Zasler ND, Katz DI, Zafonte RD, editors. Brain injury medicine: principles and practice. New York: Demos Medical Publishing; 2007. p 293.

"

92. In anoxic brain injury, which area of the brain is most suscep tible to hypoxemia and hyp otension? (a) Hippocampus (b) Pons (c) Basal ganglia (d) Cerebellum" 92 (a) The mechanism of brain damage in anoxic brain injury is ischemia due to hypoxemia or decreased cerebral perfusion. Although anoxic brain injury typically causes diffuse neuronal death and injury, there is selective vulnerability of certain neurons. Neurons in parts of the hippocampus appear to be the most vulnerable, which correlates with the high frequency of amnesia following anoxic brain injury.

93. You are seeing a 79-year-old gentleman with chronic right shoulder pain. For the past several years he has had limited shoulder movement and is diffusely tender around the shoulder. Magnetic resonance imaging demonstrates a partial tear of the supraspinatus and infraspinatus tendons with degenerative changes of the glenohumeral joint. You recommend (a) rotator cuff repair. (b) total shoulder arthroplasty. (c) intra-articular viscosupplementation injection. (d) flexibility and progressive strengthening exercises.

93 Answer: (d) Commentary: The nonsurgical management of shoulder osteoarthritis (OA) with a chronic, massive rotator cuff defect requires flexibility exercises and gentle progressive strengthening exercises to increase shoulder function. Surgical repair involves humeral hemiarthroplasty. Rotator cuff repair in partial thickness tears consists of surgical smoothing of the humeroscapular motion interface with cuff curettage. Reverse total shoulder arthroplasty is used for anterosuperior escape rotator cuff lesions. There is no role for shoulder viscosupplementation, since it has not been shown to be beneficial. Ref: Matsen FA. Rotator-cuff failure. N Engl J Med 2008;358:2138-47.

94. Which finding is most common in spinal muscular atrophy? (a) Brisk reflexes (b) Weakness and atrophy (c) Sensory loss (d) Urinary retention

94 (b) Although the various forms of spinal muscular atrophy (SMA) are clinically dissimilar, they all involve loss of the lower motor neurons, causing loss o f reflexes , rathe r than hyperreflexia, along with weakness and atrophy. Sensation re ma ins in tact. Uri nary re tent ion is no t part of the disease a nd the s ens ory ner ve s remain comp lete ly in ta ct . Th e c hil dho od fo rm s of S MA may be qu ite s eve re , a lthou gh the late- onset forms may be mild.

94. A negative prognosticator for successful s urg ic al nerve repair after trauma is (a) partial transection of the nerve. (b) distal nerve injury. (c) prior radiation therapy. (d) nerve repair within 4 months of injury.

94. (c) Negative prognosticators for successful nerve repair include advanced age, nerve injury resulting from dislocation (stretch), delay of repair beyond 5 months, prior radiation therapy, nerve discontinuity (gap) exceeding 2.5 cm, proximal nerve injury and poor condition of nerve endings. Ref: Thomas MA, Felsenthal G, Fast A, Young M. Peripheral neuropathy. In: DeLisa JA, Gans BM, Walsh, NE, editors. Physical medicine and rehabilitation: principles and practice. 4th ed. Philadelphia: Lippincott Williams &Wilkins; 2005. p 908-9.

"

96. Vacuum-assisted closure marketed as ""Wou nd VA C"" works primarily by (a) increasing blood flow in the wound and adj ace nt tissue. (b) drawing the edges of the wound together. (c) sealing out potentially harmful bacteria f rom t he wound. (d) maintaining a moist, anaerobic environment." 96 (a) The Wound VAC device increases blood flow to the wound and adjacent tissue, resulting in increased oxygen delivery, increased clearance of bacteria from infected wounds, and wound healing. Ref: O'Connor K. Pressure ulcers. In: DeLisa JA, Gans BM, Walsh NE, editors. Physical medicine and rehabilitation: principles and practice, 4th ed. Philadelphia: Lippincott Williams & Wilkins; 2005. p 1614.

97. In an otherwise healthy worker with complaints of low back pain and no neu rologic deficit on ph ysi ca l exa m, m agnet ic reso na nce i magin g of the l umbar spi ne is in dicated when low back pain (a) persists for 8 weeks. (b) first occurs, so that work restrictions can be made. (c) persists after 2 weeks of bed rest. (d) continues for 4 weeks after the worker is placed on light duty.

97 (a) A magnetic resonance imaging (MRI) provides the most accurat e and complete information during the initial assessment for lumbar disc pathology, stenosi s, infecti on, tum or, or trauma. Because many patients who present with low back pain a nd lum bar radic ular pa in m ay hav e s pontane ous improvement of sym pt oms , i t is r ec ommende d to wai t for a pp roxi mat ely 7 weeks w itho ut i mpro vem ent and wit h pro per c are before p erforming an MRI.

97. What information does a functional capacity evaluation provide the treating phy sici an ?:(a) It indicates the worker's ability to return to full duty or return to work with res tr ict ion s.:(b) It shows the worker's maximum oxygen uptake while simulating work related activ itie s.:(c) It provides standardized test results that can be compared with national normat ive da ta.:(d) It differentiates discriminates between psychiatric illness and behavioral symp tom ma gni fic ati on.

97 (a) The information provided by functional capacity evaluations (FCE) includes the worker's capability to return to full or restricted duty, the need for wo rk hard ening , o r the need for w or k condit ion ing . It can al so define di screpancies b etwe en sub jec tive com pla ints and obj ec tiv e fin dings. T he FCE can also he lp deter mine if th e wor ker has rea che d m axima l m edi cal improv ement. Although general aerobic condition may be assessed, the FCE does not include formal aerobic capacity testing. The psychological assessment is important during the FCE but formal psychiatric diagnosis is not made during an FCE. Unfortunately, functional capacity evaluations are not standardized and cannot be compared. Such standardization has been proposed but currently does not exist.

97. Workers who sustain an i njur y cau sin g cont inued impairment cannot return to work if(a) they are at or over the age of 55 ye ars.(b) their job cannot be modi fied to a cco mmodat e their impairments.(c) more than 3 months have elap sed s inc e the injury.(d) the worker was responsib le f or th e i njury.

97. (b) If job requirements cannot be modified, an injured worker may not be able to return to his/her job. The employee's age or the length of time that has elapsed since the injury does not by definition exclude the worker from returning to work. Whether the worker was responsible for the injury does not exclude him/her from returning to work.

98. Which statement regarding myotonic discharges is TRUE?:(a) They are variable in frequency.:(b) They have a constant amplitude.:(c) They arise from the anterior horn cell.:(d) They are rarely seen without clinical myotonia.

98 (a) Myotonic discharges consist of repetitive waveforms of similar configuration which wax and wane in their frequency and amplitude.

98. The inferior gluteal nerve innervates which muscle? (a) Gluteus maximus (b) Gluteus medius (c) Gluteus minimus (d) Tensor fascia latae

98. (a) The inferior gluteal nerve innervates the gluteus maximus muscle. The other 3 muscles are all innervated by the superior gluteal nerve.

98. The potentials shown in this grap h a re(a) fibrillations.(b) myopathic motor units.(c) end plate spikes.(d) complex repetitive disch arge s.

98. (c) The duration of these potentials is approximately 5ms, too short for a motor unit. The initial deflection is negative, distinguishing this potential as an end plate spike rather than a fibrillation.

99. On examination, a 3-month-old girl still has a Moro reflex, a symm etric tonic neck reflex, and plantar grasp reflex. She does not have any protective extension. You advise her parents that (a) further diagnostic evaluation is indicated. (b) she requires a physical therapy evaluation. (c) she needs a neurology evaluation. (d) these reflexes are normal reflexes.

99 (d) These are normal reflexes in a 3-month-old child. The Moro and asymmetric tonic neck reflexes (ATNR) usually are integrated by approximately 6 months. The plantar grasp reflex is integrated by 12 to 14 months after walking has begun. Protective extension in sitting is seen anteriorly at 5 to 7 months, lateral at 6 to 8 months, and posterior at 7 to 8 months.

99 Children with L4-5 spina bifida are most likely to have (a) equinus foot. (b) cavus foot. (c) knee flexion contractures. (d) knee extension contractures.

99 Answer: D Commentary: The knee extensors (quadriceps) are innervated at the L3-4 level, while the knee flexors (hamstrings) are innervated at the L5-S1 level. A child with L4-5 preserved level would have quadriceps muscles that work, while hamstrings will either be weak or absent. Foot muscles are innervated at the L5-S2 levels. Equinus and cavus feet result from asymmetric pull of foot muscles, which would be seen in sacral levels of spina bifida. Ref: Molnar GE, Murphy KP. Spina bifida. In: Molnar GE, Alexander MA, editors. Pediatric rehabilitation. 3rd ed. Philadelphia: Hanley & Belfus; 1999. p 222.

99. A child with C5 ASIA A s pina l cor d i njury should eventually become independent in which activity?(a) Intermittent catheteriza tion(b) Transfer to level surfac es(c) Feeding(d) Bathing

99. (c) A child with C5 ASIA A spinal cord injury should eventually become independent in feeding, and in upper extremity dressing with assistive devices, in driving a power wheelchair, and in propelling a manual wheelchair short distances on level surfaces.

99. A normal 6-month-old infant may demonstrate which reflex? (a) Rooting (b) Automatic walking (c) Plantar grasp (d) Posterior protective extension

99. (c) Rooting and automatic walking reflexes are present at birth, and are integrated by 4 months of age. Posterior protective extension does not appear until 7 to 8 months of age. Plantar grasp is present at birth and not integrated until after independent walking occurs at approximately 12 months of age. Reflex Age of Emergence Age of Suppression (or Integration) Moro Birth 4-6 months Rooting 4 months Asymmetric tonic neck reflex (ATNR) 1-3 months 6-7 months Plantar grasp 12-14 months-ie, when walking well Automatic walking 3-4 months Posterior protective extension 7-8 months

"9.

A 14-year-old with severe traumatic brain injury adm itt ed to your rehabilitation unit has no spontaneous movement. What is the best prevention for heterotopic ossification? (a) Passive range of motion (b) Nonsteroidal anti-inflammatory medications (c) Disodium etidronate (Didronel) (d) Radiation" 9 Answer: (a) Commentary: Heterotopic ossification is found in a high per ce nta ge of children immobilized by traumatic brain injury and spinal cord injury. The best prevention for the development of HO is an aggressive program of passive range of motion. Nonsteroidal anti-inflammatory medications and radiation are available as treatment options. Didronel is not used in pediatric patients due to risk of rickets or rachitic syndrome. Reference: Krach LE, Gormley ME, Ward M. Traumatic brain in ju ry. In: Alexander MA, Matthews DJ. Pediatric rehabilitation: principles and practice. 4th ed. New York: Demos;2010. p 244.

"121.

A 22-year-old man who is right hand-dominant prese nts t o y our office with acute onset of right shoulder pain. He is a former college tennis player without a previous history of shoulder pathology. Which test would you perform to evaluate for pathology involving the labrum of the shoulder? a) Hawkin test b) Bowstring sign c) O'Brien active compression test d) Apley scratch test" 121 Answer: (c) Commentary: The O'Brien active compression test is used to ev alu ate and differentiate labral tears (superior labrum anterior posterior -- SLAP tears) from acromioclavicular joint pathology and pain. The Hawkin test is an impingement test of the shoulder and is not intended to evaluate tears of glenoid labrum. The bowstring sign is used to identify lumbar nerve root compression. The Apley scratch test is used to assess the range of motion of the shoulder. Reference: Bowen J, Malanga GA, Pappoe T, McFarland E. P hys ic al examination of the shoulder. In: Malanga GA, Nadler SF, editors. Musculoskeletal physical examination. Philadelphia: Elsevier-Mosby; 2006. p 59-118.

"72.

A 23-year-old woman who is unresponsive after an ac ute traum atic brain injury can visually track. She periodically pushes the nurse's hand away when the nurse administers a subcutaneous heparin injection. The patient is exhibiting (a) a coma state. (b) a minimally conscious state. (c) a vegetative state. (d) a sleep/wake cycle." 72 (b) A minimally conscious state is a condition of severely altered consciousness in which minimal but definite behavioral evidence of self, or environmental awareness, is demonstrated by any or all these actions: simple gestures, purposeful behavior, appropriate smile/cry or vocalization to stimulation, reach for object, purposeful visual tracking. The vegetative state is associated with preserved hypothalamic and brainstem autonomic function and the patient exhibits a sleep/wake cycle, but there is an absence of cortical activity, judged behaviorally. The patient may exhibit visual pursuit but not in relation to meaningful behavior. The term persistent vegetative state is confusing and it is suggested that the term be abandoned, since it combines diagnosis (vegetative) with prognosis (persistent). Coma is a transient state after a traumatic brain injury (TBI) of being not awake and not aware of surroundings, and is seen in patients with a severe TBI and a Glasgow coma scale (GCS) of 8 or lower.

"32.

A 25-year-old man with a history of a traumatic bra in injury is noted to have a marked functional decline from his normal level of functioning. You order a computed tomography (CT) scan, which reveals large ventricles with flattening of the sulci and periventricular lucency. You tell the family that a ventriculoperitoneal shunt (a) is emergently needed, and immediate referral to neur osurgery is i ndicated. (b) will not be helpful, because the findings on the CT scan are due to irreversible atrophy of brain tissue (hydrocephalus ex vacuo). (c) is not indicated, because he does not have the triad of incon tine nce, gait disorder, and dementia. (d) may be helpful, because about 50% of patients with p ost-traum atic brain injury hydrocephalus experience significant improvement." 32 (d) A series reported by Tribl and Oder found that of 48 patients who underwent ventriculoperitoneal shunting for post-traumatic hydrocephalus slightly more than half experienced significant benefit.

"41.

A 26-year-old mail handler is sent to you for management of her severe sen sorimoto r carpal tunn el sy ndrom e co nfirm ed by e le ctrod iagno st ic eva lu ation. Sh e w as gi ven a splint for her presumed carpal tunnel syndrome 3 months ago, which she has worn 24 hours a day since that time without relief. She notes severe tingling in her fingers that is worse at night, and she also notes difficulty with gripping the mail, because of subjective weakness. She is now having severe pain, which radiates up her hand into her forearm. You consider that a corticosteroid injection might benefit this patient. Which statement is most correct regarding this injection? (a) The risk of intraneural injection is too high, and the patient should not be injected be injec ted. (b) So that intraneural injection can clearly be recognized, do not dilute the cortico steroid with anesthetics. cortico steroid with ane st hetic s. (c) Persisting or worsening pain and numbness or swelling normally last for more than 48 hours post injectin. re than 48 hours post inj ec tion. (d) Local tenderness and superficial hematomas are rare after this injection." 41 (b) The risk of intraneural injection is real, but in experience d hands this injection is safe. Anesthetics mixed with the corticosteroid can mask the pain associate d with needle placement into the nerve and should not be used. Numbness is a nticip ated with this i njec tion w ith out use of anesthetics, an d h el ps to conf ir m proper plac emen t. Lo ca l tend ern ess an d hem at omas a re com mon wi th th is in jec tion and do not r epresent a complication. Persistent or worsening pain or swelling lasting more than 48 hours are signs of nerve injection or neurotoxic injury.

"131.

A 30-year-old woman began running 2 weeks ago. She ru ns 4 miles a day, twice a week. She began experiencing bilateral lower leg pain. On physical examination, she reports diffuse pain along the medial tibia at the start of her run with improvement during the run. The most likely diagnosis is a) stress fracture. b) medial tibial stress syndrome. c) anterior tibialis tendinitis. d) tarsal tunnel syndrome." 131 Answer: (b) Commentary: This woman presents with symptoms most consi ste nt wi th medial tibial stress syndrome (MTSS) or what has been termed "shin splints."Pain from MTSS occurs along the lower third of the posteromedial border of the tibia. A stress fracture is unlikely in this low mileage runner who has had only 2 weeks of running activity. Stress fractures generally have a focal area of pain and are not relieved with further running. Anterior tibialis tendinitis presents with anterolateral pain along the dorsal aspect of the ankle. Tarsal tunnel syndrome is associated with numbness and tingling in the foot. Reference: Hoch AZ, Pepper M, Akuthota V. Stress fractur es an d k nee injuries in runners. In: Akuthota V, Harrast MA, Kraft GH, editors. Physical medicine and rehabilitation clinics of North America. Philadelphia: Saunders; 2005. p 749-77.

"77.

A 32-year-old maintenance worker with full-thicknes s b ur ns involving the right hand and forearm is now ready for compressive garments. Which statement regarding his case is correct? (a) In order to maximize blood flow to grafted sites, co mpr es siv e garments should not exceed 15mmHg. (b) Compressive garments should be worn a maximum of 18 hou rs a day to avoid graft site maceration. (c) In 4 to 6 months full scar maturation will be achiev ed an d c ompression garments may be discontinued. (d) To maintain adequate pressure, compression garments sho ul d b e replaced every 2 to 3 months." 77 Answer: (d) Commentary: Compressive garments should provide capillar y l ev el pressures of at least 25mmHg. Wearing time should be increased gradually to 23 hours per day. Most active scarring occurs between 4 to 6 months after injury, but full scar maturation may take 12 to 18 months. To maintain adequate pressure, compression garments should be replaced every 2 to 3 months. Reference: (a) Fitzpatrick L, Murphy P, Androwick J, Gol dbl um D, Wardius P, Wijtyk J. Burn rehabilitation. In: Grabois M, Garrison SJ, Hart KA, Lehmkuhl LD, editors. Physical medicine and rehabilitation: the complete approach. Malden: Blackwell Science; 2000. p 1771. (b) Esselman PC, Thombs BD, Magyar-Russell G, Fauerbach JA. B urn rehabilitation state of the science. Am J Phys Med Rehabil 2006;85:386.

"128.

A 33-year-old woman who had a prolonged labor 6 weeks ago, reports pain in the groin radiating along the medial aspect of the thigh. Needle electromyographic exam shows evidence of denervation in the gracilis and adductor muscles. Most likely she has a lesion in the (a) femoral nerve. (b) obturator nerve. (c) sciatic nerve. (d) genitofemoral nerve. " 128 Answer: (b) Commentary: The nerve common to the affected muscles is the obturator nerve. There is some innervation of the adductor magnus by a branch off the sciatic nerve, but the gracilis, adductor brevis, and adductor longus are supplied by the obturator nerve. Injuries of the obturator nerve are uncommon, but one cause in such cases is compression of the nerve between the fetal head and the pelvic wall during prolonged labor. Reference: (1) Dumitru D, Zwarts MJ. Lumbosacral plexopathies and proximal mononeuropathies. In: Dumitru D, Amato AA, Zwarts MJ, editors. Electrodiagnostic medicine. 2nd ed. Philadelphia: Hanley & Belfus; 2002. p 844, 847, 870-1. (2) Stewart JD. Chapter 20: Obturator nerve. In: Stewart JD. Focal Peripheral Neuropathies. 3rd ed. Montreal, Canada: Lippincott Williams & Wilkins; 2000. p 483-7.

"70.

A 33-year-old woman who is 6 months pregnant compla ins o f r ight-sided, stabbing, low back pain that is worse with movement. Which orthosis is most appropriate for her? (a) Sacroiliac belt (b) Cruciform anterior spinal hyperextension orthosis (c) Silesian belt (d) Minerva brace" 70 Answer: (a) Commentary: Pregnant women frequently develop low back p ain , and a sacroiliac belt can be helpful. A cruciform anterior spinal hyperextension (CASH) brace is generally used for osteoporotic compression fractures. A silesian belt is a type of suspension for transfemoral prostheses. The Minerva brace is a cervicothoracic orthosis. Reference: (a) Barr KP, Harrast MA. Low back pain. In: B rad do m R L, editor. Physical medicine and rehabilitation. 3rd ed. Philadelphia: Elsevier; 2007. p 922. (b) Pomerantz S, Durand E. Spinal orthotics. In: Delisa JA, G ans BM, Walsh NE, editors Physical medicine and rehabilitation: principles and practice. 4th ed. Philadelphia: Lippincott Williams & Wilkins; 2005. p 1358, 1363.

"93.

A 35-year-old woman sustained an ischemic stroke and is currently undergoing workup in the acute care hospital. Systemic lupus erythematosus (SLE) is suspected. An immunoglobulin G (IgG) or IgM anticardiolipin antibody analysis is ordered to evaluate for which associated condition? (a) Antiphospholipid antibody syndrome (b) Activated protein C resistance (c) Antinuclear antibody (d) Antithrombin III deficiency " 93 Answer: (a) Commentary: Antiphospholipid antibody is associated with systemic lupus erythematosus (SLE) and can increase risk of thrombosis. SLE is diagnosed with an abnormal serum level of IgG or IgM anticardiolipin antibodies, positive lupus anticoagulant, or false-positive serologic test for syphilis. Activated protein C resistance and antithrombin III deficiency are risk factors for thrombosis and stroke, but do not have the same association with SLE. Antinuclear antibody is usually positive in SLE and is part of the diagnostic criteria, but is not associated with thrombosis. Reference: (a) Buyon JP. Systemic lupus erythematosus. Clinical and laboratory features. In: Klippel JH, Stone JH, Crofford LJ, White PH, editors. Primer on rheumatic disease. 13th ed. Atlanta (GA): Arthritis Foundation; 2008. p 304, 313, 316.(b) Vayá A, Santaolaria M, Micó L, Calvo J, Oropesa R, Villa P, Todoli J, Simó M, Corella D, Ricart JM. Thrombotic events in systemic lupus erythematosus: its association with acquired and inherited thrombophilic defects. Clin Hemorheol Microcirc. 2008; 40(2): 79-87.

"96.

A 38-year-old woman with cystic fibrosis is schedul ed to re ceive a lung transplant for end-stage pulmonary disease. She has several questions about her pre- and posttransplant rehabilitation program. You advise her that (a) performing upper limb exercises is contraindicated. (b) interval exercise training is better than continuous tr ai nin g. (c) she should wait 5 days, postoperatively, before star tin g any out of bed activity. (d) stair-climbing activity should not start until 6 wee ks af ter surgery." 96 Answer: (b) Commentary: Preoperative rehabilitation for lung transpl ant p ati ents is essential to physically prepare them for the surgery itself, and to manage their failing strength, decreased thoracic mobility and altered posture. Before surgery, interval exercise training is better than continuous training. Upper limb exercise has been safely used in rehabilitation programs, although it can contribute to dyspnea. Lung transplant patients with end-stage pulmonary disease often do better with interval exercise training than with continuous training because less ventilatory demand is required. Progressive activity should be initiated on the first postoperative day, beginning with range of motion exercises. Before discharge from the hospital, the patient should progress to stairclimbing, which is the hallmark of recovery. Reference: Young MA, Stiens SA. Organ transplantation an d r eh abi litation. In: Braddom RL, editor. Physical medicine and rehabilitation, 3rd ed. New York: WB Saunders; 2007. p 1442-4.

"7.

A 42-year-old car mechanic with a 3-week history of low back pain and lower limb pain after lifting equipment at work is referred to you for management. He has been taking ibuprofen 800mg 4 times daily without improvement. He is unable to flex through the lumbar spine or sit without pain. Your recommendations to his employer regarding work include (a) modified duty to allow no repetitive twisting or ben ding and no p ush/pull heavier than 20 lbs. (b) return to sedentary work 8 hours daily for 1 week, a nd no pus h/pu ll heavier than 10 lbs. (c) light duty to include no pushing/pulling, or lifting more tha n 25 lbs for 1 month. (d) remain off work until lumbar flexion, sitting, and lifting are no longer painful." 7 (a) Returning the employee to modified duty that fits the impairment and avoids provocative activities is important from several aspects. One, behavioral management with the employee allows early goals to be set, so that the employee can work with restrictions. It also establishes that simply being off work until pain free is not always a logical goal. Second, the employer can fully understand the employee's capabilities during recovery. This management approach hones in on the employer to comply with the restrictions. Third, starting with reasonable restrictions allows the physician to guide the employee back to the work place by making adjustments as the worker's rehabilitation progresses.

"35.

A 42-year-old woman with multiple sclerosis comes to you describing profound afternoon fatigue. You recommend (a) tizanidine (Zanaflex). (b) amantadine (Symmetrel). (c) azathioprine (Imuran). (d) glatiramer acetate (Copaxone). " 35 Answer: (b) Commentary: Many medications are indicated for multiple issues relating to multiple sclerosis (MS). Medications for fatigue include amantadine (Symmetrel) and modafinil (Provigil). Medications for use in exacerbations include prednisone, ACTH and Solu-Medrol. These drugs are supposed to decrease the length and severity of exacerbation. Disease altering medications including the interferons beta 1A and beta 1B as well as glatiramer acetate (Copaxone) can decrease the number of exacerbations. Medications for chronic MS include cyclophosphamide (Cytoxan), azathioprine (Imuran), and cyclosporine (Sandimmune), which are supposed to slow progression in chronic MS. Medications for spasticity include baclofen (Lioresal), dantrolene (Dantrium), tizanidine (Zanaflex), and diazepam (Valium). Medications for ataxia can include clonazepam (Klonopin) as well as isoniazid (Nydrazid). Reference: Kraft GH, Cui JY. Multiple sclerosis. In: Delisa JA, Gans BM, Walsh NE, editors. Physical medicine and rehabilitation: principles and practice. 4th ed. Philadelphia: LippincottRaven; 2005. p 1758- 63.

"50.

A 45-year-old man with a history of transtibial amputation secondary to trauma presents to your office 6 months following surgery. He is successfully ambulating independently with his prosthesis. His chief complaint today is new mild phantom limb pain. Evaluation does not reveal any significant problems with his prosthesis or gait. What treatment would you recommend to decrease his phantom limb pain? (a) Cryotherapy (b) Ultrasound (c) Desensitization (d) Paraffin wax " 50 Answer (c) Commentary: First line treatment for phantom limb pain should include use of desensitization techniques (massage, friction rubbing, wrapping, etc.) The other types of therapeutics listed would not be effective in phantom limb pain management. Phantom limb pain is one of many sources of pain in an amputee and is difficult to treat. It affects anywhere from 67% to 79% of amputees. For patients whose pain interferes with function and quality of life, a biopsychosocial approach to pain management is crucial. Reference: a) Czerniecki JM, Ehde DM. Chronic pain after lower extremity amputation. Crit Rev Phys Rehabil Med. 2003;15:309-22. b) Huang ME, Miller LA, Lipschutz R, Kuiken TA. Rehabilitation and prosthetic restoration in lower limb amputation. In: Braddom RL, editor. Physical Medicine and Rehabilitation. 4th ed. Philadelphia: Elsevier Saunders; 2011:282- 284.

"47.

A 47-year-old woman with secondary progressive multiple sclerosis is applying for Social Security Disability Insurance (SSDI). She asks her primary care physician for help. His correct response to her is that (a) SSDI benefits and policies vary from state to state. (b) SSDI benefits include medical insurance. (c) he will make the final determination of disability and employability. (d) she must satisfy non-medical criteria before medical factors are considered. " 47 Answer: (d) Commentary: The Social Security Administration (SSA) provides both Social Security Disability Insurance (SSDI) and Supplemental Security Income (SSI). SSDI and SSI are federal programs with identical benefits and policies from state to state. SSDI and SSI provide financial assistance to disabled individuals, but do not provide medical insurance. Final determination of SSDI or SSI is made by the SSA, not the treating provider. However, medical information is usually requested from treating providers in order to make a determination of disability. An applicant must first meet certain non-medical (eg, economic) criteria before medical factors are considered. Reference: (a) Robinson JP, Seroussi RE. Impairment rating and disability determination. In: Braddom RL, editor. Physical medicine and rehabilitation. 3rd ed. Philadelphia: Saunders Elsevier; 2007. p111. (b) United States Social Security Administration web site. http://www.ssa.gov/disability/. Accessed June 18, 2011.

"56.

A 48-year-old woman had an acute myocardial infarction (MI) 2 weeks ago. The referring cardiologist informed you that she had a small MI and an uncomplicated hospital course. In a situation such as this, which statement is TRUE? (a) Combined aerobic and resistance training, compared to aerobic training alone, has a higher risk of adverse outcomes. (b) Beta blocker agents will attenuate the benefits of exercise training. (c) A change in left ventricular (LV) dimensions (remodeling) is associated with improving LV function. (d) Cardiac rehabilitation will improve both myocardial perfusion and LVelectrophysiologic parameters. " 56 Answer: (d) Commentary: After a myocardial infarction (MI), exercise training is initiated within 2-4 weeks. Combined resistance and aerobic training improves aerobic fitness and muscle strength more than aerobic training alone, without adverse outcomes. Beta blockers, which are a standard of care to reduce mortality after an MI, do not attenuate the benefits of exercise training. Following an MI, a change in left ventricular (LV) dimensions (remodeling) is associated with deteriorating LV function, ventricular arrhythmias, aneurysm formation, and higher mortality. Cardiac rehabilitation improves both myocardial perfusion and LV electrophysiologic parameters, reducing the risk for malignant ventricular arrhythmias and sudden cardiac death after MI. Reference: Whiteson, JH. Cardiac rehabilitation. In: Braddom RL, editor. Physical medicine and rehabilitation. 4th ed. New York: WB Saunders; 2011. p 731.

"112.

A 49-year-old man is seen in your outpatient clinic 2 years after a stroke. You notice a Trendelenberg gait and suspect weakness of which muscle? (a) Gluteus maximus (b) Quadratus lumborum (c) Quadriceps (d) Gluteus medius " 112 Answer: (d) Commentary: Weakness of the gluteus medius muscle, or reluctance to use the gluteus medius muscle because of hip pain, can cause this gait pattern. It is a pattern of either excessive pelvic obliquity during the stance phase of the affected side (uncompensated) or excessive lateral truncal lean during the stance phase on the affected side (compensated). Reference: (a)Kerrigan DC, Edelstein JE. Gait. In: Gonzalez EF, Myers SJ, editors. Downey and Darling's physiological basis of rehabilitation medicine. 3rd ed. Woburn (MA): ButterworthHeinemann; 2001. p 412.(b)Krabak BJ, Jarmain SJ, Prather H. Physical examination of the hip. In: Malanga GA and Nadler SF, eds. Musculoskeletal physical examination: An evidence-based approach. Philadelphia: Elsevier; 2006. p 252, 266-7.

"17.

A 55-year-old paramedic is under your care for a wo rk-relate d sh oulder injury. She has completed physical therapy, no longer requires pain medications, and wants to return to work. She does not have full shoulder abduction and has some pain with overhead activities. Ideally, you recommend (a) return to work without restrictions. (b) work conditioning for 4 weeks. (c) a week of work hardening. (d) functional capacity evaluation." 17 (d) The paramedic has a high demand job. A functional capacity evaluation would best determine the employee's ability to return to her job. If deficits are noted, work hardening over a period of weeks will best ensure return to work. Work hardening for 1 week may not be sufficient. Work conditioning enhances aerobic fitness and conditioning but is not job specific. The paramedic is at high risk for recurrent injury. Returning the employee to work without testing the her ability to perform her job duties may precipitate premature return and reinjury.

"33.

A 55-year-old postal worker with a 1-year history of increasing left knee pain and decreasing ability to ambulate arrives at your office. Her history is significant for 30 minutes of morning stiffness and a left medial meniscal tear that was repaired arthroscopically 5 years ago. Her exam is significant for a body mass index of 35, left knee varus deformity, and mild quadriceps weakness. Her radiograph demonstrates medial compartment narrowing and bony sclerosis. She has (a) rheumatoid arthritis. (b) osteoarthritis. (c) parvovirus infection. (d) pseudogout. " 33 Answer: (b) Commentary: Osteoarthritis (OA) is the leading cause of impaired mobility in elderly persons. Risk factors include obesity, malalignment, prior trauma or surgery, and occupational bending or lifting. Radiographs of knee OA demonstrate joint space narrowing, osteophytes, bony sclerosis and cysts. Reference: Felson DT. Osteoarthritis of the knee. N Engl J Med 2006;354:841-8.

"63.

A 60-year-old man has first metatarsophalangeal joint pain. Joint fluid analysis confirms your suspected diagnosis. What lifestyle or medication changes would reduce flare-ups of his condition? (a) Increase seafood intake (b) Decrease alcohol intake (c) Increase diuretic use (d) Decrease vitamin C use " 63 Answer: (b) Commentary: Alcohol increases uric acid production and can provoke an acute gout attack. Seafood and red meat contain purines which increase serum uric acid levels and thus increase gout flares. Gout can also be provoked by trauma and drugs such as thiazide diuretics. In contrast, vitamin C has been found to decrease gout attacks. References: (a) Nucatola TR, Freeman E, Brown DP. Rheumatology. In Cuccurullo SJ, editor. Physical medicine and rehabilitation board review. New York: Demos; 2004. p 103. (b) Edwards NL. Gout: clinical features. In: Klippel JH, Stone JH, Crofford LJ, White PH, editors. Primer on rheumatic disease. 13th ed. Atlanta (GA): Arthritis Foundation; 2008. p 246, 251-2.

"150.

A 65-year-old woman with diabetes mellitus has been sent to your clinic for evaluation of appropriate footwear. She had a diabetic foot ulcer 2 months ago over her first metatarsal head, which is now healed. Examination reveals distal sensory loss and a mild claw foot deformity. Which footwear/orthotic recommendation is the most appropriate? (a) Since the wound is healed, no specialized footwear is needed. (b) Well-fitting regular tennis shoes or sneakers are the best option. (c) Off-the-shelf diabetic shoe with a custom molded orthotic based on severity of claw foot deformity is indicated. (d) Patellar tendon-bearing ankle-foot orthoses to help unload the metatarsal head should be used. " 150 Answer (c) Commentary: The patient is at high risk for future diabetic foot ulcers based on her history of previous ulcers, claw foot deformity and peripheral neuropathy. Based on this risk profile, specialized footwear is recommended. Patients with diabetes mellitus (DM) without risk factors for ulcer could be counseled and educated regarding the use of well fitting tennis shoes and sneakers. While a custom molded shoe could benefit this patient, they are very expensive and usually needed for patients with severe foot deformity. At this point a prefabricated diabetic shoe and custom orthotic would be clinically appropriate and more cost-effective. A patella tendon bearing ankle-foot-orthosis is not indicated and would not effectively offload the metatarsal heads. A metatarsal bar or pad placed proximal to the metatarsal head is an effective way to offload the metatarsal head. Ref : Howard I . The prevention of foot ulceration in diabetic patients. Phys Med Rehabil Clin N Am. 2009;20:595-609.

"66.

A 75-year-old patient with metastatic breast cancer, hypertension, and dia betes is admitte d to the h ospit al f or ra di ation o f a p athol og ica l f em oral f rac tur e. He r severe osseous pain was controlled during the initial 72 hours of admission with intravenous morphine, 5 mg/hour. Eager to facilitate discharge, a well-meaning house officer converts her to oral sustained-release morphine sulfate, 120 mg tid. Twenty-four hours after receiving the first dose of oral morphine, the patient becomes increasingly confused and somnolent. The most likely explanation for this change in mental status is (a) the dose of oral morphine is excessive (not an accurate IV to PO conversion). (b) radiation-induced hypersomnolence. (c) accumulation of morphine metabolites. (d) hypercalcemia." #NAME?

"121. A 60-year-old man comes into your office in a stooped forward position ambulat ing wi th a c ane . He is c omp laini ng of back pai n, wh ich he ha s had since his early twenties. On examination he is unable to straighten up and appears to have a fused cervical spine, since his neck range of motion is almost nonexistent. You suspect he has ankylosing spondylitis. Work-up will reveal which finding?:(a) Negative rheumatoid factor, normal erythrocyte sedimentation rate, negative ant inuc le ar ant ibo dy:(b) A normal Gillet test showing reasonably good sacroiliac motion:(c) Sparing of the thoracic spine evidenced by plain radiographs (d)

A negative res ult fo r h uma n l eukoc yte ant igen B27" 121 (a) This patient has a seronegative spondyloarthropathy, ankylosing spondylitis. This usually begins in the patient's twenties and presents with pain in the lo w ba ck an d s acroi liac area. An advance d c ase of this di sease will s how a patient wit h fuse d s acroilia c j oints and os si fic ation of th e anterior lo ngitu di nal liga ment and p os ter ior fac ets. Th is does not sp are the th oracic spine but ascends from the sacroiliac joints to the cervical spine, resulting in a C-curve of the spine. Ankylosing spondylitis has a positive human leukocyte antigen B27 in 90% of patients and a negative rheumatoid factor, negative antinuclear antibody and a normal erythrocyte sedimentation rate.

"115.

A pharmacologic treatment for orthostatic hypoten sio n tha t involves fluid retention is a) fludrocortisone. b) ephedrine sulfate. c) midodrine hydrochloride. d) recombinant human erythropoietin." 115 Answer: (a) Commentary: Fludrocortisones (0.05 mg once daily to 0.1 mg tw ice daily) is a potent mineralocorticoid with little glucocorticoid activity. It has been used to manage orthostatic hypotension (OH) related to autonomic dysfunction for more than 40 years. The pressor action of fludrocortisones is a result of sodium retention, which occurs over several days. This delayed action needs to be understood by the clinician as well as the patient to manage expectations and time frame of benefit. Ephedrine (20mg to 30mg up to 4 times daily) acts primarily through the release of stored catecholamines and has additional direct action on adrenoreceptors. It is a nonselective and mimics epinephrine in its effects. Midodrine (2.5mg to 10mg 2 to 3 times daily) is an alpha 1-adrenorecptor agonist and directly increases blood pressure by arteriolar and venous constriction. Recombinant human erythropoietin has been shown in pilot studies to increase blood pressure by about 10mmHg to 20mmHg in patients with OH. In addition to the increase in red blood cell count and blood viscosity that occurs with epoeitien ?, it may have a yet unrecognized effect on the vasculature. Reference: (a) Sabharwal, S. Cardiovascular dysfunction in sp ina l cord disorders. In: Lin VW, editor. Spinal cord medicine principles and practice. New York: Demos; 2010. p 243-4. (b) Campagnolo DI, Merli GJ. Autonomic and cardiovascula r c om pli cation of spinal cord injury. In: Kirshblum S, Campagnola D, DeLisa J, editors. Spinal co rd me dicine. Philadelphia: Lippincott Williams & Wilkins; 2002. p 124-25.

"37. Two medical experts (Drs. A and B) have differing opinions in a medical-legal case. Dr. A accuses Dr. B of citing "junk science," and states that Dr. B's testimony fails to meet the Daubert standard. Which statement supports the opinion that Dr. B has not met the Daubert standard? (a) Dr. B's research experience and publications are less than Dr. A's. (b)

Dr. B's peer-reviewed references are all more than 10 years old (c) Dr. B's cited references did not have a known error rate. (d) Dr. B's opinions are not fully accepted by the medical community. " 37 Answer: (c) Commentary: The Daubert standard refers to a federal Supreme Court decision to prevent "junk science" from influencing juries. Information given by expert testimony must meet certain criteria, and if these criteria are not met the expert can be barred from testifying. The information provided by medical experts must meet the following four criteria: 1. Generally well accepted in the medical community 2. Published in peer-reviewed literature 3. Have a scientific basis 4. Have a known error rate In the scenario presented, Dr. B satisfied the Daubert standard except for his failure to provide a known error rate in his research. Reference: (a) Foley BS, Buschbacher RM. Occupational rehabilitation. In: Braddom RL, editor. Physical medicine and rehabilitation. 3rd ed. Philadelphia: Saunders Elsevier; 2007. p1053. (b) Daubert v Merrell Dow Pharmaceuticals, Inc., 509 U.S. 579, 589 (1993). http://caselaw.lp.findlaw.com/scripts/getcase.pl?court=us&vol=509&invol=579.

"181. Which statement describes the natural his tor y of lumbar spinal stenosis (LSS)? (a) The majority of individuals with LSS will dev el op focal weakness. (b) Of patients treated nonsurgically, 25%--50 % h av e satisfactory outcomes. (c) Ambulation worsens in the majority of indi vid ua ls with LSS. (d)

Early surgery improves long-term outcome." 181. (b) The natural history of spinal stenosis is generally benign. While decompressive surgery achieves satisfactory results in the great majority of individuals, the difference in outcomes with their nonsurgical cohorts becomes narrower with time. Ref: Vo AN, Kamen LB, Shih VC, et al. Rehabilitation of orthopedic and rheumatologic disorders: Spinal stenosis. Arch Phys Med Rehabil 2005;88 (Suppl): S74.

"2. A 67-year-old woman who h ad a left co rtical stroke 12 months ago wishes to improve her arm and hand function. She has good cognition. Sensation is only mildly decreased to light touch. Muscle strength is shoulder flexion 4-/5, elbow flexion 3/5, elbow extension 3-/5, wrist extension 3-/5, finger flexion 2/5, and finger extension 2-/5. Which technique is most likely to result in functional improvement in this patient? (a) Constraint-induced movem ent (b) Proprioceptive neuromusc ular faci lit ation (c) Electromyographic biofee dbac k to wri st and arm extensors (d)

Electrical stimulation to finger flexors" 2. (a) Constraint-induced movement is effective in persons more than a year after stroke if they have preserved wrist extension and finger movement along with good sensation. Proprioceptive neuromuscular facilitation is typically used during the acute phase of stroke and is not more effective than other traditional treatments. EMG biofeedback has a mixed record but is probably a good adjunctive treatment. Functional electrical stimulation appears to be useful in muscle retraining but would probably not be applied to the finger flexors in this patient. No randomized, controlled studies have compared these therapies for efficacy.

"28.

Electrophysiologic findings of compound muscle action potential conduction block a nd tempo ral d isp er sion, pro longe d minim um F-wa ve la te ncy , a nd reduc ed con duc ti on velocity would most likely be seen in (a) Charcot-Marie-Tooth disease. (b) myasthenic syndrome. (c) Guillain-Barré syndrome. (d) amyloidosis." 28 (c) All the findings mentioned are features associated with an a cquired demyelinating condition such as Guillain-Barré syndrome or acute inflammatory demye linating p olyradi culoneuropathy (AIDP). Hereditary motor sensory neuropathies do not u sually have tem poral d ispe rsion of compoun d muscle action pot ent ia ls. My asth en ic syndro me i s a ne uro mu sc ular ju nct ion diso rd er a nd a mylo ido sis is a sso ci ate d wit h a form of axonal peripheral neuropathy.

"67.

In persons with traumatic spinal cord injury (SCI), which statement regarding employment is TRUE? (a) The majority of patients are unemployed at the time of injury. (b) Education is most strongly associated with postinjury employment. (c) Employment status is similar between different ethnic groups. (d) Employment status is highest within the first 5 years postinjury. " 67 Answer: (b) Commentary: The National Spinal Cord Injury (SCI) Statistical Center database states that at the time of injury, 63% of people injured were employed, 19% were students, and 17% were unemployed. While unemployment at the time of injury is a negative predictor for postinjury employment, education has been found to be the factor most strongly associated with postinjury employment, with only 5% of persons with less than 12 years of education being employed, and 69% of persons with doctoral degrees being employed. Overall, only about 25% of all persons with SCI were employed. African Americans and Hispanics with SCI fared worse in employment outcomes compared to Caucasians with SCI. Employment status increased over time, with the odds of being employed at 1, 5 and 10 years after injury being 1.58, 2.55, and 3.02, respectively. Reference: (a) Bryce TN, Ragnarsson KT, Stein AB. Spinal cord injury. In: Braddom RL, editor. Physical medicine and rehabilitation. 3rd ed. Philadelphia: Saunders Elsevier; 2007. p1315. (b) Arango-Lasprilla JC, Ketchum JM, Francis K, Lewis A, Premuda P, Wehman P et al. Race, ethnicity, and employment outcomes 1, 5, and 10 years after spinal cord injury: a longitudinal analysis. PM R 2010;2:901-10.

"155. A 37-year-old woman with C5 ASIA A tetrap leg ia from trauma 1 month ago is admitted to your acute rehabilitation unit. She has a retrievable inferior vena cava (IVC) filter and no history of chemical prophylaxis for deep vein thrombosis (DVT). Her surgical team reports to you that they are no longer concerned with an acute bleeding potential related to her trauma and her hematocrit is stable. What should you do first? (a) Order a lower extremity doppler study to l ook f or DVT (b) Start mechanical prophylaxis with sequenti al co mpression devices (c) Tell the patient she is completely protect ed fr om pulmonary emboli (d)

Leave the IVC filter in place for a minimum of 4 months" 155. (a) If anticoagulation is delayed for more than 72 hours after injury, a test to exclude the presence of clots in the legs should be performed. In complete injuries, low molecular weight heparin should be used when starting chemical prophylaxis. Pulmonary embolisms may occur as a result of upper extremity DVT and are not prevented by the IVC filter. In general, the longer you wait to remove the IVC filter, the more problems you may experience in the filter retrieval process. Ref: Campagnolo DI, Merli GJ. Autonomic and cardiovascular complications of spinal cord injury. In: Kirshblum S, Campagnola D, DeLisa J, editors. Spinal cord medicine. Philadelphia: Lippincott Williams & Wilkins; 2002. p 123-34.

"62. Which statement is TRUE of pseudodementia? (a) Usually a history of previous psychiatric problems e xis ts . (b) Onset is indistinct with a long history of problems bef or e c onsultation. (c) Memory loss of recent items is worse than for remote it em s. (d)

Nocturnal accentuation of dysfunction is common." "62 Answer: (a) Commentary: Distinguishing dementia from pseudodementia (wh ic h i s really depression) is important, in order to provide appropriate treatment to your patient with memory problems. Pseudodementia's onset is fairly well demarcated with a short history and is rapidly progressive in nature. These patients usually have a history of a previous psychiatric difficulty or a recent life crisis. Their complaints of cognitive dysfunction are detailed and elaborate with an affective change and the patients expend little effort on examination items. Nocturnal exacerbations are rare and memory loss is inconsistent in recall of recent and remote items. Dementia's onset, in contrast, is indistinct with a his tor y of problems long before they seek clinical help and early deficits often go unnoticed. A history of previous psychiatric problems or emotional crisis is uncommon. These patients struggle with cognitive tasks but usually put forth good effort. Nocturnal dysfunction is common. The memory loss on recent items is worse than for remote items and there is a consistent impairment of performance. Reference: Walker-Batson D, Avent JR. Adult neurogenic com mu nic ation disorders. In: Braddom RL, editor. Physical medicine and rehabilitation. 3rd ed. Philadelphia: Elsevier; 2007. p 57-8."

"169.

On physical examination an 8-year-old patient stands on his toes and has increase d lumbar lord osi s. He h as a Tren de lenbu rg gait with c irc umd uc tion. Wha t e lse w ould you expect to find on his exam? (a) Decreased sensation in his feet (b) Anterior tibialis weakness (c) Quadriceps weakness (d) Hyperreflexia at the ankle" 169 (c) The exam describes typical findings in a boy with myopathy s uch as Duchenne muscular dystrophy (DMD). The earliest weakness in DMD is proximally in the gluteus m aximus. The boy assumes a posture of lumbar lordosis to place the center o f gravit y posteri or to t he h ip joi nt to prev ent hyperflexion of th e hip an d th us a fall. Toe walk in g i s a comp ens ato ry adapt at ion to k nee ext ens or we akn es s.

"80.

Proper positioning for a transtibial amputee should in cl ude use of a (a) pillow underneath thigh. (b) pommel between legs. (c) limb board underneath knee. (d) wedge cushion underneath buttocks." 80 Answer: (c) Commentary: A limb board placed underneath the knee will he lp to prevent knee flexion contractures. Placing a pillow underneath the thigh would encourage the development of a hip flexion and possibly a knee flexion contracture. A pommel between the legs may encourage a hip abduction contracture. A wedge cushion would promote hip flexion contractures. Reference: Kuiken TA, Miller L, Lipschutz R, Huang ME. Reh ab ili tation of people with lower limb amputation. In: Braddom RL, editor. Physical medicine and rehabilitation. 3rd ed. Philadelphia: Elsevier; 2007. p 288.

25. A 24-year-old man with T6 complete paraple gia w hose injury occurred 16 weeks ago. He is concerned he can no longer reach down to put on and tie his right shoe. Upon evaluation, he has significant loss of range of motion in the right hip with mild warmth at the hip. There is no swelling at the knee, lower leg, ankle, or foot. The most likely diagnosis is (a) hip dislocation. (b) deep vein thrombosis. (c) heterotopic ossification. (d) iliopsoas abscess.

Ref: Banovac K, Banovac F. Heterotopic ossification. In: Kirshblum S, Campagnolo DI, DeLisa JA, editors. Spinal cord medicine. Philadelphia: Lippincott Williams & Wilkins; 2002. p 253-60. 26. (c) Many chemotherapeutic agents can cause a peripheral neuropathy. Treatment for neuropathic pain includes membrane-stabilizing medications such as Neurontin. Opiates like MS Contin and nonsteroidal anti-inflammatory drugs (NSAIDs) like naproxen are not the first line treatment for neuropathic pain. Prednisone is appropriate for complex regional pain syndrome (CRPS), but CRPS is not common in cancer patients after chemotherapy. Further, this patient probably does not have CRPS, considering the absence of swelling, color changes, or temperature changes.

"99.

Risk factors for osteoporotic long bone fractures i n c hi ldr en with cerebral palsy include (a) spastic hemiplegia. (b) adequate oral nutrition. (c) vitamin D supplementation. (d) decreased mobility." 99 Answer (d) Commentary: Fracture risk is shown to increase with decr eas ed mo bility, nutrition via gastrostomy tube and tetraplegia. Nutritional status is also linked to low bone mineral density (BMD). Deficiencies in vitamin D, calcium, folate, iron and magnesium have been found in children with cerebral palsy. Vitamin D and calcium are especially important in maintaining bone mineral density (BMD). Reference: (a) Henderson RC, Kairalla J, Abbas A, Steven son R D. Predicting low bone mineral density in children and young adults with quadriplegic cerebral palsy. Dev Med Child Neurol 2004;46:416-419. (b) Henderson RC, Linn PP, Greene WB. Bone-mineral densi ty in ch ildren and adolescents who have spastic cerebral palsy. J BoneJoint Surg 1995;77:1671-81. (c) Apkon SD, Kecskemethy HH. Bone health in children wi th ce reb ral palsy. J Pediatr Rehabil Med 2008:115-21.

"118. A 70-year-old man presents with a 3-month history o f numbnes s in patchy areas over the limbs and torso. His numbness began in the left foot, then the right hand, followed by numbness over the back and all the limbs. He has no complaints of bowel or bladder problems. He has a long history of smoking. His examination reveals normal strength, normal cranial nerve function, but sensation is decreased to pin prick, vibration, and position in the limbs. Deep tendon reflexes are absent. Electrophysiologic studies show normal motor nerve conduction and needle examination of the upper and lower limb muscles. The sensory nerve conduction studies show small or absent responses. Based on this information what test would you order next? (a) Nerve and muscle biopsies (b) Radiologic studies to assess for a tumor (c)

Skin b iopsy to asse ss small nerve fibers (d) Repetitive nerve conduction studies" 118 (b) The clinical and electrophysiologic presentation is consistent with a sensory neuronopathy. With no evidence to suggest motor involvement, the numbness is likely a disorder of the dorsal root ganglion. There are only a few distinct disorders associated with acute or subacute cases described by the history and physical in this clinical vignette. They may be part of a paraneoplastic syndrome, connective tissue disorder such as Sjogren's, a postinfectious condition, pyridoxine intoxication, or as an isolated autoimmune process. In this patient with a history of smoking, a cancer work up would include obtaining anatomic studies of the chest. Biopsies of the nerve, muscle, or skin would not add much to the case. Repetitive nerve conduction studies would be considered if a neuromuscular junction disorder was suspected.

"95.

What advice would you provide to a 22-year-old man with chronic T4 ASIA A paraplegia who has ejaculatory dysfunction? (a) Avoid ejaculation because of complications related to autonomic dysreflexia (b) Use sildenafil (Viagra) 60 minutes before intercourse (c) Consider a trial of vibratory stimulation (d) Ejaculation dysfunction cannot be treated " 95 Answer: (c) Commentary: Men with an upper motor lesion (UMN) and an ejaculation reflex have a 30% to 96% ejaculation rate with vibratory stimulation, depending on the vibratory stimulator's waveform amplitude and frequency. If vibratory stimulation is unsuccessful, ejaculation can be accomplished and sperm collected using a rectal probe with electroejaculation. Sildenafil is an option for management of erectile dysfunction rather than ejaculation dysfunction. Although autonomic dysreflexia may occur with ejaculation, it is more commonly a transient phenomenon and does not lead to complications. Reference: (a) Linsenmeyer TA. Sexual function and fertility following spinal cord injury. In: Kirshblum S, Campagnola D, DeLisa J, editors. Spinal cord medicine. Philadelphia: Lippincott Williams & Wilkins; 2002. p 322-30. (b) Elliott S. Sexual dysfunction and infertility in men with spinal cord injury. In: Lin VW, editor. Spinal cord medicine: principles and practice. New York: Demos Medical; 2010. p 420-23.

"74.

What is your next course of action in a 7-year-old boy n ewl y diagnosed with Duchenne muscular dystrophy? (a) Evaluate for a wheelchair (b) Recommend spinal orthosis to correct scoliosis (c) Order an echocardiogram (d) Order physical therapy for contracture reduction" 74 Answer: (c) Commentary: Duchenne muscular dystrophy is an X-linked r ece ss ive disorder caused by the absence of dystrophin. Prognosis is poor because of cardiac involvement leading to severe heart failure. Baseline echocardiograms and electrocardiograms should be obtained after establishment of the diagnosis. Contractures are common in DMD, but typically are seen after 13 years of age. Scoliosis incidence is between 33% to 100% and presents between ages 12 and 15 years corresponding to the adolescent growth spurt. Orthotics and a wheelchair will be necessary in the course of the disease, but not at age 7. Reference: (a) Finsterner J, Stollberger C: The heart in hu ma n d ystrophinopathies. Cardiology 2003;99(1):1-19.(b) McDonald CM, Han JJ, Abresch RT, Carter G. Myopathic disorders. In: Braddom RL, editor. Physical medicine and rehabilitatio n. 3r d e d. Philadelphia: ElsevierSaunders; 2007. p 1110. (c) McDonald CM. Neuromuscular diseases. In: Alexander MA, Matthews DJ, editors. Pediatric rehabilitation: principles and practice. 4th ed. New York: Demos; 2010. p 277-333.

"16.

Which barrier is perceived by older individuals to be the LEAST significant obstacle to physical activity? (a) Time, money, family commitments (b) Illness and injury (c) Fear of injury (d) Availability of an exercise partner " 16 Answer: (a) Commentary: Perceived barriers are a powerful negative predictor of physical activity in the elderly. Although individual variation is the rule, overall obstacles to physical activity tend to change with age, and seem to increase for many aging individuals. Elderly patients report that time, money and family commitments are less significant barriers as they age. Availability of an exercise partner, illness, injury and fear of injury become more prominent concerns as they grow older. Reference: Phillips EM, Schneider JC, Mercer GR. Motivating elders to initiate and maintain exercise. Arch Phys Med Rehabil 2004;85(Suppl 3):S52.

"126.

Which burn patient has the highest risk of developing hypertrophic scars? (a) Newborn baby (b) Morbidly obese individual (c) Heavily pigmented individual (d) Elderly individual " 126 Answer: (c) Commentary: A hypertrophic scar is usually defined as a scar that is present at 3 or more months after the burn injury and is greater than or equal to 2 mm in thickness. Heavily pigmented patients tend to scar more than persons with less pigment. Little scarring has been reported in neonates, newborns, elderly and the morbidly obese. Patients with wounds that take longer than 2-3 weeks to heal, and persons requiring skin grafts, are also considered at risk for developing hypertrophic scars. Reference: Esselman PC, Moore ML. Issues in burn rehabilitation. In: Braddom RL, editor. Physical medicine and rehabilitation. 4th ed. New York: WB Saunders; 2011. p 1408-9.

"66.

Which cancer related pathological fractures require su rg ica l management? (a) Humeral, if life expectancy is less than 3 months (b) Radial, if pain resolves following radiation (c) Femoral, if life expectancy is greater than 1 month (d) Pelvic without acetabular involvement" 66 Answer: (c) Commentary: The indications for surgery for pathological fr ac tur es from cancer are life expectancy of greater than 1 month with a fracture of a weight-bearing bone, and greater than 3 months for fracture of a non-weight-bearing bone. If pain persists following radiation, fractures should be managed surgically. Healing rates are low following pathologic fractures, with 1 review of 123 patients reporting a 35% incidence of fracture healing. Fractures of the pelvis are generally treated conservatively, unless pain persists after radiation or unless they involve the acetabulum. Reference: Cheville A. Cancer rehabilitation. In: Braddo m R L, ed itor. Physical medicine and rehabilitation, 3rd ed. New York: WB Saunders; 2007. p1376.

"2.

Which clinical examination finding increases the likelihood that a stroke patient has had an ischemic stroke and NOT a hemorrhagic stroke? (a) Neck stiffness (b) Cervical bruit (c) Diastolic blood pressure greater than 110 mm Hg (d) Headache " 2 Answer: (b) Commentary: There are two fundamental types of stroke and differentiating the two types of stroke has become more important as the use of thrombolytics in the acute management of stroke has become more important. Runchey and McGee in a review of 19 prospective articles with data from 6438 patients found that the following clinical findings increased the probability of hemorrhagic stroke: coma, neck stiffness, seizures, diastolic blood pressure greater than 110 mm Hg, vomiting and headache. While other findings (cervical bruit and prior transient ischemic attack) decreased the probability of hemorrhagic stroke and made ischemic stroke more probable. However, no specific finding or combination of findings was definitively diagnostic. Reference: a) Runchey S, McGee S. Does this patient have a hemorrhagic stroke? Clinical findings distinguishing hemorrhagic stroke from ischemic stroke. JAMA 2010;303(22):22802286.b) Harvey Rl, Roth EJ, Yu DT, Celnik P. Stroke syndromes. In: Braddom RL, editor. Physical medicine and rehabilitation: principles and practice. 4th ed. Philadelphia: Elseivier Saunders; 2011. p 1180-1182.c) Brandstater ME. Stroke rehabilitation. In: DeLisa JA, Gans BM, Walsh NE, editors. Physical medicine and rehabilitation: principles and practice. 4th ed. Philadelphia: Lippincott-Raven; 2005. p 1657-1659.

"38.

Which description best localizes the extensor indic is propri us m uscle (with the forearm fully pronated) for needle electrode examination? (a) Junction of the upper and middle third of the forear m between the radius and ulna (b) Four fingerbreadths proximal to the wrist and direct ly over t he u lnar side of the radius (c) Two fingerbreadths proximal to the ulnar styloid and just rad ial to the ulna (d) Mid-forearm along the radial border of the ulna" 38. (c) Answer (a) describes the location of the extensor digitorum communis muscle; answer (b) describes the location of the extensor pollicis brevis muscle; and answer (d) describes the location of the extensor pollicis longus.

"4.

Which disorder does NOT have pes cavus as a feature? (a) Poliomyelitis (b) Cerebral palsy (c) Friedreich ataxia (d) Peroneal spastic foot" 4 Answer: (d) Commentary: The etiology of pes cavus includes malunion of ca lca neal or talar fractures, burns, sequelae resulting from compartment syndrome, residual clubfoot, and neuromuscular disease. The remaining cases are idiopathic and nonprogressive. Neuromuscular diseases, such as muscular dystrophy, Charcot-Marie-Tooth (CMT) disease, spinal dysraphism, polyneuritis, intraspinal tumors, poliomyelitis, syringomyelia, Friedreich ataxia, cerebral palsy and spinal cord tumors can cause muscle imbalances that lead to elevated arches. Multiple theories have been proposed for the pathogenesis of pes cavus. Duchenne described intrinsic muscle imbalances causing an elevated arch. Whereas, peroneal spastic foot is characterized by pain in the foot, limited subtalar motion, pes planus deformity, and shortening with spasm of the peroneal muscles often initiated by minor trauma or unusual activity. Reference:Turner, NS. Pes Cavus. E medicine 2010. Avail abl e at: http://emedicine.medscape.com/article/1236538-overview. Ac ce sse d May 14, 2010. http://emedicine.medscape.com/article/1236538-overview. Ac ce sse d May 14, 2010.

"85.

Which finding is most closely associated with favorable motor recovery after a traumatic spinal cord injury? (a) Recovery from spinal shock in less than 4 weeks after injury (b) ASIA B classification with retained pinprick sensation in the sacral dermatomes (c) Detection of somatosensory evoked potentials in the first 2 weeks after injury (d) Hemorrhage in the spinal cord of less than 1cm on MRI " 85 Answer: (b) Commentary: ASIA B patients with preservation of sacral pinprick sensation have a 70% to 90% chance of motor recovery sufficient to ambulate. The concept of spinal shock has been poorly defined and is generally not helpful to clinicians in predicting recovery. The detection of somatosensory evoked potentials is not always associated with motor recovery. Hemorrhage of any amount is generally associated with a poorer prognosis. Reference: Ditunno JF, Flanders AE, Kirshblum S, Graziani V, Tessle. Predicting outcome in traumatic spinal cord injury. In: Kirshblum S, Campagnolo DI, DeLisa JA, editors. Spinal cord medicine. Philadelphia: Lippincott-Williams & Wilkins; 2002. p 108-22.

"78.

Which finding would you expect in a 35-year-old man with type 1 hereditary motor sensory neuropathy? (a) Absence of ankle reflexes (b) Pes planus (c) Motor nerve conduction velocity slowing, with evidence of temporal dispersion (d) Absence of electromyographic spontaneous activity " 78 Answer: (a) Commentary: Hereditary motor sensory polyneuropathy type 1 (HMSN-1, the demyelinating form of Charcot-Marie-Tooth disease) presents in young adult life with insidious onset of distal weakness and sensory loss. Clinically, it typically presents with pes cavus, hammertoes and foot drop. Ankle reflexes are absent. Temporal dispersion would indicate an acquired, not hereditary, process. Secondary axonal loss is expected, which would result in positive waves and fibrillations. Reference: (1) Preston DC, Shapiro BE. Polyneuropathy. In: Preston DC, Shapiro BE, editors. Electromyography and neuromuscular disorders. 2nd ed. Philadelphia: Butterworth-Heinemann; 2005. p 398-9. (2) Albers JW. Evaluation of the patient with suspected peripheral neuropathy. In: Pease WS, Lew HL, Johnson EW, editors. Johnson's practical electromyography. 4th ed. Philadelphia: Lippincott Williams & Wilkins; 2007. p 309-11.

"14.

Which muscle group displays the earliest pattern of we ak nes s in Duchenne muscular dystrophy? (a) Ankle dorsiflexors (b) Neck flexors (c) Shoulder flexors (d) Knee extensors" 14 Answer: (b) Commentary: In Duchenne muscular dystrophy, weakness is fir st se en in the neck flexors during preschool years. Pelvic girdle weakness precedes shoulder girdle weakness by several years. Ankle dorsiflexors are weaker than plantarflexors; ankle everters are weaker than inverters; knee extensors are weaker than flexors; hip extensors are weaker than flexors. Reference:McDonald CM. Neuromuscular diseases. In: Alexa nde r MA, Matthews DJ, editors. Pediatric rehabilitation: principles and practice. 4th ed. New York: Demos; 2010. p 289.

"44.

Which pulmonary parameter is predictive of mortality in a child with Duchenne muscular dystrophy? (a) Maximal expiratory pressure (b) Peak flow rate (c) Cough peak flow (d) Forced vital capacity " 44 Answer:(d) Commentary: One simple method of assessing the interplay between pump function and load is the measurement of the forced vital capacity (FVC) and fractional lung volumes. In boys with Duchenne muscular dystrophy (DMD), the relationship between the absolute value of FVC and age can be divided into 3 epochs: one of gradual increase coincident with their ambulatory period, followed by a plateau phase at 10 to 12 years when they become confined to wheelchairs, and then a gradual but persistent decline thereafter. However, when the FVC is described as a percent of the predicted value, it is lower than normal and diverges from the normal curve over time. The decline in FVC to a value of less than 1 liter may also predict mortality in patients who do not receive assisted ventilation. Reference: Panitch H. The pathophysiology of respiratory impairment in pediatric neuromuscular disease. Pediatrics 2009;123(suppl):S215-S218.

"32.

Which statement concerning management of seizures a fte r a t raumatic brain injury is TRUE? (a) All patients with postresuscitation Glasgow Coma Sca le sc ore below 12 require 3 months of an antiepileptic medication. (b) Seizures occurring less than 24 hours postinjury req uir e an antiepileptic medication for at least 12 months. (c) Seizures occurring 24 hours to 7 days postinjury sho uld b e t reated with at least 12 months of an antiepileptic medication. (d) Seizures occurring more than 7 days postinjury shoul d b e tre ated with an antiepileptic medication for at least 3 years." 32 Answer: (c) Commentary: The American Academy of Physical Medicine an d R eh abi litation and the American Association of Neurological Surgeons recommend sei zu re prophylaxis after a traumatic brain injury as standard treatment. All patients with postresuscitation Glasgow Coma Score (GCS) below 12 require 7 days of therapeutic phenytoin sodium. Immediate posttraumatic seizures (defined as those occurring within 24 hours postinjury) do not require any additional prophylaxis after 7 days. Early (more than 24 hours but less than7 days) seizures should be treated with at least 12 months of an antiepileptic medication, unless a time-limited intracranial abnormality such as hydrocephalus, infection, or active hemorrhage, etc., was the cause. Late seizures -- those occurring more than 7 days postinjury -- should be treated with an antiepileptic medication for at least 12 months. Any seizure that lasts longer than 2 minutes is defined as "status epilepticus" and warrants treatment with an antiepileptic medication for at least 12 months. Reference: (a) Cifu DX, Kreuzer JS, Slater DN, Taylor L . R eh abi litation after traumatic brain injury. In: Braddom RL, editor. Physical medicine and rehabilitation. 3rd ed. Philadelphia: Elsevier; 2007. p 1141-2. (b) Yablon SA, Dostrow VG.. Post-traumatic seizures and epi le psy . In: Zasler ND, Katz DI, Zafonte RD, editors. Brain injury medicine: principles and practice. New York: Demos; 2007. p 443-68.

"117. You are asked to provide a brief synopsis of worker s' co mpe nsation benefits to the hospital's case management department. Which statement about the benefits and services provided in the workers' compensation system is correct? (a) The employer has to be at fault in order for the inj ure d emp loyee to seek medical care. (b) Injured workers continue to receive their full wages as l ong as they are unable to work. (c) Workers' compensation programs are designed and admi nis te red by each individual state. (d)

Workers' compensation is primarily financed by federal and state funds." 117 Answer: (c) Commentary: Workers' compensation provides benefits to w ork er s w ho are injured on the job or have a work-related illness, regardless of who is at fault for the injury or illness. Benefits include medical treatment for work-related conditions and cash payments that partially replace lost wages. In the event that symptoms do not completely resolve, financial compensation is also provided. In exchange, an injured worker gives up the right to sue the employer because of a work-related injury or illness. Workers' compensation programs are designed and administered by each state, and programs and policies vary from state to state. Workers' compensation is financed almost exclusively by employers, not federal or state funds. Reference: Reno V, Williams CT, Sengupta I. Workers' com pen sa tio n, Social Security disability insurance, and the offset: a fact sheet. Washington DC: US Social Security Administration; Social Security Bulletin 2003/2004;65(4):3-6.

"143.

You receive a call from your 70-year-old patient w ith o ste oporosis. She has been taking alendronate (Fosamax) for 3 years. The news reports and her friends are all talking about hip fractures in patients taking biphosphonates. You state that based upon scientific evidence there is (a) increased risk of femoral fractures. (b) increased risk with the initiation of bisphosphonate s a t a y ounger age. (c) no increased risk in patients with prior fractures. (d) no increased risk of femoral fractures." 143 Answer: (d) Commentary: Recent secondary analysis of 3 large, rando miz ed bi phosphonate studies did not find increased risk of subtrochanteric or femoral fractures. Proposed risk factors such as younger age upon initiation of biphosphonate treatment has not been confirmed or studied. A risk factor for future fractures is a history of prior fractures. Reference: Black DM, Kelly MP, Genant HK, Palermo L, Ea ste ll R, Bucci-Rechtweg C, Cauley J, Leung PC, et al. Biphosphonates and fractures of the subtrochanteric or diaphyseal femur. N Engl J Med 2010. Available at www.nejm.org. Accessed Mar 24, 2010. (10.1056/NEJMoa1001086).

"49.

Your 5-year-old patient with spastic tetraplegic ce rebral pa lsy needs a wheelchair prescription. He is dependent for transfers, but cognitively normal. He is able to feed himself and uses a communication device. His family transports him in their car in an adapted car seat. On examination, he is unable to sit unsupported, but sits well with minimal support; he has no scoliosis, and his passive range of motion is full. Which elements would be best to include in his wheelchair prescription? (a) Folding frame, sling seating (b) Adaptive stroller, linear seating (c) Tilt in space frame, custom seating (d) Rigid frame, contoured seating" 49 (d) While this child is totally dependent for transfers, he only requires minimal support to sit upright and has no fixed deformities. Custom seating should be used for those with fixed deformities. A tilt-in-space frame should be used when children need to have their position in space changed frequently because of deformities or medical problems. While it is tempting to prescribe a wheelchair with a folding frame for a family who transports a child in a car rather than a van, the child will be better positioned using contoured seating and a rigid frame. At age 5 years, the size of frame needed will be able to be transported in a car even without folding. Adaptive strollers usually position the child in a reclined position and should be used as a backup to a wheelchair, which is not easily transported in an automobile, or for a child who can walk but periodically needs dependent mobility for fatigue or following seizures or for similar reasons.

"130. In an upper-extremity prosthesis, an advantage of a voluntary-closing hook ter mina l dev ice th at a volu nta ry-op eni ng device d oes n ot h ave i s:(a) the ability to carry objects with less fatigue.:(b) improved ability to manipulate fragile objects.:(c) an adjustable pinch force effected by changing the number of elastic bands. (d)

a b et ter ab ili ty to vis ual ize t he obj ect bein g m an ipul ate d." 130 (b) One advantage of a voluntary-closing hook terminal device is the ability to adjust the amount of pinch force by adjusting pressure on the cable control. Th is i mprov es the a bility to m an ipulate fra gil e ob ject s. A voluntary opening termi nal device us es elast ic bands to det er min e the pinch f orce and th e num be r of ban ds wo uld h av e t o b e re duce d t o d ecrea se the pi nch for ce. When carrying objects, voluntary-closing terminal devices require constant cable tension and this can result in quicker fatigue. The ability to visualize the object being manipulated would not be dependent upon whether the terminal device is voluntary closing or voluntary opening.

"85. A 22-year-old woman with a C5 ASIA class A spinal cord injury sustained in a car c rash 2 w eeks ago c ompla ins of li gh thead ed ness, dizz in ess , a nd nause a d uri ng he r physical therapy session. In response to her therapist's call, you recommend (a) sitting the patient up and loosening tight garments. (b) placing the patient in Trendelenburg position. (c) using elastic abdominal binders and elastic stockings. . (d)

adjustment of HALO vest" 85 (c) Orthostatic hypotension (OH) is a decrease in blood pressure that results from a change in body position toward the upright posture. Symptoms include l ightheaded ness, d izziness, nausea. This form of hypotension is most likely to occur in perso ns with h igh lev els of inj ury . Treat ment involves da ily ti lt ing wi th g ra du al chan ge t o up ri ght p os ture . E las tic bind er s he lp c ompr ess th e abd ome n, th us li miting blood accumulation in the abdominal vasculature. Elastic stockings limit blood accumulation in lower extremities. Patients must be adequately hydrated. Salt tablets, 1 gram 4 times daily, ephedrine, 20-30mg up to 4 times daily, Florinef, and Midodrine may be used as pharmacologic adjuncts.

"122. Regarding central nervous system tumors in adults,:(a) meningiomas are the most common form of primary tumors.:(b) approximately 50% of these tumors are metastatic.:(c) glioblastoma multiforme has a median survival rate of 5 years. (d)

brain tumor trea tm ent si de effec ts d o n ot af fec t o utcome." 122 (b) Meningiomas are the second most common form of primary tumors. Gliomas are the most common. Glioblastoma multiforme has a median survival rate of less t han 1 y ear. Tre atmen t side effe ct s do aff ect ou tcom e. M eta static disea se comprises 0.5 of cen tra l nervou s s ystem tum ors .

"141. A high school athlete sustains a suspected concussion during a football game. The player should be (a) removed from play, evaluated and, if asymptomatic, be allowed to return to the game on the same day. (b) able to continue playing if he or she is able to perform. (c) immediately transported to the local emergency department for evaluation. (d)

evaluated on the sideline and should not return to play that same day. " 141 Answer:(d) Commentary: When an athlete sustains a concussion in a game or during practice, he or she should not return to play on the same day of the injury. The athlete should be removed from play and be evaluated on the sidelines. Standard emergency medical management principles should be applied when appropriate; serial monitoring should be performed and the athlete's disposition should be determined. The athlete should follow up with an appropriate healthcare provider before he or she is returned to play. Reference: McCrory P, Meeuwisse W, Johnston K, Dvorak J, Aubry M, Molloy M, et al. Consensus statement on concussion in sport. 3rd international conference on concussion in sport; November 2008; Zurich, Switzerland; Clin J Sport Med 2009;19(3):185-195.

"19 A 70

old man with COPD presents to your office for follow-up. His forced expiratory volume in 1 second (FEV1) is 55% of predicted normative values. What would you expect the patient's functiona l lim ita tions to be? a) No functional impairment; the patient is able to walk significant distances without difficulty. a. b) Mild functional impairment; the patient is able to walk significant distances at a slower speed. b. c) Moderate functional impairment; the patient requires intermittent rest when walking and climbing stairs. c. d) Severe functional impairment; the patient is only able to ambulate for very short distances." 19 Option c is correct. Commentary: The World Health Organization's Global Initiative for Chronic Obstructive Lung Disease classifies patients who have an FEV1 between 50%-79% o f pre dic ted values as moderately impaired. This equates to the FEV1 dropping between 1-2 liters. Functional impairment develops when the FEV1 falls below 3 liters. Patients with an FEV1 between 30%-49% of predicted values are severely impaired while those with an FEV1 less than 30% are the most impaired. Reference: (a) Keyser RE, Chan L, Woolstenhulme JG, Kennedy M, Drinkard BE. Pulmonary rehabilitation. In: Braddom RL, editor. Physical medicine and rehab ilita tio n. 4th ed. Philadelphia: Elsevier; 2011 p 744-5. (b) Manasian S. Pulmonary rehabilitation. In: Nesathurai S, Blaustein D, Editors. Essentials of inpatient rehabilitation. Blackwell Science;2001. p 42-43.

"39 A 55

old woman comes to your office with a 2month history of aching and stiffness of her neck and shoulders. Recently, she has had similar symptoms in her h ips. She describes difficulty moving in the morning f or at least 1 hou r after arising. Upon further questioning, she also has fatigue, loss of appetite, and jaw pain with eating. On exam, she has limitation in active range of motion of her shoulders and hips due to pain, but does not have joint synovitis. Plain radiographs are unremarkable. You order laboratory tests and expect to find a) normal C-reactive protein (CRP) level. b) elevated erythrocyte sedimentation rate (ESR). c) normal hemoglobin and hematocrit. d) presence of antinuclear antibodies." 39 Reference(s) Salvarani C, Cantini F, Boiardi L, Hunder GG. Medical progress: polymyalgia rheumatica and giant-cell arteritis. N Engl J Med 2002;347:261-71. Weyand CM, Goronzy JJ. Vasculitidies . Gi ant cell arte ri tis, polym yalgi a rheumatica, and Takayasu's arteritis. In: Klippel JH, Stone JH, Crofford LJ, White PH, editors. Primer on rheumatic disease. 13th e Atlanta (GA): Arthritis Foundation; 2008. p 404-6. Option b is correct. This woman has polymyalgia rheumatic Diagnostic criteria include age greater than 50 years, bilateral aching and stiffness for more than 1 month in the shou lder and hip girdle, elevated erythr ocyt e sedimentati on rate, mor ning stiffness lasting more than 1 hour, rapid response to prednisone and absence of other disease that may cause musculoskeletal symptoms. Patients also demonstrate on laboratory findings mild to moderate anemia of chronic disease and elevated CRP. Antinuclear antibodies are usually negative.

"169. An unusually high incidence of pressure ulcers is n oted on y our inpatient rehabilitation unit. As the medical director for the unit, you decide to implement a quality improvement process. The next best step in process would be to (a) understand the cause of the skin breakdown. (b) select a strategy to decrease the incidence of press ure ulcer s. (c) organize a team to investigate the problem. (d)

rep rimand th e nu rse manager for the unit." 169. (c) Various strategies can be used when implementing a quality improvement process. One widely accepted method is FOCUS PDCA. The steps in the process include finding an opportunity, organizing the team, clarifying the current process, understanding the causes of the variation, and selecting a strategy to implement it (FOCUS). Once this has been accomplished, then the strategy involves planning, doing, checking, and acting (PDCA). According to this process, the next best step in the scenario provided would be to organize a team to investigate the problem.

1. A 27-year-old female runner presents to your office with foot pain for the last 3 weeks. She reports a severe pain on the bottom of her foot which is worse with the first few steps in the morning after getting out of bed. She has no history of trauma and typically runs up to 5 miles per day. Her running has been severely limited since this pain began. What is the best initial treatment option for this patient's pain? (a) Walking cast for the affected foot (b) Complete bed rest for several days (c) Corticosteroid injection at the involved site monthly for 3 months (d) Foot orthotic and stretching

1 (d) This patient's symptoms are most consistent with plantar fasciitis. Classically, this syndrome is most painful first thing in the morning upon arising and is aggravated by overuse or change in footwear. Stretching the plantar fascia, often before getting out of bed in the morning, and use of a heel cup or medial longitudinal arch orthotic are the initial treatments for this condition.Corticosteroid injection may be indicated at the insertion of the fascia into the plantar aspect of the calcaneus; however, this is usually not required on a repeated basis. Relative rest from the aggravating activity may be useful, but bed rest is not indicated. A walking cast would not allow stretch of the plantar fascia. (This question has been eliminated from the exam, therefore, it was not scored.)

100. When comparing quadrilateral sockets with ischial containment sockets, a successful fitting ismore likely in a quadrilateral socket when (a) the adductor musculature is intact. (b) the residual limb is fleshy. (c) trunk stability demands are high at mid stance. (d) the residual limb is shorter.

100 (a) Chances of a successful fitting of a quadrilateral socket are best when the residual limb is longer with a firm residuum and intact adductor musculature. Ischial containment sockets are more successful than quadrilateral sockets for persons with shorter, fleshy, unstable residual limbs.

101. A 39-year-old mail sorter complains of severe right elbow pain for 6 months. He has tried antiinflammatory drugs without relief. He denies numbness, but does report weakness in his grip. On examination, the patient has severe pain with palpation just inferior to the lateral epicondyle. The best plan for treating the patient's condition will include (a) medical retirement because this condition is resistant to all forms of therapy. (b) exercise consisting of wrist extensor strengthening and positioning the elbow at 90( flexion and also full extension. (c) corticosteroid injection into the elbow joint. (d) surgical release of the extensor carpi ulnaris tendon to allow the wrist unresisted radial deviation.

101 (b) This condition can be treated with the use of ice after activity and ultrasound to the inflamed area at the lateral epicondyle. Steroid and local anesthetic injection into the tendinous insertion can be useful for temporary relief. A lateral epicondyle counterforce brace ("tennis elbow" band) can provide relative rest for the inflamed extensor tendons. Exercise using light weights to strengthen the wrist extensors is useful. This should be done with the elbow flexed and in full extension. If surgical release is indicated, lateral extensor release is considered to be the procedure of choice.

102. The most common neurologic disturbance associated with cranial irradiation is (a) short-term memory deficit. (b) anomia. (c) bradykinesia. (d) high-frequency hearing loss.

102 (a) Cranial irradiation typically has more side effects in children than in adults. Common side effects include short-term memory loss, fatigue, and occasional gait apraxia.

103. A 7-year-old boy is referred for evaluation and management of gradually progressive pain locatedin the right medial thigh and knee, a limp, and limited painful hip motion. There was a similar episode 3 months before which resolved spontaneously. Of the following conditions, the most likely diagnosis is (a) congenital hip dysplasia. (b) slipped capital femoral epiphysis. (c) Legg-Calvé-Perthes disease. epiphyseal fracture.

103 (c) Legg-Calvé-Perthes disease, avascular necrosis of the femoral head, occurs most commonly in boys between the ages of 4 and 8 years. It is characterized by medial thigh, groin, and knee pain, is associated with a limp, and is often preceded by a transient episode of hip synovitis. Congenital hip displasia is usually apparent much earlier; slipped capital femoral epiphysis occurs during periods of rapid growth, typically in adolescent boys, and may be preceded by trauma.

104. In a patient with bilateral hip flexion contractures, which of the following gait deviations would bemost likely? (a) Bilateral Trendelenburg gait (b) Early heel rise during stance (c) Swing-phase circumduction (d) Increased knee flexion in stance

104 (d) In normal gait, hip extension to neutral occurs during stance phase. When mild hip flexion contractures are present, a compensatory increase in lumbar lordosis occurs to maintain upright trunk posture. As the extent of the hip flexion contractures worsens, there is usually an additional compensatory increase in knee flexion during stance phase.

105. Abrupt withdrawal of which of the following medications is most likely to result in death? (a) tizanidine 8mg qid (b) diazepam 15mg qid (c) dantrolene 50mg qid (d) baclofen 40mg qid

105 (b) Symptoms of withdrawal from high-dose diazepam (>40mg/day) include anxiety and agitation, restlessness, tremor, muscle faspinal cord injuryculation, hypersensitivity to touch, taste, smell, light, and sound, hyperpyrexia, psychosis, and possibly death. The intensity of symptoms, and hence the risk of death, is related to the prewithdrawal dose.

107. You are providing testimony in a worker's compensation case. Of the following tests which, wouldprovide the most objective evidence for a lumbar radiculopathy? (a) Straight leg raise (b) Pin prick sensory examination (c) Electromyogram (d) Magnetic resonance imaging of the lumbar spine

107 (c) Of the choices provided, electromyogram/nerve conduction velocity testing would provide the most objective documentation of radiculopathy. Magnetic resonance imaging is an anatomic test. Straight leg raising and findings of manual strength of 4/5 provide more subjective information.

108. The potentials above are (see pictures table) (a) complex repetitive discharges. (b) myotonic discharges. (c) neuromyotonia. (d) myokymia.

108 (b) The potentials noted are single-fiber discharges waxing and waning in frequency and amplitude. This is characteristic of myotonic discharges.

11. Dysesthesias on the plantar aspect of the foot may be associated with peripheral neuropathy or tarsal tunnel syndrome. Which of the following would be most useful to help in distinguishing between these? (a) Reproduction of symptoms with compression inferior to the medial malleolus (b) Sensory testing using a Semmes-Weinstein monofilament on the plantar aspect of the foot (c) Plain anteroposterior and lateral radiographs of the foot, including the calcaneus (d) Slowing of the medial and lateral plantar nerves on nerve conduction studies

11 (a) Both peripheral neuropathy and tarsal tunnel syndrome can present with painful dysesthesias. Plain radiographs will not be helpful, as they can be normal with either of these conditions. Nerve conduction studies may demonstrate slowing of the medial and lateral nerves with either condition, and sensation on the plantar aspect of the foot may be decreased with either condition. Compression over the tarsal tunnel should cause increased symptoms of numbness, tingling, or burning in the plantar aspect of the foot.

111. A 21-year-old US Army recruit reports to boot camp. After 5 days of marching, he reports to thebase physiatrist with complaints of severe pain in his left shin. He states that the pain began after a 10-mile run in full gear that morning. The pain has gotten significantly worse over the last 2 hours. He is now unable to bear weight on his left leg. On examination, his left shin is shiny and edematous. He has severe pain with palpation, and the muscles seem tight. You are suspicious that this man has (a) a severe muscle strain in the tibialis anterior. (b) a muscle contusion due to a fall while running. (c) an anterior leg compartment syndrome. (d) a stress fracture in the left tibia.

111 (c) This patient has been performing excessive, unaccustomed, intense exercise. He is at risk for a compartment syndrome, which occurs when perfusion of muscle and nerve tissues decreases to a level inadequate to sustain the viability of the tissues. The intracompartmental pressure increases and produces venous obstruction. This in turn increases the intracompartmental pressures even more, and necrosis of muscle and nerve tissue may ensue in as little as 4-8 hours.

112. A 58-year-old man with Parkinson's disease has been on stable levodopa therapy for 2 years. He isbeginning to show signs of the "on-off" phenomenon. Your initial treatment considerations would be (a) referring him to neurosurgery for ablation. (b) reducing levodopa dosing intervals. (c) increasing dietary protein intake. (d) adding an anticholinergic agent.

112 (b) Motor fluctuations that occur in Parkinson's disease can be related to levodopa therapy. A proposed mechanism is degeneration of presynaptic dopaminergic (DA) nerve terminals, altered DA receptor sensitivity, or associated fluctuations in non-DA neurotransmitter systems. Best initial approaches to treatment are to decrease dosing intervals, decrease protein load (give levodopa 1-2 hours before meals) to reduce amino acid binding competition, and/or adding a dopamine agonist. Using an anticholinergic agent will have little effect on completing this "on-off" problem.

114. Recommended therapeutic interventions in orthostatic hypotension due to autonomic neuropathyinclude (a) amitriptyline use at bed time. (b) sleeping in a head-down position. (c) use of oral corticosteroids. (d) avoidance of postprandial exercise.

114 (d) Aerobic exercise programs carry the risk of exercise-induced vasodilation and worsening of the hypotension. This is more likely after meals, when hypotension is often exacerbated because of blood pooling in the gastrointestinal vasculature. Other interventions include constrictive garments, dietary modifications (high-salt diet or supplemental salt tablets), and sleeping with the head of the bed elevated to reduce early morning symptoms. Fludrocortisone (Florinef) is the most commonly used drug treatment; it works by sensitizing blood vessels to endogenous catecholamines and expanding the blood volume. Prostaglandin inhibitors such as indomethacin and ibuprofen are reported to be useful, especially for postprandial exacerbations of hypotension.

116. Elevated plasma levels of this substance have been determined to be a risk factor foratherosclerosis. (a) High-density lipoprotein cholesterol (b) Vitamin E (c) Homocysteine (d) Vitamin C

116 (c) Elevated plasma level of homocysteine is a risk factor for atherosclerosis. Antioxidants such as vitamin E may potentially have a protective effect. Likewise, the benefits of high-densitylipoprotein cholesterol are well defined.

117. A 62-year-old stevedore describes experiencing abrupt-onset shoulder pain after lifting a bale. Hehas a positive drop arm test and difficulty with overhead reaching activities. He requests magnetic resonance imaging (MRI) of his shoulder. You tell him that (a) a rotator cuff tear is unlikely and MRI is not indicated. (b) a rotator cuff tear is unlikely and MRI would be a very sensitive and specific test. (c) a rotator cuff tear is likely and MRI will probably be abnormal. (d) a rotator cuff tear is likely and MRI will probably be normal.

117 (c) Magnetic resonance imaging can identify a large percentage of tears on the rotator cuff of asymptomatic persons. These tears may be partial or complete. The overall prevalence of asymptomatic tears of the rotator cuff in all groups was 34%. Fifty-four percent of persons over age 60 had a tear of the rotator cuff (28% full thickness, 26% partial thickness).

118. Comparing the results of electrodiagnostic studies on patients with clinical evidence of postpoliosyndrome and the results of those obtained in persons with a history of polio of similar severity and duration since polio onset but no clinical evidence of postpolio syndrome, you find (a) smaller motor units in the symptomatic group. (b) more polyphasic motor units in the symptomatic group. (c) more fasciculations in the symptomatic group. (d) no significant differences between the groups.

118 (d) There are no significant differences between the groups. Electrodiagnostic studies are not performed to confirm the diagnosis of postpolio syndrome; this is a clinical diagnosis. They are performed to rule out other disorders in the differential diagnosis.

12. A primary care physician started a relative of yours on donepezil (Aricept) for the treatment of Alzheimer's disease. This medication is used to modify (a) expressive language skills. (b) behavioral and cognitive symptoms. (c) disease progression. (d) restoring normalized sleep-wake cycles.

12 (b) Donepezil is a cholinesterase inhibitor (C1) with properties shown to address cognitive behaviors, specifically behavioral disturbances. C1 has not been shown to slow or stop disease progression.

120. An amputee presents for evaluation of distal blistering and evidence of vascular congestion. Youdiagnose choke syndrome. Which of the following would NOT be an acceptable treatment for choke syndrome? (a) Expanding the proximal socket (b) Increasing the auxiliary suspension to decrease vertical pull (c) Relieving the distal socket where it interfaces with the choked surface (d) Padding the distal socket where it corresponds to the choked surface

120 (c) The choke syndrome (proximal soft tissue constriction leading to vascular congestion) may occur with suction sockets or self-suspending systems. Relieving the proximal socket to allow vascular return, providing auxiliary suspension to decrease the vertical pull on the residual limb, and improving the intimacy of the socket-limb interface correct this problem. Relieving the distal socket where it interfaces with the choked surface would increase the vacuum effect in this area and thereby increase the choke phenomenon.

121. Which of the following statements about rupture of the bicipital tendon is true? (a) Significant deficit in supination strength is usually associated with this condition. (b) Rupture of this tendon requires surgical repair to maintain functional strength in the biceps. (c) The tear occurs most commonly in the short head of the biceps and results in only a cosmetic deformity. (d) Tears most often occur in persons with long-standing shoulder impingement pain.

121 (d) Rupture of the long head of the biceps most often occurs in persons over 40 years of age with longstanding shoulder pain due to rotator cuff pathology. It may result in approximately 10% loss of supination in turning the forearm. It does result in a cosmetic deformity with a bulge in the lower arm, but most patients regain full range of motion and normal elbow flexion strength with a conservative therapy program. Usually only heavy laborers under age 40 need surgical intervention to provide the extra strength for lifting.

122. A 22-year-old man with a severe brain injury has repeated episodes of inappropriate sexualtouching of female staff. An initial treatment option for decreasing this behavior would include (a) assigning only male staff. (b) precribing clomipramine (Anafranil) 25 mg BID. (c) using of soft hand restraints for 3 minutes. (d) offering firm verbal discouragement.

122 (d) Inappropriate sexual touching occurs more commonly in patients with a history of frontal lobe damage. Treatment is best directed at staff education to make certain all team members are consistent in responses. Behavior modification is directed at telling the patient his behavior is wrong and firmly taking his hand and putting it closer to his person. Restraints are only indicated as temporary solutions to prevent acute bodily injury.

127. A wrist-hand orthosis used as symptomatic management for carpal tunnel syndrome should be (a) limited to patients with short-duration symptoms. (b) worn only during the day. (c) worn with the wrist in neutral position. (d) worn with the wrist in 30' of extension.

127 (c) A neutral position of a wrist-hand orthosis unloads the median nerve; wrist extension will aggravate nerve compression.

130. Which of the following shoe components distinguishes a blucher-style shoe from a bal-style shoe? (a) Throat (b) Toe box (c) Heel (d) Vamp

130 (a) There are two shoe types, the blucher and the bal. The components of a blucher shoe include the tongue, lace stay, open throat, toe box, toe spring, vamp, ball, shank, breast, quarter, and heel. The major difference with a bal style is that the throat is closed, limiting the ability of the shoe to open and accommodate an orthosis. For this reason, a blucher is the style of dress shoe most often recommended to patients who require an orthosis.

134. Foot ulcers in patients with diabetes (a) are frequently associated with plantar foot callus. (b) commonly occur even with clinically asymptomatic neuropathy. (c) are most frequently located over the malleolus. (d) are more common when subtalar hypermobility exists.

134 (a) Neuropathic ulcers typically occur in patients with impaired or absent sensation due to peripheral neuropathy. Plantar callus formation is indicative of excessive pressure and is a common site of ulcer formation. Ulcers most commonly occur under the metatarsal heads and toes and along the lateral borders of the forefoot. Rigidity of the subtalar and ankle joints adversely alters plantar pressures and increases the risk of ulceration. Changes in joint motion are the result of increased glycosylation of collagen and associated thickening and cross linking of collagen bundles seen in diabetes.

138. In testing a patient with suspected myasthenia gravis, needle electromyography (EMG) of the rightupper extremity and orbicularis oculi is normal. Repetitive stimulation of the right ulnar nerve at a rate of 2/second shows no decrement before or immediately after 1 minute of exercise. A 4% decrement is noted 2 minutes after exercise. Your next electrodiagnostic step should be (a) EMG of the lower extremities. (b) EMG of the frontalis before and after edrophonium (Tensilon). (c) repetitive stimulation of recording from a proximal muscle. (d) ulnar somatosensory evoked potentials.

138 (c) In patients with myasthenia gravis, repetitive nerve studies recorded from proximal muscles are more sensitive, though technically more difficult.

139. Following severe traumatic brain injury, neuroendocrine dysfunction can result in diabetesinsipidus, SIADH (syndrome of inappropriate antidiuretic hormone), and cerebral salt wasting. Cerebral salt wasting is manifested by (a) hypernatremia. (b) low serum osmolality. (c) high urine output. (d) dehydration.

139 (d) Diabetes insipidus is characterized by excessive water loss, and therefore affected patients experience hypernatremia, dehydration, polyuria, and polydipsia. SIADH is associated with decreased urine output, hyponatremia, and a decreased serum osmolarity. Cerebral salt wasting is a result of a neural effect on the renal tubules, causing loss of sodium and water and resulting in hyponatremia and dehydration.

14. A 26-year-old woman presents with the new onset of lower extremity weakness and bladder incontinence. Past medical history is remarkable for an episode of diplopia and blurred vision which resolved spontaneously. The most likely diagnosis is (a) multiple sclerosis. (b) myasthenia gravis. (c) myasthenic syndrome. (d) amyotrophic lateral sclerosis.

14 (a) Clinically, multiple sclerosis is characterized by multiple lesions separated in time and location within the central nervous system. Common presenting symptoms include diplopia, optic neuritis, weakness, sensory loss, and ataxia. Women between the ages of 20 and 40 are the most commonly affected population. Diplopia, though commonly associated with myasthenia gravis, is not seen in myasthenia syndrome or amyotrophic lateral sclerosis.

140. A 35-year-old man experiences a severe S1 radiculopathy. As a result, he loses essentially allstrength within the posterior compartment of the leg. It is 6 months later, and you are consulted to provide an orthosis to aid his ambulation. The best recommendation would be (a) a posterior leaf spring ankle-foot orthosis (AFO). (b) an articulating AFO with plantar flexion stop. (c) an articulating AFO with dorsiflexion stop. (d) an articulating AFO with dorsiflexion assist.

140 (c) In this condition, lack of active plantar flexion occurs, heel-rise is delayed, the mid-stance phase is prolonged, and push-off force is reduced. A dorsiflexion stop can accommodate for this weakness. Setting the stop at 5° of dorsiflexion substitutes best for gastrocnemius function.

142. Risk factors for the development of Alzheimer's disease include advancing age, family history,presence of the gene APO E-4, and which of the following? (a) low intelligence. (b) large head circumference. (c) male gender. (d) co-morbidity with diabetes mellitus.

142 (a) It is important to recognize these historical and clinical characteristics acting as risk factors in the development of Alzheimer's disease. It has been shown that lower intelligence, female gender, and smaller head size are all associated with increased risk. Diabetes mellitus affects cerebral vasculature and increases the risk of vascular dementia, not Alzheimer's disease.

143. The American College of Rheumatology (ACR) criteria for fibromyalgia include all of thefollowing EXCEPT (a) widespread pain. (b) nonrestorative sleep. (c) at least 11 of 18 tender points. (d) symptom duration of at least 3 months.

143 (b) The ACR criteria for fibromyalgia include the presence of widespread pain (both sides of the body, above and below the waist) for at least 3 months and tenderness at 11 of 18 points. Disturbed sleep, fatigue, various neurologic symptoms, and gastrointestinal complaints are common but not part of the criteria. A complete history and physical is usually adequate to make the diagnosis; screening laboratory testing should exclude hypothyroidism, connective tissue diseases (such as polymyalgia rheumatica or rheumatoid arthritis), and metabolic myopathies. Treatment involves restoration of sleep, reduction of psychologic variables (such as stress and depression), physical conditioning, stretching, and postural training. Nonsteroidal anti-inflammatory drugs, muscle relaxants, narcotics, and passive modalities are not useful and their use should be minimized.

144. Following a head injury, a 35-year-old woman presents with vertigo. She reports a sensation ofspinning beginning several seconds after standing up rapidly, bending over, or rolling in bed. Symptoms lasts for approximately 30 seconds. Exam is notable for nystagmus during episodes of vertigo, normal extremity coordination, and minimal increase in sway during Romberg testing. The most likely diagnosis is (a) benign positional vertigo. (b) cerebellar contusion. (c) unilateral vestibular paresis. (d) bilateral vestibular paresis.

144 (a) The symptoms are most consistent with benign positional vertigo, a disorder characterized by transient episodes of vertigo precipitated by changes in the position of the head.The underlying etiology is thought to be related to movement of otoliths or otolithic debris within the semicircular canals. It commonly occurs following head injury or viral labyrinthitis. Treatment involves a specific otolith repositioning maneuver or a series of habituation exercises.

145. Women who have sustained a spinal cord injury and become pregnant after injury are noted to have pregnancy-related complications including (a) high-birth-weight babies. (b) late-onset labor. (c) pressure sores. (d) higher than average spontaneous abortions.

145 (c) Women who become pregnant after sustaining a spinal cord injury undergo spontaneous abortions in the first trimester at the same rate as uninjured women; however, the incidence of premature and small-for-date babies is higher than normal. In addition, the spinal-cord-injured woman is known to have pregnancy-related complications such as urinary tract infections and pressure sores.

15. The best possible expected functional outcome for a person with C7 ASIA A spinal cord injury is (a) dependent with bladder, independent with bed mobility, and some assist with all transfers. (b) dependent with bladder, independent with bed mobility, and independent with level transfers. (c) independent with bladder, some assist with bed mobility, and independent with some transfers (d) independent with bladder, independent with bed mobility, and independent with level

15 (d) Expected functional outcomes after traumatic spinal cord injury have been delineated in the clinical practice guidelines for health care professionals. A person who has sustained a C7-8-level spinal cord injury can best be expected to need assistance in clearing secretions, may need partial to total assistance with a bowel program, and may be independent with respect to bladder management, bed mobility, and transfers to level surfaces. Adaptive equipment is listed in these tables (FIM (functional independent measures) purists can argue that these persons really are only modified independent).

16. Which of the following conditions is least likely to exacerbate preexistent lymphedema? (a) Scuba diving in cold water (b) Airplane travel (c) Phlebotomy (d) Sun exposure

16 (a) Conditions associated with decreased atmospheric pressure will cause lymphedema to progress. Activities or situations that lead to increased blood flow in the affected extremity (eg, burns, heat, physical exertion, trauma) exacerbate lymphedema. Atmospheric pressure increases during scuba diving, therefore, it has the capacity to ameliorate lymphedema.

17. Functional recovery programs for the injured worker with chronic pain should (a) be done on an inpatient basis to increase the likelihood of success. (b) not be started until pain medications have been discontinued. (c) focus on restoration of function. (d) focus on reducing the patient's pain symptoms.

17 (c) Functional recovery programs should focus on restoring functional ability, including return to work.

18. Of the following, somatosensory studies would be the most useful in the diagnosis of (a) tarsal tunnel syndrome. (b) motor neuron disease. (c) myasthenia gravis. multiple sclerosis.

18 (d) Somatosensory studies can be helpful in the diagnosis of multiple sclerosis. Standard nerve conduction studies and electromyography are far more useful in the diagnosis of the other disorders.

19. The leading cause of childhood disability is (a) traumatic brain injury. (b) spinal muscular atrophy. (c) spina bifida. (d) cerebral palsy.

19 (d) Cerebral palsy is the leading cause of childhood disability. The reported incidence is approximately 2-3 per 1,000 live births. The incidence of spina bifida is .5 per 1,000, of spinal muscular atrophy 1 in 25,000. The annual incidence for traumatic brain injury in children is 1-2 per 1,000. However, the great majority of cases are minor and result in no long-term disability. Approximately 15% of brain-injured children have moderate and severe injuries resulting in permanent impairment.

22. A 32-year-old woman with known multiple sclerosis has frequent bladder sensation and urgency but very little urination amounts. Nursing informs you that her void amounts are low, her post-void residual is high, and the combined voided plus catheter specimen amounts are on the low-normal side. You order urodynamic testing to verify (a) an areflexic bladder with sphincter dyssynergia (increased tone). (b) an areflexic bladder with normal sphincter tone. (c) a hyperreflexic bladder with sphincter dyssynergia. (d) a hyperreflexic bladder with normal sphincter tone.

22 (c) Bladder dysfunction in patients with multiple sclerosis can be challenging from a diagnostic perspective. The use of urodynamic testing provides useful information. Bladder function in this scenario would be hyperreflexic, and with high residual volumes it would indicate that bladder outlet pressure is high so as to block urine flow, as if squeezing on a water balloon and holding the spout. This outlet pressure is called dyssynergia, and treatment is directed at bladder neck relaxation.

23. Which of the following glucocorticoid preparations has the longest therapeutic action? (a) Triamcinolone diacetate (Aristocort) (b) Methylprednisolone acetate (Depomedrol) (c) Hydrocortisone phosphate (Hydrocortone) (d) Dexamethasone sodium phosphate (Decadron)

23 (d) The onset and length of symptomatic relief after steroid injection are related to the preparation used. Dexamethasone sodium phosphate has an intermediate solubility and a fairly long biologic half-life (36-72 hours). Methylprednisolone and triamcinolone have biologic half-lives of 12-26 hours. Hydrocortisone phosphate is not recommended for intraarticular use.

25. Outcomes of inpatient rehabilitation for neoplastic versus traumatic spinal cord injury reveal that (a) indices for depression were significantly higher in patients with neoplastic injury. (b) patients with neoplastic spinal cord injury had significantly shorter lengths of stay. (c) rate of functional change was significantly better in the traumatic population. (d) neoplastic spinal cord injury was associated with a significantly higher rate of discharge to the community.

25 (b) Patients with neoplastic spinal cord compression tend to be older than their traumatic counterparts, with a peak incidence between 50 and 70 years. Significant differences exist with regard to the level of injury; tumors involving the spinal cord tend to involve the thoracic and lumbar regions more than the cervical region. There was a shorter rehabilitation length of stay in patients with neoplasms. (This may allow patients to have more time at home with their families. These patients had an increased percentage of paraplegia and incomplete injury.) Patients with tumors did demonstrate a trend toward lower rate of discharge to the community, but this was not significant.

26. During the initial reductive phase of decongestive physiotherapy for advanced lymphedema, compressive bandaging should remain in place (a) 1-2 hours per day. (b) overnight. (c) whenever patients are not bathing or being manually drained. (d) only during exercise periods.

26 (c) During the initial phase of complete decongestive physiotherapy (CDP), compressive bandages traditionally remain in place 20 - 21 hours per day. They are removed only to permit bathing and manual lymphatic drainage. Once maximal volume reduction has been achieved, patients transfer to phase II, or maintenance, CDP indefinitely. The purpose of phase II CDP is to prevent reaccumulation of lymphedema. During phase II, patients apply compressive wrapping only at night.

27. A 35-year-old fireman with a 3-month history of shoulder bursitis is referred to you for treatment. Among the following, your initial management would include (a) ice massage to the involved area. (b) ultrasound to the involved area. (c) transcutaneous electrical nerve stimulation. (d) placing the arm in a shoulder sling.

27 (b) Ice massage is more effective for acute bursitis. Subacute or chronic bursitis may respond to deep heating modalities such as ultrasound.

29. The earliest weakness seen in skeletal muscle in Duchenne muscular dystrophy is located in (a) knee extensors. (b) hip flexors. (c) neck flexors. (d) ankle plantar flexors.

29 (c) Neck flexor weakness occurs during preschool years. Weakness is generalized but is predominantly proximal early in the disease course. Pelvic girdle weakness precedes shoulder girdle weakness by several years. Weakness progresses steadily. Quantitative strength testing is more sensitive than manual muscle testing.

30. During normal human locomotion, the center of gravity travels through a sinusoidal pathway that is modified by 6 determinants of gait. Which of the following is not considered 1 of the 6 determinants? (a) Pelvic extension (b) Foot and ankle synchronization (c) Knee flexion (d) Lateral pelvic displacement

30 (a) The 6 determinants are as follows: lateral displacement that reduces horizontal excursion from 6" down to 1.7"; knee flexion that reduces vertical excursion 7/16"; pelvic rotation that reduces vertical excursion 3/8"; pelvic tilt that reduces vertical excursion 3/16"; and foot and ankle synchronization as well as ankle and knee synchronization that both serve to smooth out the sinusoidal curve but do not decrease excursion.

35. Five weeks after sustaining a T6 spinal cord injury, your patient is noted to have urinary incontinence with intermittent catheterization volumes of less than 200mL. Urinalysis is unremarkable. You consider starting (a) sodium etidronate (Didronel). (b) oxybutynin (Ditropan). (c) urecholine (Bethanechol). (d) terazosin (Hytrin).

35 (b) This patient is probably developing spontaneous detrusor contractions but is emptying incompletely. You would consider using an anticholinergic agent to decrease detrusor (and hence intravesical) pressures. Ideally, you would obtain urodynamic studies to delineate detrusorsphincter coordination. One should not initiate a cholinergic agonist without knowing of possible detrusor -sphincter dyssynergy.

38. You are treating a 48-year-old man who has had two lumbar laminectomies for what you suspect is a recurrent right L5 radiculopathy. You perform an electromyogram to confirm the diagnosis, and it reveals 2+ positive waves and fibrillations with decreased recruitment in the right anterior tibialis. The patient informs you that he can only tolerate the examination of one more muscle. Of the following you would choose (a) extensor hallucis longus. (b) L5 paraspinals. (c) vastus medialis. (d) flexor digitorum longus.

38 (d) The history is suggestive of an L5 radiculopathy. Given the previous laminectomies, examining a single level of paraspinals would provide limited information. Although you cannot form any firm conclusions based on such a limited examination, study of the flexor digitorum longus will provide findings outside the peroneal distribution and could lend support to the clinical diagnosis.

40. A 79-year-old cachetic woman with coronary artery disease and unstable angina sustains a right hip fracture after a fall. After an open-reduction internal fixation of the hip joint with the use of a dynamic hip screw, the orthopedic surgeon determines that the patient is 25% partial weight bearing to the right side. She has weak upper body strength and good balance. Which of the following assistive devices is most appropriate? (a) Standard walker (b) Rolling walker (c) Axillary crutches (d) Quad cane

40 (b) Standard walkers require good standing balance and good upper body strength. Crutches require good upper body strength and have an increased energy expenditure of 40%-60%, which would be contraindicated in unstable angina. Quad canes are not appropriate when significant weight-bearing relief is required. Rolling walkers are most appropriate for patients who lack upper body strength and provide safer gait than crutches or canes.

41. A 16-year-old boy presents 2 days after being injured during a weekend football game. He reports receiving a direct blow on the right side from the helmet of a player. He had immediate pain in the right lower part of the posterolateral trunk. On exam, you note ecchymosis and tenderness to palpation just superior to the right iliac crest. Which of the following signs would be expected on subsequent examination? (a) Severe pain with internal rotation of the right hip (b) Pain on the right side with left lateral bending of the trunk (c) Numbness in the right femoral nerve distribution (d) A positive Gillet test

41 (b) This scenario is classic for a contusion of the iliac crest or "hip pointer." It occurs as a result of a direct blow to an unprotected iliac crest. Tenderness with swelling and ecchymosis is common, as is pain on the affected side with lateral bending away from the side of contact. This is not a hip injury, and internal rotation of the hip should be normal. The Gillet test is used to evaluate sacroiliac mobility. Numbness can be seen on the ipsilateral side because the T12-L3 lateral cutaneous nerve branches are often injured.

42. A 75-year-old man with a recent anterior communicating artery aneurysm, treated by neurosurgical clipping is admitted to the inpatient rehabilitation unit for poststroke care. Deep vein thrombosis prophylaxis should include (a) heparin 5,000 units BID. (b) warfarin doses based on INR values. (c) pneumatic compression stocking (ankle or calf). (d) continuous passive motion devices.

42 (c) Venous thromboembolic events can occur in as many as 75% of untreated patients with after stroke. Prophylaxis is typically pharmacologic or manual venous compression. In patients with documented intracerebral bleeding, anticoagulation is not recommended, and alternating pneumatic compression derives are best used.

45. The most common cause of upper limb pain in long-standing tetraplegia is (a) shoulder pain of radicular origin. (b) shoulder pain of musculoskeletal origin. (c) elbow pain of radicular origin. (d) elbow pain of musculoskeletal origin.

45 (b) In patients with quadriplegia, 55% reported pain in at least one region of the upper extremity. The shoulder was reported as painful in 46% of subjects; the most frequent diagnoses for shoulder pain were orthopedically related—tendinitis, bursitis, and osteoarthritis. Referred pain of cervical origin accounted for 33% of shoulder pain. In patients with paraplegia, symptoms of carpal tunnel syndrome were the most common complaint (66%).

46. A patient with metastatic lung cancer presents to the emergency department with new-onset back pain and lower extremity weakness suspected to be due to spinal metastases. Initial management should include (a) observation for 24 hours and careful reexamination for progressive neurologic deficits. (b) emergent irradiation prior to imaging. (c) empiric administration of high-dose dexamethasone in the absence of contraindications. (d) alteration of the patient's chemotherapy regimen.

46 (c) A randomized controlled trial of high-dose steroids (96mg dexamethasone) versus placebo concluded that steroid-treated patients with spinal cord compression from malignant epidural disease were more likely to retain or regain ambulation. Surgery and radiation may be indicated, contingent on tumor location and type and on prior radiation history. Dexamethasone should be administered to patients before imaging in order to alleviate pain and to optimize neurologic recovery.

47. A 63-year-old man was in a car accident 3 days ago. Cervical radiographs performed in the emergency department demonstrate multilevel degenerative disc disease. He is currently asymptomatic but is concerned about the radiographic findings. You inform him that (a) this is diagnostic of central spinal stenosis. (b) further neuroimaging is required. (c) cervical spine immobilization with a soft collar is recommended. (d) no evaluation or treatment is necessary until symptoms occur.

47 (d) Cervical spine degenerative changes in males over 60 are commonly seen in asymptomatic patients.

49. In children hospitalized with acute burns, early management should include (a) avoidance of sedation. (b) positioning for comfort to reduce severe pain. (c) use of a pressure garment over areas of full-thickness burns. (d) administration of narcotics and anesthetic agents.

49 (d) Although opiates should be considered the most important part of acute pain management nonopiates should be used when possible. As the needs become more chronic, other agents should be instituted to minimize the problems seen with opiates. Behavioral management and relaxation therapy should also be used when possible. Typically, the position of comfort for a burned child is the position that promotes deformity and, therefore, should be avoided. Garments are fitted later in the course of treatment.

52. A 77-year-old woman is admitted to the rehab inpatient unit with a diagnosis of general debility after urosepsis and dehydration. Clinical exam reveals impaired memory, diminished abstract reasoning, gait ataxia, and peripheral distal extremity numbness. Past medical/surgical history is positive for coronary artery disease and poor nutrition. Lab data include Na 135, K 3.5, Cl 95, C02 24, WBC 7.4, Hgb 9.5, MCV 101, urinalysis negative, RPR negative, Cobalamin assay 200 pg/mL (normal 170-900). You believe her neurologic abnormalities are attributable to (a) normal-pressure hydrocephalus. (b) latent neurosyphyilis. (c) postdehydration cerebral infarction. (d) pernicious anemia.

52 (d) Pernicious anemia can account for significant cognitive and motor disturbances in patients typically more than 60 years old. The most frequent clinical findings are paresthesias, numbness, gait ataxia, focal incontinence, leg weakness, memory disturbance, and acute dementia. Workup for cobalamin levels, intrinsic factor antibodies, and Shilling's test are useful in making a correct diagnosis. Treatment consists of cobalamin replacement.

54. A typical clinical finding in patients with alcoholic cerebellar disease is (a) gait ataxia out of proportion to extremity ataxia. (b) benign positional vertigo. (c) severe upper extremity dysdiadochokinesia. (d) vertical nystagmus.

54 (a) Alcoholic cerebellar disease preferentially affects the superior vermis of the cerebellum, resulting in gait instability, poor trunk control, and lower greater than upper extremity ataxia.

55. Functional outcomes after the use of methylprednisolone in persons with penetrating spinal cord injury as compared with blunt injury are (a) markedly improved. (b) better. (c) unchanged. (d) worse.

55 (c) The administration of methylprednisolone did not significantly improve functional outcomes in patients with gunshot wounds to the spine or increase the number of complications experienced by patients during their hospitalization.

56. A 45-year-old colon cancer patient presents with new urinary incontinence and dull pain radiating into the right buttock. Physical examination fails to reveal evidence of lower extremity motor or sensory deficits. Of the following, the MOST appropriate next step in the patient's evaluation would include (a) pelvic computed tomography with contrast. (b) lower extremity electromyogram. (c) positron-emission tomography scan. (d) urodynamics studies.

56 (a) The sacral plexus is usually involved by tumor from the colon, prostate, bladder, or uterus. Presenting symptoms usually begin as a dull, aching, midline pain, which may radiate into the buttocks. The pain may be associated with numbness in the perianal region. Numbness and aresthesias may extend to involve the buttock and posterior aspect of the thigh. Bowel and bladder

59. In considering selection of a lower limb prosthesis for a child with a congenital transfemoral amputation, a knee joint should be included (a) at initial fitting. (b) between 3 to 5 years of age. (c) when sports activities are anticipated. (d) when the child pulls to stand.

59 (b) The lower limb deficient child should be fitted with a prosthesis when he or she is ready to pull up to a standing position, usually between 9 and 12 months. A knee joint is added between 3 and 5 years.

60. Which one of the following cervical orthoses is the most restrictive to range of motion in flexion, extension, axial rotation, and lateral bending, both actively and passively? (a) Soft collar (b) Philadelphia collar (c) Philadelphia collar with thoracic extension (d) Sternal-occipital-mandibular immobilizer collar

60 (d) Measurements of the range of motion in flexion, extension, axial rotation, and lateral bending (both actively and passively) using a computerized motion analyzer for four orthoses—soft collar, Philadelphia collar, Philadelphia collar with thoracic extension, and a Sternal-occipital-mandibular immobilizer (SOMI)—found that the SOMI was most restrictive.

63. You are asked to see a 35-year-old woman with systemic lupus erythematosus who has severe left groin pain. She underwent a cadaveric renal transplant 4 years ago. She has a Trendelenburg gait and pain upon internal rotation of the hip. There is reproduction of the groin pain with hip flexion. Your diagnosis is (a) L5 radiculopathy. (b) trochanteric bursitis. (c) avascular necrosis. (d) femoral neuropathy.

63 (c) The patient has aseptic necrosis of the femoral head. Her symptoms will be resolved with a hip replacement. Before surgery, a program of isometric strengthening and endurance exercise is appropriate. Trochanteric bursitis presents as lateral hip pain extending down the leg, worse with walking or lying on that side. There is tenderness over the greater trochanter and pain with endrange adduction or resisted abduction. Femoral neuropathy presents with weakness of the knee extensors. An L5 radiculopathy typically presents with leg pain that extends to the dorsum of the foot and is not worsened with hip rotation.

64. A middle-aged man is evaluated 1 week after the onset of painless foot drop. On exam, 1/5 strength of ankle dorsiflexion and eversion is present. Electrodiagnostic testing demonstrates a 90% loss in the compound muscle action potential amplitude of the anterior tibialis without conduction block across the fibular head. Markedly reduced motor unit recruitment is present in the anterior tibialis. Of the following, the most appropriate recommendation is (a) observation and a temporary ankle-foot orthosis, as the prognosis is excellent. (b) a custom-molded ankle-foot orthosis with dorsiflexion assist. (c) functional electrical simulation of the anterior tibialis to facilitate recovery. (d) magnetic resonance imaging of the brain to rule out a lacunar stroke.

64 (b) The clinical features and electrodiagnostic studies are consistent with a peroneal neuropathy. The reduction in compound muscle action potential amplitude is indicative of significant axonal loss. In this situation, recovery is likely to be delayed and incomplete. The use of an ankle-foot orthosis to supply medial lateral stability and ankle dorsiflexion is appropriate. Functional electrical stimulation has not been shown to alter outcome after nerve injury.

65. Persons with neurogenic bowel often use laxatives such as senna (Senokot), which acts by (a) decreasing intraluminal fluid. (b) lubricating the intestinal mucosa. (c) (c) stimulating the myenteric plexus. (d) increasing the time for electrolyte resorption.

65 (c) Stimulant laxatives act by enhancing intestinal motility and thereby decreasing time available for water and electrolyte resorption. Senna is a glycoside that is split by colonic bacteria into absorbable anthraquinones. It generates increased propulsive activity by altering electrolyte transport and increasing intraluminal fluid. It exerts a direct stimulant effect on the myenteric plexus which increases intestinal motility. Senna works best in persons with upper motor neuron level injuries, and it facilitates bowel movements in 6 to 12 hours.

66. Electromyographic findings consistent with Lambert-Eaton myasthenic syndrome suggest the presence of (a) small cell lung cancer. (b) malignant melanoma. (c) inflammatory breast cancer. (d) renal cell carcinoma.

66 (a) About 60% of patients with Lambert-Eaton myasthenic syndrome have small cell lung cancer. A few others have small cell cancer elsewhere in the body, such as in the prostate or cervix. About 40%, usually women with other evidence of autoimmune dysfunction, do not have cancer.

68. The number of phases of a motor unit is related to the (a) conduction time through collateral nerve sprouts. (b) sweep speed setting on the oscilloscope screen. (c) sensitivity (gain) setting on the oscilloscope screen. (d) central conduction time and the state of the upper motor neuron.

68 (a) The number of phases of the motor unit potential represents the synchronization of firing of the individual muscle fibers in a motor unit and is related to conduction time through collateral sprouts of the nerve. The number of phases is increased under conditions in which some sprouts are poorly myelinated and conduction is slow and less synchronous. The other factors do not affect the number of phases of a motor unit potential.

69. The earliest marker of abnormal central nervous system maturation is (a) diffuse fasciculations. (b) gross motor delay. (c) delay of postural responses. (d) persistence of primitive reflexes.

69 (d) In neonates and young infants, motor behavior is influenced by primitive reflexes because of the immature central nervous system. These reflexes gradually become suppressed. Concurrently, more sophisticated postural responses emerge. Obligatory persistent primitive reflexes are the earliest markers of abnormal neurologic maturation.

7. A nurse working in a rehabilitation hospital is most likely to injure her back while (a) assisting a patient to perform a sliding-board transfer. (b) catheterizing a patient. (c) performing a wheelchair-to-toilet patient transfer. (d) helping a patient move up in bed.

7 (d) The difficulty in using proper posture and body mechanics and the forces required for pulling patients up in bed are believed to be the reasons that this task most often causes back pain among nurses.

71. Fibromyalgia is a systemic disorder that affects primarily the musculoskeletal system and results in pain and stiffness. Which of the following statements is true about the diagnosis and treatment of fibromyalgia? (a) It has an equal distribution in both men and women. (b) The "tender points" associated with this condition cause radicular pain. (c) Severe sleep disturbances and fatigue are common complaints. (d) Palpable taut bands, which exhibit local twitch responses, are characteristic.

71 (c) Fibromyalgia is found predominantly in women, and 16 paired tender points are identified by theAmerican Rheumatism Association. They do not show referred pain patterns. Palpable taut bands are seen with myofascial pain but are not specific for fibromyalgia. Sleep disturbances and depression are very common in patients with this diagnosis.

72. A 29-year-old woman with a traumatic brain injury is seen by you for consultation. On chart review, it is noted that she has frequent episodes of emesis with percutaneous endoscopic gastrostomy tube bolus feeding. Your initial recommendation would be (a) placement of a jejunostomy tube. (b) surgical consult for pyloroplasty. (c) continuous tube feeding. (d) switch to elemental formula tube feeding.

72 (c) Feeding issues for patients with traumatic brain injury can become quite complex. It is important to use stepwise approaches when investigating emesis in this population. Quite frequently a percutaneous gastrostomy tube is placed in the neurointensive care unit. Intolerance to feeding can be related to increased gastric distention, and adjusting from bolus to continuous feeding may provide relief. Other steps might then include converting to a jejunostomy or using agents to facilitate gastric emptying.

74 Which of the following symptoms is typically present in vascular claudication but not in neurogenic claudication? (a) Earlier onset of pain with uphill walking (b) Pain in the posterior calf (c) Leg paresthesias and numbness (d) Pain relief with sitting

74 (a) Calf pain that worsens with walking and is relieved with sitting occurs with both neurogenic and vascular claudication. The onset of vascular claudication occurs earlier with the increased muscular demands of uphill walking, whereas the symptoms of neurogenic claudication tend to be less severe in positions of spine flexion such as occurs during uphill walking or with the use of a shopping cart. Paresthesias and numbness are more typical of neurogenic claudication than vascular claudication.

79. A parent of an 18-month-old child reports that the child babbled as an infant but became much quieter after about 8 months of age. She has no true words, though she will wave bye-bye. She follows no verbal commands but will follow occasional pantomime commands. Her gross and fine motor skills have been normal. The most likely diagnosis is (a) autism. (b) mental retardation. (c) hearing impairment.

79 (c) A history of delay in communication development raises several diagnostic possibilities, including true language dysfunction or a motor dysfunction or significant hearing loss. Infants with hearing loss start to fall behind after 6-8 months of age, when learning of auditory-dependent vocalization begins. Oral motor dysfunction is often associated with cerebral palsy, most often spastic quadriparesis. Difficulty with drinking from a cup and difficulty with the introduction of solid food are early symptoms of oral motor dysfunction. Autism is a spectrum disorder with qualitative abnormalities in communication and in social and behavioral realms.

80. When considering realistic functional goals for the majority of transhumeral amputees, the maximal weight (in pounds) that can be carried with the body-powered prosthesis is (a) 7. (b) 15. (c) 30. (D) 50.

80 (b) Handling of heavy objects is limited in upper extremity amputees. A transhumeral amputee is expected to lift 10lb to 15lb, unless the residual limb is very short or sensitive. A transradial amputee is expected to lift 20lb to 30lb unless the residual limb is very short or sensitive.

81. What is your recommended treatment of a T11 compression fracture in a 72-year-old woman? (a) She should be referred to an orthopedic surgeon for immediate fusion because of the risk of neurologic compromise. (b) Neurologic compromise should be ruled out by physical examination, and the patient should be sent home with reassurance that "time will heal this." (c) Neurologic compromise should be ruled out, and the patient should be given an appropriate pain medication to provide adequate relief. (d) To prevent neurologic compromise, the patient should be placed in a spinal orthosis.

81 (c) Fusion of the compression fracture is not indicated unless there is evidence of instability and neurologic compromise. A thorough neurologic examination should be performed, and the patient should be given appropriate pain medication, including opioids as indicated, for adequate relief. This compression fracture pain will resolve in 6-8 weeks as the fracture heals. If physical therapy is ordered, flexion exercises should be avoided because they can increase wedging of the fractured body. An extension brace, such as the Jewett brace, may be indicated to provide pain relief, but it will not prevent bony retropulsion and subsequent neurologic compromise.

82. In the orthotic and prosthetic clinic, a patient with Parkinson's disease presents for gait analysis. The primary gait disturbance found is an alternation in stride length and altered cadence. A gaittraining program for a person with Parkinson's disease should include (a) visual cueing techniques. (b) bilateral ankle-foot orthoses. (c) patterning techniques. (d) vestibular stimulation exercises.

82 (a) Treatment options for gait disturbance in patients with Parkinson's disease include visual and auditory cueing, in addition to traditional endurance and strengthening exercises.

83. Which of the following is associated with a good outcome following surgical decompression for spinal stenosis? (a) Leg more than back pain (b) Long duration (more than 4 years) of symptoms (c) Minimal constriction of the spinal canal (d) Multilevel spondylosis and degenerative joint disease

83 (a) Prognostic factors associated with good surgical outcome following surgical decompression include pronounced constriction of the spinal canal (<6mm anteroposterior diameter), prominent leg rather than back symptoms, symptom duration of less than 4 years, and absence of concomitant disease affecting walking ability.

85. The primary advantages associated with standing frames and standing wheelchairs for persons with spinal cord injury include (a) reduction in lower extremity edema. (b) reduction in cost to make a workplace accessible. (c) ease in transportation for everyday use. (d) increased bone density in the hips.

85 (b) Standing wheelchairs are often used as second wheelchairs for a particular activity or vocation. Although their weight has been reduced over the years, they still weigh at least 50lb, which continues to be the main factor precluding their use in everyday mobility. Third-party payors and departments of rehabilitation services have funded standing wheelchairs for persons returning to careers; this can reduce the modifications and costs necessary to make a workplace accessible. Physiologic benefits include decreased spasticity, a reduction in urinary tract infections, and a reduction in pressure ulcers. One issue noted is that most standing wheelchairs do not come to a full 90° position because of instability, which may limit a person's reach. Problems reported by users include ankle instability and lower extremity edema.

86. Patients with head and neck malignancies who are receiving radiation therapy should be instructed that cervical range-of-motion activities should be (a) initiated only when radiation sessions have been completed. (b) performed throughout the course of radiation therapy to prevent contracture. (c) limited to movement planes contralateral to the involved side. (d) initiated no sooner than 1 month after radiation therapy.

86 (b) Radiation therapy causes rapid and potentially irreversible fibrosis of muscle and other soft tissue structures. In order to mitigate the inevitable fibrosis that attends high-dose external-beam radiation therapy delivered to patients with head and neck cancer, cervical range-of-motion exercises should begin at the outset of radiation and be continued through the entire course.

87. The most significant factor contributing to upper limb cumulative trauma disorders is (a) size of the equipment used. (b) age of the person. (c) macrotrauma to the area. (d) repeated forceful exertions.

87 (d) Upper limb cumulative trauma disorders, especially those involving the hand and wrist, most often result from repetitive forceful exertions.

88. Dorsal ulnar cutaneous nerve conduction studies are most useful in differentiating (a) ulnar neuropathy at the elbow from lower trunk plexopathy. (b) ulnar neuropathy at the elbow from ulnar neuropathy at the wrist. (c) lower trunk plexopathy from medial cord plexopathy. (d) ulnar neuropathy at the cubital tunnel from ulnar neuropathy at the ulnar groove.

88 (b) The fibers of the dorsal ulnar cutaneous nerve travel in the lower trunk and the medial cord of the brachial plexus. This nerve travels with the ulnar nerve to the forearm, where it branches off proximal to the wrist and supplies sensation to the ulnar aspect of the dorsum of the hand and wrist. The dorsal ulnar cutaneous sensory nerve action potential amplitude could be decreased with a lesion of the lower trunk, the medial cord, or the ulnar nerve at the elbow or proximal forearm, and would not be useful in differentiating among them. It should be normal in ulnar neuropathy at the wrist.

9. The most useful clinical criterion to distinguish Becker muscular dystrophy from Duchenne muscular dystrophy is (a) creatine kinase values at the time of diagnosis. (b) walking ability during the teen-age years. (c) Gowers' sign and calf enlargement. (d) age at onset of diagnosis.

9 (b) The most useful clinical criterion to distinguish Becker muscular dystrophy (BMD) from Duchenne muscular dystrophy (DMD) is the continued ability of the patient to walk into late teen-age years. Persons with BMD will typically remain ambulatory beyond 16 years. Outlier DMD cases generally stop ambulating between 13 and 16 years of age. Creatine kinase values cannot be used to differentiate DMD from BMD. Calf enlargement and the presence of Gowers' sign are a nonspecific findings. Studies have shown significant overlap in the observed age at onset between DMD and BMD.

91. A 33-year-old female homemaker presents with pain in the area of the left "anatomic snuff box" for the last 2 weeks. She has been knitting a blanket for her pregnant sister, and she reports that the pain has significantly increased since she began this project. On examination, she has exquisite tenderness to palpation at the site of the snuff box. There is no erythema, but mild swelling is noted in the painful area. Which of the following statements is true about the patient's condition? (a) The inflamed tendon sheath should be injected with a corticosteroid and her hand placed in a resting splint. (b) The tendons most likely to be involved are the abductor pollicis brevis and the extensor pollicis brevis. (c) This condition is so severe that it warrants referral to a hand surgeon. (d) She will have a negative Finkelstein's test.

91 (a) These symptoms are consistent with stenosing tenosynovitis, commonly referred to as deQuervain's tenosynovitis. The patient should have tenderness over the common tendon sheath for the abductor pollicis longus and the extensor pollicis brevis. Finkelstein's test is confirmative of this diagnosis. It consists of flexing the thumb under cupped fingers and flexing the wrist in an ulnar direction, stretching the thumb tendons. This condition usually responds to conservative management of steroid injection and placement in a splint. Severe refractory cases may require surgical release, but not until at least a month of conservative therapy has been tried.

93. The most successful treatment for adolescent idiopathic scoliosis with an upper thoracic spinal curve of 30( is (a) electrical stimulation. (b) physical therapy. (c) observation. (d) Milwaukee brace for 23 hours daily.

93 (d) A meta-analysis of 1910 patients with adolescent idiopathic scoliosis evaluated the effects of observation, electrical stimulation, and bracing. Of the six brace types used, the highest success was seen with the Milwaukee brace. Bracing for 23 hours was significantly more successful than any other treatment.

94. A 70-year-old man with first-degree heart block presents with a painful distal sensory neuropathy. Of the following, the most appropriate initial treatment is (a) fluoxetine. (b) gabapentin. (c) oxycodone. (d) trazodone.

94 (b) Tricyclic antidepressants such as amitriptyline and nortriptyline have traditionally been the initial drug sof choice but can lead to cardiac conduction block. Newer anticonvulsants such as gabapentin are becoming the preferred treatment. Although selective serotonin reuptake inhibitors such as fluoxetine can improve depression associated with chronic pain, they have not been shown to have a direct effect on reducing neuropathic pain. Trazodone has not been shown efficacious in the treatment of painful neuropathies. Long-acting narcotics are increasingly being used when other agents fail to adequately control pain.

95. You have been following a person with a T4 spinal cord injury for 20 years. He is now 38, and he presents with recent onset of frequent episodes of incontinence between catheterizations. His intermittent catheterization volumes have increased to 600mL every 4-6 hours (they had been 300mL). He acknowledges an increase in his fluid intake, which he feels accounts for the increased volumes. Your first priority is to order (a) a blood glucose level. (b) a urodynamic study. (c) a basic metabolic panel. (d) a prostate-specific antigen test.

95 (a) Polydipsia and polyuria are strong indicators of new-onset diabetes. Diabetes is more common in the spinal cord injured person than in the uninjured population. Although the differential diagnosis may include detrusor hyperreflexia or urinary tract infection, this would not provide a reason for the new high catheterization volumes in this person with chronic spinal cord injury. Evaluation for diabetes should be done immediately.

96. In general, the risk of fracture is lower for blastic metastases than for lytic metastases. Which one of the following tumors tends to form blastic metastases? (a) Lung (b) Multiple myeloma (c) Prostate (d) Renal cell

96 (c) In general, lytic lesions are considered more prone to fracture, although blastic lesions are not immune to fracture. Lytic lesions typically occur with primary or metastatic lesions of the following malignancies: breast, lung, kidney, thyroid, gastrointestinal tumors, neuroblastoma, lymphoma, and melanoma. Prostate cancer tends to form blastic metastases.

97. A 23-year-old painter is recovering from a rotator cuff injury after falling off a ladder. His impairment would be influenced by (a) the physical demands of his job. (b) how he injured the shoulder. (c) the treatment required. (d) the extent of limitation of shoulder range of motion.

97 (d) Impairment ratings are based on physical examination findings such as joint limitations and muscle atrophy and function loss.

99. The most severe form of mental retardation in cerebral palsy occurs in association with (a) spastic diplegia. (b) ataxia. (c) spastic quadriplegia. (d) athetosis.

99 (c) Mental retardation is the most common serious associated disability in cerebral palsy. The overall incidence of mental retardation is approximately 30%-50%. Severe mental retardation is present in about one-half of the retarded group. Approximately one-third of cases have mild cognitive deficits. The greatest retardation is seen in rigid, atonic, and severe spastic quadriplegic cerebral palsy.


Conjuntos de estudio relacionados

Milady Chapter 19: Advanced topics and treatments

View Set

PHYSIO: chapter 7 practice questions

View Set